Capstone Exam 2

¡Supera tus tareas y exámenes ahora con Quizwiz!

LV P/V Loop Changes for: A) ↑Afterload: ↓SV, ↑ESV e.g.) Aortic stenosis, hypertension B) ↑Preload (=↑EDV): ↑SV e.g.) ↑ Blood volume, venoconstriction C) ↑Contractility: ↑SV, ↑EF, ↓ESV e.g.) Catecholamine, [Ca2+]i

Remember mnemonics about how shapes of loops look like letters.

Mentzer Index (MCV/RBC): - If <13, what is the diagnosis? - If >13, what is the diagnosis?

< 13 is iron deficiency anemia (e.g., MI = 20) - because low RBCs and esp. low MCV > 13 is thalassemia trait (e.g., MI = 10) - high or normal - e.g., people with beta thalassemia trait will have elevation in hemoglobin A2 and/or F (fetal) Thalassemia = Inherited defect in the ability to produce a certain type of hemoglobin

APP and ApoE4 are genetic causes of (and BUZZwords for) what condition?

Alzheimer's Disease

Von Willebrand Disease: - vWF carries which factor - Therefore, VWD involves a prolonged PT or PTT?

Carries Factor 8 in the blood Therefore, VWD involves prolonged PTT, but a normal PT. VWD = Defect in platelet plug formation due to deficient platelet-vWF interactions on endothelium.

Hypersensitivity - Type 4 (ACID mnem)

D = delayed type (T-cell mediated) T-cells only - no antibodies, so it takes a while and is not transferable through serum.

+++++++++++++++++++++++++++++++++ Statins lower cholesterol by blocking what enzyme?

HMG-CoA Reductase Block the cholesterol synthesis pathway They reduce LDL and increase HDL

Histology finds Negri Bodies in brain. Diagnosis?

Rabies

If a prolonged QT is not fixed, it can eventually result in

Torsades de Pointes

Hypersensitivity - Type 1 (ACID mnem)

A = anaphylactic

What type of cardiomyopathy causes sudden death in athletes?

Hypertrophic Cardiomyopathy (HCM) Diastolic problem (unlike systolic problem in dilated CM)

___ of the liver is the isozyme of (same thing as) hexokinase. (Glycolysis)

Glucokinase is in the liver (and pancreatic beta islet cells) Hexokinase is in all other tissues Both catalyze the same glycolysis reaction (first rxn): Glucose to Glucose-6-Phosphate

Status of PTH, Ca2+, and Phosphate in: 1. Hypoparathyroidism 2. Pseudohypoparathyroidism 3. Pseudopseudohypoparathyroidism What phenotype is present in 2 and 3?

Hypoparathyroidism ↓ PTH ↓ Serum Ca2+ ↑ Phosphate ↑ urinary phosphate after PTH administration Pseudohypoparathyroidism AHO Phenotype (Albright Hereditary Osteodystrophy) ↓ Serum Ca2+ ↑ PTH, but kidney does NOT respond to PTH ↑ Phosphate Pseudopseudohypoparathyroidism AHO phenotype, but... NORMAL Ca2+, Phosphate, and PTH. Kidney does respond to PTH. So not even really hypoparathyroidism.

Hypersensitivity - Type 3 (ACID mnem)

I = Immune complex mediated Ag + IgG = complement activated

What medication is used to close a Patent Ductus Arteriousus

Indomethacin

Leukocyte Adhesion Deficiency Chediak-Higashi syndrome Chronic granulomatous disease Myeloperoxidase deficiency These all involve problems with the function of ...

Leukocytes So they'll get sick a lot.

What medications treat HFrEF (systolic HF)

Mainly, give an ACE-I and Beta Blocker Also, Spironalactone and ARBs are helpful. See Image to see why Digoxin can be helpful in some as it ↑Contractility and ↓AV conduction. HFrEF: EF is 40% or lower

Nitrates have what effect on vasculature. And they selectively do this to what type of vessel?

Nitrates are venodilators. They selectively dilate veins.

How do you treat Polymyositis and Dermatomyositis

These are inflammatory disorders of the muscles, so mainly Steroids (Prednisone) Overall options: - Prednisone - Methotrexate - Azathioprine - Mycophenolate mofetil - IV IgG - Plasmapheresis

ESR elevation usually indicates one of what 4 processes?

• Fibrinogen and other inflammatory mediators coat RBCs => heavier RBC's => settle faster PICS: - Pregnancy - Infection - Cancer - SLE • Doesn't happen in: - Sickle cell (altered shape prevents settling) - Polycythemia (too many RBC's relative to plasma)

What is a Single Nucleotide Polymorphism (SNP)?

A single nucleotide that differs between members of a species. Very common -- what makes us unique. But some of them are bad; tend to make you more susceptible to disease rather than causing diseases.

Classic translocation in Follicular Lymphoma (one of the most common form of non-hodgkin's lymphoma)

t(14;18) - This mutation activates BCL-2, resulting in loss of apoptosis, leading to cell proliferation and cancer. Most common form of non-hodgkin's lymphoma

What all do you need to give a pt with Hypopituitarism to treat all of the manifestations?

• GH is used for treatment in children, but hypo-pituitary adults are not normally given GH • Cortisol is used for ACTH • T4 for TSH • Testosterone for men not interested in fertility • Estrogen and progesterone for women not interested in fertility • Gonatotrophins or pulsatile GnRH when fertility is the goal • Prolactin is not available commercially for treatment

Beck's Triad for cardiac tamponade What is a cardiac tamponade? How does it present?

(1) Hypotension + Tachycardia - Due to difficulty filling heart. (2) Muffled / Distant Heart sounds - due to accumulation of fluid in pericardium -- harder to hear heart. (3) Jugular Venous Distension (JVD) - due to blood backing up into jugular vein -- heart is hard to fill. Physical Exam: - Tachycardia - Hypotension - PULSUS PARADOXUS (↓BP even more on inspiration) Cardiac tamponade is caused by the accumulation of fluid in the pericardial space, resulting in reduced ventricular filling. Usually caused by any trauma resulting in blood loss / hemorrhage. Cardiac Tamponade: - Accumulation of fluid in the pericardial sac that restricts ventricular filling. - Total blood volume in the heart is stable, but volumes in individual chambers are not. Blood can shunt to different chambers. ^ Unlike pericarditis, where chamber volumes are fixed by the crusty pericardium. - E.g., A 19-year-old man is stabbed in the left side of his chest (trauma / hemorrhage). His blood pressure on presentation is 90/60 mmHg with a pulse of 130/min. On physical exam, he has muffled heart sounds (hard to hear from fluid)and distended jugular veins. Upon inspiration, his blood pressure drops to 75/55 mmHg (pulsus paradoxus). His extremities are cool and clammy. In both cardiac tamponade and pericarditis, blood backs up into jugular, causing JVD.

USMLE 2015 Sample Question A 52-year-old man comes to the emergency department because he has had vomiting, nausea, and abdominal pain for the past 12 hours. He says he attempted suicide 3 days ago by "taking everything in the medicine cabinet." He was stuporous for approximately 12 hours after the overdose but felt better the following day. At this time, he has jaundice and pain in the right upper quadrant. Which of the following drugs is most likely to have caused the pain, vomiting, and jaundice? (A) Acetaminophen (B) Aspirin (C) Cimetidine (D) Diphenhydramine (E) Triazolam

(A) Acetaminophen This is classic acetaminophen toxicity.

A 52-year-old woman comes to the physician because of a 1-month history of headache, weakness, tingling of her extremities, muscle cramping, and fatigue. Her blood pressure is 170/110 mm Hg. Physical examination shows no other abnormalities. Laboratory studies show a decreased serum potassium concentration, metabolic alkalosis, and decreased plasma renin activity; serum sodium concentration is within the reference range. Urine catecholamine concentrations are within the reference range. Which of the following is the most likely diagnosis? (A) Adrenal adenoma (B) Focal segmental glomerulosclerosis (C) Hypothalamic tumor (D) Juxtaglomerular cell tumor (E) Renal artery stenosis

(A) Adrenal adenoma - Primary Hyperaldosteronism This patient has Conn syndrome (High Aldosterone from an adrenal tumor). Muscle weakness, HTN, low K+, low renin, and metabolic alkalosis are classic signs and symptoms. She likely has an adrenal adenoma that is secreting aldosterone. Tx with surgery or spironoloactone

USMLE 2015 Sample Question A 22-year-old man being treated with haloperidol for schizophrenia develops delusions, flattening of affect, catatonic behavior, hallucinations, and aphasia. Which of the following symptoms would be more likely to improve if this patient were treated with clozapine rather than with haloperidol? (A) Affective flattening and aphasia (B) Affective flattening and hallucinations (C) Aphasia and delusions (D) Catatonia and delusions (E) Hallucinations and catatonia

(A) Affective flattening and aphasia Take home point: NBME writers focus on negative symptoms for clozapine Clozapine is an atypical antipsychotic, along with Olanzapine and Risperidone. Typical Antipsychotics (Haloperidol) primarily treat positive symptoms. Atypical Antipsychotics (Clozapine) treat both positive and negative symptoms. FYI, Aphasia is the inability to produce (or understand) speech -- hence Broca's Aphasia and Wernicke's Aphasia.

A 17-year-old girl has never had a menstrual period. Physical examination shows a normal female body habitus, normal breast development, and normal appearing external genitalia. She has no axillary or pubic hair. The patient refuses to have a pelvic or rectal examination. Which of the following is the most likely explanation for the clinical presentation? (A) Androgen insensitivity (B) Congenital adrenal hyperplasia (C) Ectodermal dysplasia (D) A psychiatric disorder (E) A sex chromosome mosaicism

(A) Androgen insensitivity = Phenotypically Female, but no female internal parts. No ovaries, uterus, etc. - Would have been male, but androgen receptors were absent. She lacks the secondary sex characteristic of axillary and pubic hair as this is androgen driven. She is otherwise phenotypically normal. Congenital Adrenal Hyperplasia is the umbrella term for the 21, 11, and 17 hydroxylase deficiencies.

A 36-year-old woman has been trying to conceive for the past 2 years. Her menses occur every 19 to 45 days. She has a past history of some type of sexually transmitted disease, but says she was treated and cured. She is 163 cm (5 ft 4 in) tall and weighs 109 kg (240 lb); BMI is 41 kg/m2. Physical examination including a pelvic examination is unremarkable. An endometrial biopsy is performed based on the menstrual history and a negative pregnancy test. The biopsy shows stromal breakdown associated with proliferative glands. Which of the following is the most likely cause of her infertility? (A) Anovulation (B) Chronic endometritis (C) Endometrial polyps (D) Endometriosis (E) Leiomyoma

(A) Anovulation She is obese. With a BMI of 41, there is excess estrogen, which can prevent / suppress regular ovulation (not menses; happen 19 to 45 days). The endometrium grows b\c of the estrogen (which is why it is proliferative) and microclots secondary to excess estrogen cause local ischemia and breakdown. All that plus anovulation is one of the most common causes of infertility. There are no diagnostic clues for the other options.

A 15-year-old girl is brought to the physician because of a 3-week history of excessive thirst and voiding excessive amounts of urine. She shows no signs of kidney damage, and she is not taking any medications. Physical examination shows no abnormalities. She undergoes an 8-hour water deprivation test. She is also given 5 units of ADH (vasopressin), subcutaneously. Under both conditions, she continues to produce large volumes of dilute urine. Her symptoms are most likely due to a relative lack of which of the following enzymes from the apical membranes of collecting duct epithelial cells? (A) Aquaporin (B) Epithelial Na+ channel (C) Na+-K+ ATPase (D) Na+-K+-2Cl− cotransporter (E) Urea transporter

(A) Aquaporin Nephrogenic DI is caused by a lack of Aquaporin channels, which reabsorb water in response to ADH. Water is not reabsorbed, so always urinating and always thirsty.

A 16-year-old boy comes to the physician because of a rash on his left inner thigh that first appeared 2 days after he returned from a hunting trip with friends in Minnesota. The rash is large, with a target-like pattern. He is diagnosed with Lyme Disease. Without treatment, this patient is at increased risk for which of the following? (A) Carditis (B) Glomerulonephritis (C) Hepatitis (D) Pancreatitis (E) Thrombocytopenia MNEMONIC for manifestations of Lyme Disease: CAFE

(A) Carditis Lyme disease. They told you he was in the woods in the north (or northeast) and he came back with a rash. The only thing that will happen on that list is the carditis / cardiac block. MNEMONIC: CAFE: C - Cardiac Block A - Arthritis F - Facial nerve palsy E - Erythema Migrans

An otherwise healthy 45-year-old man comes to the physician because of a 3-week history of progressive epigastric heartburn and a 4.5-kg (10-lb) weight loss. The pain tends to be more severe at night and occurs 1 to 3 hours after meals during the day. He has had similar episodes with lesser intensity during the past year. Abdominal examination shows tenderness to deep palpation. Test of the stool for occult blood is positive. Endoscopy shows a bleeding 3-cm ulcer in the antrum of the stomach. A photomicrograph of Steiner silver-stained tissue (400x) from a biopsy of the gastric mucosa adjacent to the ulcer is shown. Which of the following processes is most likely to be involved? (A) Elaboration of proteases and urease with local tissue destruction (B) Hyperacidity and gastric ulcer development (C) Ingestion of preformed toxins in contaminated well water (D) Spirochete invasion of gastric cells

(A) Elaboration of proteases and urease with local tissue destruction So here we have a person with a peptic ulcer in their stomach. What we know about these is that they are almost always associated with H. pylori. Additionally, we know gastric ulcers are usually not due to increased acid (unless it is ZE syndrome) but due to a lack of protection, which is what H. pylori erodes.

A 15-year-old girl who is a ballet dancer has not had a menstrual period for the past 3 months. Menses were previously regular at 29-day intervals. She has lost weight over the past year; her weight is 70% of that expected for her height. She is afebrile and has purpuric lesions on her extremities and trunk. Platelet, absolute neutrophil, and lymphocyte counts are below the reference range. She has macrocytic anemia. The most likely cause of these symptoms is a deficiency of which of the following nutrients? (A) Folic acid (B) Iron (C) Linoleic acid (D) Magnesium (E) Niacin (F) Protein

(A) Folic acid Trigger words include ballet dancer, weight 70% of expected, macrocytic, and the lack of neurologic deficiencies.

A 16-year-old boy is admitted to the emergency department because of a knife wound to the left side of his chest. An x-ray of the chest shows an air-fluid level in the left side of the chest, partial collapse of the left lung, and elevation of the stomach bubble. The mediastinum is in the midline. Which of the following is the most likely diagnosis? (A) Hemopneumothorax, not under tension (B) Hemothorax, not under tension (C) Pneumothorax, not under tension (D) Tension hemopneumothorax (E) Tension hemothorax (F) Tension pneumothorax

(A) Hemopneumothorax, not under tension Logic: > There is an air-fluid level (due to blood from trauma entering lungs), so hemopneumothorax. A pneumothorax would only have air and a hemothorax would only have blood. That brings you to A and D. > Since there is no mediastinal shift, it's not under tension, so the answer is A. "Tension" in lung pathology always means there's a mediastinal shift. FYI: Air-Fluid Level- Dome-shaped dark area (air) in lungs, surrounded by white (fluid). The bottom of the dark area is a flat line -- that's the "air-fluid level" -- line between air and fluid.

A 33-year-old woman comes to the physician because of a 2-day history of mild nausea, increased urinary urgency and frequency, and constipation. She also has had a 4.5-kg (10-lb) weight loss during the past 2 weeks and a 3-week history of vaginal bleeding. Pelvic examination shows a nodular cervix with an irregular, friable posterior lip, and a rock-hard, irregular, immobile pelvic mass that extends across the pelvis. Examination of biopsy specimens from the cervix and anterior wall of the vagina show well-differentiated keratinizing squamous cell carcinoma. Which of the following best describes the pathogenesis of this patient's disease? (A) Inactivation of cellular p53 (B) Insertion of viral promotors adjacent to cellular growth factor genes (C) Specialized transduction (D) Transactivation of cellular growth factor genes by TAX (E) Translocation of CMYC to an Ig gene promoter

(A) Inactivation of cellular p53 This patient has cervical SCC. As you learned this is caused by HPV inserting E6 and E7 into the genome. This causes inactivation of p53 (E6) and Rb (E7). Typical 2-3 level question in that 1st you have to know it is HPV and 2nd you have to know about E6/E7 and 3rd you have to know what they inactivate!

+++++++++++ VERY COMMON STEP TOPIC +++++++++++ A 1-day-old newborn is evaluated for possible sepsis. Blood cultures grow gram-positive cocci in pairs and chains that agglutinate with group B antiserum. The most likely epidemiologic risk factor for this infection involves bacterial colonization of which of the following? (A) Mother's vagina (B) Newborn's gastrointestinal tract (C) Newborn's nasopharynx (D) Placenta (E) Umbilical cord remnant

(A) Mother's vagina Group B Strep is a very common infectious question. Every pregnant woman is screened for GBS during pregnancy. The transmission mode is via the birth canal.

A 23-year-old woman has a progressive increase in her serum β-human chorionic gonadotropin (β-hCG) concentrations during an 8-week period. A hydatidiform mole is removed, but the β-hCG concentration continues to increase. Which of the following is the most likely diagnosis? (A) Adrenal adenoma (B) Choriocarcinoma (C) Ectopic pregnancy (D) Pituitary insufficiency (E) A second noninvasive mole

(B) Choriocarcinoma This is assuming the entire mole was removed of course. What they are getting at here is the association between moles (complete moles) and choriocarcinoma. While C and E could technically happen, the odds are astronomically low.

A 75-year-old woman has increasing shortness of breath on exertion. Findings on physical examination are unremarkable. X-rays of the chest show no abnormalities of the heart or lungs. Pertinent laboratory findings include: - Hematocrit 28% (LOW) - Hemoglobin 9 g/dL (LOW) - Mean corpuscular volume 70 μm3 (slightly LOW) Which of the following is the most likely basis for these findings? (A) Acquired hemolytic anemia (B) Chronic blood loss (C) Folic acid deficiency (D) β-Thalassemia minor (E) Pernicious anemia

(B) Chronic blood loss Microcytic, hypochromic anemia in an old patient...bleeding from colon cancer must be ruled out (and considered first). Of note, C and E would cause MCV >100.

A 2-year-old girl is brought to the emergency department because of pain in her right forearm after a fall 1 hour ago. She has a history of fractures of the left femur and right tibia. Physical examination shows blue sclerae. There is tenderness to palpation over the distal right radius. 1. Diagnose 2. A mutation in which of the following genes is the most likely cause of the recurrent fractures in this patient? (A) Calcitonin (B) Collagen, type I (C) 1α-Hydroxylase (D) Parathyroid hormone (E) Vitamin D receptor 3. What are the common symptoms of this disease? (MNEM: Can't BITE) MSK Pathology

(B) Collagen, type I This is Osteogenesis Imperfecta (Brittle Bone Disease) - Blue Sclerae is the BUZZword - Also, multiple bone fractures - Brittle Bones FIRST AID: Osteogenesis Imperfecta - Genetic bone disorder (brittle bone disease) caused by a variety of gene defects (most commonly COL1A1 and COL1A2). - Caused by decreased production of otherwise normal type I collagen. - Manifestations include: Can't BITE: Bones: multiple fractures from minor trauma I (eye): blue sclerae Teeth: dental imperfections Ear: hearing loss - Multiple fractures and bone deformities after minimal trauma (eg, during birth) - Blue sclerae due to the translucent connective tissue over choroidal veins - Some forms have tooth abnormalities, including opalescent teeth that wear easily due to lack of dentin (dentinogenesis imperfecta) - Conductive hearing loss (abnormal ossicles)

USMLE 2015 Sample Question An investigator is studying the incidence of the common cold among medical students at various time points during the school year. Results show an increased incidence of upper respiratory tract infections among these students during finals week. It is hypothesized that the stress of studying for examinations adversely affects the immune system, making the students more susceptible to infection. Which of the following laboratory findings in these students during examination week is most likely to support this hypothesis? (A) Decreased AM serum cortisol concentration (B) Decreased macrophage activity (C) Increased basophil count (D) Increased lymphocyte count (E) Increased natural killer cell activity

(B) Decreased macrophage activity With stress, they usually look for increased cortisol, but decreased macrophage activity is part of stress response as well. Definitely not any of the other options.

A 7-month-old infant is brought to the physician's office because of poor weight gain despite large food intake. He has had two episodes of pneumonia and has frequent bulky stools. He coughs frequently. X-rays of the lungs show increased markings and hyperinflation. Trypsin is absent in a fresh stool sample, and the fat content is increased. Which of the following is the most likely cause of this infant's disorder? (A) Autoimmune disorder (B) Defective ion transport at epithelial surfaces (C) Disaccharidase deficiency (D) Inability to synthesize apolipoprotein B (E) Villous atrophy of the jejunum

(B) Defective ion transport at epithelial surfaces This is Cystic Fibrosis This is a question about the lung manifestations of CF. The bulky stools and presence of fat are from pancreatic deficiency. The mechanism is the chloride transporter.

USMLE 2015 Sample Question A 17-year-old girl is brought to the physician by her mother because she has not had a menstrual period for 6 months. The patient is unconcerned about the lack of menses. Menarche occurred at the age of 12 years, and menses had occurred at regular 28-day intervals until they became irregular 1 year ago. She is a member of her high school gymnastics team. She appears emaciated. She is 163 cm (5 ft 4 in) tall and weighs 40 kg (88 lb); BMI is 15 kg/m2. Her pulse is 54/min, and blood pressure is 80/50 mm Hg. Which of the following is the most likely cause of this patient's amenorrhea? (A) Hyperthyroidism (B) Hypogonadotropic hypogonadism (C) Hypothyroidism (D) Polycystic ovarian syndrome (E) Prolactinoma

(B) Hypogonadotropic hypogonadism Diagnosis: Anorexia Nervosa - Low body weight causes decreased release of LH, leading to deficiency of estrogen and androgens. This happens because the body does not have enough fat or nutrients to nourish a fetus. What to do: Hospitalize (or at least intensive outpatient tx with family therapy) Cause of death: arrhythmia

USMLE 2015 Sample Question A 26-year-old woman is brought to the emergency department by her mother 1 hour after she had a generalized tonic-clonic seizure at home. The mother states that her daughter has been talking to herself at all hours of the day and night about being thirsty. The patient was found to have schizophrenia 6 years ago. Her only medication is haloperidol. She was admitted to the hospital twice in the past year for psychotic episodes. Physical examination shows no other abnormalities. She is oriented to person but not to place or time. Her serum sodium concentration is 114 mEq/L on arrival but returns to normal with appropriate treatment. Which of the following interventions is the most appropriate next step? (A) Refer the patient for behavior therapy (B) Restrict the patient's fluid intake (C) Add lithium carbonate to the medication regimen (D) Administer furosemide therapy (E) Begin mineralocorticoid therapy

(B) Restrict the patient's fluid intake This is psychogenic polydipsia, which is when people, usually with schizophrenia, drink excessive amount of water and get hyponatremic. SIADH from medications must be ruled out.

USMLE 2015 Sample Question A 42-year-old man comes to the physician for a follow-up examination. Four months ago, he underwent repair of a Dupuytren contracture. Physical examination shows decreased range of motion in the affected hand. The patient is upset that his hand has not fully healed, and he files a malpractice suit against the physician. Which of the following is the most likely precipitating factor in this patient's decision to file a malpractice suit? (A) The patient's perception that the physician is incompetent (B) The patient's perception that the physician is uncaring (C) The patient's socioeconomic status (D) The physician's amount of experience in the medical field (E) The physician's inability to screen out problem patients

(B) The patient's perception that the physician is uncaring

A 25-year-old woman is brought to the emergency department 1 hour after she fainted. Over the last 3 days, she has had mild intermittent vaginal bleeding, sometimes associated with lower abdominal pain. She has had severe cramping pain in the right lower abdomen for 12 hours. She has not had a menstrual period for 3 months; previously, menses occurred at regular 28-day intervals. Abdominal examination shows mild tenderness to palpation in the right lower quadrant. Bimanual pelvic examination shows a tender walnut-sized mass in the right parametrium. Which of the following is the most likely diagnosis? (A) Appendicitis (B) Cancer of the ovary (C) Ectopic pregnancy (D) Endometriosis (E) Ovarian cyst (F) Placenta previa

(C) Ectopic pregnancy RLQ mass could be appendicitis, which is in the DDX, but the lack of a period screams pregnancy. Previa could cause the bleeding, but not the mass and pain (it is usually painless).

A previously healthy 19-year-old woman comes to the physician because of a 3-day history of fever, fatigue, and sore throat. She lives with a roommate who has a cat. Her temperature is 37.8°C (100°F). Physical examination shows mildly tender cervical and submental adenopathy and pharyngitis. There is mild splenomegaly. Laboratory studies show: H / H 13.3 g/dL / 43% Leukocyte count 12,500/mm3 Platelet count 250,000/mm3 Heterophile antibody titer positive The most likely cause of this patient's condition is infection with which of the following? (A) Bartonella henselae (B) Cytomegalovirus (C) Epstein-Barr virus (D) Rhinovirus (E) Toxoplasma gondii

(C) Epstein-Barr virus This is Mono Heterophile antibody positive = EBV. Don't let them fool you with the presence of a cat!!! Slightly tricky because we think of mono as profound fatigue, and here it is buried in the stem. The splenomegaly is another nice hint that it is mono

A 42-year-old woman is brought to the emergency department 4 hours after the onset of severe shortness of breath. She has no recent history of trauma, hospital admission, or operations. She had an episode of deep venous thrombosis 10 years ago that required treatment in the hospital. Her respirations are 34/min. Pulse oximetry on room air shows an oxygen saturation of 65%. A helical CT scan shows a large filling defect in the right pulmonary artery. Which of the following hypercoagulability disorders is the most likely underlying cause of these findings? (A) Antiplatelet antibody syndrome (B) Antithrombin III deficiency (C) Factor V Leiden mutation (D) Protein C deficiency (E) Protein S deficiency

(C) Factor V Leiden mutation The patient is having a PE. The most common cause of hypercoaguability in an otherwise normal patient is Factor V Leiden. Pick it first in a person with an unexplained clot! First Aid: Factor V Leiden deficiency Autosomal dominant, most common cause of inherited hypercoagulability in Caucasians. Production of mutant factor V that is resistant to degradation by protein C. Complications include DVT, cerebral vein thrombosis, recurrent pregnancy loss.

A 66-year-old man has become increasingly short-tempered with his wife. He has diarrhea, weight loss, and weakness in the proximal muscles. He has atrial fibrillation and tachycardia. Which of the following is the most likely diagnosis? (A) Congestive heart failure (B) Cushing syndrome (C) Hyperthyroidism (D) Mitral valve prolapse (E) Pheochromocytoma

(C) Hyperthyroidism Of all the things here, hyperthyroidism is the best fit. The agitation, weight loss, A Fib and tachycardia can all be attributed to that. Pheo could fit some of these symptoms, but is way more uncommon than hyperthyroidism. MVP can cause anxiety and A Fib, but not the rest of the symptoms. He lacks the other classic findings of Cushings.

A 31-year-old woman comes to the physician because of a 2-week history of malaise, nausea, vomiting, and decreased appetite. She is a known user of intravenous heroin. She appears chronically ill. She is 165 cm (5 ft 5 in) tall and weighs 47 kg (103 lb); BMI is 17 kg/m2. Her temperature is 36.7°C (98.1°F), pulse is 90/min, respirations are 18/min, and blood pressure is 114/68 mm Hg. Physical examination shows scleral icterus and a liver span of 16 cm. The spleen is not palpable. Serum studies show: Total bilirubin 3.2 mg/dL AST 774 U/L ALT 820 U/L HIV antibody negative Hepatitis B surface antigen negative Hepatitis B surface antibody positive Anti-hepatitis B core antibody positive Hepatitis B DNA negative Anti-hepatitis C virus positive Hepatitis C RNA positive Which of the following is the most likely outcome of this patient's infection? (A) Complete resolution of infection (B) Latent infection with intermittent viremia (C) Lifelong persistent infection (D) Patient death from acute infection

(C) Lifelong persistent infection This patient has hepatitis B and C. She will probably get cirrhosis and possibly cancer, but that is not an option. These would be a result of the likely lifelong infection that she will have.

A 47-year-old man comes to the physician because of a 1-week history of temperatures to 38.3°C (101°F) and occasional vomiting. He also has a 1-year history of joint and muscle pain in his calves and a 1-month history of intermittent, diffuse abdominal pain. His temperature now is 37.2°C (99°F). Abdominal examination shows mild diffuse tenderness. There is no ascites. Test of the stool for occult blood is positive. Serum studies show mildly increased urea nitrogen and creatinine concentrations. Which of the following is the most likely diagnosis? (A) Angiodysplasia (B) Mucocutaneous lymph node syndrome (Kawasaki disease) (C) Polyarteritis nodosa (D) Takayasu arteritis (E) Thromboangiitis obliterans (F) Wegener granulomatosis

(C) Polyarteritis nodosa The guy has GI and renal involvement. Should have "string of pearls appearance" in the renal artery No hint of heart (B), carotid (D) or lung/airway (F) involvement.Also no mention of smoking (E). Polyarteritis Nodosa is a vasculitis of the small to medium sized arteries. Signs and symptoms are nonspecific, and result from damage to the organs that are fed by the affected vessels. Musculoskeletal, cutaneous, gastrointestinal, cardiac, renal, and neurologic symptoms can be seen. Polyarteritis nodosa is associated with hepatitis B virus infection in up to 30% of cases. Patients often have a skin rash known as livedo reticularis, which is the purplish discoloration described in this patient. Additionally, patients may have anemia, elevated erythrocyte sedimentation rate or C-reactive protein level, and neutrophilic leukocytosis.

A 10-year-old boy is brought to a new physician by his parents for an initial examination. The patient was born with congenital glaucoma, hearing loss, and a patent ductus arteriosus that has since been surgically corrected. At the time of birth, he also had purpura, jaundice, and splenomegaly. He is in a special education class for students with mental retardation. He is at the 75th percentile for height and weight, and 5th percentile for head circumference. This patient's condition was most likely caused by in utero exposure to which of the following? (A) Cytomegalovirus infection (B) HIV infection (C) Rubella (D) Syphilis (E) Toxoplasmosis

(C) Rubella PDA is a "DEAD GIVEAWAY" for Rubella. Rubella causes PDA. You have to know this to get the Q correct. Jaundice and purpura are also c/w Rubella. Syphilis and CMV are close seconds, as they also present with many neuro / head problems.

A 40-year-old woman comes to the physician because of a 6-month history of increased facial hair growth. Her last menstrual period was 4 months ago. She is 165 cm (5 ft 5 in) tall and weighs 70 kg (154 lb); BMI is 26 kg/m2. Her pulse is 80/min, and blood pressure is 130/82 mm Hg. Physical examination shows temporal balding and coarse dark hair on the upper lip and chin. Pelvic examination shows clitoral enlargement. Her serum testosterone concentration is increased. Serum concentrations of androstenedione, dehydroepiandrosterone, and urinary 17-ketosteroids are within the reference ranges. Ultrasonography of the pelvis shows a 12-cm ovarian mass. Which of the following best describes this mass? (A) Granulosa tumor (B) Ovarian carcinoid (C) Sertoli-Leydig cell tumor (D) Teratoma (E) Thecoma

(C) Sertoli-Leydig cell tumor (sex-cord stromal tumor) This patient has signs of virilization. The only tumor in the answer choices that can cause testosterone release is a SLCT. MNEMONIC: Makes sense. Sertoli cells make sperm and Leydig cells make testosterone. So Sertoli-Leydig Cell Tumors secrete testosterone.

USMLE 2015 Sample Question A 38-year-old man who recently immigrated to the USA comes to the physician because of a 1-month history of cough and a 4.5-kg (10-lb) weight loss. Physical examination shows no abnormalities. A chest x-ray shows a right upper lobe infiltrate. One of three sputum samples is positive for acid-fast bacilli. He is diagnosed with Tuberculosis. Treatment with isoniazid, rifampin, ethambutol, and pyrazinamide is started. Which of the following should be added to the medication regimen to prevent neurologic toxicity in this patient? (A) Folic acid (B) Nicotinic acid (C) Vitamin B6 (pyridoxine) (D) Vitamin B12 (cyanocobalamin) (E) Vitamin C

(C) Vitamin B6 (pyridoxine) Anti-tuberculosis medications deplete Vitamin B6.

A 6-year-old boy is brought to the physician by his parents because of a 3-day history of fever, headache, and cough productive of a green, foul smelling discharge that also exits from his nose. He has had repeated episodes of similar symptoms during the past 4 years. He appears pale and lethargic. His height and weight are both below the 10th percentile. Coarse rhonchi are heard bilaterally. An x-ray of the chest shows scattered peripheral opacities, dilated and thickened airways consistent with bronchiectasis, and a cardiac apex that is directed toward the right. The most likely cause of his recurrent infections is a dysfunction of which of the following cell types? (A) Alveolar capillary endothelial cell (B) Alveolar macrophage (C) Chondrocyte (D) Ciliated columnar epithelial cell (E) Clara cell (F) Goblet cell (G) Kulchitsky cell (H) Squamous epithelial cell (I) Type I pneumocyte (J) Type II pneumocyte

(D) Ciliated columnar epithelial cell Classic Kartageners syndrome (dysmotile cilia and situs inversus). The patient already has bronchiectasis from what has probably been multiple bouts of pneumonia. In females this may lead to infertility due to dysmotility in the fallopian tube cilia.

A 3-year-old boy is brought to the physician because of fever, headache, altered mental status, and sores on his back and left shoulder for the past day. His temperature is 37.8°C (100°F). Physical examination shows vesicles over the back and left shoulder. Treatment with aspirin is CONTRAINDICATED in this patient because of an epidemiologic association with a syndrome that includes which of the following adverse effects? (A) Bronchoconstriction (B) Disseminated intravascular coagulation (C) Gastric irritation (D) Hepatomegaly (E) Immunosuppression

(D) Hepatomegaly Reye's Syndrome - why you NEVER give a child aspirin, especially if they have a viral infection. - Defined as acute, non-inflammatory hepatic encephalopathy resulting in fatty liver and cerebral edema. Don't give kids with a viral infection / fever aspirin. It could cause Reye's syndrome. The only choice here that is related is hepatomegaly. E.g., MedBullets: A 7-year-old girl is brought to the emergency room for lethargy and altered mental status. She was born at full term and previously healthy. She recently complained of feeling muscle soreness, headaches, and some nausea. A few of her classmates at school had tested positive for influenza A, and she was instructed to stay home from school during this time. Her parents gave her an aspirin herself this morning for her symptoms. On physical exam, she is noted to be obtunded and had hepatomegaly. On laboratory evaluation, she had elevated liver enzymes. She is admitted to the intensive care unit for supportive care and careful monitoring of her intracranial pressure.

A previously healthy 40-year-old man is brought to the emergency department because of constant substernal chest pain for 12 hours that is exacerbated by coughing and inspiration and relieved with sitting up and leaning forward. There is no family history of heart disease. His temperature is 38°C (100.4°F), pulse is 120/min, and blood pressure is 110/60 mm Hg. The lungs are clear to auscultation. Cardiac examination shows distant heart sounds. An ECG shows diffuse ST-segment elevation in all leads. An x-ray of the chest shows normal findings. The most likely cause of his condition is injury to which of the following tissues? (A) Aortic intima (B) Esophageal sphincter (C) Myocardium (D) Pericardium (E) Pleura

(D) Pericardium Chest pain that is positional in nature is usually due to pericarditis. The distant heart sounds can be due to the accumulation of pericardial fluid. The accumulation of fluid compresses the heart chambers. A friction rub can also be heard with pericarditis. Often includes sxs of right HF, including JVD and peripheral edema, as there's less room in the heart for blood to be.

A 12-year-old girl is admitted to the hospital because of marked shortness of breath, an erythematous rash, and painful, swollen hip and knee joints. She is agitated. A chest x-ray shows an enlarged heart and changes consistent with pulmonary edema. Intractable congestive heart failure develops, and she dies on the second hospital day. 1. This child most likely had a recent history of which of the following? (A) Cyanosis with chest pain (B) Jaundice (C) Meningitis (D) Pharyngitis (E) Skin infection 2. What was the likely original pathogen?

(D) Pharyngitis This is Rheumatic Fever. She probably had strep throat (S. pyogenes / GAS). She is showing the erythema marginatum (rash), arthritis, and sydenham chorea (agitated state). The carditis is evidenced by the X-ray findings. -- JONES Criteria

A 28-year-old man comes to the physician because of a 1-year history of pain with urination that has increased in severity during the past month. He also has had episodes of blood in his urine during the past 5 years. He lived in sub-Saharan Africa until he came to the USA 6 months ago for graduate school. His temperature is 38°C (100.4°F), pulse is 80/min, respirations are 16/min, and blood pressure is 110/84 mm Hg. Physical examination shows suprapubic tenderness. Laboratory studies show: Hemoglobin 12.3 g/dL, Hematocrit 37% Leukocyte count 13,400/mm3 Segmented neutrophils 65% Bands 5% Eosinophils 5% Lymphocytes 22% Monocytes 3% Serum Urea nitrogen 75 mg/dL Creatinine 3.8 mg/dL UA Blood 3+ RBC 200/hpf WBC 100/hpf RBC casts absent WBC casts absent Imaging studies show bilateral hydroureter + hydronephrosis and calcifications within the bladder. A biopsy specimen of the bladder shows marked chronic inflammation with fibrosis and scattered granulomas. Which of the following best explains the biopsy findings? (A) Exposure to a chemical toxin (B) Interstitial cystitis (C) Malacoplakia (D) Schistosomiasis (E) Vesicoureteral reflux

(D) Schistosomiasis Patient from AFRICA with a bladder infection. Notice no casts so not renal. This is classic schistosomiasis. Schistosomas (blood flukes) are the most common cause of bladder cancer and infection (cystitis).

A 64-year-old man comes to the physician because of swelling in his feet for the past 2 years. He says that his skin is dry and itchy and his feet "feel heavy." Which of the following is the most likely cause of his condition? (A) Arteriolar constriction and arteriolar hypertension (B) Arteriolar dilation and venous hypertension (C) Venous constriction and arteriolar constriction (D) Venous hypertension and incompetent valves (E) Venous hypertension and venous constriction

(D) Venous hypertension and incompetent valves Swelling in feet and an old man is venous incompetence (i.e. valve dysfunction). Probably with a side of CHF. The skin changes are due to ischemia.

A 2-week-old female newborn delivered at term is brought to the physician by her mother because of an increasingly severe diaper rash since birth. No congenital anomalies were noted after delivery. Physical examination shows a red and swollen umbilical remnant that has not separated. There are ulcerations of the skin but no purulent exudate in the area of the diaper. A culture of one of the ulcers grows Staphylococcus aureus. Despite antibiotic therapy, 1 month later she develops a perirectal fissure, culture of which grows Escherichia coli but a smear of which shows scarce segmented neutrophils. Laboratory studies now show: Hemoglobin 12.7 g/dL Hematocrit 38% Mean corpuscular volume 98 μm3 Leukocyte count 89,790/mm3 Segmented neutrophils 89% Bands 6% Lymphocytes 3% Monocytes 2% Platelet count 249,000/mm3 IgA 92 mg/dL IgG 766 mg/dL IgM 101 mg/dL A peripheral blood smear shows normochromic, normocytic erythrocytes and leukocytes with normal morpology. This patient most likely has which of the following conditions? (A) Acute myelogenous leukemia (B) AIDS (C) Chédiak-Higashi syndrome (D) Common variable immunodeficiency (E) Leukocyte adhesion deficiency

(E) Leukocyte adhesion deficiency Classic example of a wordy stem. The key here is the high WBC count, but the signs of infection and the umbilical cord not falling off. This is classic LAD. You have the WBC's they just can't get where they are needed! Sxs: • delayed separation of umbilical cord at birth to > 30 days • omphalitis • recurrent infections of skin and mucosa - most commonly due to Staphylococcus spp., enteric gram-negative bacteria, and fungi - perirectal and labial cellulitis - otitis media • absent pus formation - serosanguineous fluids may be present • impaired wound healing - poorly formed, thin, bluish scars • gingivostomatitis

A 49-year-old woman is found to have mild proteinuria and pyuria on routine screening. Urine specific gravity is 1.000. Culture of the urine grows no organisms. She has taken large doses of combination over-the-counter analgesic preparations for 10 years due to a low back injury. Which of the following renal abnormalities is most likely in this patient? (A) Acquired cystic disease (B) Acute glomerulonephritis (C) Hyperplastic arteriolitis (D) Nephrolithiasis (E) Papillary necrosis

(E) Papillary necrosis Large doses of NSAIDs for 10 years!!! That's one of the few causes of papillary necrosis of the kidney. Easy question if you have this memorized. That's why NSAIDs are bad for the kidneys! Renal Papillary Necrosis is necrosis of the renal papillae (where filtrate / urine flows into the ureters). Can also present with flank pain and gross hematuria.

A 42-year-old woman comes to the physician for a routine examination. She says that she has felt well except for occasional episodes of constipation, abdominal discomfort, and mild fatigue. She was treated for a renal calculus 10 years ago. Her pulse is 82/min, and blood pressure is 150/80 mm Hg. Physical examination shows no other abnormalities. Laboratory studies show: K+ 4.5 mEq/L (NORMAL) Cl- 107 mEq/L (HIGH) Ca2+ 12 mg/dL (HIGH) Phosphorus 2.2 mg/dL (LOW) Alkaline phosphatase 95 U/L (HIGH) The most likely cause of this patient's condition is a small, well-defined nodule in which of the following locations? (A) Adrenal gland (B) Anterior pituitary gland (C) Gallbladder (D) Kidney (E) Parathyroid gland (F) Thymus

(E) Parathyroid gland - LOOK AT THE LABS - Hyperparathyroidism will increase bone reabsorption, leading to high calcium, high alkaline phosphatase, low phosphate. This patient has hyperparathyrodism which will increase PTH levels and thus Ca and Alk Phos (from bone catabolism). Remember that PO4 moves opposite from Ca and should be low in this case. She is also displaying the groans (abdominal discomfort) and stones (kidney stones)

A 50-year-old man has headache, vertigo, and generalized pruritus. He has the recent onset of angina pectoris. His hematocrit is 65%, leukocyte count is 12,000/mm3, and erythrocyte mass is increased. Erythropoietin concentration is decreased. Which of the following is the most likely diagnosis? (A) Glucose-6-phosphate dehydrogenase deficiency (B) Hemochromatosis (C) Immune thrombocytopenic purpura (D) Pernicious anemia (E) Polycythemia vera (F) Pyruvate kinase deficiency (G) Secondary polycythemia

(E) Polycythemia vera CNS effects due to "sludging" in cerebral vasculature (may be due to clots from increased platelets or increased red cell mass). This plus the high H/H and low EPO help rule out secondary polycythemia. FIRT AID: Polycythemia Vera - Disorder of RBCs, usually due to acquired JAK2 mutation. - May present as intense itching after shower. Rare but classic symptom is erythromelalgia (severe, burning pain and red-blue coloration) due to episodic blood clots in vessels of the extremities. - DECREASED EPO (vs 2° polycythemia, which presents with endogenous or artificially increased EPO). - Treatment: phlebotomy, hydroxyurea, ruxolitinib (JAK1/2 inhibitor).

A 35-year-old man comes to the physician because of pain and swelling of his right arm where he scraped it on a tree branch 2 days ago. His temperature is 38.3°C (101°F). Examination of the right forearm shows edema around a fluctuant erythematous lesion at the site of trauma. The area is extremely tender to palpation. Which of the following is most likely the primary mechanism of the development of edema in this patient? (A) Degranulation of eosinophils (B) Disruption of vascular basement membranes (C) Increased hydrostatic pressure (D) Release of thromboxane (E) Separation of endothelial junctions

(E) Separation of endothelial junctions Tricky question, but very basic in essence. What allows the fluid to leave the vascular channels? E is the one mechanism that happens every time!

A 72-year-old man who is a retired construction worker comes to the physician because he has had a lesion on his face for 3 months. Physical examination shows a 6-mm, red, ulcerated lesion with heaped borders. A biopsy specimen of the lesion shows atypical, dysplastic keratinocytes within the epidermis and dermis. Which of the following is the most likely diagnosis? (A) Actinic keratosis (B) Discoid lupus erythematosus (C) Melanoma (D) Mycosis fungoides (E) Squamous cell carcinoma Dermatopathology

(E) Squamous cell carcinoma This person has a history of sun exposure (construction worker) and has a red, ulcerated, heaped up lesion on his face (classic description of SCC), which is sun exposed. AK could be in the DDX, except they say there are dysplastic keratinocytes in the dermis, which means there is invasion (ruling out AK).

A full-term 2-week-old male newborn has cyanosis. Pregnancy and delivery were uncomplicated. His lungs are clear, and a midsystolic murmur is heard that is loudest in the left third intercostal space and associated with a thrill. Which of the following is the most likely diagnosis? (A) Atrial septal defect (B) Bicuspid aortic valve (C) Coarctation of the aorta (D) Patent ductus arteriosus (E) Tetralogy of Fallot

(E) Tetralogy of Fallot Heart defect question! Ask yourself...blue baby (T's) or blue kid (VAP). Clearly this is a baby...so it has to start with a T.

USMLE 2015 Sample Question A 2-year-old boy is brought to the physician for a well-child examination. He was delivered at term after an uncomplicated pregnancy. His birth weight was 3500 g (7 lb 11 oz), and Apgar scores were 8 and 10 at 1 and 5 minutes, respectively. At the age of 15 months, physical examination showed no abnormalities, but he was not yet talking. Both of his parents had learning difficulties in school, and his mother stopped attending after the 10th grade. He has a maternal uncle with cognitive disabilities. He is at the 25th percentile for height, 15th percentile for weight, and 90th percentile for head circumference. He appears irritable, he resists making eye contact, and he is flapping his hands. On physical exam, the boy has a prominently long face, large jaw, and large ears. Which of the following is the most likely cause of this patient's condition? (A) Creation of an alternative splice site (B) Frameshift mutation (C) Missense mutation (D) Nonsense mutation (E) Trinucleotide repeat expansion

(E) Trinucleotide repeat expansion Fragile X syndrome causes mild to severe intellectual disability and abnormal facial features. Can affects both males and females, but males 2:1 and usually worse. Symptoms include verbal delays, anxiety, and hyperactivity. May have seizures. Physical features may include large ears, long face, prominent jaw and forehead, and flat feet. Trinucleotide repeats (CGG) in X chromosome cause reduced FMR1 protein expression. Fragile X syndrome is the most common cause of inherited intellectual disability and 2nd most common cause of genetically associated mental deficiency (after Down syndrome). Symptoms: • Developmental: developmental delay in motor, speech, and language skills • Cognitive: low IQ • Neuropsychiatric: autism-like behaviors: - hand flapping - avoidance of eye contact • Anxiety • Behavioral issues: - aggression - attention deficits - oppositional - tantrums

A 56-year-old man comes to the emergency department because of a 4-day history of hematuria and colicky right flank pain that radiates to the groin. Ultrasound examination of the kidneys shows right-sided hydronephrosis and a dilated ureter. Which of the following is most likely to be found on urinalysis? (A) Erythrocyte casts (B) Glucose (C) Leukocyte casts (D) Oval fat bodies (E) Uric acid crystals

(E) Uric acid crystals Colicky radiating flank pain that brings people the ED is the classic description for passing a kidney stone. The stone has blocked the ureter causing dilation and hydronephrosis

A 24-year-old woman comes to the physician for a follow-up examination. One week ago, she was treated in the emergency department after she accidentally spilled hot grease on her left leg while working at a fast-food restaurant. Examination of the left lower extremity shows a 7-cm, pink, soft, granular, edematous wound. The formation of this tissue was most likely caused by increased activity of which of the following? (A) Complement C3b (B) Glycosylation-dependent cell adhesion molecule-1 (C) P-selectin (D) Stromelysin (E) Vascular endothelial growth Factor

(E) Vascular endothelial growth Factor What they are describing in the stem is granulation tissue. This is evident by the timeline (7 days) and the description, especially the use of the word "granular." Since that is predominately blood vessels, E is the easy answer!

A 73-year-old woman comes to the physician because of a 2-month history of diffuse weakness and tingling of her arms and legs. Neurologic examination shows weakness of the extensor and flexor muscles of the lower extremities. Knee and ankle deep tendon reflexes are exaggerated. Sensation to vibration and position is decreased in all extremities, but the decrease is more prominent in the lower extremities than in the upper extremities. This patient most likely has a deficiency of which of the following vitamins? (A) Niacin (B) Vitamin B1 (thiamine) (C) Vitamin B2 (riboflavin) (D) Vitamin B6 (pyridoxine) (E) Vitamin B12 (cyanocobalamin)

(E) Vitamin B12 (cyanocobalamin) This is Pernicious / Megaloblastic anemia E. In this case the patient only displays neurologic symptoms, which are associated with B12 deficiency. Notice how they didn't list folate and they mention nothing about megaloblastic (or pernicious) anemia! Remember that low B12 = low folate -- they need each other.

A 25-year-old woman from Greece has a routine, preemployment physical examination. Her laboratory studies include: - Hemoglobin 11.3 g/dL (LOW) - Hematocrit 34% (LOW) - Erythrocyte/RBC count 5.2 million/mm3 (NORMAL) - Mean corpuscular volume 65 μm3 (LOW) Follow-up laboratory studies show that the serum iron concentration and iron binding capacity are within the reference ranges. Hemoglobin electrophoresis shows increased hemoglobin A2. Which of the following is the most likely diagnosis? (A) Anemia of chronic disease (B) Iron deficiency anemia (C) Sideroblastic anemia (D) α-Thalassemia minor (E) β-Thalassemia minor

(E) β-Thalassemia minor There is a compensatory rise in A2 to make up for the decrease in beta chains. This is considered B-thal trait and is very commonly confused with iron deficiency anemia The thalassemias are unique in that there is a HIGH or NORMAL RBC Count, despite low Hgb, Hct, and microcytic RBCs (MCV). Thalassemias are due to problems with making Hgb, not RBCs. Treatment: Blood tranfusion . - β-thalassemia minor (heterozygote): β chain is underproduced. Usually asymptomatic. Diagnosis confirmed by HbA2 on electrophoresis. - β-thalassemia major(homozygote): βchain is absent. Diagnosis confirmed by increased HbF (α2, γ2) and HbA2 (α2, δ2). Common among Mediterraneans. β-Thalassemia major would have marked elevation of HbF, increased HbA2, and absence of HbA1.

A 12-year-old girl with sickle cell disease has pain in her right arm. An x-ray of the right upper extremity shows bony lesions consistent with osteomyelitis. Which of the following is the most likely causal organism? (A) Clostridium septicum (B) Enterococcus faecalis (C) Listeria monocytogenes (D) Proteus mirabilis (E) Pseudomonas aeruginosa (F) Salmonella enteritidis (G) Serratia marcescens

(F) Salmonella enteritidis Classic infectious association with sickle cell disease. Here it is osteomyelitis, so the organism is salmonella. Just a reminder, parvo can cause aplastic crisis in these patients.

++++++++++++++++++++++++++++++++++++ A 14-year-old girl is brought to the physician by her mother because of a 2-month history of heavy vaginal bleeding during menstrual periods. She has had episodes of excessive periodontal bleeding while brushing her teeth and easy bruising for 6 years. She also had an episode of extended bleeding after a tooth extraction 4 years ago. Her mother and brother have had similar symptoms. Physical examination shows patchy ecchymoses over the upper and lower extremities. Laboratory studies show: Platelet count 234,000/mm3 Bleeding time 17 min (2-7) Prothrombin time 12 sec (11-15) (NORMAL) Partial thromboplastin time 46 sec (20-40) (INCREASED) Which of the following is the most likely diagnosis? (A) Factor VII (proconvertin) deficiency (B) Factor X (Stuart factor) deficiency (C) Factor XII (Hageman factor) deficiency (D) Hemophilia A (E) Vitamin K deficiency (F) von Willebrand disease

(F) von Willebrand disease This patient is displaying mild mucosa bleeding. Note mother and brother (both sexes) have had it. She has a normal platelet count and PT and a slightly long PTT. Isolated high PTT could be in D or F, but the familial distribution and symptomatology is off (not severe enough for hemophilia), so vWd it is. NORMAL PT INCREASED PTT

Histoplasmosis: - Fungus that commonly infects the lungs, forming what? - Histological appearance

- A single lung nodule - Histology: Image - Large pink circles or black spots with a green background. Histoplasma is a dimorphic fungus that is present in the midwest (so look for a state along the Mississippi river) Can cause granulomatous disease in healthy patients, and is a frequent cause of lung spots on X-ray. In immunocompromised patients can cause disseminated disease e.g., A 65 year old smoker, who lives in St. Louis, presents to your clinic with an increasing cough. A chest X-ray shows a "nodule."

Cheat sheet for cardiac function curves. MNEM for shifts resulting from: - Aerobic Exercise - Blood Transfusion - Compensated Heart Failure - Hemorrhage (bleeding out)

- Aerobic Exercise - Blood Transfusion - Compensated Heart Failure - HemorrhaGe (bleeding out)

++++++++++++++++++++++++++++ Recognize each curve on the plot: - Aortic Pressure - Left Ventricular Pressure - Left Aortic Pressure And on this combined plot, know where the Mitral and Aortic Valves open and close. (This plot represents one single contraction)

- Aortic Pressure = green - Left Ventricular Pressure = red - Left Aortic Pressure = yellow

How do each of the following fix arrhythmias (basic MoA): - BB - CCB - Adenosine & Digoxin - NCB - KCB

- BB: Slow phase 4 at the nodes - CCB: block Ca2+ channels at nodes to slow firing rate. - Adenosine & Digoxin: Block Na-K ATPase at nodes to reduce firing rate. - NCB: Block Na+ channels on heart muscle to slow the rate of contraction. - KCB: Block K+ channels on heart muscle to make it take longer before the muscle can contract again.

+++++++++++++++ Proteins involved in T-cell activation and deactivation: - CD28 - B7 interactions - CTLA-4 - B7 interactions --definitely know the opposing roles of CD28 and CTLA4 - Knockout of CTLA-4 results in ___. - PD-1 - PD-L1 interactions - Activation vs Deactivation of T cells (B7 and PDL1 are T-cell receptors located on Antigen Presenting Cells) (CD28, CTLA-4, and PD-1 are receptors located on the surfaces of T-cells)

- CD28 (on T-cells) binding to B7 (on APCs) activates T cells -- major activator. - CTLA-4 (on T-cells) binding to B7 (on APCs) inhibits T cell activation. - PD-1 (on T-cells) binding to PD-L1 (on APCs) also inhibits T cell function (PD stands for Programed Death). Knockout of CTLA-4 results in autoimmune disease - because this would inactivate the inactivator of T-cells: - multi-organ lymphocytic infiltrate, lethal by 3-4 weeks - lymphadenopathy, splenomegaly • CD28 binding of B7 is the most important costimulatory pathway • CTLA-4 interaction with B7 shuts down T cell signaling pathways • PD-1 binding of PD-L1 also delivers a negative signal to T cells.

++++++++++++++++++++++++++++++++ Type IV Hypersensitivity (Delayed-type Hypersensitivity): - Mediated by what type of cell? - Clue to Type 4 is how long it took sxs to show up. How long does it usually take? - Name 3 conditions that are famously Type 4 - Pathology: > Involves the activation of what 2 types of cells? > What inflammatory cytokine is released that causes vasodilation and extravasation of neutrophils?

- CD4+ Helper T-cells (memory cells), unlike the other 3 types, which are mediated by immunoglobins (Ig's) - Whole process takes 2-3 days, hence delayed-type hypersensitivity. It requires a prior exposure. So it doesn't happen the first time. Mediated by memory CD4 Th1 T-cells. > unlike Type I allergic reactions, which occur within minutes of initial exposure. - Examples of Type 4 Hypersensitivity: ✔ Poison Ivy ✔ Nickel Allergy ✔ Latex allergy - Type I and Type 4 occur. Pathology: • Memory CD4 Th1 T cells and macrophages are activated -- thus, it requires a prior exposure and has a slow onset (B and T-cells of the adaptive immune system are slow). • IFN-γ, secreted by Th1 cells, activates macrophages • CD4 T cells and macrophages secrete TNF-α which stimulates production of vasodilators and causes extravasation of neutrophils.

Polycystic Ovarial Syndrome (PCOS): - Commonly seen in young or old? With what feature that makes it more likely? - It's a major cause of ... - Size of ovaries - How does it present? - Hormonally, patients have an excess of __ and __, which cause the symptoms. - How does it affect insulin levels?

- Commonly seen in young, overweight women (higher estrogen levels). - Is a major cause of infertility - Ovaries are usually twice as large with numerous subcortical cysts (lining up around the edge) and stromal fibrosis. - Lack of ovulation, obesity, hirsutism. - Hormonally, patients have an excess of LH and androgens (Testosterone) leading to the symptoms of the disease. They have low estrogen and FSH. - Diabetes and hyperinsulinemia have been linked to PCOS, and studies have indicated that increased insulin can increase androgen levels.

Pseudogout - Crystals are made of what? - Usually affects which joint? - Birefringence?

- Crystals are composed of Calcium Pyrophosphate Dihydrate (CPPD) -- rather than monosodium urate in normal gout. - Usually affects the knee. - Weak, positive birefringence with compensated polarized light. Normal gout has negative birefringence.

Acute Myelocytic Leukemia (AML) - associated with what condition? - has what translocation? - VERY TESTABLE: CAN BE TREATED WITH ___.

- DIC - t(15;17) - Can be treated with ATRA!!! (as can APL and APML)

Duchenne/Becker Muscular Dystrophy: - What protein is defected in both - What inheritance pattern do these follow? - What is the easiest-to-remember difference between them?

- Dystrophin (a scaffolding protein) - X-linked recessive inheritance - Duschenne presents early, Becker presents late (teens or early adulthood) - Becker is less severe than Duschenne's.

BAD SPLEENS: - What is the most common cause trauma to the spleen? - How does Sickle Cell Anemia affect the spleen?

- Fracture of ribs 9, 10, or 11 -- bone shards can pierce the spleen and cause massive bleeding. These patients will need vaccines against Strep pneumo, N. meningitidis, and H. influenzae - Sickle Cell patients have splenic sequestrations when they are children and are Asplenic by adulthood.

Rate-Limiting Enzymes in: - Glycolysis - Gluconeogenesis - TCA/Kreb's Cycle - Glycogenesis - Glycogenolysis - Fatty Acid Beta Oxidation

- Glycolysis: PFK-1 - Gluconeogenesis: F-1,6-Bisphosphatase - TCA/Kreb's Cycle: Isocitrate Dehydrogenase - Glycogenesis: Glycogen Synthase - Glycogenolysis: Glycogen Phosphorylase - Fatty Acid Beta Oxidation: Carnitine Acyltransferase I

A 68-year-old man with emphysema comes to the physician for follow-up. The patient has a long history of heavy cigarette smoking, and pulmonary function tests show a typical obstructive pattern. He has recently developed chronic hypoxemia. When supplemental oxygen is given to this patient, assuming that respiration rate initially remains unchanged, what will happen to: - PAO2 - A-a PO2 difference - PaO2 - Difusion capacity (DLCO)

- High PAO2 - High A-a PO2 difference - High PaO2 - Same Difusion capacity (DLCO) PAO2 = partial pressure of O2 in alveoli. PaO2 = partial pressure of O2 in arteries. FIO2 (O2 levels in inhaled air) increased. No change in Ventillation, so PAO2 and PaO2 increase. But because of V/Q mismatch, A-a PO2 difference also increases. O2 in alveoli increases a lot, O2 in arteries increases some. No change in DLCO.

Prostate Cancer - Rather than the nodules seen in BPH, histology of Prostate Cancer will show proliferation of what? - While PSA is elevated in both Prostate Cancer and BPH, a PSA of at least ___ indicates it's Prostate Cancer. - Prostate Cancer is highly associated with osteo___ bone lesions. - Prostate Cancer shows up in what zone of the prostate?

- Histology: Proliferation and Infiltration of Glands. ^ Hence adenocarcinoma of the prostate gland. - PSA of at least 10 is most likely prostate cancer. >> But less than 10 doesn't mean it's not prostate cancer, as it can be rising. - Osteoblastic bone lesions - BUILDS new bone. Often on the lumbar spine. - Prostate Cancer grows in the Peripheral / Posterior zone - the part that's palpated during a DRE. - It does not show up in the central / periurethral zone (core), so it doesn't involve urinary problems like BPH does.

1. What is the most common malignancy in HIV patients? 2. It is caused by what virus? 3. How does it present? Dermatopathology

- Kaposi's sarcoma - Human herpes-virus 8 - Only infects and causes cancer in AIDS patients - Presentation: Reddish-purple lesions with varying shapes/sizes.

++++++++++++++++++++++++++++++ A 45-year-old HIV-positive man presents with a new rash. Upon inspection, you discover multiple red, raised lesions. He lost his job and insurance and ran out of his HIV medication 5 months ago. He had been feeling fine until a week ago when he developed a painless rash on his face, trunk, and legs. He has also had some night sweats and low-grade fevers. On physical exam, he has multiple scattered reddish-purple macules and papules. There are also purple nodules on his oral mucosa. A skin biopsy reveals neoplastic spindle-shaped cells and a lymphocytic infiltrate. - Diagnosis? (Most common malignancy in AIDS) - Most often caused by what pathogen? - Histology Dermatopathology

- Kaposi's sarcoma - Human herpes-virus 8 - Only infects and causes cancer in AIDS patients. Pathogenesis: HHV-8 inhibits tumor suppression pathways. Histology: 1) neoplastic spindle-shaped cells that form clefts and vascular channels (red/pink)e 2) lymphocytic infiltration (purple) MNEM: HHV8/Kaposi cause red/purple lesions and its histology is red and purple. Kaposi Sarcoma is the most common malignancy in AIDS. It is caused by a viral infection, commonly called HHV8. KS is one of the AIDS-defining malignancies (along with Burkitt's lymphoma, CNS lymphoma, and cervical cancer). Physical exam: Red or purple lesions of varying shapes and sizes. B symptoms: fevers, night sweats, weight loss (like other cancers)

IgA nephropathy/Berger's disease - Nephritic or Nephrotic? - Usually preceded by what type of infection? Why? - Systemically, IgA is associated with what condition, with what characteristic skin findings? - 2 Classic findings on LM and IF.

- Nephritic - Characterized by IgA immune deposits in the glomerular mesangium. - The most common cause of glomerulonephritis worldwide. - associated with episodic hematuria and proteinuria - Usually follows a URI or GI infection because IgA is secreted by mucosa. Circulating IgA deposits in the mesangium. Seen on LM and IF: - Mesangial Proliferation - Crescent formation Henoch-Schonlein Purpura (HSP): -Systemic disease with IgA deposits in skin, intestine, and joints. -Renal lesion in HSP is similar to lgA nephropathy -Patients present with palpable purpura that typically involves the legs and buttocks, Abdominal pain , arthritis, and arthralgias -The mean age of onset is between 6 and 7 years First Aid: Episodic hematuria that usually occurs concurrently with Respiratory or GI tract infections (IgA is secreted by mucosal linings). Renal pathology of IgA vasculitis (HSP). LM—mesangial proliferation IF—IgA-based IC deposits in mesangium; EM—mesangial IC deposition

1. How do you differentiate Primary from Secondary Hyperaldosteronism based on renin levels? 2. If a person has hyperaldosteronism, what will happen to: - Serum Na+ level - Serum Na+ concentration - Serum K+ 3. Sample Q: An observation that is not expected in primary aldosteronism is: a. hypertension b. increased renin production c. a decrease in the plasma K+ concentration d. hypernatremia e. alkalosis

- Primary Hyperaldosteronism is when an adrenal tumor (cause) causes high aldosterone (effect). Not caused by high RAAS. - Secondary Hyperaldosteronism is when increased RAAS (cause) increases aldosterone (effect). Renin will be high. Dr. Quick: -Primary: low renin; Bilateral idiopathic (most common) or neoplasm. -Secondary: high renin; Decreased renal perfusion, arterial hypoperfusion, pregnancy Remember: "Hyponatremia is a decrease in serum sodium concentration" Aldosterone reabsorbs Na+ (at the expense of K+) so that water will follow. Increases BP (part of RAAS). - Serum Na+ level will be high - Serum Na+ concentration will not change because water is also reabsorbed. - Serum K+ will be low Aldosterone causes Na+ reabsorption and water follows to maintain blood osmolarity. It also causes increased excretion of K+ into urine. Sample Q: An observation that is not expected in primary aldosteronism is: a. hypertension b. increased renin production c. a decrease in the plasma K+ concentration d. hypernatremia e. alkalosis

+++Know this like the back of your hand+++ What clotting factors are likely at fault if pt has a prolonged: - PT (Prothrombin Time) - aPTT (Activated Partial Thromboplastin Time) - Both PT and aPTT

- Prolonged PT: Factor VII (7) - Prolonged aPPT: Factors VIII, IX, XI, or XII (8, 9, 11, or 12) ^ isolated long PTT suggests Hemophilia A or B - Both Prolonged: Factors V or X (5 or 10) (there is no such thing as not enough tissue factor that we know of)

Buerger's disease (thromboangiitis obliterans): - MAIN RISK FACTOR - Symptoms

- SMOKING!!! (Imagine a guy smoking and eating a burger) - Thrombosing vasculitis of small and medium vessels - Sxs - Raynaud's phenomenon, Gangrene, severe pain Tx: Stop smoking An 31-year-old Israeli male with a history of heavy smoking presents to your office with painful ulcerations on his hands and feet. Upon examination, he is found to have hypersensitivity to intradermally injected tobacco extract. Which of the following processes is most likely responsible for his condition? A. Increased endothelial permeability B. Necrotizing inflammation involving renal arteries C. Segmental vasculitis of small and medium-sized arteries D. Eosinophil-rich granulomatous inflammation E. Concentric thickening of the arteriolar wall

Conjunctivitis / Pink Eye: - Most common bacterial pathogen (and overall most common cause) - Most common viral pathogen

- Staph Aureus is the most common cause of conjunctivitis overall. - Adenovirus is a common viral cause Bacterial conjunctivitis typically has exudate / crust Viral conjunctivitis is typically watery

What coronary artery is occluded if the ST elevations are in: - V1-V4 - V5, V6, Lead I, and avL - II, III, and avF

- V1-V4: LAD - V5, V6, Lead I, and avL: Left Circumflex - II, III, and avF: RCA

Calcium Channel Blockers: - Which main one works at the heart to decrease O2 demand and reduce angina? - Which two work in vessels, dilating them to reduce blood pressure, but often causing reflex tachycardia?

- Verapamil: Works at the heart; can treat angina. Verapamil's mechanism in all cases is to block voltage-dependent calcium channels. Verapamil is similar to beta blockers in effect. Both treat angina. - Nifedipine and Amlodipine (and Dihydropyridine) are vasodilators that prevent vasoconstriction to reduce blood pressure, but often at the expense of increased heart rate. These work on blood vessels rather than the heart itself. These are good for Prinzmetal angina, but not helpful for stable angina. From other Quizlet user: CCBs are agents that inhibit the influx of calcium ions into heart muscle cells, causing a slower heart rate, lower O2 demand, and relaxation of blood vessel smooth muscle cells to cause dilation; used to prevent or treat angina pectoris, some arrhythmias, and hypertension. From article: Verapamil's mechanism of action in the treatment of angina and hypertension is largely the same. Inhibition of calcium influx prevents the contraction of vascular smooth muscle, causing relaxation/dilation of blood vessels throughout the peripheral circulation - this lowers systemic vascular resistance (i.e. afterload) and thus blood pressure. This reduction in vascular resistance also reduces the force against which the heart must push, decreasing myocardial energy consumption and oxygen requirements and thus alleviating angina.

What is an early site of metastasis of gastric cancer? What is a Krukenberg tumor? Name 3 risk factors.

- Virchow's node (left supraclavicular) - Ovarian involvement- usually bilateral - Risk factors: > Smoked food (nitrosamines) > Atrophic (chronic) gastritis > Achloridia This is a Sister Mary Joseph's Nodule, gastric cancer

Sideroblastic Anemia: - Histology: Key finding - What is the most common cause of sideroblastic anemia? - Name 2 other causes? - Lab findings: ↑, ↓, or normal Iron, TIBC, and Ferritin - How is this different than anemia of chronic disease?

- defect in mitochondrial heme synthesis producing ringed sideroblasts. Iron is trapped in the mitochondria. - ALCOHOLISM - Lead poisoning and Vit B6 deficiency also cause this - Lab findings: ↑ iron, normal or ↓ TIBC, and ↓ ferritin. - Peripheral blood smear: basophilic stippling of RBCs - In ACD the iron is increased in the macrophages as a result of hepcidin blockage. In sideoblastic anemia it is located in the mitochondria.

What is Erb's Palsy? What part of the brachial plexus is damaged? What will be the statuses of the arm, elbow, and forearm?

- lesion in the upper trunk (C5-C6) of the brachial plexus (which contributes to the musculocutaneous and median nerves), leading to the characteristic "waiter's tip" deformity - arm is adducted - elbow is extended - forearm is pronated - also, wrist is flexed An injury to the musculocutaneous nerve would result in poor supination and weakened flexion of the elbow joint Most common neonatal brachial plexus palsy. Caused by complication during birth -- it's a deformity.

CALCIUM HOMEOSTASIS: - ___ in calcium levels means increased neuromuscular activity - Positive Chvostek and Trousseau signs indicate high or low calcium? - Relationship with Vitamin D - What happens when there's low Vit D? - Relationship with PTH -- Effect of low calcium on PTH and what the result will be. - Relationship with Phosphate - What does the medication Cinacalcet do and what does it treat?

- ↓ in Ca2+ ↑ neuromuscular activity > Chvostek sign - tapping facial nerve at jaw - twitching of facial muscles = hypOcalcemia. > Trousseau sign - increasing blood pressure with cuff ~ 20 mm Hg above systolic pressure for 3-5 minutes produces ischemia - carpal spasm = hypOcalcemia. - 1,25 (OH)2-D3 = Vitamin D: > ↑ absorption of Ca2+ and phoshate in GI tract, ↑ kidney Ca2+ absorption , ↑ bone resorption. Overall, it wants to increase Ca2+ however it can - Kidneys, GI tract, Bone. > Lack of Vit D - muscle weakness, uncalcified osteoid (rickets in children, osteomalacia in adults) - Low plasma Ca2+ increases PTH. > PTH increases osteoclastic bone resorption ➔ Brittle Bones. That's why calcium is good for bones. - Calcium and Phosphate go against e/o - Inversely proportional. > Calcium and phosphate in the body react in opposite ways: as blood calcium levels rise, phosphate levels fall. PTH regulates the levels of calcium and phosphorus in blood. > Why? Phosphate binds calcium in the blood, which, in turn, lowers your calcium levels. > When your calcium levels get too low, PTH is released to pull the extra calcium your body needs out of your bones. - Cinacalcet - sensitizes CaSRs to Ca2+ and ↓ PTH secretion > used in CKD, PTH carcinoma, and primary hyperparathyroidism.

Thin Basement Membrane Disease: - Congenital or Acquired? - What is the main symptom the patient will experience? - Prognosis

-A disease characterized by thin BM measuring ~150-250 nm (~300-400 nm is normal) and defects in genes coding for α3 and α4 chains of type IV collagen - Asymptomatic hematuria which is usually incidentally found -- thin basement membrane lets some blood through - Mild to moderate proteinuria may be present however renal function remains normal - Excellent prognosis with no long term morbidity in the vast majority of patients

Lumbar puncture for bacterial meningitis -- would expect to see high or low: -glucose -protein (MNEM)

-LOW glucose -HIGH protein (normal is <50 mg/dL) -- if >50, be suspicious. The bacteria use glucose to make lactose, which is why there's low glucose in bacterial meningitis. Just remember, Big Muscles require low glucose and high protein.

High Yield CV Equations to know for STEP - Part 2

.

Histological appearances of these fungi: 1. Cocccidiodomycosis 2. Histoplasma 3. Paracocccidiodomycosis 4. Aspergillus

.

Match each buzz word with the associated cancer: - Down syndrome - Pernicious anemia \ chronic gastritis - Actinic keratosis - Barrett's esophagus - Paget's disease of Bone - Immunodeficiency & autoimmunity - Radiation (2)

.

Mechanisms of ANS modulation of the heart in: 1. Cardiac Muscle 2. Nodes in the heart

.

Refresher on what happens in: - PCT - TAL - DCT - CD

.

Summary of capillary filtration

.

NAME THAT IMMUNODEFICIENCY! 1. Typically male infants that are about 5-8 months old that present with recurrent respiratory tract infections. - Cause - Genetic link 2. A 2-month-old baby boy is brought in for an urgent visit to the pediatrician. He has had several ear infections in his short lifetime and now seems to be struggling with a cold. On physical exam, his tongue is noted to be coated with white film. His scalp and face are covered with a flaky dandruff-like substance. Immediately concerned, his pediatrician orders a chest radiograph, which shows absence of thymic shadow. 3. A 26-year-old man with no significant past medical history presents with a prolonged period of cough and repeated sinus infections. He also reports persistent watery diarrhea. His physician orders immunoglobulin levels, which come back with decreased IgA and IgG, but normal IgM. Plasma cells are also decreased. - Heterogeneous immunodeficiency that may affect B or T cells. Patients may present years to decades after birth (mean 30 years). Recurrent pyogenic infections, diarrhea (giardia), pneumonia, and association with malignancy and inflammatory bowel disease, etc. 4. A 3-year-old girl is brought to the outpatient clinic because she has white plaques in her mouth and has been saying that her arms and legs "feel funny." Her surgical history is significant for correction of a congenital heart defect. On physical examination, diffuse white plaques are seen on her tongue and oral mucosa, and they can be scraped off easily. When an attempt to record the patient's blood pressure is made, her left wrist spasms. Thymic shadow is absent on chest x-ray. - Affected infants may die, those who live suffer from viral, bacterial and protozoal infections. Associated with 3rd & 4th pharynegeal pouch and Cr 22 abnormalities. T-cells stuck at pre-T stage. 5. An 8-year-old boy is brought to his pediatrician for easy bruising. On physical exam, he is found with petechiae and purpura in multiple areas over his body, as well as bruises over his arms. Eczematous patches are also found on his flexural surfaces. Laboratory results reveal thrombocytopenia to 30,000/mm3. Further questioning reveals a past medical history of multiple hospital stays due to pneumonia and otitis media infections as well as recurrent epistaxis. - Recurrent infections, thrombocytopenia, eczema, typically boys in first few months of life. Immunoglobulin measurements show isolated decrease in IgM.

1. "Bruton's" X-Linked Agammaglobulinemia - Recurrent "pyogenic (pus-forming)" infections - Deficiency in all types of immunoglobulin - q(21;22) defect in tyrosine kinase = no mature B-cells -- B-cells make Ig's 2. SCID - Lack Cell Mediated Immunity, i.e. no B or T cells (no immunoglobulins) - Combined B- and T-cell disorder causing immunodeficiency - Presents at a young age (always <6 months) with frequent infections of any type of pathogen. 3. Common variable immunodeficiency - CVID is an immunodeficiency arising from B-cell dysfunction - Etiology is unknown and presentation can vary. - The key is that it presents late -- 20's/30's. 4. DiGeorge Syndrome - CATCH 22 5. Wiskott-Aldrich Syndrome - Primary immunodeficiency disorder of B and T cells - Sxs: WATER: Wiskott-Aldrich: Thrombocytopenic purpura, Eczema, Recurrent infections - X-linked - Decreased IgM, rest normal (or high) - Susceptible to encapsulated bacteria and viruses

Name the congenital heart defect(s) associated with: 1. 22q11 mutation (MNEM) 2. Down's Syndrome (MNEM) 3. Rubella 4. Turner's Syndrome 5. Marfan's Syndrome 6. Diabetes

1. 22q11: Truncus Arteriosus & Tetralogy of Fallot - MNEM: Twenty Two causes Truncus and Tetralogy 2. Down's Syndrome: Cushion defects (VSD, ASD, AVSD) - MNEM: Down's Syndrome causes all of the defects with a D (except PDA) 3. Rubella: PDA, Septal defects (AVSD) 4. Turner's Syndrome: Coarctation of the Aorta 5. Marfan's: Aortic valve Insufficiency 6. Diabetes: Transposition of Great Vessels

1. A pt presents with Ventricular Tachycardia. Drug of choice? A. Amiodarone B. Amlodipine C. Dysopyramide D. Ibutilide E. Procainamide F. Quinidine G. Sotalol 2. A 63-year-old man comes to the emergency department with acute chest pain and shortness of breath. Chest X-ray, echocardiogram and ECG reveal a massive pulmonary edema and left ventricular failure. Which is the main target of the most appropriate acute diuretic treatment? A. Carbonic anhydrase B. Epithelial sodium channel C. Mineralocorticoid receptor D. Na-Ca exchanger E. Na-K ATPase F. Na-K-Cl-Cl cotransporter 3. Intravenous infusion of Angiotensin I and Angiotensin II both increase blood pressure. The animal is first pretreated with an unknown drug X. Infusion of Angiotensin I did not increase BP but Angiotensin II did. What is drug X? A. Lisinopril B. Losartan C. Metoprolol D. Prazosin 4. How does ACEI/ARB or BBs work in long-term treatment of HFrEF? A. Block maladaptive remodeling B. Something else

1. A) Amiodarone for VTach. Drug of choice questions may be asked as mechanism of action or associated side effects as second order. Amiodarone is a KCB on heart muscle, which slows repolarization, making it take longer for the muscle to contract again. 2. F) Na-K-Cl-Cl cotransporter. Furosemide, a loop diuretic acting on Na-K-Cl-Cl cotransporter is an effective way to quickly reduce pulmonary congestion in left-sided heart failure. 3. A) Lisinopril - ACE inhibitors (-pril) block conversion of Ang I to Ang II. However, if Ang II is directly given, ACEI can't do anything. On the other hand, if you have ARB (-artan) on board, Ang II action on the receptors (AT1R) will be blocked regardless of what is going on upstream. 4. A) Several questions look for this answer: blockade of maladaptive remodeling by ACEI/ARB or BB: not only in heart failure but also in other diseases as well. Ang II is one of the most powerful peptides in the world that affect many different cell types. That is partly why ACEI/ARB are useful in several different CV/Renal conditions.

1. A patient suffers an MI and is admitted to the hospital. They appear comfortable at rounds the next day, yet die shortly after morning rounds. The most likely cause of death was: A. Arrhythmia B. Rupture of cardiac muscle C. Aneurysm formation with embolization of thrombus material D. Pericardial effusion and tamponade E. Dressler's syndrome 2. A patient suffers a "MI" and is admitted to the hospital. They have an uneventful 2-day hospital course and are discharged. They are found deceased 4 days later. What is the expected finding at the time of autopsy? A. Wavy fiber change and neutrophils B. No histologic findings by light microscopy C. Scarring of heart tissue D. Dilation of the left ventricle with aneurysm formation E. Pale-yellow lesion with associated rupture

1. A. Arrhythmia Arrhythmias are the MOST LIKELY cause of death until about day 3, when rupture can happen. 2. E. Pale-yellow lesion with associated rupture After about 4 days, pale yellow center starts to form, which is weak, and increases chance of rupture. Not scar tissue because fibrosis happens after about 8 weeks.

A 13-year-old boy comes in for analysis of multiple episodes of loss of consciousness. The latest episode occurred shortly after a flu shot when he fainted for less than a minute. He is otherwise healthy and his baseline ECG is normal. Vasovagal syncope is suspected. 1. Which condition is he suffering from? A. Bradyarrhythmia B. Tachyarrhythmia 2. During a fainting episode, which of the following is the most likely change in his ECG? A. Decreased QRS width B. Decreased QT interval C. Decreased RR interval D. Increased QRS width E. Increased PR interval

1. A. Bradyarrhythmia - Heart beating too slow, not enough O2 to brain => syncope. Vasovagal maneuvers slow the heart, so this would make his condition worse, which is why he faints. 2. E. Increased PR interval - More space between heartbeats due to bradycardia. - Not C -- RR interval would also be increased, not decreased.

A 27 year old man presents to your clinic. He is 3'10'' tall. His head is normally sized and his limbs are short. 1. Diagnose 2. What is most common genetic mechanism for this disease? 3. Activation of what growth factor receptor usually causes this? (hint: starts with an F) MSK Pathology

1. Achondroplasia 2. Sporadic mutation (85%) assoc with advanced paternal age 3. Activation of Fibroblast Growth Factor Receptor 3 (FGFR 3), which inhibits chondrocyte proliferation.

A 62 year old man presents for his check up. He is a farmer and is relatively good health due to his active lifestyle. You note several raised, rough, grey white patches on his hands and ears. or A 60-year-old man with multiple "sun spots," or solar lentigos, comes to the dermatologist for his annual skin exam. He complains of several rough patches on his cheeks that has been there for a year. They feel like sandpaper. He is a gardener and often spent hours under the sun. 1. Diagnosis? 2. Is there malignant potential of this diagnosis? 3. Is it invasive? Dermatopathology

1. Actinic Keratosis 2. Yup -- about 5% can progress to SCC. 3. Nope, no invasion. Look for patches / plaques (flat), rather than raised. - UNLIKE: > BCC (waxy, pink, pearly nodules, commonly with telangiectasias, rolled borders, central crusting, or ulceration) > SCC (Ulcerative red lesions) Premalignant lesions caused by sun exposure. Small, rough, erythematous (red), grey, or brownish papules or plaques. Risk of squamous cell carcinoma is proportional to degree of epithelial dysplasia.

+++++++ High Yield Psychiatry Medications +++++++ Name the drug or drug class of choice for treating these Extrapyramidal Symptoms (often caused by Haloperidol in schizophrenic pts): 1. Acute dystonia (torticollis, "oculogyric crisis", etc.) - abnormal muscle contractions 2. Parkinson-like symptoms (tremors, bradykinesia, cogwheel rigidity, stooped posture, loss of postural reflexes) 3. Akathisia - uncomfortable restlessness, constantly moving. 4. Tardive dyskinesia (involuntary, repetitive movements of the face, tongue, etc.)

1. Acute dystonia: - Anticholinergics: Benztropine (classic) or Diphenhydramine - These reduce Ach, which will reduce muscle contractions. Blockng the effects of Ach balances the effects of reduced dopamine. 2. Parkinson-like symptoms: - Lower the dose, switch drug, or use Anticholinergics (Benztropine), or Amantadine (dopamine agonist) 3. Akathisia: - Propanolol - Lorazepam for short term symptomatic relief - Lower dose or switch to lower EPS-risk medication 4. Tardive dyskinesia: - Not much helps, so prevention is key. - Might need to switch to Clozapine, an atypical antipsychotic. - Also need to do an Abnormal Involuntary Movement Scale (AIMS) or at least document ongoing evaluation of TD as this is a frequent legal issue.

A patient with a history of hepatitis A presents exactly one year later. He has been partying all week, and in his words, "drinking a ton." He is complaining of a stabbing pain that radiates to his back along with fever, nausea, and vomiting. He hasn't eaten in 2 days (but has had plenty to drink). 1. Diagnose 2. What labs would you like to back this up? (What 2 enzymes will be high?) 3. Which is more specific?

1. Acute pancreatitis - Definition: Acute inflammation of the pancreas and surrounding tissue due to excessive release of pancreatic enzymes, causing autodigestion of the pancreas. - Sxs: sudden onset of epigastric pain radiating to the back, fever, nausea, vomiting, Steatorrhea (due to malabsorption), and Flatulence 2. Amylase and/or Lipase will be HIGH - These are two enzymes secreted by the pancreas to help digest food. 3. Lipase - Specific to the pancreas.

Diagnose each based on BUZZwords: 1. Sharp pain (worse w/ inspiration, better when leaning forward), friction rub, widespread ↑ST -Idiopathic/viral most common, effusion can eventually lead to tamponade 2. Acute hemorrhage: trauma, LV rupture post-MI, aortic dissection. Can be from acute pericarditis. -JVD, distant heart sound, hypotension, pulsus parodoxus (dec. amplitude of systole / exaggerated blood shunting to the right heart during inspiration), electrical alternans 3. Extreme form of pericarditis - Kussmaul sign (↑JVD during inspiration)

1. Acute pericarditis 2. Cardiac tamponade 3. Constrictive pericarditis Cardiac Tamponade: - accumulation of fluid in the pericardial sac that restricts ventricular filling. - Total blood volume in the heart is stable, but volumes in individual chambers are not. Blood can shunt to different chambers. - Unlike pericarditis, where chamber volumes are fixed by the crusty pericardium. - E.g., A 19-year-old man is stabbed in the left side of his chest (trauma / hemorrhage). His blood pressure on presentation is 90/60 mmHg with a pulse of 130/min. On physical exam, he has muffled heart sounds (hard to hear from fluid)and distended jugular veins. Upon inspiration, his blood pressure drops to 75/55 mmHg (pulsus paradoxus). His extremities are cool and clammy. Pericarditis: - Inflammation of the pericardium characterized by sharp pain worsened by inhalation - E.g., A 58-year-old woman with a past medical history of systemic lupus erythematosus presents to the emergency room for sharp chest pain. She reports that it is worse with inspiration and gets better when she leans forward. On physical exam, there is a friction rub that is loudest when she leans forward. An electrocardiogram shows widespread ST elevation. In both cardiac tamponade and pericarditis, blood backs up into jugular, causing JVD.

A 24 year old medical student presents to his doctor complaining of gastric pain. He is worried he has a carcinoma and demands endoscopy so he can rest easy. At endoscopy multiple small, round, brown-red, superficial ulcers are noted. 1. What is the diagnosis? 2. What kind are associated with burns? 3. What kind are associated with head trauma? 4. What are 2 other common causes?

1. Acute, erosive gastritis 2. Curling ulcers (sloughing due to hypovolemia) 3. Cushing ulcer (↑ vagal stim = ↑Ach = ↑H+) 4. EtOH and NSAIDS (very common)

+++++++++++++++++++++++++++++ More NT's, where they're made, and links to Psych DOs: 1. Addictions: - What NT is most involved? - This NT is made in what 2 structures, and in WHAT ORDER? 2. Violent suicide attempts: - What NT is most likely low? - Where is this NT made? 3. Huntington's disease: - What NT is low? - These patients have trinucleotide repeats of what 3 nucleotides (letters) -- MNEM - Brain: Atrophy of ___ and ___ and enlargement of ___ ventricles due to loss of caudate.

1. Addictions: - Dopamine - reward center - 2 parts of Basal Ganglia are involved: Ventral tegmental area and Nucleus accumbens (reward center). > Pleasurable stimulus ==> Ventral Tegmental Area (VTA) ==> Nucleus Acumbens (NAc) ==> dopamine ==> interpretation as reward by the frontal lobe. - Use bupropion (Wellbutrin) for nicotine addiction / smoking / vaping. 2. Suicide Attempts: - Serotonin (aka 5HT) - Made in the Raphe Nucleus (brainstem) 3. Huntington's Disease: - Low GABA - makes sense - less inhibition produces sxs like chorea. - Atrophy of caudate (main loss) and putamen, enlargement of lateral ventricles due to loss of caudate. Caudate + Putamen = Dorsal Striatum. - They have trinucleotide repeats of CAG (CAG-CAG-CAG-...) - MNEM: No Caudate => low ACh and GABA.

A newborn girl is found to have ambiguous genitalia. Genetic workups reveal a normal chromosome analysis. Ultrasound reveals the presence of enlarged adrenal glands bilaterally. 1. What is the most likely diagnosis? 2. What is the most common enzyme deficiency causing this?

1. Adrenogenital syndrome (Congenital Adrenal Hyperplasia) 2. 21-Hydroxylase Deficiency

++++++++++++++++++++++++++++++ Site of action in the nephron of each hormone: 1. Angiotensin II 2. ANP 3. PTH and Vitamin D 4. PTH also works at 5. Aldosterone 6. ADH

1. Afferent arteriole of the glomerulus 2. Glomerulus 3. Thick Ascending limb - prevents loss of Ca2+ 4. DCT -- prevents loss of Ca2+ there too 5. Collecting Duct -- where the K+ channel is 6. Collecting Duct -- where water is reabsorbed if needed

++++++++ HIGH YIELD NT's = EASY POINTS ++++++++ Rapid Fire - Name the NT abnormalities in each: 1. Alzheimer's 2. Anxiety 3. Depression 4. Huntington's 5. Parkinson's 6. Schizophrenia

1. Alzheimer's - High: glutamate - Low: Acetylcholine 2. Anxiety - High: Norepinephrine (NE) - Low: Serotonin, GABA 3. Depression - High: none (MNEM: NTs are depressed) - Low: Serotonin, Norepinephrine, Dopamine > MNEM: Hence, SNDRIs 4. Huntington's - High: dopamine (early) - Low: GABA & Acetylcholine 5. Parkinson's - High: Ach - Low: Dopamine 6. Schizophrenia - High: Dopamine - Low: glutamate

A 16 year old girl is worried because she hasn't started her period. She is concerned that she might have cancer, since her aunt and grandmother died from ovarian cancer. She presents to the doctor who performs an examination and discovers that the patient has a shortened vagina and no cervix. An ultrasound reveals absence of the uterus, fallopian tubes and ovaries. 1. What is the diagnosis? 2. What is the mechanism of this syndrome? I.e., what is messed up? 3. What are the expected levels of testosterone, LH, and estrogen?

1. Androgen insensitivity syndrome (testicular feminization) 2. Defective androgen receptor 3. Testosterone is high and LH and estridiol are slightly increased (estrogen is unopposed b/c receptors work fine, no negative feedback). (Image from google, not PPT)

Liver Neoplasias -- Name the liver neoplasia associated with each toxin / drug / chemical: 1. Vinyl chloride (also arsenic, thorium dioxide\contrast) 2. Thorium dioxide 3. Aflatoxin (aspergillus mold) and Vinyl Chloride 4. Birth control pills

1. Angiosarcoma - E.g., A 75-year-old man presents to the clinic with complaints of generalized fatigue and weight loss. He states that over the past 3 years he lost a total of 15 lbs. without changes to his diet or his exercise. He notes that he can no longer copmlete his daily 2 mile walks because he would get extremely tired half way. Laboratory studies revealed mild normocytic anemia and mild elevations in transaminases. Physical examination reveals moderate tenderness to palpation at the right upper quadrant. 2. Cholangiocarcinoma - E.g., A 35-year-old man with a history of primary sclerosing cholangitis presents with two weeks of jaundice, pruritus, and abdominal pain. He also has noticed that his urine is dark and stools are clay-colored. On physical exam, he has generalized jaundice and a palpable right upper quadrant mass. Lab studies are remarkable for elevated alkaline phosphatase, total bilirubin, alanine aminotransferase, aspartate aminotransferase, and CA 19-9. Magnetic resonance cholangiopancreatography reveals a contrast-enhancing intrahepatic lesion with rim enhancement. 3. Hepatocellular carcinoma (HCC) 4. Hepatic adenoma

++++++++++++++ LIKELY ON STEP ++++++++++++++ A 21-year-old man presents to his physician's clinic with lower back pain. He reports the pain is accompanied by stiffness, is worse in the morning, and improves with exercise. He denies any radiation of pain or bowel and bladder incontinence. On physical exam, there is decreased anterior flexion of the lumbar spine. Core sxs: Bamboo spine, uveitis, and aortic regurgitation 1. Diagnose 2. This condition is strongly associated with a positive titer for HLA-___. MSK Pathology

1. Ankylosing Spondylitis 2. HLA-B27 One of the 4 types of Seronegative Spondyloarthritis that all have positive HLA-B27 (called seronegative because serum lacks rheumatoid factor): - MNEMONIC: PAIR: Psoriatic arthritis Ankylosing Spondylitis Inflammatory Bowel Disease (ulcerative colitis and crohn's) Reactive arthritis / Reiter's syndrome - All of these share symptoms of inflammatory back pain (associated with morning stiffness, improves with exercise), peripheral arthritis, enthesitis (inflamed tendons, e.g., Achilles), dactylitis ("sausage fingers"), and uveitis.

A 60 y/o man with a 30 year 1 ppd smoking history and a PMH of hyperlipidemia comes to ER with severe tearing chest pain. The pain gradually moves to his back during the 2-3 hour drive to the ER. He says "Damn it Doc, it's a 12/10 and tearing me up Doc...DO SOMETHING!" BP 160/110 UA: hematuria 1. Diagnose 2. What is causing hematuria?

1. Aortic dissection 2. Bilateral renal infarction from aortic dissection.

NAME THAT VIRUS! 1. Transmitted by mosquitoes. Birds are the reservoir. West nile virus is one of these. 2. Most common cause of viral meningitis. Peaks in late summer early fall. Also causes myocarditis. 3. Targets the Ach receptor. Causes hydrophobia, fever, encephalitis. Is universally fatal if not treated 4. Destroys upper and lower motor neurons. Causes muscle paralysis. 5. Most common viral CNS infection in HIV positive patients. Causes periventricular calcification in newborns. Has an "owl's eye" basophilic inclusion. Histopathology of lung shows a pneumocyte containing characteristic intranuclear inclusion.

1. Arbovirus 2. Coxsackievirus 3. Rabies 4. Polio 5. Cytomegalovirus (CMV)

A study is conducted to evaluate physiologic adaptation to high altitude. Two groups of 20 individuals participate. The first group comprises long-term residents of communities in the Himalayan Mountains, located at an elevation of 4,267 m (14,000 ft). Blood samples, which were collected in their home communities, were found to have an arterial O2 content of ~20 mL O2/dL blood. A second group comprises individuals who are long-term residents of communities located at sea level. Blood samples from these individuals, which were collected at sea level, were also found to have an arterial O2content of ~20 mL O2/dL blood). In blood taken from the group living at high altitude, as compared with samples taken from those living at sea level, you would expect high or low: 1. Arterial PO2 2. Arterial O2 Saturation 3. Hematocrit

1. Arterial PO2 - LOW 2. Arterial O2 Saturation - LOW 3. Hematocrit - HIGH Increased ventilation and increased hematocrit do not return arterial PO2 and Saturation to normal.

1. An otherwise ethical physician asks extra questions and writes a longer note to upcode a bill to generate more money for the practice without thinking about the questionable behavior. A. Suppression B. Compartmentalization C. Splitting D. Rationalization E. Dissociation 2. A 55-year-old man who is a business executive is admitted to the hospital for evaluation of abdominal pain. He is polite to the physician but berates the nurses and other staff. The patient's wife and two of his three adult children arrive for a visit. The patient says with disgust that the missing child is and always has been worthless. Which of the following is the most likely explanation for this patient's behavior? (A) Countertransference (B) Projection (C) Projective identification (D) Reaction formation (E) Splitting

1. B. Compartmentalization 2. (E) Splitting

A 52-year-old woman with fair skin is concerned about a pink, pearly lesion that is growing slowly on her cheek. She is a varsity water polo coach at her local high school and spends many hours under the sun. She admits to not using sunscreen consistently. Physical examination shows a pearly pink to white dome-shaped lesion with central ulceration and telangiectasias. 1. Diagnose this skin cancer. 2. Will this invade locally? 3. Will this metastasize? 4. Histology will show what feature that isn't present in other skin cancers? 5. Gross appearance of lesion Dermatopathology

1. Basal Cell Carcinoma of the skin 2. Invades locally 3. Does not metastasize (just like SCC of skin) Slow-Growing skin cancer suggests BCC > SCC. Gross Appearance (First Aid) - Waxy, pink, pearly nodules, commonly with telangiectasias, rolled borders, or central crusting / ulceration. - or, Pearly white domed lesion with central ulceration or telangiectasias. - "ucleration" is usually SCC > BCC, but if it says "central ulceration," you can start thinking more BCC > SCC. Histology: - basophilic palisading cells on histology - nests of basaloid cells in dermis - So look for pouches of purple cells and white areas that look like they're moving to the top. (a palisade is a single layer of relatively long cells, arranged loosely perpendicular to a surface and parallel to each other)

+++++++++++++++++ HIGH YIELD +++++++++++++++++++ GABA Agonists: Important MoA's 1. Do Benzodiazepines increase the duration or frequency of Chloride channel opening? 2. Do Barbituates increase the duration or frequency of Chloride channel opening? 3. Which receptor do Z-drugs (zolpidem, zopiclone, and zaleplon) bind to?

1. Benzodiazepines increase the FREQUENCY of chloride channel opening. 2. Barbituates increase the DURATION of chloride channel opening (That's how those differ, even though they bind to the same site). 3. Z-drugs bind to the BZ-1 (AKA Omega 1) receptor - "Z-drugs" all start with Z. Prototype is Zolpidem. - "BZ" for benzodiazepine - Appear to require an α1 subunit on the GABA receptor as well. - "These ZZZs put you to sleep." Zolpidem, Zaleplon, etc.

Name each platelet adhesion defect: 1. ___ - ___ ◦Defect in plug formation ◦Platelets lack Gp1b ◦Platelets don't stick to collagen ◦Can have decreased platelet count ◦Platelets can clump on peripheral smear 2. _____ _____ ◦Defect in plug formation ◦Platelets lack GpIIb\IIIa ◦Platelets don't stick to other platelets ◦Platelet count usually normal ◦No platelet clumping on peripheral smear

1. Bernard-Soulier 2. Glanzmann's thrombasthenia

Achalasia: 1. Barium swallow imaging looks like what??? 2. How will the symptoms differ from partial obstruction (mass effect)? 3. What is an infectious cause? 4. Which autoimmune dz can cause? 5. What is the mechanism? 6. If untreated, can lead to?

1. Bird's Beak 2. Trouble with solids and liquids, Regurgitation of undigested food 3. Chagas' disease 4. Scleroderma \ CREST 5. Loss of myenteric plexus (Auerbach's) 6. Esophageal carcinoma

A 40-year-old man presents to the emergency room for blurry vision. He reports that the blurry and double vision started this morning along with nausea and vomiting. He also reports feeling weak in his arms. Upon further questioning, he recently ate a can of beans from his aunt in Alaska, where they run their own food storage business. On physical exam, there is bilateral ptosis and facial weakness. There is also bilateral upper arm weakness with absent deep tendon reflexes. His mental status is intact. His physician immediately administers antitoxin treatment for the disease. 1. Diagnose 2. This disease is caused by antibodies to what part of the NMJ? 3. What are the hallmark symptoms of this disease?

1. Botulinism (Clostridium botulinum) - 2 main sources are (1) improperly canned foods and (2) honey in babies (floppy baby syndrome). 2. Blocks the release of Ach from the pre-synaptic neuron into the NMJ. 3. Causes flaccid paralysis, may affect the pupils, often described as "descending" paralysis - usually affects face, eyes, eyelids, vision, arms, hands. - might also see nausea and vomiting. First Aid: Symptoms of botulism = (the 4 D's): Diplopia, Dysarthria, Dysphagia, Dyspnea. Botulinum is from bad bottles of food, juice, and honey (causes a descending flaccid paralysis). If babies injest honey, can get botulinism, where it's called floppy baby syndrome.

A 62 year old presents to the ED complaining of severe abdominal pain, scleral icterus, and jaundice. Ultrasonography reveals a blockage in the hepatic venous connection to the inferior vena cava and the absence of any waveform in the hepatic veins. His activated partial thromboplastin time (aPTT) is prolonged and his H/H is 21/64. Labs also reveal elevations in transaminases (AST and ALT). Physical examination reveals an enlarged and tender liver, ascites, and a palpable spleen. 1. What is the most likely diagnosis? 2. If this was a young woman, what type of medication would be the most likely cause? 3. What imaging modalities can confirm diagnosis? 4. Classic pathology finding / description

1. Budd-Chiari Syndrome (Thrombosis of the Hepatic vein that drains the liver) - Causes blood to back up into the liver - Sxs: abdominal pain (always), jaundice/icterus, hepatomegaly, varices, ascites 2. OCP's, a thrombotic disorder like Protein C \ S deficiency or APLA syndrome. All of these would cause a hypercoagulable state, and Budd-Chiari is from a thrombosis in the hepatic vein. Polycythemia vera is another common cause (makes sense -- too many RBCs). 3. Ultrasound (Doppler) or MRI 4. Classicaly described as a "nutmeg liver"

A 55-year-old man comes to the physician because of a 2-week history of recurrent, widespread blister formation. Physical examination shows lesions that are most numerous in the flexural areas including the axillae and groin. The blisters do not break easily, and there are no oral lesions. 1. Diagnosis 2. These blisters are most likely the result of adhesion failure involving which of the following? (A) Basement membrane (B) Dermal papillae (C) Langerhans cells (D) Melanocytes (E) Merkel cells 3. Would this patient have a positive or negative Nikolsky sign? Dermatopathology

1. Bullous pemphigoid - Autoimmune skin condition that causes large, fluid-filled blisters. - They develop on areas of skin that often flex — such as the lower abdomen, upper thighs or armpits. - Type II hypersensitivity reaction - IgG antibodies against hemidesmosomes (epidermal basement membrane); antibodies are "bullow" the epidermis). - Bulla = a large blister filled with fluid. 2. (A) Basement membrane (AKA hemidesmosomes) - IgG against basement membrane - MNEM: They are "Bullow" (below) the basement membrane. 3. Negative Nikolsky sign (i.e., they do not slough off when rubbed), unlike Pemphigus Vulgaris (the other autoimmune skin blistering condition) The blisters are tense because they are below the level of the basement membrane, which is what the antibody targets. The only other site that commonly has "adhesion failure" due to antibodies would be choice B, dermal papillae, which describes dermatitis herpetiformis, but those are classically on the extensor surfaces and associated with GI symptoms (celiac dz).

1. A 7 yo boy is brought to the physician by his mom because of recurrent epistaxis for the past 2 days. He recently recovered from a mild illness with sore throat and cough. PE shows multiple mucosal petechiae, and several bruises on his arms and legs, but no other abnormalities. Lab tests show: Hb 14 g/dl (normal); PT: 13 sec (~inc.); PTT: 30 sec (normal); platelet count 8,000/mm3 (very low). Remainder of CBC is normal. Which of the following is the most likely underlying cause of these findings? A. Bernard-Soulier syndrome B. Glanzmann thrombasthenia C. Immune thrombocytopenia D. Von-Willebrand disease E. Wiskott-Aldrich syndrome 2. A primigravid 32 yr old females at 37 weeks gestation develops severe lower abdominal pain and vaginal bleeding and is brought to the ED. After examination, she is rushed to the OR for emergency C-section. Following the surgery, the patient develops shortness of breath, cyanosis, and copious bleeding from her surgical wounds. Labs show: platelet count of 75,000 (low), PT of 30 sec (inc.), and PTT of 56 sec (inc.). D-dimers are elevated and fibrinogen is decreased. Which of the following factors best accounts for her bleeding disorder? A. Antiplatelet antibodies B. Bacterial toxins C. Lidocaine anesthetic D. Tissue factor E. Von Willebrand factor

1. C. Immune thrombocytopenia purpura (ITP) - Classic presentation - isolated thrombocytopenia) - Kid was normal and healthy, got sick, and then developed platelet problems. 2. D. Tissue factor - If a pregnant woman develops platelet problems after delivery, it's because she's releasing too much Tissue Factor (which starts the whole coagulation cascade)

Complement Deficiencies: 1. Increased neisseria infections (gonorrhea or meningitis) are associated with deficiency of C__ - C__. 2. Hereditary Angioedema and SLE are associated with deficiency of the C__ esterase inhibitor. 3. Paroxysmal nocturnal hemoglobinuria is associated with deficience of CD__. 4. SLE is associated with lack of C__ and C__ 5. Vasculitis is associated with lack of C__ and C__

1. C5-C9 (MAC) - increased neisseria infection 2. C1 esterase inhibitor - SLE and "Hereditary Angioedema" - Edema / swelling of skin, especially periorbital skin, and mucosal surfaces. 3. CD55 - Paroxysmal nocturnal hemoglobinuria 4. C2 and C4 - SLE 5. C5 and C6 - Vasculitis

+++++++++ HIGH YIELD, EASY POINTS +++++++++ T-Cells and their Cytokines: 1. CD8+ Cytotoxic T-cells express which 2 cytokines? 2. Th1 CD4+ Helper T-cells express which 2 cytokines? 3. Th2 CD4+ Helper T-cells express which 3 cytokines? 4. Regulatory T-cells (T-reg) have what function and express which 2 cytokines?

1. CD8+ Cytotoxic T-cells express IFNγ, TNFα 2. Th1 CD4+ Helper T-cells express IL-2, IFNγ 3. Th2 CD4+ Helper T-cells express IL-4, IL-5, IL-13 4. Regulatory T-cells express TGF-ß & IL-10 (which tone down the immune response) - Genetic deficiency of FoxP3, which makes those cytokines, results in autoimmunity because there's no toning down of the inflammatory response.

A 65 year old man presents with new onset diarrhea, hot flashes, facial flushing, difficulty breathing, and weight loss. He's had a decreased appetite for the last 2 months and has lost 15 pounds. 1. What is the most likely diagnosis? 2. What drug can be used for treatment?

1. Carcinoid syndrome 2. Octreotide Carcinoid Syndrome is a rare syndrome that is caused by the metastasis of carcinoid tumors that secrete high levels of serotonin. Can go to liver, etc. but that doesn't matter. Key Features always in stems: Diarrhea, Flushing. Due to increased serotonin. Carcinoid tumors most commonly originate in the ileum and appendix (90%) -- i.e., GI tract

A 21 year old woman presents to her gynecologist for a routine exam. An inflammed, granular cervix is noted and a biopsy is taken. She is diagnosed with chronic cervicitis. 1. What is the most likely cause? 2. What condition can it turn into if not treated?

1. Chlamydia 2. Pelvic inflammatory disease Chain Reaction: Chlamydia ➙ PID (cervical motion tendernes) ➙ Ectopic pregnancy and Infertility - Chlamydia is the #1 cause of PID and PID is the #1 cause of both ectopic pregnancies and sterility.

A 43 year old woman presents to her family practice doctor. She states that she is turning yellow and she itches all over. She has been having crampy abdominal pain for the last couple of months, but attributed it to her diet of Taco Bell and gas station sushi. 1. What is the likely diagnosis? 2. What would you expect the labs to show? 3. Why does she itch? 4. Is the bilirubin conjugated or unconjugated? 5. The patient is scheduled for surgery in 3 weeks and sent home. 1 week later she develops a high fever that spikes intermittantly, along with worsening jaundice and RUQ pain. What is the diagnosis? 6. The patient is brought in for a drain to be placed and during an ultrasound a solitary cyst in the right liver lobe is noted. It is tapped returning pus, what is the most likely organism (and group of organisms)? 7. Worldwide what is the most common organism causing liver abscesses (in the colon too!)?

1. Cholestatic liver disease due to a gallstone 2. Increased alk phos and GGT. 3. Bile salts deposited in the skin 4. Conjugated 5. Ascending cholangitis (Charcot's triad) 6. E. Coli (gram negative bacteria) 7. Entamoeba histolytica (amebiasis)

++++++++ MEMORIZE HOW TO DRAW THIS ++++++++ MNEMONIC drawing that shows the toxicities of: 1. Cisplatin 2. Vincristine 3. Doxorubicin 4. Bleomicin 5. Cyclophosphamide 6. Methotrexate

1. Cisplatin: ototoxicity, nephrotoxicity 2. Vincristine: Peripheral neuropathy 3. Doxorubicin: Cardiotoxicity 4. Bleomicin: Pulmonary fibrosis 5. Cyclophosphamide: Hemorrhagic cystitis 6. Methotrexate: Myelosuppression

+++++++++++++++++++++ What type of necrosis goes with each? 1. Heart, liver, kidney, tumors (most common) 2. Brain, abscesses 3. TB, and then fungi 4. pancreas 5. vessels 6. extremities, bowel

1. Coagulative - Heart, liver, kidney, tumors (most common) 2. Liquefactive - Brain, abscesses 3. Caseous - TB, TB, TB, and then fungi 4. Fatty - AKA saponification, pancreas 5. Fibroid - AKA fibrinoid (cause its pink), vessels 6. Gangrenous - extremities, bowel - Wet - associated with bacteria - Dry - ischemic (AKA mummification)

1. A 69 year old male presents with acute onset of chest pain and shortness of breath. He is diagnosed with acute myocardial infarction and undergoes percutaneous coronary intervention. After the procedure the attending tells the resident to start a P2Y12 inhibitor. Which of the following should the resident choose? A. Aspirin B. Rivaroxaban C. Abciximab D. Clopidogrel E. Dipyridimole 2. A 38-year-old woman returning to the US from a trip to Japan complains of sudden chest pain and shortness of breath. She is taking no medication other than an oral contraceptive pill. She is tachycardic and tachypneic, and her right calf is swollen and tender. She is admitted for appropriate therapy and started on unfractionated heparin. Which of the following laboratory tests is important in monitoring her initial therapy? A. Bleeding time B. International Normalized Ratio C. No monitoring is needed D. Partial thromboplastin time E. Prothrombin time 3. A 42 year old man is admitted for pneumonia. On day 6 of antibiotic treatment he is found to have the following CBC: WBC 15.3 (high); Hb 14.6 (normal); platelet 71,000 (low) The CBC on the prior day was normal with platelets of 187,000. PT/PTT are normal. CMP is normal. On day 7 he develops a left lower extremity DVT despite prophylaxis with low dose heparin. What is the most likely cause of his thrombocytopenia? A. Anti-platelet ab against GP IIb/IIIa B. Sepsis C. Antibody against heparin/PF4 complex D. Antibiotic reaction E. Amegakaryocytic thrombocytopenia

1. D. Clopidogrel - P2Y12 inhibitor 2. D. Partial thromboplastin time (PTT) - ALWAYS monitor PTT for anyone on heparin - This pt had a DVT 3. C. Antibody against heparin/PF4 complex - If Heparin did not work, must be a problem with Heparin's MoA. - Heparin is involved in inactivation of coagulation by stimulating antithrombin.

A 64-year-old man with a history of obesity, diabetes, hypertension, and hyperlipidemia. Acute onset of nausea, vomiting, diaphoresis, and crushing substernal chest pain. 37°C, HR 110, BP 149/87, and RR 22 99% O2 on room air. Auscultation reveals an S4 sound. Labs reveal elevated troponin. ECG shows ST elevations. 1. Which is the most likely underlying pathology? A. Partial occlusion by plaque B. Partial occlusion by thrombus C. Partial occlusion by vasospasm D. Full occlusion by thrombus E. Full occlusion by vasospasm 2. What is the most likely diagnosis? A. Stable angina B. Unstable angina C. Variant (Prinzmetal) angina D. NSTEMI E. STEMI

1. D. Full occlusion by thrombus 2. E. STEMI - STEMIs are due to COMPLETE occlusion of a coronary artery by a plaque. - Partial occlusion results in unstable angina or Non-STEMIs.

1. You are caring for a newly diagnosed patient with a brain tumor and reviewing the common side effects of the therapy they are about to receive. The standard chemotherapy regimen for this disease is cisplatin, vincristine, and cyclophosphamide. Which of the following severe side effects will this patient likely have from the cisplatin? A. Hemorrhagic cystitis B. Cardiac failure C. Severe diarrhea D. Ototoxicity E. Pulmonary fibrosis 2. You see a 15-year-old young man for respiratory distress and discover that he has a large anterior mediastinal mass and pancytopenia with circulating blasts. Which specific subtype of acute leukemia is most likely in this clinical situation? A. Pre-B ALL B. M3 (acute promyelocytic) AML C. M1 (undifferentiated) AML D. T-ALL E. M7 (acute megakaryoblastic) AML 3. You are evaluating a 15-year-old child who presents with a 1 week history of abdominal pain, fever and night sweats. A CT scan of the abdomen shows a 8.5 x 7.7cm mass in the cecum. You order a biopsy of the mass and the pathologist reports that the histology shows a "starry sky" appearance and that 99% of the malignant cells are positive for CD20 and Ki-67. You request routine cytogenetics and FISH studies. What is the most likely result from the cytogenetic and FISH studies? A. Positive FISH for t(8;14) B. Positive FISH for t(15;17) C. Positive FISH for t(14;18) D. Positive FISH for t(12;21) E. Positive FISH for t(9;22) 4. You see a 67-year-old man for a routine yearly physical exam and he complains that he feels tired all the time and he has a tingling sensation in his feet at times. He is generally healthy, and the only medication that he takes on a daily basis is a long-acting NSAID for increasingly bothersome back pain. Physical exam is remarkable only for pallor and some tenderness to percussion over the lumbar spine. Screening labs reveal a normal WBC and platelet count, a mild normocytic normochromic anemia with a Hb of 9.1 g/dL, an elevated serum calcium of 12.1 mg/dL, and an elevated serum creatinine of 2.3 mg/dL. Which of the following tests is most likely to yield the correct diagnosis in this patient? A. Nerve conduction testing B. Lymph node biopsy C. Lumbar puncture D. Sleep study E. Serum protein electrophoresis (SPEP)

1. D. Ototoxicity 2. D. T-ALL 3. A. Positive FISH for t(8;14) - Burkitt Lymphoma 4. E. Serum protein electrophoresis (SPEP) - M-Spike for Multiple Myeloma

3 possible causes of prolonged PT and PTT?

1. DIC Consumption of multiple coagulation factors 2. Vitamin K deficiency Decrease in Vitamin K-dependent factors, including FII (thrombin), VII, IX, and X 3. Liver disease All clotting factors, except FVIII, synthesized in liver

Creutzfeldt-Jakob Disease (CJD) (Mad Cow Disease) has two BIG clinical hallmarks that are always observed in infected pts. What are they? (Two buzz words) What is the clinical picture of these patients in the terminal stage of CJD? Brain biopsy will show ___-like lesions

1. Dementia that rapidly gets worse and worse. 2. Myoclonus- a sudden, involuntary muscle jerk or spasm (twitch). Terminal stage: Mute, rigid, unresponsive The prions gradually eat away at your brain matter. Brain biopsy will show sponge-like lesions

A 41 year old man presents complaining with problems with his golf swing. He states that lately he hasn't had any power and his friends are laughing at his driving distance. Upon examination, you note symmetrical weakness of his shoulders and upper arms. You also note a rash, which he attributes to being in the sun all the time, on his cheeks and around his eyes. 1. Diagnose 2. What should he be checked for...? 3. What antibodies will be positive? 4. This condition is mediated by which type of T-cell? MSK Pathology

1. Dermatomyositis - Like Polymyositis, but also involves skin rashes (hence dermato) - Symmetric proximal muscle weakness 2. Increased risk of any type of malignancy -- ALWAYS need to screen them for cancer. 3. Positive ANA and Anti-Jo-1 4. CD8 T-cell mediated E.g., (USMLE-Rx): A 38-year-old woman presents to the emergency department complaining of increasing muscle weakness and pain. She first noticed the muscle weakness approximately 1 month ago, and it has gradually worsened since then. During the same time she has had increasing difficulty swallowing her meals. Two weeks prior to this visit, she recalls swelling and a rash over her eyelids. On physical examination, deltoid and quadriceps strength are 2/5 bilaterally. Serum analysis reveals a creatinine kinase level of 2105 U/L, lactate dehydrogenase of 105 U/L, and an elevated aldolase level. Which of the following autoantibodies would diagnostic testing most likely find to be elevated? A. Anti-IgG B. Anti-Jo-1 C. Anti-dsDNA D. Antimicrosomal E. Antimitochondrial

1. What is the effect of Dopamine on Prolactin? 2. What 2 medications (and type) are used to treat hyperprolactinemia? 3. What type of medication increases Prolactin, and can thus lead to hypogonadism?

1. Dopamine inhibits the secretion of prolactin from the anterior pituitary. 2. Bromocriptine and Cabergoline are dopamine agonists used to treat hyperprolactinemia. - They can shrink prolactin secreting tumors and are normally considered to be standard, first line therapy. 3. Antipsychotics reduce dopamine and can thus lead to elevated prolactin (loss of prolactin inhibitor). - High prolactin suppresses GnRH, and thus suppresses TSH and LH, and can thus lead to hypogonadism (side effect of antipsychotics).

++++++++++++++++++++++++++++++++++++++++ The Big 3 Spinal Tracts: Which one is responsible for: 1. Proprioception, vibration 2. Pinprick, temperature 3. Voluntary movement Label each on a cross-section of the spinal cord.

1. Dorsal Columns: Proprioception, vibration - On cross section, these are at the top, on the dorsal side. 2. Spinothalamic Tract: Pinprick, temperature - On cross section, these are at the bottom, on the anterior side. 3. Corticospinal tract: Voluntary movement - On cross section, these are on the sides in the middle (vertically).

++++++++++++++++++++++++++++ Specific Diseases w/ Type 2 Hypersensitivity: 1. Drug-induced hemolytic anemia is usually caused by what drug class? 2. Autoimmune hemolytic anemia involves an antibody against what? 3. Myasthenia gravis involves an antibody against what? 4. Grave's disease involves an antibody against what? 5. Goodpasture's syndrome involves an antibody against what? 6. Acute rheumatic fever involves an antibody against what? 7. Type 2 diabetes involves an antibody against what?

1. Drug-induced hemolytic anemia - Usually cephalosporins 2. Autoimmune hemolytic anemia - Ab to Rh blood group antigens 3. Myasthenia gravis - Ab to the post-synaptic Ach receptor, leading to progressive weakness 4. Grave's disease - Stimulating Ab to thyroid-stimulating hormone receptor => hyperthyroidism 5. Goodpasture's syndrome - Ab to basement membrane => glomerulonephritis + pulmonary hemorrhage 6. Acute rheumatic fever - Ab to streptoccal Ag, cross-reacts with cardiac muscle 7. Type 2 diabetes - Ab against the insulin receptor => hyperglycemia

A 4-year-old boy is brought to his pediatrician by his mother for muscle weakness. She reports that he has trouble running with his friends and climbing the stairs. She also states that the patient has increased difficulty arising from a seated position on the ground and walks in a "waddling" way. On physical exam, there is significant proximal muscle weakness and pseudohypertrophy of the bilateral calves. He also has Gower sign. Laboratory testing is notable for a significantly elevated creatine kinase and aldolase. A sample is obtained for genetic analysis of the dystrophin gene. 1. Diagnose 2. What protein is defected? 3. What are the major hallmarks of this disease? 4. What is his prognosis?

1. Duschenne Muscular Dystrophy - because it's presenting in a 4 y/o; Duschenne presents early, Becker presents late. 2. Dystrophin is messed up. 3. Major signs: Delayed motor milestones; Gower's Sign; Proximal muscle weakness. 4. Prognosis: • Loss of ambulation by teens • Life expectancy <20 y/o • Respiratory failure, Cardiac disease

Diagnose: 1. You hear a wide S2 split that grows even wider during inspiration. 2. Wide S2 that improves during inspiration. 3. Fixed S2 split (always the same) A. Aortic regurg B. Aortic stenosis C. Atrial septal defect D. Pulmonic regurg E. Pulmonic stenosis F. Ventricular septal defect

1. E. Pulmonic stenosis 2. B. Aortic stenosis 3. C. Atrial septal defect ***SEE IMAGE - focus on distance between red and blue blocks (red = aortic valve, blue = pulmonary valve) - Widened S2 means the pulmonary valve is closing late. - This can occur normally in a young pt (<15 y/o) and it's very mild. "Wide S2 that worsens with inspiration" = PS (or RBBB) - see left side of IMAGE "Wide S2 that improves during inspiration" (backwards) = AS (or LBBB) - P comes before A here (PAradoxical) because aortic valve has stenosis, so slower to close. - see right side of IMAGE "Fixed S2 split" = ASD

In an animal model of myocardial infarctions, the left anterior descending artery is ligated (tied off with a suture), significantly reducing blood flow through it. After this successful induction of an MI.... 1. ...administration of Adenosine is most likely to: A. Constrict coronary collateral arteries B. Dilate arterioles distal to LAD C. Increase resistance of obstructed coronary arteries D. Increase venous capacitance E. Reduce resistance in the normal coronary arteries 2. ...blood flow in the region distal to obstruction (O) and the rest of ventricle (R) is measured while the animal runs on an exercise wheel. Compared to baseline, the most likely changes in blood flow and oxygen consumption during exercise are: A. O: constant, R: constant B. O: constant, R: increased C. O: increased, R: reduced D. O: increased, R: constant E. O: reduced, R: increased F. O: reduced, R: reduced

1. E. Reduce resistance in the normal coronary arteries 2. E. O: reduced, R: increased This is about how the heart compensates and how adenosine can help during an MI. If you are blood, where would you go? The path of least resistance. The "point to remember": Blood flow in the healthy coronary arteries will increase and oxygen consumption in non-affected parts of the ventricle will increase after an MI. - The area that was supplied by the blocked artery will likely die after a couple of days from ischemia. But the other areas will be okay thanks to collateral circulation. Ligating a coronary artery mimics an MI as it obstructs flow through one of the coronary arteries. This is a common method used to observe the effects of an MI on the heart. Ligating a coronary artery does induce an MI, but the effects of this are minimized by the collateral circulation of the heart...other arteries take over. But the area that would have been perfused by the blocked artery doesn't get anything. The healthy arteries "STEAL" the heart's blood flow (arrow in left of Image). (1) Adenosine has potent vasodilator effects. It can be administered during an MI to increase blood delivery to the heart in the non-affected coronary arteries. It does this by vasodilation, which decreases the resistance in the other coronary arteries. (2) During and after an MI, the other vessels compensate for the vessel that was obstructed. Blood flow to the bad artery will be reduced while blood flow to the healthy coronary arteries will be increased. This helps protect the heart from ischemic injury. - Don't get fixated on the exercise part. That's just to increase the work of the heart for the purpose of seeing the effects better. These changes in blood flow are present even without exercise.

Polymyositis and Dermatomyositis Diagnosis: 1. Which muscle enzymes would you expect to be elevated? 2. Muscle Biopsy: What type of T-cell is elevated in PM vs. DM? 3. EMG (in 85-90% patients) will show what typical findings? 4. MRI will show what finding in the affected muscles?

1. Elevated muscle enzymes: CPK (usually in 1000's), Aldolase, SGOT, SGPT, LDH 2. Muscle Biopsy: - PM: CD8+ T cells endomysial infiltrate - DM: CD4+ T cells perifascicular infiltrate with vasculitis - Both will have muscle fiber necrosis and regeneration 3. EMG in 85-90% of patients will show: > Increased insertional activity fibrillations > Sharp positive waves > High frequency discharges > Low amplitude, short duration polyphasic motor unit potentials. 4. MRI will show muscle edema (i.e., inflammation) Other Quizlet User: 1- symmetric proximal muscle weakness 2- increased CPK, ALT, APT< LDH 3- myopathic EMG abnormalities 4- typical changes on muscle biopsy 5- typical rash of dermatomyositis (shiny which scaly on joints extensor, subcutaneous calcification)

Diagnose each Elimination Disorder: 1. Repeated passage of feces into inappropriate places (involuntary or intentional). - Chronological age at least 4 years (so if 3 y/o, normal). - Not due to laxatives or medical problem - At least one event each month for at least 3 months 2. Repeated urination into bed or clothes - Must be at least 5 y/o (so if 4 y/o, it's normal) - Frequency is 2x/week for at least 3 months 3. What would you tell a parent (i.e., what will be the correct exam answer) if their kids are excessively pooping before the age of 4 or excessively urinating before the age of 5 (i.e., before the age cut-offs)?

1. Encopresis (incontinence of feces) 2. Enuresis (incontinence of urine) 3. If age is less than cutoff, Answer is "normal" or "reassurance."

A 12 year old boy is brought to the clinic by his mother. She states he has been complaining of pain for months and he has trouble using his left arm. X-ray reveals a mass involving the diaphysis of his left humerus with an onion-like appearance. Biopsy of the tumor reveals small, round blue cells. 1. Diagnose 2. What is the most common associated translocation? MSK Pathology

1. Ewing's Sarcoma: Onion-like appearance of bone is a BUZZword 2. t11;22 - First Aid MNEM: 11 + 22 = 33 (Patrick Ewing's jersey number). Demographics: Most common in Caucasians. Generally boys < 15 years old. Location: Diaphysis of long bones (especially femur), pelvic flat bones. Biopsy Findings: Anaplastic small blue cells of neuroectodermal origin (resemble lymphocytes)

Babies with Jaundice 1. Jaundice at birth? 2. Treatment (2) 3. Jaundice at 3 days old? 4. Treatment?

1. Extrahepatic biliary atresia 2. Kasai procedure (hepatoporto-enterostomy) or Liver transplant 3. Physiologic jaundice of newborn 4. UV light therapy

+++++++ High Yield Psychiatry Medications +++++++ 1. Haloperidol (antipsychotic) is famous for causing what adverse side effect? (Think about what the medication does / MoA). 2. Lithium has what 2 main medical side effects to look out for? First Aid MNEMONIC for Lithium SEs ("LiTHIUM") 3. Which 3 benzos are safe on the liver and in the elderly as they don't require oxidative P450 metabolism in the liver? (Really good MNEMONIC from Dr. Spollen)

1. Extrapyramidal Symptoms (Parkinson's-like Symptoms) On STEP, look for EPS (haloperidol) due to dopamine / substantia nigra / midbrain as mechanism -- lowers dopamine, which can cause Parkinson's-like Symptoms. - Remember how antipsychotics do that? - MNEM: Haloperidol causes extrapyramidal symptoms - So tremors, slowed movement (bradykinesia), rigid muscles, impaired posture / balance, difficulty initiating movements, etc. 2. Li causes Hypothyroidism and decreased renal clearance (Low GFR, High Creatinine) - i.e., decreased function of the Kidneys & Thyroid - Hypothyroidism: e.g., cold intolerance, fatigue, constipation, weight gain, etc. - Renal: Li is an ADH antagonist, resulting in nephrogenic diabetes insipidus (polyuria). >> First Aid: Lithium is excreted by kidneys, and then reabsorbed in the PCT with Na+. Thiazides are implicated in lithium toxicity. To help you remember, Lithium is also implicated in treating SIADH (but not 1st line), so it must increase urination. - First Aid Mnemonic for Lithium SEs: LiTHIUM: > Low Thyroid (hypothyroidism) > Heart - teratogenic; Ebstein anomaly (defective tricuspid valve) in newborns if taken during pregnancy. So DON't give to pregnant mothers. > Insipidus (nephrogenic diabetes insipidus) > Unwanted Movements (tremor) 3. Oxazepam, Temazepam, and Lorazepam are safe On The Liver

Thrombotic Diseases to Know: 1. Most common cause of inherited hypercoagulable state 2. ___ gene mutation = 3'-UTR mutation = venous clots 3. Reduced effect after heparin (i.e. the PTT doesn't go up much) 4. Can't inactivate FV or FVIII...so give warfarin and watch their skin fall off

1. Factor V Leiden - mutated form of factor 5 - Protein C cannot break down this mutated form of Factor V. Leads to hypercoagulability. 2. Prothrombin gene mutation 3. ATIII deficiency 4. Protein C or S deficiency Protein C has a short half life and the imbalance in coag factors leads to coagulation and no C to stop it!

Common Breast Masses / Tumors: 1. Small, mobile, firm, well demarcated. Young women. Not a precursor to cancer. May be painful with ↑ estrogen (varies through mentrual cycle). 2. Occurs in the lactiferous duct, below the areola. All ages. Causes bloody nipple discharge, slight risk for Ca (2x). 3. Large, bulky, leaf like projections. Older women (6th decade). Some malignant potential (sarcoma). 4. Firm, fibrous mass. Calcifications on X-ray. Older woman, younger if BRCA associated. 5. Scaly, erythematous nipple, may bleed. Middle aged to older women. Usually associated with underlying breast cancer. 6. Peau D'orange. Older women. Lymphatics blocked by cancer, bad prognosis.

1. Fibroadenoma 2. Intraductal papilloma 3. Phyllodes tumor 4. Breast Carcinoma - Ductal Ca most common, with worst prognosis. Lobular may be bilateral 5. Paget's Disease of the Nipple 6. Inflammatory Breast Cancer

Name each Sex Cord Stromal Tumors of the Ovaries/Testes: 1. Common, made of fibroblasts, Meigs' syndrome (ascites, hydrothroax) 2. Makes steroids (androgens), gynecomastia, Reinke Crystals. 3. Makes lots of estrogen (endometrial hyperplasia \ ca). Has Call-Exner bodies 4. Often seen with leydig cells (to the left) forming a sertoli leydig cell tumor

1. Fibroma - common, made of fibroblasts, Meigs' syndrome (ascites, hydrothroax) 2. Leydig cell tumor - makes steroids (androgens), gynecomastia, Reinke Crystals. MAKES SENSE - Leydig cells secrete testosterone. 3. Granulosa cell tumor - makes lots of estrogen (endometrial hyperplasia \ ca). Has Call-Exner bodies ("Give Granny a Call). 4. Sertoli cell tumor - often seen with leydig cells (to the left) forming a sertoli leydig cell tumor

Ovarian Cysts - Name each: 1. Common cyst Usually 1-5 cm Lined by follicle / granulosa cells Can secrete estrogen and be associated with endometrial hyperplasia May bleed or rupture 2. Most common cyst during pregnancy 5-20 cm! May bleed or rupture

1. Follicular cyst 2. Corpus Luteal Cyst / Luteal Cyst

+++++++++++++++++ HIGH YIELD +++++++++++++++++++ 1. GABA receptors are ___ ion channels. 2. Does GABA activation hyperpolarize or hypopolarize neurons to deactivate them? 3. What 3 types of drugs act on GABA-a receptors?

1. GABA receptors are Chloride ion channel 2. GABA receptor binding will hyperpolarize the neuron, preventing it from firing. Hyperpolarizes via the Cl- ion channels. 3. Barbiturates, Benzodiazepines, and "Z-drugs" (BBZ) (zolpidem, zopiclone, and zaleplon) are GABA agonists (also Alcohol).

Diagnose each: 1. Histology shows negatively birefringent needle-shaped crystals (yellow under parallel light). 2. Histology shows positively birefringent rhomboid-shaped crystals (blue under parallel light) MSK Pathology

1. GOUT - Monosodium urate crystal deposition due to hyperuricemia 2. Pseudogout - Pseudogout has Positive birefringence - Calcium pyrophosphate crystals - Commonly in large joints in elderly - Clinically may mimic gout

A 55-year-old woman with a history of osteoarthritis comes into her primary care physician's office with a complaint of new indigestion. She notes that, for the past week or so, she has been experiencing discomfort in her upper abdomen, associated with meals. This has not affected her appetite, and she has not vomited up any blood or experienced any weight loss. She denies any heartburn/GERD symptoms or substance use. Her past medical history is noteworthy for osteoarthritis in her knees, and she says she has been taking naproxen as prescribed everyday for the past year and a half. She takes no other medications and is otherwise relatively healthy. 1. What is the diagnosis? 2. What is the most likely cause? 3. What malignancy is she at risk for?

1. Gastritis (can be acute or chronic) - protective mucosal barrier is disrupted and stomach lining is exposed to acid, resulting in inflammation - usually due to H. pylori, NSAIDs or alcohol 2. H. Pylori infection - Also, NSAIDs and Alcohol iritate the stomach lining. 3. MALT Lymphoma is more likely (if untreated). - Gastric lymphoma is less likely Tx with PPIs -- makes sense -- decrease the acidity.

Categorization of Glomerular Diseases: 1. Which two glomerular disorders are genetic (inherited)? 2. Which two glomerular disorders are immune-mediated? 3. Which two glomerular disorders are caused by metabolism defects? 4. Which glomerular disorder occurs after an infection?

1. Genetic: Alport's disease, Familial forms of nephrotic syndrome 2. Immune mediated: Lupus nephritis, MPGN 3. Metabolic: Diabetic nephropathy, Amyloidosis 4. Infection related: Post streptococcal GN

++++++++++++++++++++++++++++++ For each Hyperbilirubinemia, name the disease and cause: 1. ↑ UBR and slight jaundice that increases with stress and fasting. Asymptomatic. - Caused by decreased amount of what enzyme? 2. ↑↑ UBR, full jaundice, kernicterus. These patients die young (therefore pt will be young). - Caused by total lack of what enzyme? 3. ↑ CBR, defective excretion leads to black liver (4. ↑ CBR, however, liver is NOT black)

1. Gilbert's Disease - ↓UDP-glucuronyl transferase (less bilirubin is conjugated), or ↓ bilirubin uptake - Unconjugated bilirubin is therefore high, causing some jaundice (only conjugated BR is water soluble and can be excreted) 2. Crigler-Najjar type I - Abstent UDP-glucuronyl transferase; bilirubin is not conjugated. - Unconjugated bilirubin is therefore high, causing jaundice 3. Dubin-Johnson Syndrome - Defective bilirubin excretion by liver - Leads to buildup of bilirubin in liver, causing a black liver - Conjugated bilirubin is high -- bilirubin is still conjugated by UDP-GUT, but it isn't being excreted. 4. Rotor's Syndrome - Defective bilirubin excretion by liver - but liver is not black

+++++++++++++++++++++++++++++++++++++++++++++++++++++ Aphasias - Name the type of aphasia and where the damage is for each scenario: Note: Here, "speak" = produce own original speech 1. Can't speak, Can't understand, Can't repeat. 2. Can't speak, Can't understand, but CAN repeat. 3. CAN understand, but Can't speak, Can't repeat. 4. Can't speak, but CAN understand and CAN repeat. 5. CAN speak, but can't understand or repeat. 6. Can speak, CAN'T understand, CAN repeat. 7. Can speak, Can understand, but CAN'T repeat.

1. Global Aphasia; Large damage in the dominant hemisphere -- No language functions at all. 2. Transcortical Mixed Aphasia; Both transcortical areas: Mesial frontal and Temporal-parietal junction 3. Broca's Aphasia; Inferior frontal gyrus 4. Transcortical Motor Aphasia; Mesial Frontal 5. Wernicke's Aphasia; Posterior-Superior Temporal gyrus. 6. Transcortical Sensory Aphasia; Temporal-Parietal junction 7. Conduction Aphasia; Arcuate Fasciculus All and only the transcortical lesions can repeat.

ARTHRITIS: 1. Which STD is associated with arthritis / joint problems? 2. What are the two common pathogens that cause infectious arthritis? 3. Arthritis associated with tick bite? 4. What condition includes arthritis associated with granulomas? MSK Pathology

1. Gonorrhea -- arthritis is a potential complication 2. Staph and Strep -- can cause infectious arthritis 3. Lyme Disease -- can cause arthritis 4. Tuberculosis -- can cause arthritis

+++++++++ HIGH YIELD, EASY POINTS +++++++++ 1. HLA-B7 is associated with what 4 major diesases? (MNEM) 2. HLA-D3 is associated with what major disease? 3. HLA-D4 is associated with what 2 major diseases?

1. HLA-B7: PAIR ✔ Psoriasis ✔ Ankylosising Spondylitis ✔ Irritable Bowel Diseases ✔ Reiter's syndrome (Reactive Arthritis- ureitis, conjunctivitis, athritis - can't see, pee, or climb a tree) 2. HLA-D3: ✔ Type 1 Diabetes, also SLE. 3. HLA-D4: ✔ Type 1 Diabetes ✔ Rheumatoid Arthritis (MNEM: There are 4 walls in a RHEUM).

+++++++++++++++++++++++++++ What is the rate-limiting enzyme of cholesterol synthesis? What is the primary reducing agent used in cholesterol synthesis?

1. HMG-CoA Reductase! 2. NADPH!

Basal Ganglia Diseases: 1. Subthalamic nucleus lesion results in ... 2. Substantia nigra compacta degeneration results in ...

1. Hemiballismus - type of chorea caused by a decrease in activity of the subthalamic nucleus of the basal ganglia, resulting in the appearance of flailing, ballistic, undesired movements of the limbs 2. Parkinson's disease - Substantia nigra degenerates, reducing the amount of dopamine released - Loss of smooth movements -- parkinson jerky movements.

Hepatitis -- Things to Remember: 1. Hepatitis B can progress to ___ or ___. 2. How do you get Hepatitis A? 3. Which Hepatitis usually becomes chronic? 4. Which hepatitis only occurs if you also have Hep B? 5. Which hepatitis has a severe and sudden onset in pregnant women?

1. Hep B can progress to Cirrhosis or Hepatocellular Carcinoma (HCC) 2. Fecal to Oral route; Hep A is very Acute; 1\3 of acute hepatitis, increased incidence in children, day-cares, prisons, travelers, no chronic state, most recover 3. Hep C becomes Chronic; HCC and cirrhosis can develop. 4. Hep D requires the Hep B surface antigen 5. Hep E has a severe and sudden onset in pregnant women.

Diagnose each by the liver inclusions listed: 1. "ground glass" pink cytoplasm 2. Mallory Hyaline - chunky pink inclusions 3. pink cytoplasmic inclusions 4. Iron - blue on iron stain 5. Fat - very common

1. Hepatitis B - "ground glass" pink cytoplasm 2. Alcoholism -- Mallory Hyaline aka Alcoholism Hyaline - chunky pink inclusions -- AST > ALT 3. Alpha 1 antitrypsin deficiency - pink cytoplasmic inclusions 4. Hemochromatosis - blue on iron stain 5. Alcoholism and Obesity - Fat - very common (Fatty liver)

A 55 year old alcoholic woman presents to the ED. She has been drinking heavily since the age of 16. She has a 25 year history of cirrhosis. She presents altered mental status. MRI shows the presence of 2 masses within the liver, measuring 5 and 3 cm. Her AFP is elevated. 1. What is the diagnosis? 2. Why is there altered mental status?

1. Hepatocellular carcinoma. - Cirrhosis has a high chance of turning into HCC. - Jaundice, poor skin turgor, ascites, caput medusae (bulging veins around the umbilicus), fatigue, weight loss, palpable abdominal mass are all signs of chronic liver disease and resulting cirrhosis and portal hypertension. - Aflatoxin (from aspergillus mold) is a big risk factor. - Spreads hematogenously (bones, lungs) via hepatic vein infiltration. 2. Hepatic encephalopathy -- ammonia going to the brain.

+++++++++++++++ GET THESE DOWN +++++++++++++++ ToRCHeS - Congenital Infections - Name the neonatal infection described by each (out of order on purpose): 1. Baby is born with Encephalitis and Vesicular (fluid-filled) lesions. Mother also has vesicular lesions. 2. Mother is asymptomatic, but her child is born with chorioretinitis, hydrocephalus, and intracranial calcifications. 3. Stillbirth or baby is born with hydrops fetalis, facial abnormalities, saber shins, and CN 8 sensorineural deafness (lots of head problems). Mother has a chancre and a disseminated rash. 4. Child is born with PDA (or pulmonary artery hypoplasia), deafness, cataracts, "blueberry muffin rash." The mother has a rash, arthritis, and LAD. 5. Baby is born with hearing loss, seizures, a petechial rash, and a "blueberry muffin" rash. Mother is asymptomatic.

1. Herpes Simplex 2. Toxoplasmosis. - This is why mothers should avoid cats / cat feces. 3. Syphilis 4. Rubella 5. Cytomegalovirus (CMV)

A previously healthy 50-year-old man hyperventilates in the waiting room while waiting for a colonoscopy. He develops blurred vision and begins to feel faint. Compared with a normal individual, how will each of these be affected in this patient? 1. Arterial pH 2. Arterial PCO2 3. Cerebral Blood Flow 4. Cerebral Vascular Resistance

1. High Arterial pH 2. Low Arterial PCO2 3. Low Cerebral Blood Flow 4. High Cerebral Vascular Resistance Hyperventilation, so pH increases, CO2 decreases. CO2 is local regulator of cerebral blood flow, so if CO2 decreases, then vascular resistance increases, and cerebral blood flow decreases.

++++++++++++ Often on STEP in some way +++++++++++++++ "Calcitonin Escape" / Effects of Calcitonin: What happens to the rate of bone resorption in each scenario: 1. High PTH alone (Hyperparathyroidism) 2. Calcitonin was administered to a pt with High PTH. 3. Calcitonin was administered with normal PTH KNOW THE GRAPH: Bone Resorption over Time for each ^ scenario. Calcitonin Escape: Describe sensitization to Calcitonin and how this affects the graph.

1. High PTH (Hyperparathyroidism) = Rapid bone resorption (that's what PTH does) 2 & 3. Calcitonin slows bone resorption no matter what. - Calcitonin keeps Calcium IN the bones. BOTTOM LINE: Calcitonin (CT) inhibits osteoclastic bone resorption Calcitonin Escape: - Continuous treatment with CT causes a loss of its inhibitory effects on bone resorption. - IMAGE: In the Red and Yellow curves (Calcitonin administered), Calcitonin has a strong effect in the early days, but as Calcitonin treatment continues, bone resorption slowly starts coming back. Especially if PTH is high. So Calcitonin isn't the perfect treatment.

Clinical abnormalities involving PTH and Vitamin D3 -- name the condition: 1. High PTH and Low Calcium 2. High PTH and High Calcium 3. Low PTH and Low Calcium 4. Low PTH and High Calcium Options: Vitamin D3 deficiency vs. intoxication. Hyper vs. Hypoparathyroidism.

1. High PTH and Low Calcium: Vitamin D3 deficiency (Secondary Hyperparathyroidism) Pt likely has CKD 2. High PTH and High Calcium: Primary Hyperparathyroidism 3. Low PTH and Low Calcium: Hypoparathyroidism 4. Low PTH and High Calcium: Vitamin D3 Intoxication (&/or malignancy)

+++++++++ HIGH YIELD, EASY POINTS +++++++++ Cytokines - Practice Q's: 1. Which cytokine released from TH1 cells inhibits TH2 cells and activates macrophages, increasing their phagocytic activity? 2. Which cytokine (IL-#) is released from TH2 cells inhibits TH1 cells? 3. Which cytokine (IL-#) leads to the production fo TH1 cells? 4. Which cytokine (IL-#) leads to the production of TH2 cells?

1. IFN-gamma -- Interferon interferes with Th2. - Made by Th1 cells. - Shuts off Th2 cell production. - Activates macrophages and NK cells. 2. IL-10 3. IL-12 4. IL-4

+++++++++ HIGH YIELD, EASY POINTS +++++++++ ANTIBODIES: IgA, IgE, IgG, IgM, IgD 1. Which immunoglobulin isotype is the main antibody in the delayed response to an antigen, is abundant in blood, fixes complement, and can cross the placenta? 2. Which immunoglobulin isotype prevents attachment of bacteria and viruses at mucous membrane, does not fix complement, forms a dimer when secreted, is found in tears, saliva, mucus and breast milk, and picks up a secretory component from epithelial cells before secretion? 3. Which immunoglobulin isotype has an unclear function and is found on the surface of many B cells and in the serum at low amounts? 4. Which immunoglobulin isotype is produces in the immediate response to an antigen, fixes complement, does not cross the placenta, is either a monomer or a pentamer and therefore capable of binding up to 10 Ag? 5. Which immunoglobulin isotype binds mast cells and basophils, cross-link when exposed to an allergen, mediates type 1 hypersensitivity through release of histamine, mediates immunity to parasites by activating eosinophils, and has the lowest concentration in the serum?

1. IgG 2. IgA 3. IgD 4. IgM - M for Mega -- huge -- other ABs are made from IgM, so IgM is the default precursor before Ig differentiation. Too big to cross the placenta. 5. IgE

DermatoPath - Skin Infection Warmup: 1. Sores with honey colored crusting = ___ 2. Most common pathogens that cause the above and cellulitis (2)? 3. Deep tissue infection, crepitus, pain out of proportion to appearance, requires surgery = ___. 4. Above caused by what 2 pathogens? 5. What virus causes chickenpox and later shingles?

1. Impetigo = honey colored crust 2. S. aureus and Strep pyogenes (GAS). - MNEM: Impetigo c/b Staph and Strep - "I got ISS for spreading impetigo at school." - Highly contagious skin infection that causes red sores with yellow/brown crusts, usually on the face (around the nose). - e.g., A 10-day-old girl is brought to the pediatrician by her mother after noticing multiple blisters. The mother states that some of the blisters have ruptured and left behind a ring of crust. - There's a subtype called Bullous impetigo that has bullae and is usually caused by S. aureus. 3. Necrotizing fasciitis ("Flesh-Eating Bacteria") 4. Anaerobes and Strep pyogenes (GAS) cause Necrotizing Fasciitis - Stem might say "tenderness to palpation of the affected area is out of proportion to his superficial skin findings." - Usually associated with some explainable cause, like injecting drugs at the site of infx, etc. i.e., something to explain how bacteria got there. - First Aid: Deeper tissue injury, usually from anaerobic bacteria or S. pyogenes. Pain may be out of proportion to exam findings. Results in crepitus from methane and CO2 production. "Flesh-eating bacteria." Causes bullae and a purple color to the skin. Surgical emergency. 5. Varicella Zoster virus: Causes varicella (chickenpox) and zoster (shingles).

Neonatal Respiratory Distress Syndrome: 1. What is the L:S ratio usually in neonatal respiratory distress syndrome? 2. Surfactant is made by what cell? 3. What are the risk factors (3)?

1. L:S is <1.5 2. Type II pneumocytes 3. preterm <35 weeks, maternal diabetes, C-section

A 40-year-old man presents with increasing muscle weakness. He reports difficulty rising from seated position and difficulty climbing stairs. He denies any blurry vision or any rashes on his body. He has a 100-pack-year history of smoking. Physical exam reveals proximal muscle weakness, decreased deep tendon reflexes, and dry mucous membranes. A bedside edrophonium test is conducted, which is negative. His chest radiography shows a suspicious finding in the lungs. 1. Diagnose 2. This disease is caused by antibodies to what part of the NMJ? 3. What are the hallmark symptoms of this disease? 4. This disease is strongly correlated to what type of cancer?

1. Lambert-Eaton Syndrome 2. Antibodies to Pre-Synaptic Motor Neuron Voltage-Gated Ca2+ Channels - Type II hypersensitivity reaction 3. Proximal weakness that improves with use. Autonomic symptoms (dry mouth), reduced reflexes 4. Small cell carcinoma of lung

1. What is the most common tumor arising in the uterus? 2. What is it composed of? 3. Does it ever develop malignancy? 4. What symptoms are associated with it? 5. What hormone is it sensitive to?

1. Leiomyoma (uterine fibroid) 2. Benign smooth muscle cells 3. Rarely if ever (sarcomas arise de novo) 4. Pelvic pain, bleeding (AUB), infertility 5. Estrogen Leiomyomas have a broad range of clinical presentations including: - Menorrhagia (submucosal) - Obstructive childbirth - Painful menses - Constipation (subserosal) - Urinary urgency (subserosal) - Pelvic pain - Asymptomatic

Diagnose each disease: 1. This X-linked disorder is limited almost exclusively to males and is associated with a severe or complete deficiency of HGPRTase. Inhibition of purine salvage pathways causes accumulation of uric acid. Characterized by hyperuricemia, excessive uric acid synthesis, and neurological problems, including spasticity, mental retardation, and self-mutilation, such as finger biting. A mild HGPRTase deficiency can be a cause of gout. 2. Caused by deficiencies in orotate phophoribosyl transferase and/or OMP decarboxylase (together known as UMP synthase). Patients exhibit severe anemia, growth retardation, and high level of orotic acid in the urine. This disease can be successfully treated by feeding with uridine. 3. Patients undergoing cancer treatment can show increased serum and urine uric acid due to the degradation of nucleic acids and nucleotides from destruction of tumor cells. Allopurinol is frequently included in treatment protocols to limit uric acid buildup.

1. Lesch-Nyhan Syndrome 2. Hereditary Orotic Aciduria 3. Tumor Lysis Syndrome

Dermatology BUZZwords - Name each condition: 1. Pruritic, polygonal, purple papules (associated with Hep C) 2. Cutaneous horn, off-colored plaques 3. Dark, thick, plaque like lesions, especially in the armpit or on neck, that's associated with increased insulin or a visceral malignancy 4. Erythematous nodular lesion on the anterior shin Dermatopathology

1. Lichen Planus - Pruritic, Polygonal, Purple Papules (associated with Hep C) - "Pruritic, Purple, Polygonal Planar Papules and Plaques are the 6 P's of Lichen Planus" (First Aid) 2. Actinic keratosis - Often have a cutaneous horn - Premalignant lesions caused by sun exposure. Small, rough, erythematous or brownish papules or plaques. Risk of squamous cell carcinoma. 3. Acanthosis nigricans - Epidermal hyperplasia causing symmetric hyperpigmentation and thickening of skin, especially in axilla or on neck. Associated with insulin resistance (diabetes, obesity, Cushing syndrome, PCOS) or a visceral malignancy (e.g., Gastric adenocarcinoma). 4. Erythema nodosum - Painful, raised inflammatory lesions of subcutaneous fat, usually on anterior shins.

++++++++ HIGH YIELD NT's = EASY POINTS ++++++++ Dopamine: 1. Location of production 2. 2 main Functions, and where the dopamine comes for each. 3. Associated psychological disorders

1. Location of Production: - Substantia Nigra - Ventral Tegmental Area (VTA) (Midbrain) 2. Function: - Caudate/Basal ganglia need it to produce Smooth Movements - comes from Substantia nigra. - Nucleus Accumbens needs it for the Reward / Drive / Addiction - comes from Ventral Tegmental Area. 3. Associated Psychological Disorders: - Parkinson's (too low) - Schizophrenia (too high) - Addiction: Craving and Reward feelings

++++++++ HIGH YIELD NT's = EASY POINTS ++++++++ GABA: 1. Location of production 2. Function 3. Associated psychological disorders

1. Location of Production: Everywhere 2. Function: Inhibitory effects on NTs - whereas glutamate is excitatory. - Can produce anxiolysis & hypnosis 3. Associated Psychological Disorders: - "stiff-person" syndrome - Huntington's symptoms (remember caudate with HD)

++++++++ HIGH YIELD NT's = EASY POINTS ++++++++ Norepinephrine: 1. Location of production 2. Function 3. Associated psychological disorders (2)

1. Location of Production: Locus Coeruleus (Pons) 2. Function: Fight-or-flight 3. Associated Psychological Disorders: - Panic attacks: Inappropriately active SNS / NE - PTSD: Hyperarousal and nightmares

++++++++ HIGH YIELD NT's = EASY POINTS ++++++++ Acetylcholine: 1. Location of production 2. Function 3. Associated psychological disorder 4. Excessive nicotinic effects of Ach result in what sxs? (MNEM) 5. Excessive muscarinic effects of Ach result in what sxs? (MNEM)

1. Location of Production: Nucleus Basalis (of Meynert) (Basal Forebrain) 2. Function: Movement - muscle contraction; muscarinic vs. nicotinic receptors. 3. Associated Psychological Disorders: - Alzheimer's Disease 4. Excessive Nicotinic Effects: SLUDGE symptoms: Salivation, Lacrimation, Urination, Diarrhea, GI upset, Emesis - All the fluids come out - Nicotinic receptors -- glands, GI 5. Excessive Muscarinic Effects: - "Blind as a bat, Mad as a hatter, Red as a beet, Hot as Hades, Dry as a bone..."

++++++++ HIGH YIELD NT's = EASY POINTS ++++++++ Serotonin: 1. Location of production 2. Functions - 3 - 5HT1a vs. 5HT2 vs. 5HT3 3. Associated psychological disorders

1. Location of Production: Raphe Nucleus (Brainstem) 2. Functions: - Mood/Depression (5HT1a) - Appetite, Sex (5HT2) - Nausea (5HT3) 3. Associated Psychological Disorders: Depression, Appetite, Sexual Function, Nausea, Risk for violent suicide

1. Marfan's disease is a defect in? 2. Ehlers-Danlos syndrome is a defect in? 3. What do most of these patients die from? MSK Pathology

1. Marfan's: defect in Elastic tissue (fibrillin is defective) 2. Ehlers-Danlos: Collagen defect - hyperflexible joints. 3. Most die from Aortic dissection / heart disease - Both are associated with MVP. Both Marfan's and EDS cause defects in connective tissue. The reason they are brought up here is that the patients often present with multiple musculoskeletal problems including dislocations and poor wound healing. It is the cardiovascular problems that are associated with these diseases that cause the most mortality (valve problems like MVP, dissection, etc).

Name each Germ Cell Ovarian/Testicular Tumor: 1. 90% of all germ cell tumors; 20% of all tumors; can have neural elements. Has elements from all 3 germ layers (eg, teeth, hair, sebum). Sometimes includes thyroid-like tissue and hyperthyroidism. 2. Malignant (seminoma in males), radiosensitive (excellent prognosis). Most common male tumor, fried egg cells. 3. Aggressive. Common in children. Produce AFP. Have Schiller-Duval bodies. In males, what condition increases the risk of all 3 germ cell tumors?

1. Mature cystic teratoma (dermoid), 90% of germ cell tumors, Immature (neural) elements = malignant. Sometimes presents with thyroid-like tissue and hyperthyroidism. Has elements from all 3 germ layers (eg, teeth, hair, sebum). 2. Dysgerminoma - malignant (seminoma in males), radiosensitive (excellent prognosis). Most common male tumor, fried egg cells. 3. Yolk sac tumor (endodermal sinus tumor) aggressive. May arise in the sacroccygeal area in children. Produce AFP. Have schiller duval bodies - Yolk SAC: Schiller-duval bodies, AFP, Children Germ cell tumors are associated with gonadal problems (cryptorchidism)

Name each childhood brain tumor: 1. Highly malignant tumor, that is a form of PNET (composed of small blue cells), cerebellum 2. Tumor commonly found in the 4th ventricle that can cause hydrocephalus? 3. Tumor that can calcify and is in the region of the pituitary?

1. Medulloblastoma 2. Ependymoma 3. Crainopharyngioma

A 57 year old woman is found dead in her bathroom (Elvis position) at 8:30 AM on a Saturday. The family, frantic to know why calls the ED and speaks to the resident on call. It is noted she had a "mid-systolic click" on her last physical. Additionally, at that time, she had complained of SOB while climbing the stairs. 1. What caused the mid systolic click? 2. What do people with ___ die from? 3. Why did she have a hard time with the stairs?

1. Mitral valve prolapse 2. Arrhythmia 3. She was suffering from CHF due to her incompetent mitral valve

A 42-year-old woman presents to her primary care physician for fatigue. She reports that her fatigue is worse towards the end of the day. She also notes that while bathing her nephew in the shower her head would "drop." At times when she is watching television or reading a book she sees double. Lastly, she would see her left or right eyelid droop after returning from work. On physical exam, there is right-sided ptosis after sustaining upward gaze for a few minutes. She cannot keep her right eye open unless it is held open. 1. Diagnose 2. This disease is caused by antibodies to what part of the NMJ? 3. What are the hallmark symptoms of this disease?

1. Myesthenia Gravis 2. Antibodies against post-synaptic acetylcholine receptors - antibodies binding these Ach receptors will make the muscle less responsive to Ach. - for the muscle to contract, Ach must bind to post-synaptic nicotinic Ach receptors. 3. Fluctuating symptoms, worsen with fatigue, improve with rest • Ocular (ptosis, diplopia) • Bulbar weakness (dysarthria, dysphagia) • Respiratory (shortness of breath) • Limb weakness

A 6 year old boy is brought to the pediatrician complaining of headaches and brown urine. His PMH is unremarkable except for a recent cold. UA is positive for RBCs and proteinuria. 1. What is the most likely diagnosis? 2. Why did he have a headache? 3. Why was his pee brown? 4. What is the prognosis? 5. What will histology and IF show? 6. What is the pathogenesis of this disease?

1. Nephritic syndrome - IgA glomerulopathy / Berger's disease - because hematuria is present, it's nephritic. - if it was protein only after a cold, it would be minimal change disease. 2. HTN, secondary to disease, can cause a headache. 3. Hematuria (may be microscopic, his was gross). Hematuria is present in Nephritic syndrome, but not nephrotic syndrome (which is proteinuria only). 4. Unlike MCD, this can be slowly progressive to chronic renal failure in a large number of cases! MCD can be treated with steroids; this can't. 5. Histology and IF: IgA depositis in the mesangium. Mesangial Proliferation. 6. Berger's disease is the most common nephropathy. It is caused by increased production and decreased clearance of IgA. This leads to accumulation of IgA in the glomerulus (mesangium). This can be identified by immunofloursence for IgA. The IgA immune complexes that form can activate the complement system and this leads to damage and blood is able to get into the urine. - i.e., immune system attacks mesangium

A 6 year old boy is brought to the pediatrician with marked swelling. His PMH is unremarkable except for a recent cold. Urinalysis is negative for hematuria but positive for high protein. 1. What is the most likely diagnosis? 2. What is the treatment and effectiveness? 3. Why are these pts so puffy? 4. What will LM and EM classically show? 5. Why are proteins lost?

1. Nephrotic syndrome - minimal change disease 2. Steroids are VERY effective 3. Edema due to selective loss of albumin -- swelling 4. LM will be normal, EM will show podocyte effacement. 5. Loss of negative charge in basement membrane results in proteins being able to get across. In a normal kidney, negative charge repels proteins.

++++++++++++++++++ HIGH YIELD, EASY POINTS ++++++++++++++++++ ANTIBODIES: IgA, IgE, IgG, IgM, IgD Which one(s) is/are does each of the following? 1. Neutralizes the pathogen (2) 2. Opsonization of the pathogen (marking it for macrophage destruction) (1) 3. Marks a cell for killing by Natural Killer Cells (1) 4. Tells Mast Cells to release histamine (1) 5. Activates the complement system (2) 6. Can transport across the placenta (1)

1. Neutralization: IgG and IgA 2. Opsonization: IgG - Remember, IgG is involved in Type 2 Hypersensitivity reactions which kills cells (cytotoxic) 3. Sensitization for killing by Natural Killer Cells: IgG - Again, remember that IgG is involved in Type 2 Hypersensitivity reactions which kills cells (cytotoxic) 4. Sensitization of Mast Cells to release histamine: IgE - This describes Type 1 Hypersensitivity reactions 5. Activation of the complement system: IgM and IgG 6. Can transport across the placenta: IgG only Notice that IgD does nothing. Has no known function.

++++++++++++++++++++++++++++++ Site of action in the nephron of each diuretic: 1. Acetazolamide 2. Osmotic diuretics (e.g., Mannitol) 3. Loop diuretics (e.g., Furosemide) 4. Thiazides (HCTZ) 5. Potassium-Sparing diuretics (Spironalactone)

1. PCT 2. Descending Loop of Henle 3. Ascending limb of the Loop of Henle 4. DCT 5. Collecting duct - prevents excretion of K+

DDx for Depression -- Big 5 illnesses that cause the same symptoms as depression (must rule out before diagnosing)

1. Pancreatic cancer 2. Vitamin B12 deficiency 3. Parkinson's 4. Hypothyroidism (elevated TSH) 5. Cushing's Disease (excess endogenous steroids)

A 19 year old female presents to her doctor with a newly found lump in her neck. The doctor palpates a 4 cm, well circumscribed, round mass in the midline of the patients neck. A Iodine uptake scan shows the nodule to be non-functioning. She is diagnosed with an Adenoma of the thyroid, which has a high risk of turning into a Carcinoma. 1. What is the most common type of thyroid carcinoma? 2. What is the classic cause? 3. How does it spread and where to? 4. Which thyroid cancer is associated with increased levels of calcitonin? 5. Which thyroid carcinoma has the worst outcome? 6. Which thyroid carcinoma has the best outcome? 7. Which thyroid tumor usually presents as a solitary, cold nodule? 8. How does follicular carcinoma spread? 9. Papillary, follicular and medullary carcinoma may develop into what type of carcinoma? 10. Which thyroid cancer is assocaited with MEN syndrome?

1. Papillary Thyroid Carcinoma 2. Radiation exposure 3. Lymphatics & lymph nodes 4. Medullary carcinoma -- medulla of thyroid produces calcitonin. 5. Anaplastic carcinoma 6. Papillary carcinoma 7. Follicular (papillary is multinodular) (thus, this pt probably has follicular) 8. Via the blood vessels 9. Anaplastic carcinoma 10. Medullary carcinoma - MEN2a and MEN2b

A 30-year-old woman comes for evaluation because of dark-colored urine every morning for the past week. The patient states the urine gradually returns to a normal yellow color throughout the day. She denies burning or pain during urination. She is not taking any medications nor uses supplements. She states that she has been more fatigued than usual lately and has had abdominal pain. Blood pressure is 115/70 mm Hg, pulse is 65/min, respiratory rate is 15/min, and temperature is 98°F (36.6°C). On physical examination, she appears pale and fatigued with a palpable liver edge. Her urinalysis is unremarkable. Results of a complete blood count (CBC) shows: Hgb: 8.6 g/dL Hematocrit: 32% RBC: 3000/mm3 MCV: 82 μm3 MCHC: 32% Hb/cell WBC: 9200/mm3 Platelets: 202,000/mm3 Flow cytometry shows that the RBCs are negative for CD55/59. 1. Diagnose 2. Related to what mutation? This results in loss of what cell surface anchoring protein? 3. Which two complement regulatory proteins are deficient? 4. Treatment - classic medication to know.

1. Paroxysmal Nocturnal Hemoglobinuria - Hgb in urine only at night, so hematuria in morning. - Type of Normocytic Hemolytic Anemia 2. PIGA gene (X-linked) is often found in PNH - Result is loss of Glycosylphosphatidylinositol on the surface of RBCs, which anchors a lot of membrane proteins. 3. CD55 and CD59 are deficient in PNH RBCs. Without these regulatory proteins, the complement system is able to destroy RBCs more easily. 4. Eculizumab • Just remember E-umab. • MoA: - Decreases RBC destruction - Inactivates C5 in complement pathway, > The complement cascade is a part of the immune system that enhances (complements) the ability of antibodies and phagocytes to clear microbes and damaged cells from an organism, promote inflammation, and attack the pathogen's cell membrane. > The end result is formation of a cylinder that punches a hole in the cell's wall. With fluids and molecules flowing in and out, the cell swells and bursts.

A 50 year old man with a long history of alcoholism and cirrhosis presents to the ED complaining of diffuse abdominal pain and a fever. A tap is performed and the fluid (all 1.5 liters of it) is sent to pathology for analysis. The patient is diagnosed with acute (spontaneous) peritonitis. 1. What is peritonitis? 2. What is the most likely infectious pathogen? 3. For children with Spontaneous bacterial peritonitis (SBP), what is the most common infectious pathogen?

1. Peritonitis- Inflammation of the membrane lining the abdominal wall and covering the abdominal organs. It's caused by leakage, or a hole in the intestines, such as from a burst appendix. - presents with ascites, abdominal pain and tenderness, fever, N/V 2. E. Coli - makes sense -- from GI tract 3. Strep pneumoniae

Diagnose each dementia: 1. A 59-year-old male is brought to the physician by his wife due to "odd behavior." She reports that he would attempt to kiss strangers, or would urinate in public. At times, he would grow distant and show no interest in maintaining a conversation. She has also noticed that he has been frequently binge eating sweets. He does not have insight into his symptoms. On exam, the patient displays disinhibition. There is impairment in executive functioning with intact visuospatial functioning. 2. A 67-year-old male presents with poor attention and getting lost while driving. He is accompanied by his wife. She reports that he would have episodes of staring into space and disorganized speech. His wife also reports that he would have full conversations with himself in the living room. She says that he was talking to his brother, who is deceased. Postural instability and bradykinesia is noted on physical exam. 3. Rapidly progressive dementia (weeks to months) and Myoclonus. Findings include Spogiform changes, prion changes, alpha helix to B-pleated sheet, which cannot be broken down

1. Pick's Disease (frontotemporal dementia) - Aphasia, parkinsonism, personality changes. Spares parietal lobe, posterior superior temporal gyrus. - Findings include Pick bodies (intracellular tau protein) in the frontal and temporal areas & frontotemporal atrophy. 2. Lewy Body Dementia - Hallucinations, parkinsonism. - Findings include an Alpha synuclein defect (causes the Lewy Bodies). 3. CJD (mad cow's disease)

Glomurelonephritis - Most Commons 1. Most common postinfectious glomerular disease 2. Most common glomerular disease in SLE? 3. Most common cause of nephrotic syndrome in adults 4. Most commonly assocaited with hepatitis C infection 5. Most commonly associated with young black males, HIV, and IV Drug Use

1. Post-strep GN (group A strep) 2. Diffuse proliferative GN - Due to antibodies from SLE diffusely depositing on gloms. 3. FSGN 4. Type I membranoproliferative GN 5. FSGN

++++++++++++++++ EASY POINTS / EASY TO GRASP ++++++++++++++++ Hereditary Fructose Intolerance: 1. Main symptoms and what foods/drinks exacerbate sxs? 2. Caused by a defect in what enzyme? 3. Follows what pattern of inheritance? 4. The accumulation of what substrate results in a decrease in available phosphate? 5. What 2 metabolic pathways are inhibited in patients with hereditary fructose intolerance as a result of depletion of available phosphate? 6. Will urine dipstick for glucose in a patient with hereditary fructose intolerance be positive or negative? 7. Treatment: Dietary limitation (makes sense)

1. Presents with: Hypoglycemia (see #5 below), jaundice, cirrhosis, vomiting. - Really hurts the liver, which is where fructose is metabolized. Jaundice and Cirrhosis. - Symptoms are exacerbated following consumption of fruit, juice, honey, etc (high in fructose). 2. Caused by a defect in Aldolase B. - Catalyzes second step in the breakdown of fructose (into glyceraldehyde and dihydroxyacetone phosphate). 3. Autosomal Recessive 4. Accumulation of Fructose-1-Phosphate results in a decrease in available phosphate. Phosphate is held in the form of F1P, so free phosphate is depleted. 5. Depletion of available phosphate impairs Glycolysis and Gluconeogenesis. This results in Hypoglycemia. 6. Negative -- See #5 above -- there's a lack of glucose as a result of this disease. 7. Treatment: Decrease intake of fructose and sucrose (which is glucose + fructose). - The more fructose you have, the more phosphate will be stored as F1P and glycolysis and gluconeogenesis will be impaired. Summary: Fructose-1-phosphate accumulates, leading to decrease in available phosphate, which inhibits glycogenolysis and gluconeognesis, which causes hypoglycemia and damages the liver.

1. A 40-year-old woman presents to the urgent care for excessive fatigue and jaundice. She reports that she had felt fatigued for months. Recently, she noticed that her skin is more orange. On physical exam, she is noted to have scleral icterus and jaundice. Laboratory results reveal transaminitis. Anti-smooth muscle antibodies were negative, but a liver biopsy showed granulomatous inflammation of the bile ducts. 2. A 62-year-old man presents to his primary care physician complaining of constant itchings and yellow skin and eyes. On exam, there is no evidence of hepatomegaly, however, laboratory findings are of concern for elevated serum ALP and GGT.

1. Primary Biliary Cholangitis: - Granulomatous inflammation of the bile ducts - Autoimmune damage to the bile ducts. - Lymphocytes - Granulomas are key/unique - Anti-mitochondrial Ab is HIGH YIELD - Associated with other autoimmune dz's (RA, CREST, celiac) 2. Primary Sclerosing Cholangitis: - Unknown cause. - ALP & GGT are elevated! - Onion skin fibrosis of ducts - Alternating strictures and dilation (beading) - Diagnosed radiologically -- look for narrowed bile ducts - Associated with Ulcerative Colitis - Causes biliary cirrhosis Both problems are due to bile duct damage, just by different mechanisms. Beads on a string appearance is diagnostic of PSC. Both diseases present with jaundice, pruritus, dark urine, light stool, and hepatosplenomegaly. The levels of conjugated BR, cholesterol and Alk Phos will be up in both conditions (so labs and sx don't distinguish)

A 60 year old woman presents with problems reading and a milky discharge from her nipples. 1. What tumor does she likely have? 2. How would this tumor present in a young woman? 3. What other pituitary disease could cause infertility in a young woman?

1. Prolactinoma (pituitary tumor) 2. In a young woman, these present with infertility, missed periods, milk discharge 3. Apoplexy or Sheehan syndrome

A 60 year old woman presents with well circumscribed, salmon colored plaques with a fine silvery scale. When scratched there is fine, pinpoint bleeding noted. Additionally, the patient is noted to have pitting of the fingernails. 1. Diagnose 2. What is the pathologic mechanism of the clinical lesion? 3. Classic histological feature. 4. Often associated with what other symptoms / what condition? 5. Treatment Dermatopathology

1. Psoriasis - Know that nail pits are common. 2. Unregulated keratinocyte proliferation (hyperkeratosis) - new skin keeps growing and growing. 3. Classic histology: Psoriasiform • The rete ridges push down and become tall • The papillary dermis moves up and is close to the epidermal surface • Hyperkeratosis 4. Often occurs with psoriatic arthritis - psoriasis + arthritis symptoms. 5. Treatment: Topical steroids

Vomit! 1. Projectile vomiting in a baby? 2. Vomiting with a fever? 3. Vomiting bright red blood 4. Repeatedly vomiting undigested food 5. Vomiting brown, digested (coffee grounds) blood

1. Pyloric stenosis 2. Gastroenteritis (stomach flu) 3. Varicies, gastric bleed, Mallory Weiss tear 4. Achalasia (LES doesn't open when you swallow), gastroparesis (diabetic or iatrogenic) 5. Gastric or duodenal ulcer with slow bleed

- - - - - - - Barely in First Aid - - - - - - - Diagnose each Childhood Trauma Disorder: 1. Inhibited, emotionally withdrawn behavior toward adult caregivers - Rarely seeks or responds to comfort when distressed - Minimal social responsiveness, limited positive affect, unexplained irritability, sadness, and fearfulness. 2. Disinhibited, overly familiar behavior with relative strangers - Diminished "checking back", willingness to go off w/ unfamiliar adults

1. Reactive Attachment Disorder (inhibited) 2. Disinhibited Social Engagement Disorder (disinhibited) DSM-5 says that in both of these conditions, the child must have experienced a pattern of inconsistency and/or insufficient care, such as: - Social, emotional, or physical neglect - Repeated changes in primary caregivers - Growing up in an unusual setting that limited selective attachments Comparison: - They both involve a hx of an unstable or neglected childhood, but are opposite responses to that adversity. - Reactive Attachment Disorder kids are inhibited. They are withdrawn, indifferent, and unresponsive to comfort. - Disinhibited Social Engagement Disorder kids are disinhibited. They indiscriminately attach to strangers.

+++++++++++++++ LIKELY ON STEP +++++++++++++++ A 20 year old woman presents to her family practice doc complaining of an itchy, weeping left eye. She feels feverish and complains of aches, specifically in her shoulders and knees. She states that she recently got over a bladder infection that was treated with cranberry juice, but she still has dysuria. 1. Diagnose the syndrome/condition that's causing all of her symptoms. 2. What pathogen / infection did she likely have? MNEM for the most common pathogens causing this. 3. Classic triad of symptoms: Classic MNEMONIC 4. This disease, as well as the other 3 type of seronegative spondyloarthritis, is strongly associated with HLA-__. MSK Pathology

1. Reiter's Syndrome (reactive arthritis) - Arthritis that is reactive to a recent infection -- pathogen gets in joints and causes swelling (= arthritis). 2. ShYChiCS: Shigella, Yersinia, Chlamydia, Campylobacter, Salmonella. - Since she recently had a bladder infection, it's probably chlamydia. 3. Classic triad MNEMONIC - Can't See, Can't Pee, Can't Climb a Tree > Urethritis > Conjunctivitis > Arthritis 4. HLA-B27 (MHC class I serotype) -- just like all 4 types of seronegative spondyloarthritis (PAIR: Psoriatic arthritis, Ankylosing Spondylitis, Inflammatory Bowel Disease [ulc. col. and crohn's] and Reactive arthritis / Reiter's syndrome)

Name the defense mechanism displayed in each: 1. Extreme anxiety when around someone who looks like a person you hate/fear without realizing why you're anxious. 2. Mad at your own parents/spouse/ children and get angry at someone who unconsciously reminds you of them. 3. Dealing with stress by excessively using abstract thinking and generalizations to avoid or minimize unpleasant feelings. E.g., we may focus on pathophysiology, instead of the patient's suffering and our own fears. 4. "I would like to spend more time with my daughter, but I am so busy" or "It makes sense I make this much money because I worked hard for it" or "Everyone cheats on their taxes." 5. Achieving impulse gratification and retention of goals but altering a socially unacceptable aim to a socially acceptable one. E.g., Aggressive person with authority figure issues becomes a police officer. 6. Alleviating guilty feelings by demonstrating unsolicited generosity towards others 7. "If I don't pass Step I, I can always go flip burgers!" or cracking a joke to deal with illness. 8. "I'm not great at standardized tests but I'm better with people than some other students"

1. Repression - Unconscious blocking of unacceptable thoughts, feelings, and impulses. - It's like suppression, but unconscious and less mature version of suppression - Often involves not remembering adverse experiences. 2. Displacement - Another Ex: Mad at the boss, a person goes home and kicks the dog or yells at his family. - Displacement involves an actual event or action that is observable (like kicking the dog, yelling at spouse, getting frustrated with your patient, etc.), while projection doesn't usually involve anything other than blaming, accusing, etc. - Displacement is when feelings or impulses are inappropriately transferred to a neutral person or object. 3. Intellectualization - Overemphasis on thinking ("using facts and logic") when it comes to something hard to deal with. - Obsessive-compulsive personality DO trait - Similar to "isolation of affect", where feelings are not connected to events 4. Rationalization 5. Sublimination 6. Altruism 7. Humor - MATURE MECHANISM 8. Compensation (not rationalization!) - Def: Psychologically counterbalancing perceived weaknesses by emphasizing strength in other arenas

Name the defense mechanism displayed in each: 1. Putting your late rent notices in the trash and not thinking about it as it is too stressful. 2. Deciding not to worry about whether you can pay the late rent in 2 weeks before you get evicted because you have to study for a test next week. 3. "I like to drink but it isn't a problem" in a person with cirrhosis, GI bleed. 4. Sleeping with a transitional object (teddy bear, "blankie") 5. Punching walls when you're mad at your partner, or an adolescent using drugs and having sex when parents are getting divorced 6. Temporary but drastic modification of a person's character or sense of identity to avoid distress; parts of oneself are separated from awareness of other parts and behaving as if one had separate sets of values - E.g., an honest man cheats on taxes; womanizers who hit it and quit it. 7. "You're angry with me, I can tell" (when really they the angry one) 8. You want to play with your feces, but instead you wash compulsively; or Acting like you hate someone you secretly love (to avoid being hurt) 9. When the idealized aspects of the self are preserved, and its limitations ignored or denied. E.g., People who talk about their college degree, jobs, possessions, etc., may be trying to cover up feelings of inadequacy. 10. Internalizing the qualities of another person. E.g., identifying with the aggressor (like kids in concentration camps boot stepping like Nazi guards), or an abused person believing they deserve abuse and abusers as mates. Can also be healthy, such as taking on best aspects of your parents

1. Repression or Denial 2. Suppression 3. Denial 4. Regression 5. Acting Out 6. Compartamentalization (if it sounds a little sociopathic) 7. Projection (if it sounds a little like paranoid PD) 8. Reaction Formation 9. Narcissism - People can be narcissistic as a defense, such as in narcissistic personality disorder, or actually really believe they're that great 10. Introjection

DIARRHEA! Buzz Words 1. What virus is the most common cause of isolated (i.e., non-recurrent) periods of diarrhea in children? 2. What virus causes conjunctivitis and bloody diarrhea? 3. Most likely pathogen if diarrhea is associated with ABx use? 4. Most common cause of diarrhea in AIDS patients? 5. Most common pathogen and best antibiotic if a person returns from a hiking or camping trip with large amounts of watery, nonbloody diarrhea, abdominal cramping, and lots of flatulence 6. Most likely immunodeficiency and pathogen in a kid with recurrent diarrhea. 7. Most common pathogen that causes bloody or watery diarrhea + RUQ abd. pain

1. Rotavirus - diarrhea in children 2. Adenovirus - conjunctivitis and bloody diarrhea 3. Clostridium difficile (C. diff) - diarrhea from ABx - e.g., An 89-year-old man with a past medical history of gastritis presents to the hospital for dehydration and watery diarrhea. He was was recently treated with clindamycin for a soft tissue infection. 4. Cryptosporidium parvum - diarrhea in AIDS pt. 5. Giardia lamblia - fatty and watery diarrhea with flatulence after a hiking trip. - Tx with Metronidazole (in 3 MedBullets Q's) 6. IgA Deficiency, causes recurrent Giardia lamblia infections. 7. Entamoeba histolytica: bloody/watery diarrhea + RUQ pain; very often involves a liver abscess. Expect neutrophilia and a "left shift" to immature WBCs in anyone with acute diarrhea (e.g., gastroenteritis)

A 55 year old man presents to his family doctor because he has recently turned yellow "like a highlighter." He has expierenced a 25 pound weight loss and states that he occasionally has pain shooting to his back. You can palpate his gallbladder on abdominal exam. 1. What should you absolutely rule out? 2. What is Trousseau's syndrome?

1. Rule out Pancreatic adenocarcinoma - pain that radiates to the back and signs of liver or alcohol involvement point to Acute Pancreatitis. 2. Migratory thrombophlebitis (pain in extremities on palpation). - medical sign involving episodes of vessel inflammation due to blood clot (thrombophlebitis) which are recurrent or appearing in different locations over time (thrombophlebitis migrans or migratory thrombophlebitis).

A 42 year old male is involved in a car accident and sustains severe injuries to his chest, pelvis and forehead. During his stay in the hospital he develops mental status changes. The nurse notes that he has produced very little, highly concentrated urine in the last 2 days. 1. What is the likely diagnosis? 2. What is the most common cause? 3. Why did the patient have mental status changes? 4. Will he have hypernatremia or hyponatremia? 5. Will his urine osmolality be high or low? 6. If this exact same patient instead presents with intense thirst and polyuria, what would be the most likely diagnosis?

1. SIADH (secondary to head trauma) 2. Ectopic production due to small cell lung cancer 3. Cerebral Edema 4. Hyponatremia -- Too much ADH. Water reabsorption dilutes blood. ADH causes increased excretion of Na+, which lowers Na+ in blood. - i.e., decreased serum osmolality. 5. High Urine Osmolality (unlike diabetes insipidus) - urine osmolality > serum osmolality - due to inc. excretion of sodium 6. Diabetes insipidus - Central vs. Nephrogenic with water deprivation test - Desmopressin for Central only

Diagnose each Autoimmune Skin Disease: 1. Eythematous maculo-papular eruption, butterfly rash, worse with UV exposure. Immune-complex deposits along basement membrane. Deep dermal infiltrate of lymphocytes. 2. Vesicles \ bullae on skin and oral mucosa. IgG against desmosomes between keratinocytes (netlike pattern). Histology shows Basal cell tombstones, acantholytic keratinocytes in blister. (+) Nikolsky sign (sloughs with applied pressure) 3. Tense bullae on the skin (esp. flexor surfaces) and oral mucosa. IgG against basement membrane (linear deposition). Subepidermal blister, no acantholytic cells. Type II HS rxn. (-) Nikolsky sign (does not slough with applied pressure). 4. Vesicles with erythema on extensor surfaces (e.g., elbows). Deposits of IgA at tips of dermal papillae. Histology shows subepidermal blisters with neutrophils. Strongly associated with celiac disease (look for GI sxs too) and responds will to a gluten-free diet. Treatment: dapsone, gluten-free diet. Dermatopathology

1. SLE Eythematous maculo-papular eruption, butterfly rash, worse with UV exposure. Immune-complex deposits along basement membrane. Deep dermal infiltrate of lymphocytes. 2. Pemphigus vulgaris Vesicles \ bullae on skin and oral mucosa. IgG against desmosomes between keratinocytes (netlike pattern). Histology shows basal cell tombstones, acantholytic keratinocytes in blister. (+) Nikolsky sign (sloughs with applied pressure) 3. Bullous pemphigoid Tense bullae on the skin (esp. flexor surfaces) and oral mucosa. IgG against basement membrane (linear deposition). Subepidermal blister, no acantholytic cells. Type II HS rxn. (-) Nikolsky sign (does not slough with applied pressure). 4. Dermatitis herpetiformis Vesicles with erythema on extensor surfaces (e.g., elbows). Deposits of IgA at tips of dermal papillae. Histology shows subepidermal blisters with neutrophils. Associated with celiac disease (look for GI sxs too) and responds will to a gluten-free diet. Treatment: dapsone, gluten-free diet. - This is the classic skin symptom of people with Celiac Disease.

++++++++++++++++++++++++++++++++++++++++ Squamous Cell Carcinoma of the Skin: 1. What is the #1 cause / risk factor? 2. Can it invade? Can it metastasize? 3. What do the lesions look like? (How the stem will describe it?) 4. Histology classically shows ___ ___. 5. How can you distinguish it from an Actinic Keratosis? (!!!!!) 6. How can you distinguish it from Basal Cell Carcinoma? (!!!!!) Dermatopathology

1. SUN EXPOSURE 2. Can invade locally, but does NOT metastasize. 3. Red, ulcerated lesion (sometimes with "heaped up borders"). 4. Keratin Pearls on histology -- white circular shapes on a purple background OR normal keratin pearls are pink and smooth. - NOTE: The abnormalities tend to be up high, closer to the surface, where the keratinocytes are. BCC histology shows pallisading deeper down, below / not including the top layer. DDx: You have to look at the description of the lesion. 5. Actinic keratosis can seem like an SCC, but it's described in stems as a scaly plaque or patch, rather than raised. It's a precursor to SCC, but does NOT have local invasion (i.e., will not have any dysplastic findings on biopsy). 6. Basal Cell Carcinomas are described in stems as pink, pearly lesions that grow slowly. They often have telangiectasia and can have some central ulceration that looks like SCC. Sun exposure is still a risk. Also, histology for BCC will show pallisading that is below the top layer, whereas SCC is closer to the top layer and should have keratin pearls. Both are "locally invasive but rarely metastasize." Also, SCC's tend to be very ugly / nasty looking, compared to BCC's. So the worse it looks, the more you can lean towards SCC > BCC. Etiology UV radiation produces Thymidine Dimers in DNA => Leads to mismatched base pairing. MedBullets Stem for SCC of Skin: A 70-year-old man presents to the dermatologist's office for his annual skin exam. He complains of a large pink lesion on his cheek that oozes and bleeds. It never seemed to heal properly. He is concerned, because he knows that he should have used more sunscreen throughout his life. MedBullets Stem for BCC of Skin: A 52-year-old woman with fair skin is concerned about a pink pearly lesion that is growing slowly on her cheek. She is a varsity water polo coach at her local high school and spends many hours under the sun. She admits to not using sunscreen consistently. MedBullets Stem for an Actinic Keratosis: A 60-year-old man with multiple "sun spots," or solar lentigos, comes to the dermatologist for his annual skin exam. He complains of several rough patches on his cheeks that has been there for a year. They feel like sandpaper. He is a gardener and often spent hours under the sun.

Family Therapy will only be a correct treatment option for what two psychological disorders?

1. Schizophrenia - Like counseling the family on taking care of and understanding them. 2. Anorexia - Like home instructions, encouragement, etc.

Name the 5 surface epithelial ovarian tumors: 1. Most common! Tumor spreads by exfoliation, causing Ascites. BRCA and p53 associations. 2. Mimics endometrial gland tissue 3. Can cause pseudomyxoma peritoneii, always suspect appendix 4. High grade, hypercalcemia, endometriosis 5. Transitional cells and stroma, mostly benign, looks like bladder

1. Serous Carcinoma (tubal differentiation, highly malignant), p53, tumor spreads by exfoliation, BRCA association 2. Endometrioid tumors - mimic endometrial glands 3. Mucinous tumors can cause pseudomyxoma peritoneii, always suspect appendix 4. Clear cell tumors - high grade, clear cells, hypercalcemia, endometriosis 5. Brenner tumor - transitional cells and stroma, mostly benign, looks like bladder

A 43 year old woman presents to her family practice doctor complaining of eye discomfort. She states that they feel dry and she has had to use a lot of eye-drops. Further questioning reveals that she also has a hard time eating because her mouth gets dry. Her review of systems reveals that she has sore knees. 1. What is the most likely diagnosis? 2. What malignancies is she at risk for? 3. What auto-antibodies can you test for? (MNEM - easy) MSK Pathology

1. Sjogren's Syndrome 2. B-cell lymphoma and Mucosa-Associated Lymphoid Tissue (MALT) lymphoma 3. SS-A (Ro) & SS-B (La): MNEM: SS for Sjogren's Syndrome. Autoimmune disease of the exocrine glands => Decrease in saliva and tear production. First Aid: Sjogren's Syndrome - Autoimmune disorder characterized by destruction of exocrine glands (especially lacrimal and salivary) by lymphocytic infiltrates. - Predominantly affects women 40-60 years old. - Clinical Findings: ✔ Inflammatory joint pain ✔ Keratoconjunctivitis sicca (tear production and subsequent corneal damage) ✔ Xerostomia (dec. saliva production), mucosal atrophy, fissuring of the tongue ✔ Bilateral parotid enlargement Antibodies: ✔ Presence of antinuclear antibodies, rheumatoid factor (can be positive in the absence of rheumatoid arthritis), antiribonucleoprotein antibodies: SS-A (anti-Ro) and/or SS-B (anti-La) ✔ Anti-SSA and anti-SSB may also be seen in SLE

Diabetes Flash Quiz: 1. What is the cause of the small vessel disease seen in diabetes? 2. What is the cause of cataracts associated with diabetes? 3. What happens to Beta Islets in DM1 vs. DM2? 4. Which type of DM is associated with HLA DR-3 and DR-4?

1. Small vessel thickening of basement membrane by nonenzymatic glycosylation. Most common cause of morbidity! 2. Sorbitol accumulation 3. In DM1, they are attacked (lymphocytic infiltration and loss). In DM2, they get burnt out; amyloid deposition, eventual loss. 4. Type 1 is a/w HLA DR-3 and DR-4 - DR3 + Type 1 = DR4

Virchow's triad of DVT's -- 3 causes

1. Stasis (Long flights, sitting, lying, surgery) 2. Hypercoagulable state (OCP) 3. Endothelial damage (smoking, cholesterol, surgery)

Rheumatic Fever: 1. What pathogen usually causes the infx that precedes it? 2. This is caused by formation of antibodies to ___. 3. Most common clinical finding (symptom) in RF? 4. How can it affect the heart? 5. What are the JONES Criteria for diagnosis? Particularly some unique ones.

1. Strep pyogenes pharyngitis (recent strep throat) 2. M-protein (antibodies that cause all the problems with RF are made to this ag) 3. Migratory polyarthritis 4. INFECTIVE ENDOCARDITIS 5. Jone's Criteria: IMAGE; unique ones: - Endocarditis - Migratory polyarthritis - Chorea: Dance-like movements of extremities - Erythema marginatum - Subcutaneous nodules

Temporal Lobe lesion found on MRI -- Most likely Dx if found: 1. In a patient with coronary artery disease and atherosclerosis 2. In a young patient with seziures

1. Stroke (brain ischemia) in the Middle Cerebral Artery region. 2. Herpes Encephalitis - Can cause temporal lobe lesions and Seizures - Just remember it as temporal lobe seizures in herpes encephalitis.

Recognize these BUZZ stems: 1. A child comes to the clinic with multiple fractures from various times and spiral fractures 2. Family member with demented pt. with many bruises, or malnourished / in a bad state 3. It seems like one of your patients is suffering from spousal abuse. What is the proper course of action for each?

1. Suspected child abuse - especially spiral fractures. - A spiral fracture is a bone fracture that occurs when a long bone is broken by a twisting force. - REPORT THIS 2. Suspected elder abuse - REPORT THIS - if you think an elder is being neglected or intentionally mistreated. BOTH OF THESE ARE REPORTABLE - YOU ARE LEGALLY OBLIGATED TO REPORT THESE 2 TYPES OF ABUSE. 3. But not required to report suspected spousal abuse, oddly enough. In that situation, you want to prioritize the abused spouse's safety however you can.

Name the cytokine(s) produced by each of the following transcription factors: 1. T-bet (1) 2. GATA-3 (3) 3. FoxP3 (2) -- and what happens if there's FoxP3 deficiency? 4. RORγt (1)

1. T-bet: IFNγ - Released by CD8+ cytotoxic T-cells and Th1 CD4+ Helper T-cells. 2. GATA-3: IL-4, IL-5, IL-13 - GATA-3 makes 3 interleukens. - These are the 3 cytokines released by Th2 Helper T-cells. 3. FoxP3: TGF-ß & IL-10 (The T-regulatory cells, which tone down the immune response) - Genetic deficiency of FoxP3 results in autoimmunity. 4. RORγt: IL-17 - IL-17 is released by Th17 Helper T-cells and is involved in inflammation and neutrophilia.

+++++++ High Yield Psychiatry Medications +++++++ Anti-Depressants - Name the prototypes and main side effects (1-2) to know for each class: 1. TCAs (MNEM for main side effect) 2. MAO-Is - and DO NOT ingest ___ if taking a MAO-I. 3. SSRIs

1. TCAs: Amitriptyline, Imipramine, Nortriptyline, etc. - MoA: block reuptake of NE and Serotonin (i.e., ↑ NE & Serotonin) - Main SE: Anticholinergic effects (atropine-like effects from low Ach — tachycardia, urinary retention, dry mouth, constipation, blurry vision, confusion & hallucinations in the elderly). - MNEM: TCAs Take Choline Away - (Extra): 3 C's of TCA Overdose = Convulsions, Coma, and Cardiotoxicity (arrhythmias) ^ Rx for TCA OD: NaHCO3 — Na+ will fix arrhythmia (best answer for mechanism) 2. MAO-I's: Phenelzine, Tranylcypromine, Isocarboxazid, Selegiline - MoA: ↓ MAO → ↓ amine degradation → ↑ levels of amine neurotransmitters (mainly NE and serotonin) - DO NOT combine with Wine or Cheese (tyramine) - Increases BP => Hypertensive crisis (due to tyramine ingestion) - Serotonin syndrome, esp. when combined with SSRIs or tramadol. 3. SSRIs: Sertraline, Fluoxetine, Paroxetine, Citalopram - Sexual dysfunction (anorgasmia, decreased libido) - Serotonin syndrome, esp. when combined with MAO-I's or tramadol.

+++++++++ HIGH YIELD, EASY POINTS +++++++++ 1. What is HLA? 2. HLA-A, HLA-B, HLA-C code for what type of major histocompatibility complex? 3. HLA-DR, HLA-DQ, HLA-DP code for what type of major histocompatibility complex?

1. The HLAs (human leukocyte antigen) are genes that code for the major histocompatibility complex (MHC I & II) proteins, which regulate the immune system. 2. MHC I - MHC I loci have 1 letter - A, B, C 3. MHC II - MHC II loci have 2 letters - DR, DQ, DP

1. What is the complement system? 2. Which components of the complement system (C_ _) is crucial for opsonization (marking a pathogen for destruction), as it binds bacteria? 3. What 2 components of the complement system (both = C_ _) are crucial for inflammation and anaphylactic reactions? E.g., USMLE-Rx: A 34-year-old woman is brought to the emergency department after collapsing during a picnic at a local park. Her blood pressure is 88/75 mm Hg, pulse is 124/min, and temperature is 37.3°C (99.1°F). Oxygen saturation is 85% on room air. She is using accessory muscles to breathe and has high-pitched breath sounds on inspiration. Physical examination reveals hives on her skin and swelling around her eyes. Which of the following is involved in causing this patient's symptoms? check A. C3a B. C3b C. IL-3 D. IL-8 E. IgG

1. The complement system is a part of the innate immune system that consists of a number of small proteins synthesized by the liver that enhances (complements) the ability of antibodies and phagocytic cells to attack the pathogen's cell membrane, clear all the microbes and damaged cells, and promote inflammation. It is part of the innate immune system, which generally attacks pathogens. Complement components circulate in the blood as inactive precursors. When stimulated by a trigger, cytokines are released to initiate the amplifying complement cascade. Activation of the complement system achieves 3 main things: - stimulates phagocytes to clear foreign and damaged material - triggers inflammation to attract additional phagocytes - attacks the cell membranes of pathogens (cell-killing membrane attack complex). 2. C3b - opsonization and clearance of immune complexes - C3b binds to lipopolysaccharides on bacteria. 3. C3a and C5a - anaphylaxis/allergies and inflammation. Sample Question: This is an anaphylactic reaction, so it's C3a.

1. What is the most common type of tumor of the urinary tract system? 2. Painless hematuria suggests cancer where? 3. What are some risk factors of the above (5)?

1. Transitional cell carcinoma (Urothelial carcinoma) 2. Bladder cancer - painless hematuria 3. Bladder cancer risk factors: Phenacetin (banned now), smoking, aniline dye, cyclophosphamide, schistosoma infection.

Female GU - High Yield BUZZword Associations: Name the pathogen: 1. Strawberry cervix, green and foul-smelling discharge 2. Solitary painless ulcer/lump 3. Squamous dysplasia of the cervix -- #1 cause 4. Painful vesicles that ulcerate 5. Painful ulcer with suppurative inguinal lymph nodes 6. Most common vaginitis, malodorous, "fishy" smell 7. Most common pathogens associated with PID (2)

1. Trichomonas 2. Syphilis (T. pallidum) 3. HPV - 16 or 18 4. Herpes (usually type 2) 5. H. ducreyi 6. Gardnerella vaginalis 7. Chlamydia & Gonorrhea

NAME THAT INFECTION / PATHOGEN! 1. Causes a cyclical fever spike because of antigen variation. Spread by the bite of the tsetse fly. Causes sleeping sickness. 2. Gram positive rod!!! Has actin "rockets" that cause tumbling motility. Why expectant mothers cannot have unpasteurized dairy. 3. Broad, ribbon like fungal hyphae. Infects the frontal sinuses. Preys on people with DKA. 4. Ring-enhancing brain lesion in an AIDS patient. 5. One of the only helminthic infections in the brain. Causes cysts and seizures. Comes from undercooked pork.

1. Trypanosoma gambiense / rhodesiense 2. Listeria monocytogenes 3. Mucor 4. Toxoplasmosis - RING-SHAPED LESION 5. Taenia solium (cysticercosis)

+++++++++++++++++++++++++++++++++ A 21 y/o woman was transported to the ER after the sudden onset of shortness of breath, wheezing, and stridor associated with a generalized pruritic rash and facial swelling. A friend reports that the woman was stung by a wasp 10 minutes prior to the onset of symptoms. 1. What type of hypersensitivity reaction is this? 2. What antibody mediates this hypersensitivity reaction? 3. That antibody causes a massive release of ___ from ___ cells, causing the symptoms.

1. Type 1 Hypersensitivity - Allergic rxn / Anaphylaxis 2. IgE-mediated 3. IgE causes massive release of Histamine from Mast Cells. That's why anti-histamines work for allergies. Features/Clues: • Sudden onset (minutes) • Generalized edematous, itchy rash • Respiratory distress Wheal and flare reaction • Swelling caused by increased permeability of blood vessels (wheal) • Redness due to increased blood flow (flare)

++++++++++++++++++++++++++++++++ A woman presents to an antenatal clinic at 28 weeks of pregnancy. In the past, she has had one healthy child and a subsequent miscarriage in mid pregnancy. Over the next few weeks the pregnancy runs smoothly, but the woman is found to be Rh negative. Further tests show that she has antibodies to the rhesus D antigen. At 34 weeks of pregnancy, an ultrasound scan shows signs of fetal distress. Labor is induced and a baby girl is born. The baby's red blood cells are Rh+. She is profoundly anemic. A direct Coomb's test on the cord blood is positive, confirming a diagnosis of hemolytic disease of the newborn. 1. What type of hypersensitivity is hemolytic disease of the newborn? 2. How could this have been prevented?

1. Type 2 Hypersensitivity 2. RhoGAM - Gives the mother IgG antibodies (only type that crosses placenta, so makes sense) against the RhD antigen - This will deplete any fetal RhD+ RBCs that enter the maternal circulation before they have a chance to sensitize the mother and form anti-Rh. If a mother is Rh- and the baby is Rh+, she develops antibodies to Rh, so if the baby is Rh+, the mother's immune system (anti-Rh) will attack and kill fetal RBCs that have Rh. This will kill the fetal RBCs and cause hemolytic disease of the newborn.

Acute Coronary Syndrome (angina and MIs): Diagnose each from Classic descriptions of occlusions: 1. Partially occlusive thrombus with no cardiomyocyte death/necrosis 2. Partially occlusive thrombus with cardiomyocyte death/necrosis 3. Completely occlusive thrombus with cardiomyocyte death/necrosis

1. Unstable angina 2. Non-STEMI (no ST elevation; in fact, ST depression) 3. STEMI - MI with ST elevations Both 2 and 3 are heart attacks

Common Vitamin Deficiencies: 1. Vitamin deficiency that causes bleeding? What clotting factors does it work on? 2. Common vitamin deficiency associated with heart disease in alcoholics? 3. What vitamin is given to pregnant women? Most common cause of deficiency? Deficiency can cause what? 4. In malnutrition, what is the most common mechanism of poor wound healing?

1. Vitamin K - Vit K comes from gut bacteria & leafy greens - 2,7,9,10, C, S (all of these need Vit K for clotting) - Antibiotics can cause deficiency (wipe out gut bacteria) 2. Thiamine (B1) - Wernicke's and Korsakoff's -- ataxia, confusion, confabulation, psychosis, etc. - Beriberi -- peripheral neuropathy, heart failure 3. Folate - Alcoholism causes deficiency - Deficiency causes megaloblastic anemia 4. Vitamin C deficiency leads to poor wound healing due to lack of collagen cross-linking.

What small vessel disease is it if labs reveal: 1. c-ANCA 2. p-ANCA

1. c-ANCA = Wegener Granulomatosis (Granulomatosis with Polyangiitis) 2. p-ANCA = Churg Strauss Syndrome (Eosinophilic Granulomatosis with Polyangiitis)

Repro Question 7 A 23-yo female (G2P1) presents in her first trimester of an unwanted pregnancy. What is the most suitable drug to use to terminate the pregnancy? A. Estradiol B. Mifeprostone C. Oxytocin D. Progesterone E. Tamoxifen Repro Question 8 A 48-yo woman who has already undergone radiation therapy for breast cancer and has a family history of breast cancer is prescribed tamoxifen. What is the possible side-effect of this drug? A. Gynecomastia B. Infertility C. Mania D. McCune-Albright syndrome E. Endometrial cancer

7. B. Mifeprostone Mifepristone is an antiprogestin, acting to block progesterone receptor action in the uterus. It prevents implantation of the embryo/adherence of the fetus into the uterine wall and increases uterine contractions to expel the fetus. Basically induces a period. 8. E. Endometrial cancer Tamoxifen is a selective estrogen receptor modulator (SERM) that treats breast cancer, but increases the risk of uterine cancer. Tamoxifen acts as an anti-estrogen in breast tissue, but it acts like an estrogen in the uterus. In women who have gone through menopause, it can cause the uterine lining to grow (and doesn't shed because post-menopause), which increases the risk of endometrial cancer.

A 4-month-old boy is brought to the physician by his parents because of twitching of his facial muscles. A review of his records shows that he has previously been seen for several severe episodes of Candida infections. Physical examination shows low-set ears, increased distance between the eyes, and a shortened upper lip. Which of the following additional findings is most likely to be seen in this patient? A. Absence of Type IV hypersensitivity B. Decreased alpha-fetoprotein C. Elevated IgE levels D. Elevated IgM levels E. Prominent ocular telangiectasias

A. Absence of Type IV hypersensitivity This patient has DiGeorge Syndrome. - thymus and parathyroids fail to develop - e.g., twitching of facial muscles -- hypocalcemia (no PTH) What is the immunological impact of DiGeorge Syndrome? - Lack of thymus, therefore T-cell immunodeficiency. What's the only option that would result from a lack of T-cells? -- Type 4 Hypersensitivity (mediated by CD4+ memory T-cells).

A 46-year-old woman with a history of intravenous drug use presents to a community clinic complaining of "yellow eyes," which she has noticed for the past several days. She also complains of right upper quadrant abdominal pain and nausea. Laboratory tests show: ALT: 630 U/L AST: 400 U/L Total bilirubin: 5 mg/dL A viral hepatitis panel is negative for antibodies against hepatitis C virus. Which of the following is the most likely diagnosis? A. Acute viral infection B. Chronic viral infection C. Fulminant hepatitis D. Hepatocellular carcinoma E. Liver cirrhosis

A. Acute viral infection This patient presents with "yellow eyes" (jaundice), right upper quadrant pain, nausea, and elevated alanine aminotransferase (ALT) and aspartate aminotransferase (AST) (ALT > AST) and total bilirubin. These physical exam findings and lab abnormalities are most consistent with a diagnosis of acute viral hepatitis. In viral hepatitis, the ALT level is higher than the AST level; this is in contrast to elevations seen with alcoholic hepatitis in which the AST level is higher than the ALT level. Given this patient's history of intravenous drug use, a likely causative agent is the hepatitis C virus (HCV). Although only 20%-30% of acutely infected patients have symptoms, acute HCV is associated with jaundice and aminotransferase levels > 500 U/L. The HCV genome consists of single-stranded RNA. HCV RNA levels may be very high early in the course of acute infection. The presence of symptoms indicates an immune response to the virus. Because of the delay in the adaptive immune response, antibodies will not be present early in the course of the infection. Although many people who inject drugs are chronically infected, not all are. Consequently, there is a continued risk of acute infection. Women are more likely than men to experience spontaneous resolution of hepatitis C. Could also be Hep A or B. NOT: Chronic hepatitis C infection - serum antiviral antibodies would be present, and ALT levels would be < 500 U/L, a value that is not seen in this patient. Fulminant hepatitis is very rare in patients with acute hepatitis infection. Hepatocellular carcinoma occurs in up to 3% of patients with HCV-induced cirrhosis and can manifest with right upper quadrant pain. However, this diagnosis would not explain the patient's ALT elevation >500 U/L. Liver cirrhosis is seen in patients with chronic hepatitis infection; however, there is no evidence of chronicity in this patient.

An 11-month-old infant has had upper and lower respiratory tract infections almost continuously since the time of birth, with organisms including Pneumocystis jiroveci and Pseudomonas aeruginosa identified. The baby also has oropharyngeal candidiasis. The baby succumbs to a cytomegalovirus pneumonitis. At autopsy, the thymus is markedly hypoplastic, and lymph nodes throughout the body are small, with absent germinal centers on microscopic examination. Which of the following mechanisms is most likely to explain these findings? A. Adenosine deaminase deficiency B. Failure of B cell maturation to plasma cells C. HIV infection D. Autoantibodies to both T and B cell lymphocytes E. Failure of development of the 3rd and 4th pharyngeal pouches

A. Adenosine deaminase deficiency Severe combined immunodeficiency (SCID) may be due to adenosine deaminase (ADA) deficiency in about half of cases. This disorder leads to malfunction of both humoral (B-cell) and cell-mediated (T-cell) immunity. Viral, fungal, and bacterial infections occur frequently in the first year of life.

++++++++++++++++++++++++ A Hispanic 57-year-old male with adult T-lymphocyte leukemia received a bone marrow transplant from an unrelated donor 1 month ago. The patient is on an immunosuppressive posttransplant treatment regimen that includes cyclosporine, mycophenolate, and prednisone. Over the course of several weeks, the patient develops a severe cutaneous rash over his back and intractable watery diarrhea. What immunologic cells/factors are most directly responsible for the patient's current presentation? A. Alloreactive donor T lymphocytes B. Immunoglobulins and fibrinoid necrosis C. Recipient antigen-presenting cells presenting donor peptides D. Recipient's cytotoxic T lymphocytes

A. Alloreactive donor T lymphocytes This is GVHD, which is when the donor's T-cells attack the host. The organs most often affected are the gut, skin, and liver, as evidenced by the rash, diarrhea, elevated alkaline phosphatase, elevated transaminases, and elevated LDH in this case. Human leukocyte antigen (HLA) matching of the donor and recipient can help reduce the severity of GVHD, but the disease may still occur due to a minor histocompatibility mismatch. NOT: - Acute graft rejection occurs when the recipient's cytotoxic T-lymphocytes destroy the graft within weeks to months of the transplant. - Hyperacute graft rejection results in vessel occlusion, ischemia, or fibrinoid necrosis, occurs within minutes to hours after a transplant, and does not typically feature rashes or watery diarrhea. - Chronic organ rejection features donor peptides being displayed by recipient antigen-presenting cells, resulting in interstitial fibrosis, atherosclerosis, and vanishing bile duct syndrome, not symptoms seen in this patient.

A 12 year old male presents to the ED with a 5 day history of increasing fatigue. He states that several kids in his class have been out sick with fever and rash. He has a history of anemia with scleral icterus requiring PRBC transfusion when he was 4 years old but missed his follow up in hematology clinic and mom does not remember what the physicians told her during that hospitalization. His father had a history of splenectomy when he was 15 years old for unknown reason and his paternal aunt underwent cholecystectomy at age 15 years. His laboratory values and peripheral smear today are as follows: - Hemoglobin 6.5 g/dL - Hematocrit 19% - MCV 83 fL - WBC Count 8.0 x 10^3/mm3 - Platelet Count 185,000/mm3 - Reticulocyte count 0.4% What is the most likely etiology of his anemia today? A. Aplastic crisis secondary to parvovirus B. Hyperhemolytic crisis C. Thalassemia major D. Iron deficiency E. Autoimmune hemolytic anemia secondary to IgM Ab

A. Aplastic crisis secondary to parvovirus Parvo B19 can cause aplastic crisis in patients with red blood cell disorders. From lab values, this is normocytic with a low retic (hence "aplastic")

A 35-year-old man who works at a facility processing highly radioactive substances accidentally receives a high, whole-body dose of ionizing radiation estimated to be 1500 rads (15 gray). He dies 1 week later. At autopsy, histologic examination of the skin shows scattered, individual epidermal cells with shrunken, markedly eosinophilic cytoplasm and pyknotic, fragmented nuclei. These morphologic changes most likely indicate which of the following processes? (A) Apoptosis (B) Coagulation necrosis (C) Liquefaction necrosis (D) Mutagenesis (E) Tumor initiation

A. Apoptosis What really sticks out here is the patient gets radiated, dies, and the autopsy shows CELLS. Apoptosis is a process that occurs in cells, generally classifying necrosis is done on tissue!

A 62-year-old woman comes to the physician because of fatigue and mild exertional dyspnea (after walking 3 to 4 blocks) over the past several weeks. She has lost 3.2 kg (7 lb) in the past two months, despite no change in appetite. The remainder of the history is unremarkable. Her temperature is 37.2°C (98.9°F), pulse is 96/min, respirations are 18/min, and blood pressure is 110/70 mm Hg supine, and there is no significant orthostasis. Physical examination is unremarkable except for generalized pallor. Laboratory studies show hemoglobin is 6.8g/dL, hematocrit is 23%, leukocyte count 6,000 cells/mm , platelet count 160,000 cells/mm ; serum creatinine 0.7 mg/dL. Carcinoembryonic antigen (CEA) level is twice the limit of normal. Her stool is guaiac-positive. Colonoscopy shows a large fungating mass in her ascending colon. Which of the following is most likely to be decreased in this patient? A. Arterial O2 content B. Arterial O2 saturation C. Arterial PO2 D. Cardiac output E. Heart rate F. Stroke volume

A. Arterial O2 content Hemoglobin 6.8 = Anemia. Arterial O2 content must be low. Arterial O2 content depends on both hemoglobin concentration and saturation with oxygen. Not O2 Saturation -- the small amount of Hgb will be very saturated with O2.

++++++++++++++++++++++++ A 57-year-old man comes for evaluation because of a 4-month history of a tactile fever, worsening fatigue, and generalized weakness. He states that his clothes fit him more loosely now than they have in the past. Physical examination reveals enlarged cervical, supraclavicular, axillary, and inguinal lymph nodes, along with a palpable spleen and 20-cm lateral lobe of the liver. Lymph node biopsy specimens are sent to the pathologist. Which of the following types of neoplastic cells are most common in this patient's condition? A. B lymphocyte B. Myeloblast C. Plasma cell D. Reed-Sternberg cell E. T lymphocyte

A. B lymphocyte The patient presents with weight loss, signs of anemia, generalized lymphadenopathy, and hepatosplenomegaly, which strongly suggests a non-Hodgkin lymphoma. Systemic symptoms of Hodgkin lymphoma may include "B" symptoms, such as fever, weight loss, and drenching night sweats. These symptoms—which can be seen in both Hodgkin and non-Hodgkin lymphoma—are observed in approximately 40% of patients with non-Hodgkin lymphoma and are a poor prognostic indicator. Non-Hodgkin lymphoma often involves multiple lymph nodes, while extranodal involvement is also common. One risk factor for non-Hodgkin lymphoma is immunosuppression. Neoplastic B lymphocytes are the cells of origin in most non-Hodgkin lymphomas (90% of cases). The subtypes of non-Hodgkin lymphomas include Burkitt lymphoma, mantle cell lymphoma, follicular lymphoma, diffuse large B-cell lymphoma, and marginal zone lymphoma. Approximately 10% of lymphomas are Hodgkin lymphomas. NOT: Myeloblasts - Acute myelogenous and promyelocytic leukemia feature increases in myeloblasts. Acute promyelocytic leukemia is caused by a t(15;17) mutation and features symptoms of marrow failure, such as anemia and thrombocytopenia, along with splenomegaly and bleeding from the gums. Plasma cells are mutated and overactive in multiple myeloma, which can present with hypercalcemia, generalized weakness, and renal failure. Reed-Sternberg cells are CD15 and CD30 positive neoplastic cells in Hodgkin lymphoma, which can present with fever, night sweats, and localized lymphadenopathy, unlike the generalized lymphadenopathy of non-Hodgkin lymphoma. T-cell lymphomas such as Sézary syndrome and T-cell lymphoblastic leukemia can present with a mediastinal mass.

A 37-year-old man is brought to the emergency department by his wife, who is concerned about his recent strange behavior. Over the past few months, he has developed difficulty speaking and a new-onset tremor at rest in both upper extremities. His wife also mentions that he becomes confused and she sometimes finds him talking to himself. He takes ibuprofen occasionally for headaches but takes no other medications. Temperature is 37° C (99° F), blood pressure is 140/90 mm Hg, pulse is 70/min, and respiratory rate is 12/min. An irregular brown discoloration in the cornea and scleral icterus are noted on fundoscopic examination. On neurologic examination, the patient has difficulty pronouncing words, appears lethargic, and slurs some of his words. His abdomen is nontender, distended, and a positive fluid wave is found. The liver is palpated 3 cm below the costal margin. Ceruloplasmin: 8 mg/dL ALT: 200 U/L AST: 200 U/L Alkaline phosphatase: 170 U/L Total bilirubin: 30 mg/dL Direct bilirubin: 1.2 mg/dL Which of the following structures is most likely affected in this patient? A. Basal ganglia B. Cerebellum C. Frontal cortex D. Occipital lobe E. Parietal cortex F. Pons

A. Basal ganglia This patient is a middle-aged man who presents with new-onset difficulty speaking and a tremor. The labs show modest elevations in ALT, AST, and alkaline phosphatase. Physical exam findings reveal irregular brown discoloration in the cornea, and he has difficulty pronouncing words with a tremor at rest in both upper extremities. This constellation of symptoms is concerning for a diagnosis of Wilson disease. Wilson disease, also called hepatolenticular degeneration, is caused by an autosomal recessive mutation in the hepatocyte copper-transporting ATPase (ATP7B gene; chromosome 13) gene. The ATP7B gene prevents excretion of copper from the body. In Wilson disease, there is a decrease in copper incorporation into apoceruloplasmin, and a decrease of copper excretion into the bile, causing a decrease in serum ceruloplasmin. The Parkinsonian-like movement abnormalities seen in Wilson disease are a direct result of copper accumulation in the basal ganglia. Wilson disease typically presents before age 40 with liver disease (hepatitis, acute liver failure, cirrhosis). Additional signs and symptoms include neurologic disease (eg, dysarthria, dystonia, tremor, parkinsonism), psychiatric disease, and Kayser-Fleischer rings (deposits in descemet membrane of cornea, shown in the photograph), which are a pathognomonic finding. Lab findings include hemolytic anemia and possible indications of renal disease. Treatment includes chelation with either penicillamine, trientine, or oral zinc.

A 57-year-old man with a history of alcoholism is admitted to the intensive care unit after being involved in a motor vehicle collision. One day later, he develops a fever and a cough productive of purulent sputum. His temperature is 38.1°C (100.6°F), pulse is 98/min, respirations are 22/min, and blood pressure is 128/80 mm Hg. Within hours, the patient's respiratory status declines, with increased work of breathing and cyanosis, necessitating intubation. A Gram stain of sputum shows gram-negative encapsulated rods. A chest x-ray shows right upper lobe consolidation. Which of the following drugs is most appropriate for treatment of this patient assuming no bacterial resistance? A. Cefotaxime B. Erythromycin C. Gentamicin + vancomycin D. Pentamidine E. Ticarcillin-clavulanic acid + vancomycin F. Trimethoprim-sulfamethoxazole

A. Cefotaxime Pneumonia; Alcoholic; Klebsiella (Gram Neg Encapsulated Rods) Use a 3rd generation cephalosporin B- Macrolide; Legionella (less used now) C- Pseudomonas, MRSA D- Pneumocystis jirovecii E- MRSA F- Oral agent; intubated patient

If a man's serum creatinine level increased from 0.5 to 2 mg/dL over the past year, which of the following about creatinine is decreased by four fold? A. Clearance B. Excretion rate C. Filtered load D. Production E. Urine concentration

A. Clearance C = UV/P Here, P (plasma Cr) increased by 4-fold, so C (Clearance of Cr) must decrease by 4-fold. Serum Creatinine is INVERSELY PROPORTIONAL to GFR. If Cr is high, it means GFR is low (low kidney function)

Which of the following is a key mechanism of immediate hypersensitivity reactions? A. Cross-linking of mast cell Fc receptor-bound IgE B. Formation of hapten-protein complexes C. Polymorphonuclear leukocyte inflammation of the tissue site within 15 minutes D. Neutrophil release of reactive oxygen radicals E. Release of vasoactive amines following eosinophil binding of allergen-specific IgG

A. Cross-linking of mast cell Fc receptor-bound IgE • Immediate (Type I) hypersensitivity is mediated by IgE-mediated degranulation of mast cells and release of histamine. • Eosinophils could contribute, but the mast cell is of paramount importance. IgG is not involved.

A 65-year-old man comes in with a few days of decreased urine output. Current BP 180/100 mmHg. BUN 50 mg/dL, creatinine 1.2 mg/dL. Which is the most appropriate treatment? A. Decrease afterload B. Decrease contractility C. Decrease preload D. Increase afterload E. Increase contractility F. Increase preload

A. Decrease afterload

A healthy 25-year-old man lives at the beach on the Pacific coast of the United States. His twin brother has been living in a mountain cabin for the past 2 years. Which of the following indices is likely to be greater in the man living at sea level? A. Diameter of pulmonary arterioles B. Erythropoietin production C. Muscle capillary density D. Renal bicarbonate excretion E. Respiratory rate

A. Diameter of pulmonary arterioles Hypoxic vasoconstriction at high altitude lowers vessel diameter. Everything else changes in opposite direction. Renal bicarbonate excretion (NOT secretion) increases at altitude to correct respiratory alkalosis. So kidneys get rid of more bicarbonate.

A 65-year-old man with a history of coronary artery disease and myocardial infarction status post coronary artery bypass graft (CABG) surgery presents to his cardiologist for a routine appointment. On physical exam, the cardiologist appreciates a holosystolic, high-pitched blowing murmur heard loudest at the apex and radiating towards the axilla. Which of the following is the best predictor of the severity of this patient's condition? A. Enhancement with hand grip maneuver B. Enhancement with expiration C. Presence of audible S3 D. Enhancement with inspiration E. Presence of audible S4

A. Enhancement with hand grip maneuver Holosystolic = Mitral Regurgitation - Backflow into LA during contraction / systole Hand Grip increases the afterload, and thus will enhance the sound of ANY regurg murmur.

A 5-year-old boy is brought to the emergency department because of a severe reaction to a bee sting. His right hand and arm are very swollen and itchy. His temperature is 37.9°C (100.2°F), blood pressure is 100/75 mm Hg, pulse is 100/min, respirations are 20/min, and oxygen saturation is 96% on room air. On physical examination, the patient is able to speak in complete sentences and is non-toxic appearing. Which of the following substances is most likely responsible for this patient's symptoms? A. Histamine B. Eosinophil chemotactic factor-A C. Anaphylatoxins D. Interleukin-4 E. Interleukin-6

A. Histamine • This child is suffering from the effects of histamine release, resulting from mast cell degranulation. • Histamine, heparin, and eosinophil-chemotactic factor-A are found in preformed mast cell granules. These granules are released in response to physical injury, heat, cold, type I hypersensitivity reactions, C3a and C5a anaphylatoxins, proteins released from leukocytes, substance P, and melittin found in bee venom. In the present case, the direct degranulation of mast cells by melittin and subsequent release of histamine is the most direct cause of the boy's symptoms. • This type of reaction is called an anaphylactoid reaction because it uses the same cells and mediators found in anaphylaxis (type I hypersensitivity), but does not activate mast cells through IgE. Histamine is the principal mediator of the immediate transient phase of increased permeability of venules in acute inflammation, an effect mediated by H1 histamine receptors.

A 71-year-old man complains of shortness of breath with exertion. He was previously able to walk one mile without stopping but has recently started to feel short of breath after walking a few blocks. He also complains of a choking sensation when supine that is relieved by sitting up. His medical history is notable for coronary artery disease, diabetes mellitus, and alcohol abuse. Physical examination reveals a diffuse, left-displaced point of maximal impact (PMI). Auscultation demonstrates an early diastolic gallop best heard over the apex with the patient in the left lateral decubitus position. What is this? A. Left HF B. Right HF C. Both

A. Left HF Early diastolic gallop = S3

A 16-year-old female presents with complaints of difficulty breathing during exercise. In your office, she has no difficulty with breathing. A physical exam reveals no abnormalities. You suspect asthma and perform a methacholine challenge test. Which of the following is methacholine's mechanism of action? A. Muscarinic receptor agonist B. Endothelin-1 receptor antagonist C. 5-lipoxygenase pathway inhibitor D. Bronchial smooth muscle relaxer E. H1 Histamine inhibitor

A. Muscarinic receptor agonist (methacholine) - Causes bronchial smooth muscle contraction Methacholine acts as a non-selective muscarinic receptor agonist to stimulate the parasympathetic nervous system. It is most commonly used for diagnosing bronchial hyperreactivity, using the bronchial challenge test. NOT: B- Relaxes pulmonary vascular smooth muscle; used for pulm HTN C- Not how methacholine acts D- Beta 2 agonists relax bronchial smooth muscle E- Not how methacholine acts

A 28-year-old woman has noted occasional dyspnea for the past 13 months. She has also noted a non-productive cough and mild fever. Her symptoms improve when she is on vacation. A chest radiograph reveals nodular infiltrates. A transbronchial lung biopsy reveals microscopic findings consistent with an extrinsic allergic alveolitis with interstitial infiltrates composed of lymphocytes along with a few neutrophils and eosinophils. Which of the following is most likely to explain her findings? A. Parrots in her house B. Heavy cigarette smoking C. Downwind from a lead smelter D. Abusing crack cocaine E. Systemic lupus erythematosus

A. Parrots in her house The history suggests exposure to an allergen at home to which she is not exposed elsewhere. Bird fancier's disease is similar to farmer's lung. Both are forms of extrinsic allergic alveolitis, a form of localized antigen-antibody complex disease (Arthus reaction). Persistence of exposure may eventually lead to granulomatous inflammation as well.

Repro Question 22 A 22-yo college student presents due to amenorrhea. She reports significant stress from school. She is not sexually active. She runs 1 hour daily. She reports that she restricts her food intake due to concern for weight gain. Her BMI is 16.1. Over time, she is at risk for developing which of the following? A. Low bone density B. Hyperkalemia C. Hyperthyroidism D. Hypothyroidism

A. low bone density She has hypogonadotropic hypogonadism ("athlete's triad") which causes persistently low estrogen. In the long-term, estrogen deficiency is associated with bone loss. Clues for this were amenorrhea in a young female who is stressed, athletic, and low weight.

Acute Interstitial Nephritis / Tubulo-interstitial nephritis (TIN): 1. Most common drug causing drug-induced TIN? 2. What are 4 causes of renal papillary necrosis? 3. What causes most cases of TIN? 4. What organism causes most Acute Pyelonephritis (APN)? 5. What symptoms (2) and sign (1) differentiate APN from cystitis?

AIN is a drug-induced hypersensitivity reaction involving the interstitium and tubules of the kidneys. Basically an allergic reaction to a drug that occurs in the kidneys. 1. NSAIDs -- That's why NSAIDs are bad for the kidneys! 2. Most common causes: - Chronic analgesic (NSAID) use - Sickle cell - Acute pyelonephritis - Diabetes mellitus (not common) 3. Acute pyelonephritis (APN) 4. E. Coli 5. Flank pain, fever, WBC casts -- in pyelonephritis, but not cystitis.

Acute Renal Failure: - Main causes - Downstream consequences of (what you would expect to find) this turning into chronic renal failure. MNEM: MAD HUNGER Glomerulonephritis and Pyelonephritis -- what do these classically have in the urine?

ARF Causes: - Acute Tubular Necrosis (most common preceding renal cause; usually from ischemia) - Sepsis / Septic shock (complication in 50% of cases) - Post-renal obstruction (BPH, cervix cancer) - Vascular disease (malignant HTN) - RPGN - Drugs - DIC Consequences (MAD HUNGER): -Metabolic Acidosis - Dyslipidemia (especially triglycerides) - High potassium - Uremia - Na+/H2O retention (HF, pulmonary edema, hypertension) - Growth retardation and developmental delay - EPO deficiency (anemia) (EPO made by kidneys) - Renalosteodystrophy What urine findings do you get in GN and pyelonephritis? - RBC casts and WBC casts

What are the two biggest complications of an MI? Which one is most common?

ARRHYTHMIA is the most common cause of death from a heart attack. 50% don't make it to hospital in time and die from arrhythmia. RUPTURE -- Yellow/soft material that forms from ischemia can rupture and hemorrhage. Can occur within 2-14 days (i.e., up to 2 weeks after), which is the time most people would be discharged. Common cause of sudden death after an MI, usually around a week later.

A 13-year-old boy is brought to the emergency department after collapsing while playing soccer at school. He states that he suddenly had difficulty breathing and fell to the ground but did not lose consciousness. His coach states that he developed high-pitched inspiratory noises soon after collapsing. The boy appears lethargic and in respiratory distress. His temperature is 37.1°C (98.7°F), pulse is 120/min, respirations are 30/min, and blood pressure is 110/70 mm Hg. The remainder of the physical examination is most notable for decreased breath sounds with a markedly prolonged expiratory phase and end-expiratory wheezes. Arterial blood gas on 28% oxygen via venti-mask shows pH of 7.34, PCO2 of 38 mm/Hg, PO2 of 55 mm/Hg, and bicarbonate of 20 mEq/L. This patient most likely has high or low: - Forced Vital Capacity FVC - Forced Exp Vol in 1 sec (FEV1) - Total Lung Capacity

ASTHMA FVC and FEV1 decrease in asthma. TLC increases in all obstructive disease, especially emphysema but also in asthma.

++++++++++++++++ HIGH YIELD, EASY POINTS ++++++++++++++++ Porphyrias (Heme Synthesis Defects) - Know the defective enzyme, accumulated product, and presentation for each: - Sideroblastic anemia (X-LINKED) - Lead Poisoning - Acute Intermittent Porphyria - Porphyria Cutanea Tarda

Acute Intermittent Porphyria - Defective Enzyme: Porphobilinogen deaminase - Accumulated Substrate: Porphobilinogen, ALA - Presenting Sxs: (5 P's): Painful abdomen, Port wine-colored Pee, Polyneuropathy, Psychological disturbances, Precipitated by drugs (eg, cytochrome P-450 inducers), alcohol, starvation Porphyria Cutanea Tarda - Defective Enzyme: Uroporphyrinogen decarboxylase - Accumulated Substrate: Uroporphyrin (tea-colored urine) - Presenting Sxs: Blistering cutaneous photosensitivity and hyperpigmentation (hence, "cutanea"). - Most common porphyria. - Exacerbated with alcohol consumption. Lead Poisoning - Defective Enzyme: Ferrochelatase and/or ALA dehydratase - Accumulated Substrate: Protoporphyrin and/or ALA (blood) - Presenting Sxs: Microcytic anemia (basophilic stippling in peripheral smear, ringed sideroblasts in bone marrow). GI and kidney disease. Children—exposure to lead paint; mental deterioration. Adults—environmental exposure (eg, batteries, ammunition); headache, memory loss, demyelination (peripheral neuropathy).

An 18-year-old woman presents to the emergency department with acute-onset severe abdominal pain and a seizure. She says she had a similar attack 1 year earlier after taking some phenobarbital. At that time she underwent an exploratory laparotomy, which revealed nothing. The patient no longer takes barbiturates but recently started an extremely low-calorie diet for weight loss. Her temperature is 37°C (98.6°F), respiratory rate 16/min, and blood pressure 128/83 mm Hg. On physical examination it is noted that, in addition to severe abdominal tenderness, the patient has numbness in her fingers and is actively hallucinating. Porphobilinogen was found to be positive in high titers in her urine. 1. Diagnose 2. Classic triad of symptoms or the 5 P's of this condition 3. Due to loss of what enzyme in what pathway? 4. What are three common triggers for episodes in this condition? 5. Role of ALA synthase 6. Treatment

Acute Intermittent Porphyria (AIP) Triad of sxs: - Abdominal pain - Psychiatric symptoms - Seizures ^ Sxs are due to accumulation of toxic heme synthesis intermediates (such as porphobilinogen in stem). You should recognize the classic "5 P's" of AIP: 1. Painful abdomen 2. Port wine-colored urine due to excretion of porphobilinogen 3. Polyneuropathy (in this case, numbness in her fingers) 4. Psychological disturbances (hallucinations, seizures) 5. Precipitated by drugs (especially cytochrome P-450 inducers such as phenobarbital) or other causes (eg, starvation, alcohol use) AIP is a blood disorder caused by a deficiency of porphobilinogen deaminase, an enzyme involved in the heme synthesis pathway. As a result, porphobilinogen accumulates in the body, leading to various symptoms. Triggers: Usually triggered by an exposure 1. Reduced intake of calories and carbohydrates 2. Antibiotics (rifampin, sulfa drugs) 3. OCPs ^ These triggers can precipitate AIP attacks by promoting the activation of aminolevulinic acid (ALA) synthase, causing an accumulation of downstream products in the heme synthesis pathway. ALA synthase is inhibited by glucose, so a lack of glucose caused by this patient's restrictive diet contributed to the overactivity of ALA synthase. This is why glucose and hemin (an ALA synthase inhibitor) are given to patients with AIP attacks. - ALA synthase irreversibly catalyzes the first reaction of hemoglobin synthesis: glycine + succinyl-coenzyme A → ALA. This is the rate-limiting step of heme synthesis. Treatment: - IV glucose and saline - Hemin (ALA synthase inhibitor) - Pain control

Adenocarcinoma vs. Squamous Cell Carcinoma of the Esophagus: - Where in the esophagus each occurs - Primary cause - Increasing or Decreasing in frequency in the U.S.?

Adenocarcinoma: - Distal 3rd of esophagus - Cause: GERD => Barretts => Adenocarcinoma - Typically exophytic \ nodular - Increasing in frequency in US (now on par with squamous cell carcinoma) Squamous Cell Carcinoma: - Middle esophagus - Causes: Smoking, Drinking, Hot liquids - Typically ulcerative - Decreasing in frequency in US (high worldwide)

++++++++++++++++++++++++++++ Know the neurotransmitter, nucleus, area of the brain for various vignettes: Alzheimer's: 1. What NT is deficient? 2. Involves damage to which two parts of the brain? (Which causes #1) 3. Treatment = ___ (first line - enhances cognition) (MNEM) 4. If an Alzheimer's pt accidentally takes an anticholinergic and has severe muscarinic effects / anticholinergic syndrome, you can reverse these effects using ___. 5. Example case -- How does Alzheimer's present (sometimes kind of vague)

Alzheimer's Disease 1. Acetylcholine deficiency 2. Brain damage: both have "basal" ✔ Nucleus basalis - Ach production site. ✔ Basal forebrain - another Ach production site. ✔ Also, remember that there is shrinkage of brain tissue -- the sulci widen and the gyri diminish. 3. Donepezil: Treatment for Alzheimer's - MNEM: "Done with Alzheimer's symptoms" - Donepazil is an Acetylcholinesterase inhibitor (increases Ach) and serves as a "cognitive-enhancing drug" in Alzheimer's pts - improves memory, cognition, etc.. 4. Physostigmine will reverse the effects of anticholinergic syndrome, such as an atropine overdose in an Alzheimer's patient. - Alzheimer's pts shouldn't take atropine or other anticholinergics since already deficient in Ach. > Recall: Physostigmine is an Acetylcholinesterase Inhibitor used for reversing anticholinergic syndrome induced by anticholinergics, such as Atropine. Thus, it increases / restores Ach by blocking the Ach-degrading enzyme. Patients with Alzheimer's are at increased risk for being sensitive to anticholinergic medicines due to their already low Ach. Ex: MedBullets: What does Alzheimer's look like? A 65-year-old man is brought by his daughter to the neurologist. The daughter reports foregetfulness at times and has noticed few instances where he was confused in familiar places. She denies any major changes in his personality or hallucinations. The patient was diagnosed with mild Alzheimer's dementia and started on Donepezil.

Diagnose A 55-year-old man presents to the emergency department with chest pain. He describes the pain as excruciating and knife-like that began earlier in the morning. His pain is radiates to the back. Medical history is significant for hypertension, for which he is taking lisinopril. Physical examination is significant for unequal blood pressures in the arm and a diastolic murmur in the left sternal border. An electrocardiogram is unremarkable, his cardiac troponins are not elevated, and a chest radiograph demonstrates a widened mediastinum. Preparations are made to obtain a CT angiography.

Aortic Dissection How will this patient present? - Tearing \ stabbing \ sharp pain that radiates to the back - Loss of upper extremity pulses - Aortic valve regurgitation (X ray with wide aortic valve root) - Widened mediastinum What other conditions are associated with dissection? HYPERTENSION - main cause Marfans disease Ehlers-Danlos syndrome

Repro Question 4 When does maternal serum hCG reach peak levels? A. 20-40 days post-conception B. 45-90 days post-conception C. 90-110 days post conception D. 110-120 days post-conception E. 120-150 days post-conception Bonus Questions - Likely diagnosis if: - High hcg but no gestational sac is seen on ultrasound. - Very high hcg, but large uterus and bleeding.

B. 45-90 days post-conception hCG is produced by trophoblasts. Its job is to support the corpus luteum so that it produces progesterone and estrogen to maintain pregnancy. While it can be detected in maternal serum or plasma as early as 8-9 days post conception and in urine after 12-14 days, it does not peak until 45-90 days. Its levels begin to decline at 10 weeks when the placenta is developed enough to produce estrogen and progesterone (in lieu of the corpus luteum). When hCG is detected in maternal serum but no gestational sac is found by ultrasound, the differential diagnosis includes very early pregnancy, very early pregnancy failure, and ectopic pregnancy. In this case, a repeat ultrasound is necessary for diagnosis. Bonus: - High hCG (>6000 mIU/ml) without an intrauterine sac is strongly suggestive of ectopic gestation. - Very high hCG levels in early pregnancy (>200,000 mIU/ml) with bleeding and larger than expected uterus may indicate molar pregnancy. - Molar pregnancy is an abnormal form of pregnancy in which a non-viable fertilized egg implants in the uterus and will fail to come to term. A molar pregnancy is a gestational trophoblastic disease which grows into a mass in the uterus that has swollen chorionic villi.

+++++++++ HIGH YIELD, KNOW +++++++++ A 20-year-old woman presents to the emergency department with severe nausea, vomiting, and abdominal pain. She tells the emergency physician that her last menstrual period was 3 months ago. The patient claims not to use alcohol but has been smoking a pack of cigarettes per day for 5 years. On physical examination, abdominal swelling is observed. Laboratory tests indicate that her ß-human chorionic gonadotropin (ß-hCG) level is 200,000 mlU/mL. Ultrasonography reveals numerous vesicles without any fetal parts in the uterus. Which of the following is the most likely karyotype of the uterine contents? A. 45,X0 B. 46,XX C. 47,XXY D. 48,XXYY E. 69,XXX

B. 46,XX This is a complete molar pregnancy (aka complete hydatiform mole) because there are no fetal parts at all. Caused by too much paternal DNA and not enough maternal DNA, so fetal parts don't develop (need maternal DNA to grow fetus). The karyotype is going to be 46,XX 99% of the time, unless it's clearly a partial mole (e.g., some fetal parts), then it might be 69,XXX or 69,XXY.

+++++++++++++++++++++++++++++ Repro Question 17 A 25-yo old woman who is 8 weeks pregnant presents to the emergency room with vaginal bleeding and excessive vomiting. Her hCG levels are 250,00 mIU/ml and a uterine sonogram reveals a 'snowstorm pattern' inside an enlarged uterus, chromosomal analysis confirms a complete molar pregnancy. What karyotype did the chromosomal analysis show? A. 23X B. 46XY C. 46YY D. 69XXY E. 92XXYY

B. 46XY A complete molar pregnancy occurs when 2 sperms fertilize an empty ovum or when a sperm fertilizes an empty ovum and subsequently duplicates. As a result the ovum contains 2 paternal chromosomes leading to karyotypes 46XX or 46XY. Because the genetic material comes solely from the sperm, there is no fetus in complete molar pregnancy. Google: Molar pregnancy is an abnormal form of pregnancy in which a non-viable fertilized egg implants in the uterus and will fail to come to term. A molar pregnancy is a gestational trophoblastic disease which grows into a mass in the uterus that has swollen chorionic villi.

A 23-year-old man comes to the emergency department because of sudden-onset shortness of breath and chest pain. The chest pain is retro-sternal and left-sided, and is exacerbated by deep breathing and by cough. He has a history of asthma and uses albuterol only when needed. He is in moderate distress, with a temperature of 98°F (36.7°C) and respiratory rate of 28/min. Physical examination shows normal breath sounds on the right, but decreased breath sounds on the left. Percussion of the left chest shows hyper-resonance. Arterial blood gas on room air shows: PO2 = 78 mm Hg PCO2 = 25 mm Hg pH = 7.52 Which of the following is most likely to be seen on chest x-ray? A. An infiltrate in the left lower lobe B. A radiolucency along the left chest wall C. A wedge-shaped opacity in the left lateral lung field D. Fluid along the left costophrenic angle E. Hyperinflation of both lung fields

B. A radiolucency along the left chest wall - radiolucency = darker area (as opposed to radiopaque) Hyperventiliation, chest pain that gets worse with deep breathing, cough, elevated A-a gradient, young male = spontaneous pneumothorax. A- Sudden onset, pneumonia unlikely. C- PE unlikely because of hyper-resonance and no risk factors. D- Unlikely because of hyper-resonance. E- Unlikely if normal breath sounds on right and decreased on left.

A 51-year-old woman suffering from diabetic nephropathy receives a kidney transplant from an unrelated donor. Transplant was followed with a standard regimen of tacrolimus, mycophenolate, and prednisone. One month after surgery, she develops a fever and her renal function declines. A renal biopsy reveals extensive lymphocytic infiltration. Her dose of tacrolimus is increased, and her renal function markedly improves. The clinical findings in this episode indicate a diagnosis of: A. Hyperacute antibody-mediated rejection B. Acute T cell-mediated rejection C. Accelerated T cell-mediated rejection D. Delayed-type hypersensitivity to mycophenolate E. Prednisone toxicity

B. Acute T cell-mediated rejection The timing of the clinical observations and pathology and the responsiveness to increased tacrolimus point to an episode of acute T cell-mediated rejection.

A 57-year-old high school teacher visits the physician because of pain in her left knee and right hip that has been ongoing for the past 3 years. She notices some stiffness in the morning when getting out of bed, but it lasts no longer than 5 minutes. The pain is most severe at the end of the school day, after she has spent several hours standing. She agrees that aspirin partially relieves the pain. She denies recent fevers, changes in weight, changes in stool, back pain, or vision changes. She mentions a family history of "joint problems" in her mother and father. Physical examination reveals a blood pressure of 140/70 mm Hg, heart rate of 78, respiratory rate of 16, temperature of 37.5°C (99.5°F), and body mass index of 27 kg/m2. There is no tenderness, warmth, or fluctuance on palpation of her hips or knee joints bilaterally. Lower extremities show full range of motion, 4/5 strength, and brisk patellar and ankle reflexes. No skin lesions or rashes are appreciated. Laboratory values are unremarkable. What additional finding is most likely in this patient? A. An HLA-B27 allele B. Bulges in the distal interphalangeal joints C. Elevated levels of serum rheumatoid factor D. Generalized systemic symptoms E. Hyperuricemia

B. Bulges in the distal interphalangeal joints This question tests knowledge of rheumatoid vs. osteoarthritis. This patient presents with increasing pain in the large joints that worsens during the day and with increased use. This picture is consistent with osteoarthritis (OA), a degenerative joint disease with onset in a patient's fifth decade of life, after years of wear and tear. There is no hereditary component, although increased weight is a risk factor. Pathologic changes include the presence of joint "mice" (the worn, dislodged pieces of cartilage and subchondral bone in the joint spaces) and osteophytes (bony outgrowths that may develop at the margins of the affected articular surfaces). A classic physical examination finding in osteoarthritis, especially in affected women, is the presence of Bouchard and Heberden nodes, which are grossly visible bulges in the proximal and distal interphalangeal joints, respectively caused by osteophyte formation. The image on the left shows the physical appearance of Heberden and Bouchard nodes. The image on the right shows x-rays of healthy joints (identified in the red box) and joints with Heberden nodes.

A 27-year-old woman with breast cancer undergoes bilateral mastectomy and radiation therapy. One year later, she returns to the physician because of pain in her neck and anterior thorax. The physician explains that radiation therapy can cause adhesions of muscles and nerves. The patient undergoes surgery to remove the scar tissue of the supraclavicular region but the phrenic nerves are accidentally damaged. The patient suddenly develops dyspnea and cyanosis and the abdominal wall paradoxically moves inward during inspiration. Which of the following is the best explanation for this observation? A. Contraction of the diaphragm during inspiration causes the abdominal wall to move inward B. Contraction of the intercostal muscles during inspiration causes the diaphragm to be drawn upward and the abdominal wall to be drawn inward C. Contraction of the rectus abdominus muscles during inspiration causes inward motion of the abdomen D. In the horizontal position, the reduction in thoracic volume required for inspiration is caused by upward movement of the abdominal viscera E. Relaxation of the muscles of the thoracic wall during inspiration causes the abdominal wall to be drawn inward

B. Contraction of the intercostal muscles during inspiration causes the diaphragm to be drawn upward and the abdominal wall to be drawn inward Using external intercostals instead of diaphragm for inspiration, so diaphragm moves passively upward, which draws abdomen inward. No longer using diaphragm because the phrenic nerves were severed.

A 28-year-old man presents to the emergency department complaining of increasing muscle weakness. He states he first noticed that he had some mild weakness in his feet and legs 4 days ago. He initially attributed the weakness to "being tired," but became concerned when he began to have difficulty walking. He states that his symptoms have progressed over the past few days, and the weakness has now spread to his arms and hands. Other than a recent episode of diarrhea and vomiting, the patient has been in good health. His neurologic exam is notable for 1+ Achilles and patellar reflexes, 2+ reflexes in the brachioradialis and biceps, 3/5 strength on foot dorsiflexion and plantarflexion, and 4/5 strength on knee extension. Which of the following organisms is most commonly associated with this patient's neurologic symptoms? A. α-Hemolytic, encapsulated, gram-positive cocci that produce an IgA protease B. Curved, oxidase-positive, gram-negative bacteria that can be grown at 42°C C. Non-lactose-fermenting, oxidase-positive, gram-negative, aerobic bacilli D. Rod-shaped, gram-positive, spore-forming anaerobe that produces a heat-labile toxin E. Spiral-shaped bacteria with axial filaments, visualized using dark-field microscopy

B. Curved, oxidase-positive, gram-negative bacteria that can be grown at 42°C This is Guillain-Barré Syndrome, which is a rapid-onset muscle weakness caused by the immune system (autoantibodies) attacking the myelin of the peripheral nervous system, making it hard for action potentials to get all the way down to the distal limbs and for sensory impulses to ascend up nerves. Typically occurs after a Campylobacter infection. - MAKES SENSE: an illness could form harmful antibodies. - Curved, oxidase-positive, gram-negative bacteria that can be grown at 42°C - This explains his GI symptoms -- diarrhea and vomiting.

A 71-year-old man complains of shortness of breath with exertion. He was previously able to walk one mile without stopping but has recently started to feel short of breath after walking a few blocks. He also complains of a choking sensation when supine that is relieved by sitting up. His medical history is notable for coronary artery disease, diabetes mellitus, and alcohol abuse. Physical examination reveals a diffuse, left-displaced point of maximal impact (PMI). Auscultation demonstrates an early diastolic gallop best heard over the apex with the patient in the left lateral decubitus position. Which set of cardiovascular parameters are most likely in this patient? A. Decreased EF, decreased LVEDV B. Decreased EF, increased LVEDV C. Increased EF, decreased LVEDV D. Preserved EF, normal compliance E. Preserved EF, decreased compliance

B. Decreased EF, increased LVEDV This is Left Heart Failure Early diastolic gallop = S3

A 35-year-old woman with a bicuspid aortic valve comes to the physician because of a 1-week history of a swollen, painful left knee. She has had occasional fever and progressive fatigue during the past 4 months. Her temperature is 38.2°C (100.8°F). Physical examination shows a tender left knee with an effusion. A grade 2/6 systolic murmur is heard. Echocardiography shows a vegetation on the aortic valve. Analysis of synovial fluid aspirated from the left knee shows many segmented neutrophils; a Gram stain shows no organisms, and culture is negative. Microscopic examination of the urine shows RBCs and RBC casts, but culture of the urine grows no organisms. Four blood cultures grow an α-hemolytic streptococcus. Which of the following is the most likely cause of the joint and renal disease? (A) Autoimmune response triggered by a bacterial antigen (B) Deposition of antigen-antibody complexes (C) IgA nephropathy (D) Neutrophil response to replicating bacteria (E) Type IV (delayed) hypersensitivity to a bacterial antigen

B. Deposition of antigen-antibody complexes Infective endocarditis here. Some of the hints are the bicuspid valve (surrogate for damaged valves) and the presence of alpha heme strep. This is subacute and can seed distant sites, but here clearly hasn't. The lack of bacteria in culture points to something else...that being the deposition of IC's (kidney is commonly affected by IC's)

At which part of the cardiac cycle is S3 heard? A. Atrial contraction B. Early diastolic filling C. Ejection D. Isovolumetric contraction E. Isovolumetric relaxation When is S4 heard?

B. Early diastolic filling S3 is the sound of valves opening and blood flowing into the empty ventricles. S4 is heard in late diastolic filling -- see IMAGE - S4 is the sound of atrial contraction distending the left ventricle

A 30-year-old woman comes to the physician because of progressively worsening shortness of breath and a dry cough. A chest x-ray shows hilar lymphadenopathy and diffuse abnormalities of the lung parenchyma. A biopsy specimen of a segment of lung shows noncaseating granulomas. Acid-fast, silver, and periodic acid-Schiff (PAS) stains of the granulomas are negative. Which of the following is most likely to be increased in this patient? A. Arterial partial pressure of oxygen B. Elastic recoil C. Forced expiratory volume in 1 second D. Forced vital capacity E. Minute alveolar ventilation F. Residual volume

B. Elastic recoil This is Sarcoidosis; Restrictive; Compliance decreases; Recoil increases. - characterized by granulmomatous inflammation of multiple organs, esp. the lungs. Pulmonary Function Tests: restrictive pattern is common (normal FEV1/FVC with normal TLC) A,C,D,F would all decrease in this patient E would not increase.

Repro Question 18 A 43-yo woman presents with amenorrhea over the past 6 months. Physical examination reveals that the woman is obese, and has acne and hirsutism. A pelvic ultrasound shows a 'string of pearls' arrangement on her ovaries. Which of the following hormones is NOT elevated in this patient? A. Androstenedione B. FSH C. Testosterone D. Estrone E. LH

B. FSH The patient has PCOS and all hormones are elevated except FSH. In PCOS, there's a high ratio of LH to FSH due to increase in LH levels. The pathogenesis of PCOS isn't really understood, but it does involve ovarian stimulation that leads to excess androgen production. High: LH and Testosterone Low: FSH and Estrogen

Venous blood from which organ most likely has the lowest O2 content at baseline? A. Brain B. Heart C. Kidney D. Skeletal muscle Why do systemic blood vessels dilate during sympathetic activation, exercise, or any other time the heart is working hard?

B. Heart The heart works the hardest out of any organ. It uses up the most oxygen, so venous blood leaving the heart has very low oxygen. "This is why when the heart is working hard, systemic vasodilation occurs so that muscles and organs can try to get as much oxygen from blood as they can while the heart is using up so much of it."

A 25-year-old medical student presents to the clinic after a possible needlestick injury. He states that the incident occurred 2 hours earlier in the operating room as he was closing an incision. Although it is unclear whether there was any break in his skin, he is concerned about exposure to the hepatitis B virus. The student relates that he thinks he has had his hepatitis B vaccinations, but he has never had his titers measured. Which of the following is most similar to the mechanism of the appropriate treatment for this patient? A. Allogeneic bone marrow transplantation B. IgG delivered transplacentally from mother to fetus C. Interferon therapy D. Natural killer cell response to bacterial pathogen E. Vaccination with an attenuated strain of influenza virus

B. IgG delivered transplacentally from mother to fetus This medical student sustained a potential needlestick injury and has uncertain immunization status against hepatitis B. The mainstay of postexposure prophylaxis for hepatitis B is administration of hepatitis B immunoglobulin (HBIG). This is a form of passive immunity as pre-made antibodies are being delivered directly to the patient. Transplacental delivery of IgG antibodies is also an example of passive immunization because the fetus receives preformed maternally derived antibodies rather than producing its own. Remarkably, a neonate possesses the same serum level and diversity of IgG as his or her mother. However, these maternally derived immunoglobulins persist for only several months, and eventually the infant's own immune system must produce antibodies. Postexposure prophylaxis for hepatitis B depends on the immunization status of the patient. Scenarios in which postexposure prophylaxis with HBIG and hepatitis B immunization are warranted include the following: - Absent or incomplete hepatitis B immunization series - Lack of response to the hepatitis B vaccine - Unknown response to the hepatitis B vaccine - Unclear immunization status Passive immunization is also used for exposures to other infectious pathogens or toxins in addition to hepatitis B. Examples include early treatments for tetanus, botulism, hepatitis B, varicella, and rabies; a helpful way to remember these conditions is with the mnemonic "To Be Healed Very Rapidly."

An 8-year-old boy is brought to the clinic with a complaint of a runny nose and difficulty breathing. His mother says the boy has had many respiratory tract infections throughout his childhood, often with a productive cough. The child's symptoms are tolerable in the morning but progressively worsen as the day continues. Chest auscultation reveals crackles and wheezing, but the physician is unable to auscultate normal S1 and S2 heart sounds. Radiographic examination reveals large, dilated airways and a right-sided cardiac shadow. What other complication is this patient likely to have? A. Eisenmenger syndrome B. Impaired fertility C. Panacinar emphysema D. Pancreatitis E. Pulsus paradoxus

B. Impaired fertility The patient's sinus symptoms, x-ray findings of bronchiectasis, and history of respiratory tract infections, combined with the physician's inability to auscultate normal S1 and S2 (so can't hear any heart sounds), suggest a diagnosis of Kartagener syndrome. This inherited disease is characterized by a dynein arm defect (dyenin is a protein on microtubules). This results in immotile cilia. Microtubules are hollow, fibrous shafts whose main function is to help support and give shape to the cell. They also serve a transportation function, as they are the routes upon which organelles move through the cell. Sperm flagella in men and fallopian tube cilia in women are affected, leading to impaired fertility in both sexes. Approximately 50% of patients with Kartagener syndrome, including this patient, have situs inversus, abnormal placement of the internal organs on the opposite side of the body. There are usually no serious adverse health consequences associated with this feature of the syndrome. NOT: Pancreatitis would be observed in a young patient with cystic fibrosis; it is not associated with Kartagener syndrome. Although this patient has pulmonary symptoms, which are common to both Kartagener syndrome and panacinar emphysema, the latter is associated with α1-antitrypsin deficiency. Symptoms of panacinar emphysema develop in the third and fourth decades and would not be present in an 8-year-old child. Eisenmenger syndrome would be identified in a patient with an uncorrected congenital cardiac defect, such as a ventricular septal defect or patent ductus arteriosus. No murmur was reported for this patient. Pulsus paradoxus may be observed in a young patient with asthma; however, apart from respiratory involvement, this patient's symptoms are not consistent with asthma.

A 26-year-old woman is brought to the emergency department because of a 4-day history of flu-like symptoms accompanied by vomiting following each attempt to eat or drink. Her temperature is 38.5°C (101.3°F), pulse is 93/min, respirations are 24/min, and blood pressure is 105/70 mm Hg. Physical examination shows no other abnormalities. Which of the following additional findings is most likely in this patient? A. Decreased serum ADH (vasopressin) concentration B. Increased serum aldosterone concentration C. Increased serum atrial natriuretic peptide D. Increased urine sodium and chloride concentrations E. Increased urine volume

B. Increased serum aldosterone concentration With continued vomiting, she is losing volume and HCl. RAAS and ADH kick in. ANP is decreased due to low volume. All of these changes act to conserve water. At first urine sodium concentration may be high but, eventually (by day 4), sodium is conserved and keeps osmolarity.

Repro Question 19 A 17 yo female presents to the ED with acute right lower quadrant pain. Her vital signs are BP 130/90, T 102, RR 17, and HR 115. Blood work demonstrates leukocytosis (WBC 18,000). Urine pregnancy test is negative. She reports a history of chlamydia two years ago which was treated. She has a new sexual partner and uses condoms inconsistently. Pelvic ultrasound does not show any ovarian cysts. On examination, she has significant lower abdominal tenderness and sensitive cervical motion tenderness. What is she at risk of developing? A. Endometriosis B. Infertility C. Ovarian torsion D. Adenomyosis

B. Infertility Scar tissue formation from PID leads to infertility, ectopic pregnancy, and chronic pain (RLQ; cervical motion tenderness). She likely has pelvic inflammatory disease (PID) which is a well-established risk factor for infertility due to potential scarring and blockage of the fallopian tubes. MedBullets - PID: - PID = inflammation of the uterus, fallopian tubes, or ovaries - Etiology: Most often caused by an ascending UTI infection of these reproductive organs. The most common preceding UTIs are Chlamydia and Gonorrhea - Sxs: Scar tissue formation leads to infertility, ectopic pregnancy, and chronic pain

You are volunteering at a school Halloween carnival when a 5-year-old kindergarten student with known egg allergy develops sudden onset of generalized urticaria (hives), facial swelling, wheezing and shortness of breath after she ate a cupcake. When you arrive on the scene, she says her tummy hurts, and then vomits. She then becomes lethargic and loses consciousness. Which of the following is the best immediate treatment option? A. Oral antihistamines B. Intramuscular epinephrine C. Inhaled albuterol D. Oral corticosteroids E. An emetic

B. Intramuscular epinephrine • Epinephrine (adrenalin) injection works by increasing cardiac output, reversal of blood pressure loss, and relaxation of airway smooth muscle. • None of the other choices are appropriate or act quickly enough in an emergency situation.

A 28-year-old woman undergoes allogeneic bone marrow transplantation for acute leukemia. There is a 5/6 HLA match. Three weeks later, she has marrow engraftment and her hemoglobin and WBC count are returning to normal. However, she has the appearance of a fine, scaling skin rash over her trunk and upper extremities. These findings are most consistent with which of the following complications? A. Reduced numbers of megakaryocytes B. Keratinocyte apoptosis C. Dry gangrenous necrosis D. Contact dermatitis E. Abrasions from a fight in the ICU

B. Keratinocyte apoptosis The rash is typical for acute graft versus host disease (GVHD). The donor lymphocytes engraft and attack host tissues, including epidermal keratinocytes, causing apoptosis. The slight antigen mismatch predisposes to GVHD, though most cases are acute and mild and can be treated. The two other tissues often affected by GVHD include liver and GI tract. GVHD is a Type IV hypersensitivity reaction secondary to the donor's T-cells attacking the recipient's cells leading to rash, jaundice, diarrhea, hepatosplenomegaly.

++++++++++++++++++++++++++++++ Repro Question 5 A 29-yo female presented to her family physician with acne on her face, chest and back. At 5'3" and 180 lbs, the patient was also concerned that her last menstrual period was over 3 months ago. Physical exam in the patient's neck and inguinal area revealed velvety skin and patches of thick hair throughout her body. The patient was diagnosed with PCOS and started on hormone replacement therapy. In addition to prescribing Accutane to control the patient's acne, the doctor looked over her recent lab work. Which of the following hormonal values most likely represents a patient suffering from PCOS? A. LH↑, FSH↑, E↓, T- B. LH↑, FSH↓, E↓, T↑ C. LH↑, FSH↓, E-, T ↓ D. LH↓, FSH↓, E↓, T↓ E. LH↑, FSH↑, E↑, T↓ (Note: E- and T- mean no change)

B. LH↑, FSH↓, E↓, T↑ Polycystic Ovarian Syndrome is characterized by high androgens (ovarian cysts produce androgens), low estrogens, and a high LH/FSH ratio. High: LH and Testosterone (later or higher in alphabet) Low: FSH and Estrogen (earlier or lower in alphabet)

A 55-year-old woman comes to the emergency department (ED) because of left-sided facial droop and slurring of speech that began 1 hour ago. Her medical history is significant for poorly controlled hypertension. In the ED, the patient's pulse is 88/min, and her blood pressure is 160/80 mm Hg. She is alert and cooperative on examination. Strength is 3/5 in the left upper extremity, 2/5 in the right upper extremity, 4/5 in the left lower extremity, and 5/5 in the right lower extremity. A noncontrast CT scan of the head is noncontributory. This patient most likely had an occlusion of which of the following arteries? A. ACA B. MCA C. PCA D. AICA E. PICA

B. MCA The middle cerebral artery is the artery most often occluded in stroke. Website that shows signs/sxs of strokes in different arteries: https://www.myamericannurse.com/identify-the-vessel-recognize-the-stroke/ Poorly controlled hypertension has an acute unilateral facial droop and dysarthria (difficulty using facial muscles). Together with her reduced strength in her left upper arm (3/5), reduced right arm strength (2/5), and preserved strength of the lower extremities, these findings suggest an acute ischemic stroke involving a branch of the right middle cerebral artery (MCA) that supplies the primary motor/sensory cortices of the face and tongue. Signs/Sxs of an MCA stroke include: • Hemiplegia (paralysis) of the contralateral side, affecting the lower part of the face, arm, and hand while largely sparing the leg • Contralateral (opposite-side) sensory loss in the same areas • Contralateral homonymous hemianopia—visual-field deficits affecting the same half of the visual field in both eyes. The superficial MCA branches distribute blood to the lateral aspects of the cerebral hemispheres, including the motor and sensory cortices of face, throat, hand, and arm—and in the dominant hemisphere, the areas for speech (eg, the Broca and Wernicke areas). Therefore strokes affecting the large MCA branches typically cause motor and sensory deficits in the face/upper extremities, speech deficits (dysarthria and aphasia in dominant, usually left hemisphere strokes, and solely dysarthria in nondominant, usually right hemisphere strokes), or inability to attend to one side of space (nondominant, usually right hemisphere strokes). Small deep penetrating arteries arising from the MCA branch to the internal capsule and basal ganglia, would result in symptoms of contralateral hemiparesis of both the upper and lower extremities, usually in the absence of sensory symptoms. Strokes to the deep penetrating arteries of the MCA are often hemorrhagic, caused by Charcot-Bouchard aneurysm rupture. The other answers are incorrect for the following reasons: A: Individuals with anterior cerebral artery occlusions would show signs of motor and sensory impairment of the lower extremities with or without urinary incontinence. Occlusions of the ACA are the least common type of stroke. C: A posterior cerebral artery stroke would result in contralateral hemianopia with macular sparing. D: An individual with an anterior inferior cerebellar artery (AICA) occlusion would have facial paralysis, as demonstrated in this patient. However, there would be several other symptoms including Horner syndrome, vertigo, ataxia, and more. Patients with AICA would likely have the classic "crossed symptoms" associated with brainstem stroke, including ipsilateral loss of pain/temperature sensation on the face (spinal trigeminal nucleus) with contralateral loss of pain/temperature sensation of the rest of the body. E: An individual with a posterior inferior cerebral artery (PICA) occlusion would also show signs of dysphagia, hoarseness, impaired gag reflex, and possibly hiccups—the so-called "lateral medullary syndrome." Ipsilateral facial pain and temperature sensation is typically affected as well as contralateral body pain and temperature sensation.

While on a hike with her husband Harry, a female stumbles and falls into a patch of poison ivy. She does not develop an immediate reaction and thinks nothing of it. However, 2-3 days later she develops a skin rash on her left hand. This is most likely mediated by: A. Histamine release from IgE-activated mast cells B. Memory CD4+ T-cell response to allergenic hapten-protein complexes C. Large molecular weight antigens D. Immune complex deposition in peripheral blood vessels E. Primary IgM antibody response to poison ivy antigens

B. Memory CD4 T cell response to allergenic hapten-protein complexes This scenario describes a T cell-mediated delayed-type hypersensitivity reaction to poison ivy, the active ingredient of which is a small molecule hapten. This is the type that requires a prior exposure, which is when the memory CD4 T-cells form, causing an allergic reaction with any future exposures. It's called delayed-type because the symptoms show up in 2-3 days. So this is mediated by T-cells, specifically Helper CD4+ T-cells. NOT: A - IgE and Histamine are type I HS, which is an immediate allergic reaction. C - Immune complexes are Type 3 hypersensitivity. E - Mediated by T-cells, not IgM

How do acetylcholine (ACh) & Muscarinic Agonists cause vasodilation? A. Adrenergic activation in Endothelial Cells B. Muscarinic activation in Endothelial Cells C. Nicotinic activation in Endothelial Cells D. Muscarinic activation in Smooth Muscle E. Muscarinic blockade in Smooth Muscle F. Nicotinic activation in Smooth Muscle

B. Muscarinic activation in ECs Direct M agonist / ACh at ECs → Nitrous Oxide released from ECs → cGMP released at SM → Vasodilation Ach and Muscarinic agonists don't directly bind to smooth muscle. NO and NO meds directly activate smooth muscle. In studies, ACh relaxes arteries preconstricted with NE - So ACh vasodilation is stronger than NE vasoconstriction Learn Image: Ach is a direct muscarinic agonist in ECs, which triggers NO release in ECs, and then cGMP release in smooth muscles to vasodilate.

++++++++++++++++ HIGH YIELD ++++++++++++++++++ A 67-year-old woman presents to the emergency room two hours after an acute onset of dizziness. When trying to swallow a medication for this, she noted difficulty swallowing water. Upon arrival to the emergency department, her examination showed a right ptosis. She has nystagmus upon right lateral gaze. Muscle Strength is normal. Sensation of temperature is reduced along the right side of the face and the left side of the body. What is the most likely location of the patient's vascular occlusion? A. Right middle cerebral a. B. Right posterior inferior cerebellar a. C. Right posterior cerebral a. D. Left middle cerebral a. E. Left posterior inferior cerebellar a. F. Left posterior cerebral a.

B. Right posterior inferior cerebellar a. LATERAL MEDULLARY SYNDROME - Affects the Nucleus Ambiguus (CNs 9, 10, 11), which is located in the LATERAL part of the MEDULLA -- hence, lateral medullary syndrome. Brainstem lesions: Ipsilateral effects in face and head, contralateral in body. Symptoms of Lateral Medullary Syndrome: 1. Ipsilateral manifestations - facial loss of pain and temperature - ataxia (arm and leg, gait) - hoarseness (specific) - dysphagia (specific) - Horner syndrome 2. Contralateral manifestations - pain and temperature hemisensory loss 3. Nausea, nystagmus, vomiting, vertigo

A 30-year-old male immigrant from India was diagnosed with pulmonary tuberculosis after presenting with weight loss, night sweats, and hemoptysis. The patient began an anti-tuberculosis drug regimen with regular follow-up visits 2x monthly. He makes steady progress in subsequent weeks with repeat negative sputum cultures. At 2 months of treatment, the patient starts complaining of leg and hand numbness and weakness. On examination, his lips appear cracked and dry. What could have prevented these symptoms? A. Vitamin B1 supplementation B. Vitamin B6 supplementation C. Vitamin B12 supplementation D. Regular blood glucose measurement and carbohydrate restriction

B. Vitamin B6 supplementation Anti-TB drugs can cause numbness, tingling, or weakness. Isoniazid impedes the actions of pyridoxine (Vitamin B6) by forming an inactive complex and by forming hydrazones that block conversion of pyridoxine to pyridoxal 5- phosphate. To overcome these effects, Vitamin B6 is commonly co-prescribed with isoniazid.

Your patient has suffered a stroke and seems to be suddenly unable to understand what is said to him. His speech seems normal, but does not make sense. What does the patient have? A. Broca's aphasia B. Wernicke's aphasia C. Gerstmann's Syndrome D. Unilateral neglect E. Conduction aphasia

B. Wernicke's aphasia Heschl's gyrus: - Primary auditory cortex - tonotopic organization - If lesioned, would not have any language abilities -- comes before broca's and wernicke's areas. Wernicke's Area: - Superior temporal gyrus - Comprehension, extracting meaning from language Angular gyrus: - links written words with language (reading) Broca's area: - left inferior frontal lobe - Speech production

Goblet cells in the esophagus indicate what diagnosis?

Barret's Esophagus

Locked-in syndrome is due to a lesion in what artery? What is the clinical picture? Why?

Basilar a. !!! Can only move eyes (slightly) > because CNs 3 and 4 are the only ones spared; Basilar a. is pons and medulla; it also gets the LCST, leading to paralysis. Definitely know this!

Circle of Willis labeled: - MCA - Basilar a. - ACA - PCA - AICA - PICA (Brainstem Vasculature)

Be able to label each.

Hypocalcemia will have what effect on: - PTH - Vitamin D

Both will INCREASE Trying to restore calcium levels

Breakdown of what types of questions are asked on STEP1 (and Dr. Rhee's Questions too) CV resource links

Breakdown: • 25% Physiology • 50% Pathology • 25% Pharmacology Dr. Rhee's Available Resources: • Murmur online quiz https://tinyurl.com/uw5nyap • Murmur practice https://tinyurl.com/w2o5lmk • Murmur CBL part 1, part 2 • CV final CBL part 1, part 2, part 3, part 4 • ANS ILA https://tinyurl.com/y2zjkcaw • M2 CV YouTube https://tinyurl.com/y5j6j7l4 • Library online textbook list: https://libguides.uams.edu/COM_eBooks • They have a list of buzzwords they actively avoid -Q writers are asked to replace buzzwords in their Qs • Test-taking skills may work against you - They know all the tricks, actively remove any obvious material pointing to right answer. • Qs with good discrimination index are kept - Qs that everyone does equally are removed (= they catch up with new common resources but high-yield is still high-yield) - Understand and integrate! • They love their audiovisual Qs - Pathology images, heart murmurs, ... - Newer Qs do not repeat the info in the stem or in AV media • Qs in a single block (1 h) is balanced out - Hard Qs that require more time mixed with easier Qs in one block

A 24 year old woman comes to your clinic complaining of increased frequency of urination, along with dysuria and some pelvic pain with voiding. She has just returned from her honeymoon in Mexico and wonders if it could have been something she ate or drank. Acute Cystitis: 1. Most common pathogen in: > Catheterized patients; most common overall. > Young, sexually active females > Acute urethral syndrome 2. Biggest risk factors

Bugs (know the demographic groups!!!): - E. coli - most common, catheters, sepsis --- E. coli from GI tract can get into female GU tract - Staph Saphrophyticus (Staph Saph) - young, sexually active - Chlamydia - most common cause of acute urethral syndrome. Risk factors: - Female (short urethra) - Catheters (common cause of sepsis) - Schistosoma (blood fluke) can cause SCC of bladder! - Intercourse (urethral trauma) - Diabetes (neurogenic bladder) - Cyclophosphamide (bleeding)

Diagnose: • Hypochloremic, hypokalemic metabolic alkalosis with elevated serum bicarbonate • Mildly elevated serum amylase (increased salivary amylase) • Erosion of tooth enamel, especially lingual surfaces of front teeth • Parotid gland enlargement

Bulimia Nervosa Vomiting makes you lose stomach acid and potassium, so you become hypochloremic, hypokalemic and develop metabolic alkalosis with elevated serum bicarbonate.

Hypersensitivity - Type 2 (ACID mnem)

C = cytotoxic, meaning antibody-mediated toxicity to cells IgM, IgG, complement binding lead to lysis or phagocytosis

Serum from an infant suffering from recurrent otitis media is analyzed by serum protein electrophoresis. The gamma globulin peak is twenty times lower than that of a normal healthy individual. Your most probable conclusion is that: A) The patient has not received childhood vaccinations B) The result is normal because not all people have similar levels of IgG C) The patient may have a B-cell immunodeficiency D) The patient has a defect in antibody affinity maturation E) The patient may have a monoclonal gammopathy

C) The patient may have a B-cell immunodeficiency B-cells make antibodies • This is a very basic question. The information presented tells you that the patient has very low levels of IgG antibodies • Normal variations in IgG levels would not account for this lab result • Infants commonly have recurrent otitis media, so that's not diagnostic in itself • Affinity maturation would not account for the low levels of IgG, since that occurs at a later stage in the antibody response • A monoclonal gammopathy would probably give a spike in IgG levels, although it may be associated with immune deficiency • A primary B cell deficiency is the most likely conclusion

A man with blood pressure of 125/80 mmHg. What is his mean arterial pressure (MAP) in mmHg? A. 80 B. 90 C. 95 D. 100 E. 110 F. 125

C. 95 MAP = ⅓ systolic P + ⅔ diastolic P But the trick is to do Systole - Diastole, divide that by 3, and add that number to the diastole. 125-80 = 45 45/3 = 15 80+15 = 95 MAP will always be closer to diastole than systole because you're counting 2/3 systole.

A 30-year-old woman has experienced myalgias for the past 3 months. On physical examination she has 5/5 motor strength in all extremities. She has dullness to percussion at lung bases. A chest x-ray shows bilateral pleural effusions. Laboratory studies show a positive antinuclear antibody test at a titer of 1:1024. Her serum urea nitrogen is 30 mg/dL (HIGH). A renal biopsy is performed and microscopic examination shows a granular pattern of immunofluorescence staining with antibody to complement component C1q. This pattern is most commonly produced as a consequence of which of the following immunologic mechanisms? A. IgE coating mast cells B. Antiglomerular basement membrane antibody C. Antigen-antibody complexes D. Macrophage release of cytokines E. Release of prostaglandins

C. Antigen-antibody complexes • A granular pattern of glomerular immunofluorescence staining is typical of immune complex deposition that occurs with a type III hypersensitivity reaction. This patient's findings suggest SLE. NOT: • Anti-GBM would be a feature of Goodpasture syndrome, which produces a linear pattern of immunofluorescence from antibodies attacking basement membranes in lungs and kidneys. Goodpasture syndrome is an example of a Type II hypersensitivity reaction

A patient presents with bilateral weakness and wasting of all of the hand intrinsic muscles. Sensation examination reveals loss of pin and temperature sensation in the arms and hands. Other sensation is normal. Reflex examination demonstrates hyperreflexia in the legs. Which of the following is the most likely cause for these findings? A. Polyneuropathy B. Median and ulnar entrapment neuropathies C. Cervical syringomyelia D. Amyotrophic Lateral Sclerosis E. B12 deficiency

C. Cervical syringomyelia = Cavitation / cyst within the spinal cord that damages surrounding spinal tracts. - This cyst, called a syrinx, can expand and elongate over time, destroying the spinal cord. The damage may result in loss of feeling, paralysis, weakness, and stiffness in the back, shoulders, and extremities. MNEM: Like sticking a syringe down the middle of the spinal cord. • Damage to anterior white commissure of spinothalamic tract "cape-like" deficit in pain and temperature - preserves proprioception and vibration sense (dorsal columns) • Cavitation expansion can compress ventral horn motor neurons - bilateral flaccid paralysis • May involve descending hypothalamic fibers to preganglionic sympathetic neurons of T1-T4 - Horner's syndrome

A 46-year-old woman receives a non-T lymphocyte- depleted, allogeneic bone marrow transplant from a matched, unrelated donor. Immunosuppressive therapy with cyclosporine is started. One month later, she has fever. Cytolytic destruction of the skin, gastrointestinal tract, and liver is seen, with associated dermatitis, enteritis, and hepatitis. Which of the following best explains these findings? (A) C3b deposition (B) Cytomegalovirus infection (C) Graft-versus-host disease (D) Tolerance induction (E) Type I (immediate) hypersensitivity

C. Classic GVHD. The trigger is the BMT followed by widespread cell death (apoptosis). Note that it happens in areas of high cell turnover.

A 25-year-old woman with a history of recurrent pneumonia and chronic sinusitis over the past 5 years has been referred to your office. She reports smoking a pack per day for the past 2 years. A CT scan shows evidence of chronic lung damage and bronchiectasis. She has a history of pneumonia due to Streptococcus pnemoniae and Haemophilus sp. Flow cytometry shows normal CD3+, CD4+ and CD8+ T cell counts, but her CD19+ and CD20+ B cell counts are low. In addition, she has low IgG, IgA, and IgM levels. When pre- and post-immunization titers were measured to diptheria, tetanus, and pneumococcal immunizations, she did not develop a 4-fold rise in titers or generate a protective specific antibody titers. Subcutaneous immunoglobulin replacement results in reduced frequency of sinopulmonary infections. The most likely diagnosis is: A. Hyper-IgM syndrome B. Chronic emphysema C. Common variable immunodeficiency D. HLA class II deficiency E. X-linked agammaglobulinemia

C. Common variable immunodeficiency The low B cell count, the generally low (but not absent) Ig titers and the poor vaccine responses point to a diagnosis of CVID. In reaching this conclusion, it's important that you eliminate other T cell or B cell deficiencies. Also note the age of diagnosis (25 yo): CVID is commonly diagnosed at a much later age than other primary immunodeficiencies. It's tempting to choose XLA as the correct answer, but: the subject is female and XLA is X-linked. a patient with XLA would not reach adulthood without the deficiency being apparent. Most other primary immunodeficiencies are diagnosed at a much earlier age.

An 8-year-old boy is brought by his parents to the pediatrician because of pain and swelling in various joints over the past 48 hours. The symptoms began in his left knee, then moved to his left hip and most recently to his right shoulder. He had a fever and sore throat 3 weeks ago, but otherwise, he has been healthy with no changes in appetite, stools, urination, or activity level.Physical examination reveals an uncomfortable but alert boy with a blood pressure of 110/70 mm Hg, pulse of 95, respiratory rate of 18, and temperature of 38.1°C (100.6°F). Heart rate and rhythm are regular. A 2/6 holosystolic murmur is appreciated at the apex. Lungs are clear to auscultation bilaterally. Several nodules are appreciated under the skin of his elbows. Several erythematous, nonpruritic, nontender annular macules are appreciated on both forearms. Which of the following is the most likely mechanism of this disease? A. Active disseminated bacteremia caused by Streptococcus pyogenes B. Autoimmune reaction induced by prior skin infection due to Staphylococcus aureus C. Cross reactivity between antibodies against foreign and host antigens D. Immune complex deposition E. Reaction from an exotoxin released by Streptococcus pyogenes F. Toxicity from ingestion of preformed exotoxins produced by certain strains of Staphylococcus aureus

C. Cross reactivity between antibodies against foreign and host antigens This patient presents with a holosystolic murmur, skin nodules, rash consistent with erythema marginatum, and migratory polyarthritis. Because of the history of pharyngitis 3 weeks earlier, these symptoms are most consistent with rheumatic fever (RF). The patient's fever and pharyngitis 3 weeks ago were likely due to a group A streptococcus (GAS) infection. RF manifests after infection with GAS. GAS carries a virulence factor called M protein. Antibodies targeting M protein also cross-react with self-antigens, a concept called molecular mimicry. Specifically, the antibodies against M protein target the structural proteins of the heart and central nervous system, clinically manifesting as a new murmur and Sydenham chorea, respectively. The diagnosis of RF is often made by using the Jones criteria, which requires evidence of a prior S. pyogenes infection and two of the following five major manifestations: - Migratory polyarthritis of large joints - Carditis - Subcutaneous nodules located under the skin overlying bony prominences - Erythema marginatum of the skin (a circular erythematous lesion that spreads out from its center), shown in the image - Sydenham chorea (involuntary, purposeless movements).

A patient's arterial blood gas sample showed that O2 saturation was reduced compared with a sample taken from a normal, healthy individual despite a normal PaO2. Which of the following most likely accounts for this shift of the patient's O2 -hemoglobin dissociation curve? A. Decreased 2,3-bisphosphoglycerate B. Decreased PCO2 C. Decreased pH D. Decreased temperature E. Hereditary persistence of fetal hemoglobin

C. Decreased pH - More acidic - Hgb is less saturated because it's unloading more O2. - Acidic conditions promote O2 unloading, which would explain the decreased O2 Saturation of Hgb. O2 saturation decreased, PaO2 normal, must be right shift. All other choices produce left shift. O2 saturation = how much O2 is bound to Hgb. PaO2 = how much O2 is in the blood.

A 77-year-old man comes to the physician because of swelling of his legs and feet for 6 months. He has a 40-year history of alcoholism and a 5-year history of hepatic disease. Physical examination shows ascites and a 2+ edema of the lower extremities. Which of the following most likely promotes edema formation in this patient? A. Decreased capillary permeability B. Decreased interstitial hydrostatic pressure C. Decreased plasma colloid oncotic pressure D. Increased capillary hydrostatic pressure E. Increased capillary permeability F. Increased plasma colloid oncotic pressure During a study of renal glomeruli, a healthy animal kidney is kept in a vascular bath preparation at a constant afferent arterial pressure of 100 mm Hg. If the efferent arteriole is constricted with a vascular clamp, which of the following Starling force changes is most likely in the glomeruli? A. Decreased filtration coefficient (Kf) B. Decreased hydrostatic pressure C. Decreased oncotic pressure D. Increased hydrostatic pressure E. Increased oncotic pressure

C. Decreased plasma colloid oncotic pressure D. Increased hydrostatic pressure

A 45-year-old man presents to a psychiatrist at his wife's prompting. He is a college professor and regularly wins accolades for his well-organized and articulate lectures. In the past 3 months, he has become convinced that his wife is having an affair with a co-worker, despite her protests to the contrary. His wife recently discovered that he hired a private investigator to track her whereabouts. He is very defensive when the counselor questions his suspicions about his wife. Mental status examination reveals a well-dressed, middle-aged man without hallucinations or other mood disturbances. He has no psychiatric history. His speech is normal and he displays an appropriate range of affect. Which of the following is the most likely diagnosis? A. Antisocial personality disorder B. Borderline personality disorder C. Delusional disorder D. Narcissistic personality disorder E. Paranoid personality disorder F. Schizophreniform disorder G. Schizotypal personality disorder

C. Delusional disorder It sounds like it would be paranoid personality disorder, but this guy is just paranoid about one thing and it's not affecting his daily life. He's fixated on one single belief that has no evidence yet he cannot be convinced otherwise (= Delusion). Not schizophrenia or schizophreniform because he is well-organized, a college professor, no hallucinations, etc. USMLE-Rx Explanation: This patient's 3-month history of a persistent false belief that his wife is having an affair is suggestive of delusional disorder. Delusional disorder is diagnosed following 1 month of nonbizarre delusions (persistent, fixed, false beliefs) that are usually focused around a particular topic, in this case, the fidelity of the patient's wife. The delusions are not attributable to another psychiatric disorder such as schizophrenia. Delusional disorder does not markedly impair the person's functioning in daily activities, and the ramifications are limited to the delusional content. Given that this patient is a well-respected college professor who appears well-dressed with no psychiatric disturbance, it is apparent that his daily function is not significantly impaired by his delusion.

A 2-day-old infant presents with a generalized seizure due to hypocalcemia. On physical examination, you note that he has a cleft lip and palate, low set, posteriorly rotated ears, and a systolic cardiac murmur. PA and lateral chest X-rays reveal an absent thymic shadow. The infant is very lymphopenic with both the absolute lymphocyte count and CD3 count less than the age-adjusted normals and no naïve T-cells were detected. What is the most likely diagnosis? A. X-linked severe combined immune deficiency (SCID) B. X-linked lymphoproliferative disease C. DiGeorge syndrome D. Congenital HIV infection E. Wiskott-Aldrich syndrome

C. DiGeorge syndrome The lack of thymic shadow, the cardiac murmur, the hypocalcemia, and the palatal abnormalities are all characteristic of DiGeorge syndrome. Defective: - Thymus - Parathyroid (resulting in hypocalcemia) - Heart One of the CATCH-22 diseases: A/w chrom. 22 - Cleft palate - Abnormal facies - T-cell deficiencydue to thymic aplasia - Cardiac abnormalities - Hypocalcemia, due to parathyroid aplasia

27-year-old primigravid woman has been seen regularly for prenatal care and has had an uneventful pregnancy. At 28 weeks' gestation, she complains of a malodorous vaginal discharge. Bacterial vaginosis is diagnosed and she is treated with antibiotics. At 30 weeks' gestation, she goes into labor, and despite the use of tocolytics, she delivers the baby the next day. The neonate develops tachycardia, nasal flaring, and grunting, and requires intubation. A chest x-ray is shown. Which of the following is most likely decreased in the alveoli of this newborn? A. Arachidonic acid B. Corticosteroids C. Dipalmitoylphosphatidylcholine D. α-Ketoglutarate E. Phenylalanine

C. Dipalmitoylphosphatidylcholine (AKA surfactant) 30 Weeks, not enough surfactant => Respiratory Distress Syndrome. Lecithin:Sphingomyelin (L:S) ratio will be low (<1.5). Supplementation with O2 can cause Retinopathy of prematurity, Intraventricular hemorrhage, and Bronchopulmonary dysplasia (RIB). Alveolar collapse => "ground-glass" appearance of lung fields

A 10-year-old boy is brought to the pediatrician for the first time by his parents. The parents report that he has had frequent nosebleeds, bleeding gums when he brushes his teeth, and frequent sinus and skin infections. He says he often has episodes where he "feels dizzy" and his "chest feels funny." On examination, the child appears pale and is found to have small and wide thumbs, small eyes, and pigmented spots on his abdomen. Laboratory results are as follows: Erythrocyte count: 3.5 x 10^6/mm3 (LOW) Hemoglobin: 9.7 (LOW) Hematocrit: 31% (LOW) Leukocyte count: 2.5 x 10^3/mm3 (LOW) Platelet count: 70 x 10^3/mm3 (LOW) What congenital disorder is most consistent with this patient's presentation? A. Diamond-Blackfan anemia B. Edwards syndrome C. Fanconi anemia D. Hereditary spherocytosis E. Neurofibromatosis type 1

C. Fanconi anemia = Aplastic anemia (pancytopenia) with small eyes, wide thumbs, and cafe-au-lait spots. - so anemia + eye and thumb abnormalities + cafe-au-lait spots = Fanconi Anemia. This preadolescent patient presents with signs and symptoms of pancytopenia, or aplastic anemia, which occurs when a person experiences a decrease in all three blood cell types: red blood cells, white blood cells, and platelets. His "dizzy spells" and the "funny feeling" in his chest are presyncope and palpitations caused by the anemia. His bleeding gums and nosebleeds are a symptom of thrombocytopenia (platelets), while his frequent infections suggest leukocytopenia (WBCs). The combination of pancytopenia, abnormal skin (pigmented cafe-au-lait spots), skeletal (wide thumbs), and facial findings (small eyes) suggest Fanconi anemia. Fanconi anemia is the most common cause of hereditary aplastic anemia and is an important cause of acute myeloid leukemia (AML; excess immature white blood cells; interferes with blood cells and causes pancytopenia) in children. It is characterized by defective DNA repair that results in increased chromosomal breakage, rearrangements, and deletions. Patients with this condition are particularly sensitive to alkylating agents. Other characteristics of Fanconi anemia include predisposition to malignancy, and physical abnormalities, including short stature, microcephaly, developmental delay, and café-au-lait skin lesions. Radial ray abnormalities result in abnormal or absent radii or thumbs. Kidney malformations, hypogonadism, microcephaly, and high fetal hemoglobin concentration (relative to normal) are often seen as well. WRONG ANSWERS: Diamond-Blackfan anemia is a congenital anemia caused by RBC aplasia. It's characterized by progressive normochromic, macrocytic anemia with no other significant cytopenias. Edwards syndrome is a trisomy of chromosome 18 and is associated with a number of anomalies, including rocker-bottom feet, intellectual disability, clenched hands at birth, and heart defects. Hereditary spherocytosis is the most common hemolytic anemia; due to a RBC membrane defect. Clinical presentation includes anemia, jaundice, and splenomegaly with a positive family history of hemolytic anemia (hence, "hereditary"). Neurofibromatosis type 1 (NF1) is an autosomal dominant neurocutaneous syndrome characterized by café-au-lait macules, axillary and/or inguinal freckling, Lisch nodules (iris hamartomas), osseous dysplasias, and neurofibromas (a tumor on a nerve that forms soft bumps on or under the skin).

A 21-year-old woman presents to her male family practitioner complaining of sleep deprivation and severe depression. When asked about her sleeping habits in greater depth, she reports sleeping for 7-9 hours per night, but states that it is "just not right." On further questioning, she claims to not have any problems with her personal life and is excited about her recent promotion at work. She is wearing a lot of makeup and a large hat, and she asks the physician to comment on her new orange dress. Throughout the course of the visit, the patient becomes progressively more animated and begins making sexual advances toward the physician and the staff. She mentions how upset she is that the staff isn't paying any attention to her in the waiting room. She reports that her mood has been good, but not overly elevated, expansive, or elated. She tells the physician, "You are my best friend. You understand my problems so well." Which of the following is the most likely diagnosis? A. Bipolar disorder type I B. Borderline personality disorder C. Histrionic personality disorder D. Narcissistic personality disorder E. Obsessive-compulsive personality disorder F. Schizotypal personality disorder

C. Histrionic personality disorder Histrionic personality disorder is one of the cluster B personality disorders. It manifests early in adulthood as excessive emotionality, and attention-seeking and sexually provocative behavior. Not narcissism - The patient in this vignette does not show grandiosity or arrogant behavior. Patients with narcissistic personality disorder have a grandiose sense of self-importance and are preoccupied with fantasies of unlimited success, power, brilliance, beauty, or ideal love.

A biotechnology company is attempting to develop a new rapid-acting reversal agent for anaphylactic shock. The company's goal is to prevent the reaction from occurring by stopping the cascade of reactions that leads to the symptoms of anaphylactic shock. Multiple targets are considered for investigation. Which of the following molecules is the most promising target for blocking this pathway? A. C5b B. Hageman factor C. Immunoglobulin surface receptors D. Interferon-γ E. Interleukin-1 F. Nitric oxide

C. Immunoglobulin surface receptors Anaphylactic shock occurs due to cross-linking of the IgE receptors present on mast cells and basophils; these mast cells and basophils release histamines, serotonins, and other compounds stored in their granules. These compounds lead to vasodilation and leaky capillaries, which can result in the life-threatening condition of anaphylactic shock. Therefore the company should consider finding a way to prevent the initial cross-linking of the IgE receptors. If a surface receptor of IgE is blocked, the cross-linking cannot occur, and the anaphylactic reaction can be stopped.Symptoms of anaphylactic shock include itchy skin, difficulty breathing as airways swell, dizziness, fainting, and nausea or diarrhea.The other answer choices are not related to the cascade seen in anaphylactic shock. NOT: Interferon-γ is important for immunity against viruses and bacteria and, among many functions, acts to activate macrophages. C5b is a component of the membrane attack complex, which is an important part of the complement system. Hageman factor is the name for a form of factor XII that can activate the intrinsic pathway of the coagulation cascade. Interleukin-1 causes fever, which is not seen in anaphylaxis. Nitric oxide is usually an important mediator of septic shock but is not the primary mediator involved in the mechanism of anaphylaxis.

A 48-year-old man is experiencing marked pain in his left first metatarsophalangeal joint. On examination this joint is swollen and markedly tender. Joint aspiration yields cloudy fluid that on microscopic examination has many neutrophils and needle-shaped crystals. His acute response is most likely mediated through crystal-induced NOD-like receptor signaling of which of the following? A. Antigen-antibody complex B. Complement membrane attack complex C. Inflammasome D. Mast cell E. Toll-like receptor

C. Inflammasome The disease is acute gouty arthritis, and the needle-shaped crystals are sodium urate. NOD-like receptors can recognize many substances and they signal inflammasomes to activate caspase, which cleaves IL-1 to an active form, promoting inflammation. - Complement can be part of the inflammatory process, but not the membrane attack complex - Toll-like receptors are on macrophages, dendritic cells, etc. and recognize pathogens.

A 55-year-old man with 15-year history of hypertension that is poorly controlled. Progressively worsening shortness of breath over the last few months. Now he has difficulty walking to his mailbox. Current BP 160/100. S4. Bilateral crackles. Ankle edema. Increased amplitude of QRS complex with some ST- and T- abnormalities on ECG. Echo shows normal ejection fraction and left ventricular thickening. What is most likely causing his dyspnea? A. Aortic stenosis B. Hypertrophic cardiomyopathy C. Left ventricular diastolic dysfunction D. Pulmonary hypertension E. Viral myocarditis

C. Left ventricular diastolic dysfunction Hypertrophic cardiomyopathy is NOT the same as LVH. Hypertrophic cardiomyopathy is when the intraventricular septum enlarges. Also, its histology has those white disarrays in it.

Repro Question 14 A 3-year-old girl presented with hirsutism, breast enlargement, and a height and bone age that was consistent with an age of 9. Diagnostic testing revealed precocious puberty. Which of the following is the most appropriate drug for treatment of this patient's precocious puberty? A. Atosiban B. Follitropin C. Leuprolide D. Octreotide O. Pegvisomant

C. Leuprolide In precocious puberty, the hypothalamic-pituitary-gonadal axis becomes prematurely active for reasons that are not understood. Treatment involves suppressing gonadotropin secretion with continuous administration of a GnRH agonist such as leuprolide. This will decrease the amount of testosterone.

A healthy young runner is running in a marathon. Besides the huge increase in cardiac output, what is the most important factor in the increased blood flow to his/her leg muscles? A. Alpha1-mediated constriction of other arteries B. Beta2-mediated dilation of muscle arteries C. Local metabolite-induced dilation

C. Local metabolite-induced dilation Local Metabolites released during exercise ="CHALK" CO2 H+ Adenosine - Vasodilation Lactate K+ - Def. not Alpha 1, which slows HR. - Not Beta 2. Beta 2 works with normal SNS at rest.

A 30-year-old male biology graduate student was dismissed from his PhD program after 8 years because he was not able to produce a thesis, claiming that his data was never exactly how he wanted it. He would spend weeks planning out a simple experiment, since everything had to be just right. For many experiments, he would start over because he felt he went out of order in adding the reagents to his media for his cells. He has had similar problems in his undergraduate courses, often failing to complete his assignments on time because he had to revise them until they were perfect. Which of the following disorders does this patient potentially suffer from? A. Schizoid personality disorder B. Narcissistic personality disorder C. Obsessive compulsive personality disorder D. Paranoid personality disorder E. Obsessive compulsive disorder

C. Obsessive Compulsive Personality Disorder - Very perfectionistic, to the point where it's slowing him and others down. He seems to have no insight that there's anything wrong with him. This male has OCPD due to his preoccupation with order, perfectionism, and control. Cluster C personality disorders are characterized by being worried, anxious, and fearful. They are often associated with anxiety disorders and they all have an element of anxiety, just in different forms. Why is it not OCD? Obsessive compulsive disorder is characterized by ritualistic behavior and abnormal behaviors that are often completed to relieve anxiety related to some underlying cause in the patient's life. This can be excessive hand washing or excoriation disorder. However, they have insight -- they are aware that these behaviors are odd. People with OCPD lack insight; they don't think there's anything wrong with them. OCPD is typically egosyntonic (i.e., they're cool with being like that; syncs with their ego), whereas OCD is egodystonic (i.e., their behaviors aren't in line with how they want to be).

In allogeneic solid organ transplantation, acute rejection is mediated by: A. Pre-existing antibodies to HLA antigens B. Activation of a Type I Hypersensitivity reaction C. Primary T cell response to HLA antigens D. Antibodies specific for RhD antigens E. Platelet activation and microvascular thrombosis

C. Primary T cell response to HLA antigens • Acute rejection (rejection in the first few weeks to months) is typically triggered by a primary T cell response. •Antibodies may also be produced, but pre-existing antibodies would cause hyperacute rejection. Platelet activation and thrombosis may contribute to the pathology of chronic rejection, but they're not initiating events. Type I hypersensitivity or Rhesus-specific antibodies are not involved.

A 24-year-old man is brought to your emergency department under arrest by the local police. The patient was found naked at a busy intersection jumping up and down on top of a car. Interviewing the patient, you discover that he has not slept in 2 days because he does not feel tired. He reports hearing voices. The patient was previously hospitalized 1 year ago with auditory hallucinations, paranoia, and a normal mood. What is the most likely diagnosis? A. Schizophrenia B. Schizotypal disorder C. Schizoaffective disorder D. Bipolar disorder E. Unipolar mania

C. Schizoaffective disorder This patient is suffering from schizoaffective disorder. Schizoaffective disorder is characterized by symptoms of schizophrenia plus mood changes such as mania or depression. Importantly, a diagnosis of schizoaffective disorder requires that the patient experience psychotic symptoms WITHOUT mood disorder for at least 2 weeks. The patient's history is consistent with this requirement. In this case, the patient is hearing voices, consistent with schizophrenia, and is experiencing a manic episode with the classic symptoms of public nudity and decreased need for sleep. Patients who consistently experience mood symptoms with symptoms of schizophrenia should receive a diagnosis of bipolar disorder with psychotic features.

A 55-year-old man presents to his primary care physician with the complaint of increasing muscle weakness, especially in his legs, over the past 3 weeks. He also admits to some difficulty in swallowing, and at times, he finds himself having difficulty remaining alert and keeping his eyes open. He states that his legs feel less weak as he walks for a short distance. He has a 100-pack-year history of smoking. What type of malignant disease is most likely responsible for this patient's symptoms? A. Breast carcinoma B. Renal cell carcinoma C. Small cell lung carcinoma D. Squamous cell lung carcinoma E. Thyroid carcinoma

C. Small cell lung carcinoma - Lambert Eaton is similar to myesthenia gravis (see differences below), but LEMS is pretty much always caused by small cell lung carcinoma. This cancer is also associated with SIADH. This patient presents with muscle weakness (esp. in legs and face) that improves with use. He has weakness of the facial muscles (in this case, the eyelids). These symptoms are typical of Lambert-Eaton myasthenic syndrome, a paraneoplastic syndrome usually found in patients with underlying small cell lung cancer. This syndrome is distinct from, but similar in presentation to, myasthenia gravis. The mechanism involves antibodies to presynaptic calcium channels at the neuromuscular junction. Unlike myasthenia gravis, LEMS symptoms often start in the limbs. The legs will often feel heavy. It can also affect the arms and sometimes affects facial muscles. In contrast to myasthenia gravis, the symptoms of LEMS do not improve with administration of anticholinesterase agents (because the problem is the pre-synaptic Ca2+ channels), but should improve with treatment of the underlying cancer. Common symptoms include proximal muscle weakness and sometimes facial muscle weakness that diminish with use. Ocular symptoms such as ptosis and extraocular muscle weakness can occur in both Lambert-Eaton myasthenic syndrome and Myasthenia Gravis. Ocular symptoms are more commonly seen in myasthenia gravis. Breast carcinoma, Renal Cell carcinoma, and Squamous cell lung carcinoma are associated with hypercalcemia. Medullary thyroid cancer is associated with Cushing syndrome.

A 56-year-old man is hospitalized in the neurology intensive care unit after suffering a stroke. The physicians explain to his wife that he has experienced a thrombosis in the basilar artery, causing complete occlusion of the vessel. She asks for information regarding his prognosis. Which of the following statements best describes this patient's current condition? A. The patient cannot hear or comprehend anything around him B. The patient has become blind and will not regain his sight C. The patient is fully conscious but permanently paralyzed D. The patient most likely will regain movement in his extremities E. The patient will no longer have normal sleep-wake cycles

C. The patient is fully conscious but permanently paralyzed This patient suffers from basilar artery thrombosis, which results in "locked-in syndrome," a rare neurologic disorder characterized by complete paralysis of voluntary muscles except for those that control vertical eye movements and blinking. The condition is caused by infarct or other injury to the ventral pons, affecting the corticospinal and corticobulbar tracts, bilaterally. In locked-in syndrome, the patient's consciousness remains fully intact; he can think and reason, but can only communicate through blinking and/or vertical eye movements. Because the lesion that causes this syndrome is confined to the basilar pons, all sensory pathways and the reticular activating system of patients with locked-in syndrome are spared; therefore, patients are fully conscious and are able to feel, see, hear, and understand everything in their environment. However, since the descending corticospinal and corticobulbar tracts—which carry motor commands to the body and face, respectively—pass through this area, the patient is immobile and "locked in" by the paralysis. The center for vertical gaze and its connections with the oculomotor and trochlear nuclei (all of which are located in the midbrain) are spared, permitting the patient to communicate by moving his eyes up and down to indicate "yes" or "no." Corticobulbar fibers that supply facial lower motor neurons that innervate the orbicularis oculi muscle travel through the brainstem tegmentum (not through the basis pons like other corticobulbar fibers) and are also spared. This allows the patient to communicate by blinking in Morse code.

Epstein-Barr virus infects/enters __-cells (lymphocytes) by binding to CD___ on their surfaces. (MNEMONIC)

CD21 on B-cells MNEMONIC: You must be 21 to drink Beer in a Barr - Epstein-Barr virus infects B-cells by binding to CD21. E.g., Mono would therefore involve CD21. Epstein-Barr virus infects normal B-cells by binding to CD21, a complement receptor, which allows it to enter the B-cell. So EBV infects resting lymphocytes (actually attacks immune cells).

Ectopic Pregnancy: - 3 common causes (which one is most common?) - Usually occurs in what part of the fallopian tube? - How does it present? - Complications - how do people die from them?

Causes: - PID (most common) - Salpingitis Isthmica Nodosum (SIN=tics) - Endometriosis Most commonly occurs in the ampulla of the fallopian tube Pts present with abdominal pain, late or missing periods, periodic spotting / non-mentrual bleeding, abdominal tenderness to palpation, hemorrhage / shock. Often, you can actually palpate the mass in the lower abdomen. May rupture and cause pelvic hemorrhage, shock and death

Central vs. Nephrogenic Diabetes Insipidus - Water Deprivation Test: What type is it if this fixes the problem vs. not fixing the problem. - Which type would Desmopressin work for? - Both will present the same: with ___ and ___

Central DI is due to a lack of the hormone vasopressin (antidiuretic hormone). Nephrogenic DI (NDI) occurs when the kidneys do not respond properly to vasopressin. Both present with intense thirst / polydipsia and excessive urination / polyuria. Water deprivation: - Central DI if this does not decrease urinary frequency and concentration. Can be tx'd with Desmopressin. - Nephrogenic DI if water deprivation does fix it. Impaired renal response to ADH, thus they will not respond to Desmopressin.

+++++ "This needs to be instilled in your mind" +++++ If an infant (~12-15 months old) has signs of iron deficiency anemia, what is the first thing you should ask about in terms of diet? (Note: This can also be a problem in toddlers; but it's a major factor in infants developing Fe deficiency anemia) (i.e., what dietary factor in children usually leads to iron deficiency anemia?)

Cow's Milk! Infants whose diets consist of a lot of or mostly cow's milk are likely to develop ireon deficiency anemia. This is why you should not/do not feed infants milk -- they drink breast milk or formula.

Repro Question 1 A 42-yo female presents to the emergency room complaining that her heart is racing. She has noticed a 10 lb weight loss over the last 2 weeks along with frequent diarrhea. Her vital signs are T 98.6 F, BP 126/78, HR104 and RR14. She appears anxious but there are no murmurs, rubs or gallops on cardiac auscultation. Breath sounds are clear. The patient's abdomen is soft, non-tender and non-distended. She denies sick contacts or ingesting any new foods. She does not have any rashes and her skin appears moist. The pituitary hormone implicated in this patient's diagnosis shares a common subunit with a hormone that has which of the following features? A. Inhibits gonadotrophin releasing hormone B. Inhibits growth hormone C. Is derived from the neuroectoderm D. Acts through the cAMP signaling pathway E. Stimulates secretion of the zona reticularis

D. Acts through the cAMP signaling pathway This patient's clinical picture is hyperthyroidism. TSH shares a common alpha subunit with LH, FSH and hCG. They differ only in their beta subunits. (MNEM: all 3 of those are reproductive hormones). Each of these hormones binds to their respective receptors (beta-subunit specific) to activate the cAMP signaling pathway to have their biologic response. The gonadotropin hormones, human chorionic gonadotropin (hCG), luteinizing hormone (LH), follicle-stimulating hormone (FSH), and thyroid-stimulating hormone (TSH) are heterodimers consisting of alpha and beta subunits that are associated non-covalently. The alpha subunits of these four human glycoprotein hormones are identical; however, their beta chains are unique and confer biological specificity.

You are working in the emergency department when a 19-year-old girl is brought in by her family for severe abdominal pain and "talking out of her head". While you are evaluating her she has a 30 second seizure that stops spontaneously. Labs indicate that the CBC is normal and the electrolytes are normal except for hyponatremia (low serum sodium). You start IV dextrose and saline and send a spot urine of porphobilinogen (PBG), which returns as markedly elevated. What is the most appropriate action to take at this time? A. Send a porphyrin DNA panel and await the results before adding additional therapy B. Send urine, serum, and stool porphyrins and wait on the results before adding additional therapy C. Avoid treating her pain since pain medications may worsen the severity of the attack D. Begin therapy with hemin and continue this daily for a total of 4 days E. Begin therapy with chloroquine

D. Begin therapy with hemin and continue this daily for a total of 4 days This is Acute Intermittent Porphyria - Abdominal pain - Psychiatric sxs - Seizure AIP is treated with Hemin (ALA Synthase Inhibitor). Also treated with IV glucose and saline. Also treated with pain meds for abdominal pain (so not C) Spot urine showing high porphobilinogen is enough for diagnosis and you should begin treatment immediately.

A 42-year-old man has lost 7 kg over the past two years. During that time he has had pain and swelling around the small joints of his hands and feet. He has had a series of upper respiratory tract infections, with organisms including Staphylococcus aureus, Streptococcus pneumoniae, and Pseudomonas aeruginosa cultured. He has a chronic diarrhea. A stool specimen is found to have Giardia lamblia cysts. Laboratory studies show a hemoglobin of 8.8 g/dL with a reticulocyte count of 3.2%. His serum urea nitrogen is 17 mg/dL with creatinine 1.0 mg/dL. His antinuclear antibody test is negative. Quantitative serum immunoglobulins show IgA 70 mg/dL (LOW), IgG 303 mg/dL (LOW), and IgM 64 mg/dL (LOW). By flow cytometry there are normal numbers of T cells and slightly reduced numbers of B cells. Which of the following conditions is most likely to be present in this man? A. HIV infection B. Polymyositis C. Bruton agammaglobulinemia D. Common variable immunodeficiency E. Diabetes mellitus

D. Common variable immunodeficiency • He has common variable immunodeficiency (CVID), with hypogammaglobulinemia that predisposes him to bacterial infections and also Giardia infection. Patients with CVID can have rheumatoid arthritis and hemolytic anemia. Most patients have normal to slightly reduced numbers of B lymphocytes that cannot differentiate into immunoglobulin-secreting plasma cells. • Persons with diabetes mellitus are prone to bacterial infections, but they do not get Giardia diarrhea or hemolytic anemia or arthritis

A 12-year-old boy is brought to the physician because of a skin abscess. The patient's mother states that the patient has had several abscesses and Aspergillus infections during the course of his childhood. The area around the abscess is indurated. Wound cultures are obtained during incision and drainage of the abscess. The cultures grow Staphylococcus aureus. Which of the following best explains this patient's repeated infections? A. C5 deficiency B. Defective chemotactic response C. IgA deficiency D. Defective NADPH oxidase E. Thymic hypoplasia

D. Defective NADPH oxidase A history of recurrent infection with Staphylococcus aureus (or strep pneumo) and Aspergillus species (fungal pneumonia) suggests a diagnosis of chronic granulomatous disease (CGD). Deficiency of NADPH means ROSs can't be eliminated as there is no oxidative burst of phagocytes after they've consumed pathogens. Deficiency of NADPH oxidase: • X-linked (most common) or autosomal recessive. • Susceptible to catalase-positive infections, particularly those caused by S. aureus, Aspergillus, Nocardia, Salmonella, and Candida. • Phagocytes require the enzyme NADPH oxidase to produce superoxide radicals to kill ingested bacteria. • Without NADPH oxidase, phagocytes rely on hydrogen peroxide (H2O2) to kill microbes. • H2O2 is used by myeloperoxidase to oxidize chloride into hypochlorous acid (HOCl), which is toxic to bacteria. • CGD diagnosis: Nitroblue tetrazolium (NBT) dye test.

Repro Question 3 A 29-yo woman presents with an inability to become pregnant. She has been menstruating every other month and has a milky discharge from her nipples. Lab test reveals a prolactin level of 334 ng/ml and a subsequent MRI shows the presence of a pituitary adenoma measuring 0.5 cm in diameter. Which of the following therapeutic option is most appropriate at this time? A. Transsphenoidal pituitary surgery B. Radiation therapy C. No treatment D. Dopamine agonist E. Tricyclic antidepressant

D. Dopamine agonist The patient has prolactinoma. The first treatment of choice in patients with prolactin-secreting macroadenomas (<1 cm) who desire fertility is dopamine agonist. Bromocriptine and Cabergoline are dopamine agonists used in the management of prolactinoma. If she is not interested in childbearing, then treatment is not necessary unless the adenoma reached >3 cm in diameter.

A 35-year-old woman comes to the clinic because of progressive weakness of the shoulders and legs for the past 3 months. She has been having difficulty combing her hair, buttoning her blouses, walking upstairs, and lifting objects. She especially has great difficulty in getting up from the seated position. She does not notice anything that improves or worsens her weakness, it does not seem to change throughout the day or with repeated efforts, and she has no pain. She does not take any medications. Her mother has systemic lupus erythematosus. Vital signs are within normal limits. Physical examination shows symmetrical weakness of the proximal muscles in the both upper and lower extremities. There are no rashes, lesions, or other remarkable findings. Which of the following is the most likely cause of this patient's condition? A. Anti-U1 RNP (ribonucleoprotein) antibodies B. Autoantibodies to postsynaptic ACh receptor C. Autoantibodies to presynaptic Ca2+ channel D. Endomysial inflammatory infiltration E. Endoneurial inflammatory infiltration F. Periarticular inflammation G. Perimysial inflammatory infiltration

D. Endomysial inflammatory infiltration The patient is a 35-year-old woman with signs of proximal muscle weakness (difficulty standing from a seated position, raising arms above head, etc.). Her physical examination is significant only for weakness with no signs of a rash. Therefore, the most likely diagnosis is polymyositis. Polymyositis is an autoimmune disorder that has progressive onset of proximal muscle weakness without pain. It is commonly associated with other autoimmune diseases. It is caused by endomysial infiltration with CD8+ T cells and is associated with overexpression of major histocompatibility complex (MHC) class I, which allows the muscle to become susceptible to destruction. Workup for polymyositis shows increased creatinine kinase enzymes and positive for antinuclear antibodies (ANA), anti-Jo-1, anti-SRP, and anti-Mi-2 antibodies. Results of a muscle biopsy would show inflammation, patchy necrosis, and regeneration and fibrosis of muscle fibers. Unlike dermatomyositis, there is no cutaneous involvement. NOT: Perimysial inflammatory infiltration occurs in patients with dermatomyositis, which is a myopathy that involves proximal muscle weakness and a malar rash. Infiltration is with CD4+ T cells. Unlike polymyositis, there is cutaneous involvement in dermatomyositis. Polymyalgia rheumatica is characterized by proximal muscle stiffness associated with severe pain, fever, and weight loss. It is an inflammatory disorder which involves periarticular inflammation, especially of the shoulders. Endoneurial inflammatory infiltration is a demyelinating polyneuropathy associated with ascending weakness and decreased reflexes that occurs 2-4 weeks after a respiratory or gastrointestinal illness. This is seen in patients with Guillain-Barré syndrome. Anti-U1 RNP antibodies are suggestive of mixed connective tissue disease (MCTD), a disorder that involves polymyositis, systemic lupus erythematosus, and systemic sclerosis. Patients may have muscle weakness, joint pain, or Raynaud's phenomenon. Proximal muscle weakness in Lambert-Eaton myasthenic syndrome is caused by autoantibodies to presynaptic Ca2+ channel that lead to insufficient muscle contractions. Continued use of the limbs will relieve muscle weakness. Autoantibodies to postsynaptic ACh receptor describes the cause of myasthenia gravis, which presents as muscle weakness that worsens with use.

Repro Question 13 A 52-year-old postmenopausal patient has evidence of low bone mineral density. She and her physician are considering therapy with raloxifene OR a combination of conjugated estrogens and medroxyprogesterone acetate. Which of the following patient characteristics is most likely to lead them to select raloxifene? A. Previous hysterectomy B. Recurrent vaginitis C. Rheumatoid arthritis D. Family history of breast cancer E. Troublesome hot flashes

D. Family history of breast cancer Estrogen excess can increase the risk of breast cancer. Raloxifene is a SERM, which is an anti-estrogen in breast tissue, but it will prevent bone resorption.

A 4-year-old boy is brought to the clinic for an annual check-up. He had cardiac surgery at 6 months of age and has been followed up by a pediatric cardiologist since that time. The boy has also had recurrent infections with opportunistic pathogens, including an episode of Pneumocystis jirovecii pneumonia. Physical examination reveals a unilateral cleft lip and hard palate on the left. Which serum electrolyte abnormality is most likely to be found in this patient? A. Hypercalcemia B. Hyperkalemia C. Hypernatremia D. Hypocalcemia E. Hypokalemia F. Hyponatremia

D. Hypocalcemia This child has a history of a cardiac surgery as an infant, recurrent infections, and a cleft lip and palate, which are all consistent with DiGeorge syndrome. DiGeorge syndrome is most often caused by a microdeletion at chromosome 22q11, resulting in the loss of a critical region responsible for the development of the third and fourth pharyngeal pouches. These embryonic structures give rise to the thymus and parathyroid glands. DiGeorge syndrome specifically includes thymic, parathyroid, and cardiac defects. This patient's recurrent infections indicate a deficiency of T cells, which are normally produced by the thymus, and his cardiac surgery was likely performed to repair a congenital defect, such as a ventricular septal defect. Patients with absent or defective parathyroid glands would be expected to have hypocalcemia, which can be severe and even cause tetanic seizures. The common findings in the 22q11 syndromes can be remembered by using the mnemonic CATCH-22, which stands for Cleft palate, Abnormal facies, Thymic aplasia, Cardiac defects, Hypocalcemia, and deletion of chromosome 22.

A 65-year-old female is transferred from her nursing home to the hospital in respiratory distress. A portable chest x-ray demonstrates heavy shadowing of the right middle and left lower lobes. A Gram stain of her sputum shows many lance-shaped, gram-positive diplococci. Arterial blood gases reveal a PO2 of 52 mm Hg. Which of the following likely accounts for this patient's hypoxia? A. Decreased surface area of alveolar-capillary membranes B. Decreased PO2 of inspired air C. Hypoventilation of central origin D. Hypoventilation of peripheral origin E. Inequalities of ventilation and perfusion

D. Hypoventilation of peripheral origin From x-ray and sputum, most likely pneumococcal pneumonia, causing V/Q mismatch and physiological shunt or absolute shunt if consolidated. Both absolute and physiologic shunt cause elevated A-a difference and low PaO2.

An otherwise healthy 26-year-old man presents with petechiae on her legs during the last 24 hours, epistaxis, and gingival bleeding. He reports his symptoms began one day prior to presentation and has never occurred before. He denies starting any new medications. Social history is notable for current intravenous drug use. On physical exam, there is blood in the nares and mild bleeding of the gums. A peripheral blood smear shows normal red cell morphology. A bone marrow smear shows mature megakaryocytic hyperplasia. Laboratory studies show: Hemoglobin 13.1 g/dL (NORMAL) Hematocrit 39.7% (NORMAL) Leukocyte count 8500/mm3 Neutrophils 65% (a little HIGH) Lymphocytes 30% (NORMAL) Monocytes 5% (NORMAL) Mean corpuscular volume 82.2 μm3 (NORMAL) Platelet count 20,000/mm3 (LOW) ^ i.e., all normal except for low platelets Which of the following is the most likely diagnosis? (A) Acute megakaryocytic leukemia (B) Acute myelogenous leukemia (C) Aplastic anemia (D) Immune thrombocytopenic purpura (E) Epstein-Barr viral infection (F) Papovavirus infection (G) Thrombotic thrombocytopenic purpura

D. ITP The most common cause of isolated low plt count in a healthy person is always ITP! Hence, Immune THROMBOCYTOPENIC Purpura. Low platelets, due to immune destruction. ITP = immune-mediated destruction of otherwise normal platelets Megakaryocyte Hyperplasia because megakaryocytes are trying to make more platelets because they keep getting destroyed. (Megakaryocytes make platelets) So look for normal or high megakaryocytes, yet isolated low platelets. That means platelets are being destroyed. Pathogenesis: IgG antibodies directed against platelet membrane glycoproteins such as GPIIb/IIIa (a type II hypersensitivity reaction) The normal mature, but increased megakaryocytes in the bone marrow rules out leukemia.

A pressure-volume curves shows a very low TLC (total lung capacity) in a 56-year-old patient who presented to his primary care physician with shortness of breath and a cough. Physical exam shows diffuse crackles at the lung bases. Which of the following conditions best accounts for the low TLC observed in the patient? A. Alpha-1-antitrypsin deficiency B. Asthma C. Bronchospasm D. Idiopathic pulmonary fibrosis E. Old age

D. Idiopathic pulmonary fibrosis TLC is basically lung compliance. IPF is Restrictive. Compliance (= ΔV/ΔP) would decrease. Lung is stiffer than normal. A - A1AT deficiency would increase Compliance / TLC B - Asthma affects resistance, not compliance C - Definitely not E - Would Increase Compliance

In an animal experiment, sympathetic nerves to the heart are externally stimulated and an increase in coronary blood flow is observed. Assuming coronary perfusion pressure does not change, which is the most likely mechanism for this increase in coronary blood flow? A. Alpha1 receptor-mediated constriction in other arteries B. Beta2 receptor-mediated dilation of coronary arteries C. Decrease in ventricular stiffness D. Increase in myocardial metabolism E. Muscarinic receptor-mediated dilation of coronary arteries

D. Increase in myocardial metabolism What's released when SNS is activated? NE. - NE does not work on arteries, so not Beta 2 (B). Heart works hard and needs a lot of metabolic resources to keep going. Sympathetic activation would increase blood flow to the heart via NE. Then the heart releases more metabolites to help with vasodilation, like nitrous oxide.

A Decrease in which of the following is the main mechanism of angina relief by sublingual nitroglycerin? A. Coronary artery resistance B. Extravascular fluid volume C. Heart rate D. LV end diastolic volume E. Peripheral artery resistance

D. LV end diastolic volume = preload Angina is caused by increased O2 demand by the heart. Preload (= LVEDV) is decreased by nitroglycerin (nitrate = venodilator). More room for blood in the veins. Less oxygen demand so less angina. Oxygen can be diverted more towards heart than oxygenating blood. By diminishing preload, nitroglycerin decreases left ventricular end diastolic volume and pressure and myocardial wall tension, thus reducing myocardial oxygen consumption / demand. By reducing the O2 demand, angina is relieved.

A 34-year-old woman is brought to the emergency department by ambulance because of dizziness, confusion, and fatigue. She has no history of significant illness, but reports 2 to 3 similar episodes in the past few months. On evaluation today, the patient appears anxious. Pulse is 120/min. Physical examination discloses and a mild tremor in both hands. Blood glucose level is 45 mg/dL. The patient reports normal food intake during that day, having had lunch just a few hours prior to the episode. She is accompanied by her husband, who has type 1 diabetes mellitus requiring daily insulin injections. On further questioning, he reports he has caught her stealing his insulin pens on more than one occasion. Which of the following laboratory values would confirm the diagnosis of factitious hypoglycemia? A. High glucagon level B. High insulin level C. High proinsulin level D. Low C-peptide level

D. Low C-peptide level Factitious Disorder (AKA Munchausen Syndrome) - Intentionally faking an illness for no clear reason, or possibly just because they crave the attention from the doctor. But they know they are lying. • Done to yourself or another ("by proxy" - e.g., child) • Often leave AMA (against medical advice) when confronted • NBME likes hypoglycemia but may also be wounds/infection/stones • For hypoglycemia, check for increased serum insulin/C-peptide ratio (>1.0) during a hypoglycemic episode to see if they are injecting themselves with insulin to look like they're hypoglycemic. NOTE: Factitious disorder is different from malingering, which looks identical except malingering is doing it for some sort of personal gain (e.g., getting out of jail, getting out of school, financial gains, getting drugs, etc.)

A 12-year-old girl is brought to the physician for a follow-up examination because of aplastic anemia. Physical examination shows no abnormalities except for pallor. Laboratory studies show hemoglobin of 6.0 g/dL and hematocrit of 24%. Three months ago, her hemoglobin was 6.2 g/dL and hematocrit was 25%. Which of the following is most likely to be decreased in this patient during resting conditions? A. Arterial PO2 B. 2,3-Bisphosphoglycerate C. Cardiac output D. Mixed venous PO2 E. Red blood cell H+ concentration

D. Mixed venous PO2 Due to more oxygen unloading in tissues; tissues are using up as much as they can. Lower arterial O2 content, lower venous O2 content, lower venous PO2 NOT: A- Will be normal if vent and gas exchange normal B- Increased in anemia C- Increased in anemia E- RBC H+ concentration higher because of increased 2,3-bisphosphoglycerate 2,3-BPG and blood acidity will be higher to promote O2 unloading to tissues.

A 37-year-old man presents to your clinic with shortness of breath and lower extremity edema. He was born in Southeast Asia and emigrated to America ten years ago. Physical exam reveals 2+ pitting edema to the level of his knees, ascites, and bibasilar crackles, as well as a late diastolic murmur best heard at the cardiac apex. The patient undergoes a right heart catheterization that demonstrates a pulmonary capillary wedge pressure (PCWP) of 24mmHg (HIGH). The patient is most likely to have which of the following? A. Increased pulmonary vascular compliance B. Decreased pulmonary artery systolic pressure C. Increased LVEDP D. Normal or decreased LVEDP E. Decreased trans-mitral gradient

D. Normal or decreased LVEDP This is clearly Mitral Stenosis Pt has diastolic HF / right HF / HFpEF The answer cannot be C because systole / EF is preserved in right heart failure. This patient's problem is that his heart can't fill. Stenosis Rule: ↓P distal, ↑P proximal (adaptive*), ↑ change in P across a stenosis So pressure before Mitral Valve will be high (so high LAP, which is c/w high PCWP). Pressure after Mitral Valve will be low (so low LVP). PCWP = LAP -- Pressure in Left Atrium and Pulmonary Capillaries essentially the same.

A 32-year-old severely obese male non-smoker with no significant past medical history is brought to the emergency department in respiratory distress. After 2 hours in the waiting room, he becomes confused and somnolent. Initial vital signs are temperature 37.5°C (99.5°F), heart rate 105/min, respiratory rate 25/min, and blood pressure 154/102 mm Hg. A stat arterial blood gas on room air shows his PaO2 is 65 mm Hg and PaCO2 75 mm Hg. A chest radiograph shows basilar atelectasis with minimal symmetric diaphragmatic elevation and without pneumonia, pneumothorax, or pleural effusion. CT of the chest without contrast shows no lung abnormalities, but does indicate right ventricular enlargement. ECG shows sinus tachycardia. Labs are remarkable for elevated D-dimer, elevated red blood cells, and elevated serum bicarbonate. Which of the following is the most likely diagnosis? A. Chronic obstructive pulmonary disease B. Diaphragmatic paralysis C. Interstitial lung disease D. Obesity hypoventilation syndrome E. Pulmonary embolism

D. Obesity hypoventilation syndrome Young, obese patient, non-smoker; hypoventilating (elevated PaCO2), normal A-a gradient, enlarged RV, pulmonary hypertension from hypoxia. NOT: A- CT shows no abnormalities expected of either emphysema or chronic bronchitis. B- Symmetric diaphragm movement, no history suggesting phrenic nerve damage. C- CT would be abnormal. E- Elevated D-dimer is not conclusive for PE. Elevated RBC, bicarb indicate chronic condition.

Amyloidosis. What would you most likely hear? A. Fixed S2 split B. Paradoxical S2 split C. S3 D. S4

D. S4 Amyloidosis would thicken the heart. This is restrictive, very much like Hypertrophy. Remember the MNEM: 4H -- hypertrophy will produce an S4 sound.

A 24-year-old man sustained a stab wound to the right shoulder 3 months ago. Since then, he has been profoundly weak in right deltoid, triceps, brachioradialis, finger extensors, supination, and wrist extensors. He is numb on the lateral upper arm, and on a narrow strip down the lateral arm to the dorsal hand. This lesion localizes best to: A. The proximal radial nerve B. The lateral cord of the brachial plexus C. The middle trunk of the brachial plexus D. The posterior cord of the brachial plexus E. The superior trunk of the brachial plexus

D. The posterior cord of the brachial plexus With the extent of weaknesses, the lesion is probably further up in the brachial plexus. A lesion to the posterior cord would impair the axillary and radial nerves. Radial nerve damage causes Wrist Drop (can't extend wrist). Axillary damage makes abduction of arms past 15 degrees impossible.

A 12-year-old girl with generalized aching pain in both knees two weeks ago. Pain moved to her right shoulder and now in both her wrists. She was sick with a sore throat a month ago but recovered without medical attention. There is redness of skin with pink rings on the torso and inner surfaces of the limbs. Which is the most likely late complication of this condition? A. A continuous crescendo-decrescendo murmur best heard at P B. A friction rub heard throughout the precordium C. A harsh crescendo-decrescendo early systolic murmur heard best at A with radiation to neck D. An S4 gallop heard at the apex E. An opening snap with a diastolic rumble at M

E. An opening snap with a diastolic rumble at M This is Rheumatic Fever. Meets JONES criteria. Early, there's mitral regurg (holosystolic). Late, there's Mitral Stenosis (opening snap)

The second pregnancy for a 23-year-old woman appears uncomplicated until ultrasound performed at 19 weeks shows hydrops fetalis. The fetal organ development is consistent for 19 weeks, and no congenital anomalies are noted. Her first pregnancy was uncomplicated and resulted in the birth of a normal girl at term. The current pregnancy ends with birth of a baby boy at 32 weeks gestation. On examination the baby has marked jaundice. Laboratory studies show a hemoglobin of 7.5 g/dL, and neonatal exchange transfusion is performed. Which of the following immunologic mechanisms best explains this infant's findings? A. Anti-receptor antibody B. Loss of self tolerance C. Immune complex formation D. Delayed type hypersensitivity E. Complement mediated cell destruction

E. Complement mediated cell destruction The baby has hydrops fetalis: Severe edema in the fetus and newborn. - Caused by fluid leaving the bloodstream and entering tissues - Immune hydrops caused by maternal destruction of fetal red blood cells (hence, low hemoglobin) by complement-mediated lysis - Form of Type II hypersensitivity reaction -- involves complement.

During a transfusion in a 4 year old boy with leukemia, he develops fever to 38.3°C and chills. The transfusion is stopped and testing does not reveal intravascular hemolysis. He is given acetaminophen and antihistamine and the transfusion is restarted. He does not have any further fever or other reactions. What was the most likely cause of the fever? A. ABO incompatibility B. Infected blood product C. Minor Rh antigen mismatch D. Transfusion acquired graft vs host disease E. Cytokine release

E. Cytokine release Febrile Non-Hemolytic Transfusion Reaction If there's a minor fever after a blood transfusion, no need to panic. It's because the WBCs can sometimes release cytokines during storage, which causes a mild immune response during transfusion.

Repro Question 10 A 37-yo female is interested in oral contraception. She is newly married but she has 2 children from a previous relationship and does not desire any additional children from her new husband. She drinks alcohol intermittently and smokes a pack of cigarettes a day. This patient is at risk for which of the following complications if she starts oral contraceptives? A. Hypertension B. Endometrial carcinoma C. Trichomoniasis D. Unwanted pregnancy E. Deep venous thromboembolism

E. Deep venous thromboembolism Woman over the age of 35 who smoke and use oral contraceptives are at increased risk of deep venous thromboembolism. Oral contraceptives should be avoided in this patient population or the patient should be counseled to stop smoking completely prior to starting OCP.

A 76-year-old man is being treated with digoxin, captopril, and furosemide for systolic heart failure and hypertension. Today he presents to the emergency department with dizziness and fatigue. An ECG recording indicates the presence of sinus bradycardia and intermittent third-degree atrioventricular conduction block. Which of the following is the most likely explanation for this patient's heart rhythm disturbance? A. Captopril overdose B. Calcium overload caused by digoxin C. Hyperkalemia caused by furosemide D. Hypokalemia caused by furosemide E. Increased vagal tone caused by digoxin F. Nonadherence to prescribed medications

E. Increased vagal tone caused by digoxin Digoxin has two distinct actions: 1. Blockade of Na-K ATPase at nodes to reduce firing rate 2. Activation of vagal tone (i.e., PSNS). - The side effects of this include sinus bradycardia and AV conduction block.

A healthy 25-year-old man with no known cardiovascular disease participates in an exercise study where multiple cardiovascular parameters are measured. After running lightly on a treadmill for 10 minutes, which of the following is most likely decreased? A. Blood flow to the brain B. Blood flow to the heart C. Blood pressure D. Left ventricular end diastolic pressure E. PR interval on ECG

E. PR interval on ECG The AV node causes the heart to contract, so on an ECG, the AV node fires in the P to R region. Since the heart is beating faster, the AV node is firing faster and everything is closer together. So the PR interval on ECG will be shorter. Not D because blood is filling the LV faster after each contraction.

Repro Question 2 During an infertility evaluation, a 38-yo female undergoes a blood test on day 19 of her normal 28-day cycle. If ovulation has occurred, relatively high levels of which hormone will be detected in her serum? A. Follicle-stimulating hormone B. Gonadotrophin-releasing hormone C. Prolactin D. Testosterone E. Progesterone

E. Progesterone During ovulation, the corpus luteum is producing progesterone. It is the dominant hormone during the luteal phase (second half) of the menstrual cycle. It inhibits the pituitary secretion of FSH and LH. Estrogen is also somewhat elevated in this part of the cycle, but it's highest in the first half. Progesterone is the classic hormone of the luteal phase. FSH and LH (and therefore GnRH) would be high in the first half of the cycle. The LH surge induces ovulation.

An otherwise healthy 34-year-old man has a history of petty theft. He has stolen lunches, loose change, and office supplies from his coworkers over the past 3 years. Today, he cannot find his phone. He quickly and loudly accuses his coworker of stealing it. It is later discovered that the man had misplaced his phone in the bathroom. When he gets home, he complains to his wife that his phone disappeared because "someone at work is a dirty liar." Which of the following defense mechanisms best explains this patient's behavior? A. Acting out B. Denial C. Displacement D. Passive aggression E. Projection F. Rationalization G. Reaction formation H. Sublimation

E. Projection Projection is defined as the attribution of unacceptable internal feelings to an external source. A classic example is the individual with adulterous desires who accuses his/her partner of infidelity. Basically, projection is usually blaming someone else / villainizing another person.

A 28-year-old woman comes to the clinic to establish care as a new patient. She reports a 1-year history of generalized abdominal pain, fatigue, and headaches. She also reports that she often worries about her symptoms so much that she has difficulty sleeping. The patient's mother has rheumatoid arthritis and an aunt has systemic lupus erythematosus. The patient has no nausea, vomiting, diarrhea, constipation, fever, chills, weight loss, chest pain, or shortness of breath. Her menstrual cycles are regular. She reports having seen multiple doctors over the past year and recalls having "negative blood work." CT scan of the abdomen showed no abnormalities. She takes no medications. Vital signs are within normal limits. Heart rate is regular, and the lungs are clear bilaterally on auscultation. The abdomen is soft and nontender without masses; there is no rebound or guarding. Range of motion is full in all extremities. Which of the following is the most appropriate next step? A. Abdominal x-ray study B. Admit for hospitalization C. Referral for gastroenterology consultation D. Referral for psychiatry consultation E. Regular outpatient visits with same provider F. Reassure her that it's all in her head

E. Regular outpatient visits with same provider This is Somatic Symptom Disorder. The best way to handle that is to let her keep coming to you for checkups. This patient is a 28-year-old woman with a 1-year history of increasing anxiety regarding her vague reports of generalized abdominal pain, fatigue, and headaches. Laboratory tests and imaging have been negative in the setting of a normal physical examination, which makes the diagnosis of somatic symptom disorder most likely. Somatic Symptom Disorder: • Somatic symptoms: 1 or more, distressing, & disrupting of daily life • Often mentions that all tests were negative. • Excessive thoughts, feelings, behaviors with at least 2 of the following symptoms: ✔ Disproportionate & persistent thoughts about the seriousness of the symptom(s). ✔ Persistently high anxiety about symptoms (but won't accept anxiety as the diagnosis). ✔ Excessive time, thought, & energy devoted to the symptoms or healthcare (but that's also in Illness Anxiety DO so not helpful in distinguishing).

A 55-year-old woman is bothered by dry mouth and eye discomfort with blurry vision for 2 years. On examination she has xerophthalmia and xerostomia. There is bilateral diffuse painless parotid gland enlargement. A year later she develops bilateral swelling, warmth, and pain on movement of her hands. Which of the following autoantibodies is most likely to be detected in this woman? A. Centromere B. Double stranded DNA C. Histone D. RNP E. SS-A

E. SS-A SS-A is characteristic for Sjogren Syndrome involving salivary glands and lacrimal glands. The presence of one autoimmune condition increases the risk for others, and she likely has rheumatoid arthritis. NOT: Anti-RNP is more characteristic of mixed connective tissue disease Double stranded DNA antibodies would be more characteristic of SLE, and histone antibodies may be seen with drug-induced SLE Anti-centromeric Ab is associated with limited systemic sclerosis (CREST)

A 15-year-old girl with a history of moderate persistent asthma is brought to the physician because of progressively worsening asthma that requires daily therapy with a rescue inhaler despite the use of twice daily inhaled fluticasone. Treatment is subsequently begun with a drug that will produce a prolonged increase in cAMP levels for prophylaxis of her symptoms. Which of the following drugs was most likely added to her therapeutic regimen? A. Albuterol B. Ipratropium C. Metaproterenol D. Nedocromil E. Salmeterol F. Zafirlukast

E. Salmeterol - LABA A- SABA B- M3 Cholinergic blocker C- SABA D- Not bronchodilator; mast cell inhibitor F- Leukotriene receptor LTD4 antagonist

++++++++++++++++++++++++++ A 10-year-old boy from Finland comes to the physician saying that he has been having diarrhea and increased flatulence. He notes that his stools smell foul, and sometimes they seem to float. He reports seeing no blood on the toilet paper or in the toilet bowl. He also describes feeling more tired and weak lately. On physical examination, he appears pale and thin. On cardiac auscultation, the patient has a regular rate and rhythm; pulse is 92/min. The abdomen is mildly tender to palpation in all four quadrants. Lungs are clear to auscultation. A CBC reveals the following data: Leukocyte count: 6000/mm3 Hemoglobin: 9.0 g/dL (LOW) Hematocrit: 27% (LOW) RBC count: 2000/mm3 RBC distribution width index: 20.2% Mean corpuscular volume: 73 μm3 (LOW) Ferritin: 27 ng/mL (LOW) Platelet count: 212,000/mm3 This condition is associated with which of the following findings? A. Ascending weakness B. Frequent bloody stools C. Hyperpigmented skin lesions D. Painful tender nodules on the shins E. Small clusters of fluid-filled papules and vesicles

E. Small clusters of fluid-filled papules and vesicles This is Celiac Disease. This patient presents with increased frequency of stools that are not bloody but are foul smelling. Foul-smelling stools are commonly due to malabsorption or infection. Furthermore, the patient is presenting with iron deficiency anemia (low mean corpuscular volume, low ferritin, high RBC distribution width in addition to low hemoglobin/hematocrit). These findings are consistent with celiac disease (i.e., gluten-sensitive enteropathy), which tends to occur in individuals of Northern European descent. An intestinal biopsy specimen, as shown in the image provided with the question, reveals villous atrophy / blunting, crypt hyperplasia, increased intraepithelial cells, and lymphocytic infiltration into the lamina propria. Celiac disease is associated with fat malabsorption and subsequently vitamin B12 deficiency. It is also associated with decreased iron and folate absorption due to damage to the small intestine. Of the possible types of anemia, iron deficiency anemia (microcytic) unresponsive to iron therapy is the most common. Celiac disease is an autoimmune disorder associated with intolerance to gluten and resulting in malabsorption. Diagnosis is made on the basis of reduction of symptoms on withdrawal of gluten, as well as the presence of antiendomysial and/or antitissue transglutaminase antibodies. Celiac disease is associated with dermatitis herpetiformis (~10% of cases), a skin disorder named because the lesions cluster in a manner similar to those of herpes. It is characterized by clusters of pruritic, papular, and sometimes bullous lesions located symmetrically on the trunk and extensor surfaces. NOT: Ascending weakness, associated with Guillain-Barré syndrome (GBS), is not likely in this patient as GBS typically follows an antecedent gastrointestinal or genitourinary tract infection. Frequent bloody stools, associated with an inflammatory bowel disease such as ulcerative colitis, are less likely because the patient has not noted any blood in his bowel movements. Hyperpigmented lesions are associated with Whipple disease, which would present with more systemic signs affecting the joints and nervous system in an older patient. Painful tender nodules on the shins describes erythema nodosum, a disorder classically associated with inflammatory bowel disease that is not likely in a patient who does not report bloody stools.

Repro Question 16 Regarding the placental transfer of drugs, what is the correct statement? A. The fetal plasma is slightly more acidic than the mothers, so ionic trapping of acidic drugs occurs. B. The placenta is an absolute barrier to drugs C. The extent of plasma binding and degree of ionization of acids and bases do not affect the transfer of drugs across the placenta. D. P-glycoprotein and other efflux transporters are not present in the placenta. E. Some tocolytics cross the placenta and may have adverse effects of the fetus.

E. Some tocolytics cross the placenta and may have adverse effects of the fetus.

A 3-year-old boy is brought to the emergency department with recent onset fever and chest pain. The symptoms began after a recent episode of aspiration 3 days ago while the patient was drinking water in bed. His mother reports he was lying supine at the time of the choking event. Chest x-ray reveals dense consolidation in one of the lobes of the lung. In which of the following bronchopulmonary segments did the aspiration event most likely occur? A. Apical segment of the left upper lobe B. Apical segment of the right upper lobe C. Medial segment of the right middle lobe D. Posterior basal segment of the left lower lobe E. Superior segment of the right lower lobe

E. Superior segment of the right lower lobe Right main bronchus is wider and less angled from trachea. Superior segment exits from posterior wall of lobar bronchus, so aspirated material in supine person most likely here rather than in medial segment of right middle lobe (C).

A newborn is born with Apgar scores of 8 and 9. Bottle feedings are begun and the newborn quickly vomits the formula. A nasogastric tube is placed and resistance is felt after several centimeters. A developmental defect of which of the following is most likely responsible for this finding? A. Bronchial buds B. Buccopharyngeal membrane C. Laryngotracheal diverticulum D. Rathke's pouch E. Tracheoesophageal septum

E. Tracheoesophageal septum - Separates foregut from trachea This is Esophageal atresia with distal tracheoesophageal fistula. Proximal atresia produces immediate vomiting after feeding. Air in stomach from distal fistula.

A 36-year-old man presents with fever, anorexia, abdominal pain, weight loss, and arthritis that began several weeks ago. Medical history includes an episode of hepatitis B. Some cutaneous ulcerations and a purplish discoloration with some ulceration are evident on the lower extremities. Laboratory results are negative for antinuclear, anti-double-stranded DNA and antiphospholipid antibodies. Other laboratory findings include: Sodium: 138 mEq/L Potassium: 4.9 mEq/L Chloride: 96 mEq/L Bicarbonate: 24 mEq/L Creatinine: 3.2 mg/dL BUN: 29 mg/dL Glucose: 109 mg/dL Calcium: 9.2 mg/dL Which of the following is another characteristic feature of the most likely diagnosis for this patient? A. Association with polymyalgia rheumatica B. Heart murmur C. Necrotizing granulomas in the lung and upper airway D. Same stage of inflammation in different vessels E. Transmural inflammation of blood vessels

E. Transmural inflammation of blood vessels This patient presents with fever, anorexia, abdominal pain, weight loss, and arthritis. With a history of hepatitis B, this presentation is most suggestive of polyarteritis nodosa. The disease is a vasculitis of the small- to medium-sized arteries. Signs and symptoms are nonspecific, and result from damage to the organs that are fed by the affected vessels. Musculoskeletal, cutaneous, gastrointestinal, cardiac, renal, and neurologic symptoms can be seen. Polyarteritis nodosa is associated with hepatitis B virus infection in up to 30% of cases. Patients often have a skin rash known as livedo reticularis, which is the purplish discoloration described in this patient. Additionally, patients may have anemia, elevated erythrocyte sedimentation rate or C-reactive protein level, and neutrophilic leukocytosis. Histologically, transmural necrotizing inflammation is seen in blood vessels and spares the pulmonary system. This inflammation may progress to fibrosis and thickening of vessel walls (see image). Treatment of polyarteritis nodosa consists of corticosteroids and cyclophosphamide.

A 48-year-old man has had a chronic cough with fever for 2 months. On physical examination his temperature is 37.9°C. A chest radiograph reveals a diffuse bilateral reticulonodular pattern. A transbronchial biopsy is performed and microscopic examination shows focal areas of inflammation containing epithelioid macrophages, Langhans giant cells, and lymphocytes. These findings are most typical for which of the following immunologic responses? A. Type I hypersensitivity B. Type II hypersensitivity C. Graft versus host disease D. Polyclonal B cell activation E. Type IV hypersensitivity

E. Type IV hypersensitivity Granuloma formation with reactivation or reinfection tuberculosis in an adult is a classic type IV hypersensitivity reaction Langhans giant cells are large cells found in granulomatous conditions.

A 72-year-old man collapses while playing golf. He has a 5-year history of angina and type 2 diabetes mellitus. Paramedics arrive in 10 minutes. Examination shows no respirations or blood pressure; an ECG shows asystole. Cardiopulmonary resuscitation is attempted for 10 minutes without success. Which of the following is the most likely cause of death in this patient? A. Cardiac tamponade B. Embolus to the right middle cerebral artery C. Necrosis of the myocardium D. Rupture of the papillary muscle E. Ventricular fibrillation

E. Ventricular fibrillation Kills very quickly Asystole -- no systole -- only ventricles are contracting

A 14-year-old boy comes with his parents to the family physician and reports muscle weakness that has been affecting his ability to play soccer. Physical examination reveals gross enlargement of his calf muscles, and a biopsy specimen reveals active fiber degeneration and significant presence of fat and connective tissue. The boy's family history is positive for similar symptoms in a maternal uncle, whose condition progressed to a wheelchair-bound state by the age of 35 years. The boy's parents are both healthy, as are his two older sisters. Which of the following is the pattern of genetic inheritance of this patient's disorder? A. Autosomal dominant B. Autosomal recessive C. Mitochondrial inheritance D. X-linked dominant E. X-linked recessive

E. X-linked recessive Based on the patient's reports of muscle weakness in adolescence and calf muscle hypertrophy with fiber degeneration, the diagnosis is most likely Becker muscular dystrophy. Becker muscular dystrophy is an X-linked recessive disorder caused by a partially functioning dystrophin gene, resulting in muscular weakness and replacement of muscle with fatty tissue. That's why his maternal uncle had it, but not his mom. His father would be normal and his sisters would not have been affected because they have another X chromosome to balance out the X-linked recessive mutation. Mother to son transmission. The distribution of weakness can be similar to that of Duchenne muscular dystrophy. However, Duchenne muscular dystrophy presents in a younger age group, and its symptoms are more severe and progress much faster than those of Becker muscular dystrophy. The remaining modes of inheritance—autosomal dominant, autosomal recessive, mitochondrial inheritance, and X-linked dominant—do not explain the inheritance pattern of Becker muscular dystrophy.

A 20-year-old man is brought to the emergency department by his friends after becoming very agitated at a party. His friends report that the patient threatened to harm his girlfriend when he saw her speaking privately with another man at the party. He subsequently cursed at other party goers who attempted to calm him down. The friends do not know what caused the patient's change in behavior, but they did see him having several beers while at the party. In the waiting room of the emergency department, he is belligerent and uncooperative. His temperature is 99.5°F (37.5°C), pulse is 120/min, respirations are 16/min, and blood pressure is 156/90 mm Hg. A physical examination reveals horizontal nystagmus and hyperacusis. His pupil size is normal, and there is no conjunctival injection. Which of the following substances is most likely responsible for this patient's clinical presentation? A. Alcohol B. Amphetamines C. Cocaine D. Lysergic acid diethylamide E. Morphine F. Phencyclidine G. Phenobarbital

F. Phencyclidine A presentation of aggression, hypertension, tachycardia, mild fever, horizontal nystagmus, and hyperacusis (ie, increased sensitivity to certain frequencies and volumes) strongly suggests that this patient has taken phencyclidine (PCP). PCP is an illicit dissociative anesthetic that acts as an antagonist at N-methyl-d-aspartate (NMDA) receptors and as an agonist at α-receptors; it also prevents the reuptake of dopamine, norepinephrine, and serotonin within the central nervous system (CNS). Signs of PCP use often reflect its sympathomimetic effects and include belligerence, impulsiveness, psychosis, fever, psychomotor agitation, nystagmus, tachycardia, hypertension, and ataxia. Both horizontal and vertical nystagmus can be seen in patients with PCP intoxication, but vertical nystagmus is the more classic presentation. First Aid: PCP Violence, impulsivity, psychomotor agitation, nystagmus, tachycardia, hypertension, analgesia, psychosis, delirium, seizures. Trauma is most common complication.

A 60-year-old woman is brought to the emergency department 20 minutes after being involved in a motor vehicle collision. She sustained several lacerations on her face and hands. Two days later, she develops a temperature of 39°C (102.2°F) and the facial injuries become hot, red, and swollen. Physical examination shows anterior cervical lymphadenopathy, increasing edema and erythema of the facial wounds, a petechial rash in the same area, and hypotension. The patient's fingertips are also darkly discolored and cool to the touch. Cultures of the wounds grow catalase-negative, beta hemolytic, gram-positive cocci. or A 23-year-old woman presents to the emergency department with fever, headache, vomiting, rash, and confusion. She was in her usual state of health approximately two days prior to presentation. She is currently in menses and uses tampons. Her temperature of 38.9°C (102.0°F), blood pressure is 88/58 mmHg, pulse is 110/min, and respirations are 22/min. On physical exam, there is diffuse macular erythroderma and vaginal hyperemia. Which of the following types of immune effector cells have most likely been stimulated in this patient? A. B cells B. Cytotoxic T cells C. Eosinophils D. Natural killer cells E. Plasma cells F. T-helper cells

F. T-helper cells This is Toxic Shock Syndrome. - often related to tampon use - superantigens directly link MHC II cells to CD4 helper T cells - this causes massive release of cytokines. - also releases dangerous amounts of IL-1, 2, TNF-alpha and beta, and IFN-gamma, leading to systemic toxicity. • Caused by a response to superantigens produced by two gram-positive cocci, Streptococcus pyogenes (a group A Streptococcus) and Staphylococcus aureus. The pathogen in this case is likely S. pyogenes, as the culture reveals a catalase-negative pathogen. • S. pyogenes produces the superantigen exotoxin A. S. aureus produces toxic shock syndrome toxin (TSST-1). • Superantigens initiate release of dangerous amounts of interleukin (IL)-1, IL-6, and tumor necrosis factor (TNF)-alpha from macrophages and interferon (IFN)-gamma from T-helper (TH) cells • Bacterial superantigens crosslink the variable-beta region of the T-cell receptor with major histocompatibility complex (MHC) class II molecules on antigen-presenting cells. This stimulates large numbers of CD4 helper T cells in the absence of antigen specificity

FISH and Microarrays - good at detecting ___ but not very good for ___

FISH and microarray tests are very helpful in identifying duplications / deletions of specific loci. But won't detect small changes, point mutations, etc.

Diagnose: A 62-year-old woman presents to her primary care physician due to an unremitting headache. She says she has been having this headache for the last few days and is nonresponsive to ibuprofen. Her headache is left-sided and affects the temporal region of her head. She notes to having jaw pain when chewing food or brushing her teeth. On physical exam, there is tenderness to palpation of the left temporal artery. or A 69-year-old woman presents to her primary care physician complaining of a new-onset, left-sided, throbbing headache that has lasted for several days. She reports that the pain is worsened by eating. Which of the following is a complication of this condition if not treated promptly? A. Skin nodules B. Intestinal ischemia C. Monocular blindness D. Myocardial infarction E. Ischemic stroke HIGH YIELD: Proper treatment regimen is to give them ___ BEFORE the biopsy

Giant cell temporal arteritis / Temporal Arteritis - a large artery vasculitis High Yield Features: - Caused by INTIMAL THICKENING and fragmentation of the lamina. - ESR is elevated - Polymyalgia rheumatica - Jaw pain worse with chewing - Vision problems (due to ophthalmic artery) Usually presents with a one-sided headache that is worse with chewing. Caused by arteritis of a temporal artery. Inflammatory labs show very high ESR C. Monocular blindness - Most common manifestation; due to decreased perfusion of the head. HIGH YIELD: First give steroids (IMMEDIATELY) Then do a biopsy The reason for giving steroids right away is to prevent blindness in Temporal Arteritis. MNEM: Giant Cell Arteritis -- Give Corticosteroids Ammediately - and Gaw Claudication Always

Hemolytic Anemia is a result of what enzyme deficiency? What two histological findings are classic for this condition? - ___ cells and ___ bodies

Glucose 6-Phosphate Dehydrogenase deficiency => Hemolytic Anemia G6PD is part of the PPP. Takes some of the G6P from glycolysis and shunts it into the PPP to form ribose-5-phosphate used to make DNA and RNA and regenerate NADPH. Histology: Bite cells Heinz bodies

Von Gierke Disease - One of the ___ storage diseases - Enzyme deficiency - Sxs

Glycogen storage disease. Can't make glycogen. - Von Gierke disease (type I), Pompe disease (type II), Cori disease (type III), McArdle disease (type V); Very Poor Carbohydrate Metabolism Lack of glucose-6-phosphatase. Presentation: Sxs result from lack of glycogen • Chronic, severe hypoglycemia - liver cannot release stored glucose - body must rely on fat/protein catabolism for energy • Hepatomegaly • Hyperlipidemia • Hyperuricemia • Lactic acidosis

Regulatory Enzymes in Glycolysis and Gluconeogenesis - Name the Gluconeogenesis enzyme that does the opposite of each of these glycolysis enzymes: 1. Hexokinase 2. Phosphofructokinase-1 3. Pyruvate kinase (3 Bypass Rxns of Gluconeogenesis)

Glycolysis: Gluconeogenesis (3 Bypass Rxns of Gluconeogenesis) 1. Hexokinase: Glucose 6-phosphatase 2. Phosphofructokinase-1: Fructose 1,6-bisphosphatase 3. Pyruvate kinase: Pyruvate carboxyalse and PEPCK

++++++++++++++++++ Diagnose: A 34 y/o woman presents to the physician's office with complaints of weight loss and sweatiness. She states that despite a ravenous appetite, she has lost 4 pounds this past month. In addition she states that she has been more sweaty lately, and unable to cool down in rooms that others find comfortable. On physical exam you see an anxious and fidgety woman who has a very prominent gaze with protuberant eyes. Her thyroid is diffusely enlarged. She has pre-tibial edema. Histology: Diffuse papillary hyperplasia. Decreased colloid.

Grave's Disease = Stimulatory IgG antibodies to the TSH receptor on the thyroid gland. Release a bunch of thyroid hormones. - Goiter: Diffuse thyroid enlargement - Hyperthyroidism (Grave's is the most common cause) - Infiltrative ophthalmopathy with exopthalmosis (bulging eyes) - Pre-tibial edema

Define these respiratory physiology values: - IRV - TV - ERV - RV - IC - FRC - VC - TLC

IRV - inspiratory reserve vol - air that can still be inhaled after normal inspiration TV - Tidal volume - air movement with normal breath (500 ml) ERV - Expiratory reserve vol - Air that can still be exhaled after normal exhale RV - Residual volume - air left after max exhale (never measured, can't be exhaled) IC - Inspiratory capacity - IRV + TV FRC - Functional reserve capacity - RV + ERV VC - Vital Capacity - TV + IRV + ERV (amount forcefully exhaled after max inhale) TLC - total lung capacity - volume in lungs after max inhale

Ig__ mediates Type I Hypersensitivity reactions (basic allergic reactions) by inducing the release of histamines from mast cells. Ig__ and Ig__ mediate Type II Hypersensitivty reactions, in which they are directed against antigens on cells (such as circulating red blood cells) or extracellular material (such as basement membrane) and kill those cells (cytotoxic).

IgE mediates Type I Hypersensitivity reactions (basic allergic reactions) by inducing the release of histamines from mast cells. IgG and IgM mediate Type II Hypersensitivity reactions. They are directed against antigens on cells (such as circulating red blood cells) or extracellular material (such as basement membrane) and kill those cells (cytotoxic).

Systemic Lupus Erythematosus (SLE) is an autoimmune disease with Ig__ antibodies to various tissues. What type of Hypersensitivity is it?

IgG antibodies to various tissues. Type III Hypersensitivity -- immune complexes form since there is chronic exposure to antigen (self-antigen in this case). Causes Sxs: Glomerulonephritis, Arthritis, characteristic butterfly rash on face.

A 38-year-old man presents to his primary care physician's office for episodes of epistaxis and gingival bleeding. He reports his symptoms began one day prior to presentation and has never occurred before. He denies starting any new medications. Social history is notable for current intravenous drug use. On physical exam, there is blood in the nares and mild bleeding of the gums. There are petechiae distal to where the blood pressure cuff was placed. Abdominal exam is unremarkable. A complete blood count is significant for a platelet count of 28,000/μL and peripheral blood smear demonstrates enlarged platelets. 1. Diagnose 2. Best treatment is ___

Immune Thrombocytopenia Purpura (ITP) Acquired immune-mediated destruction of otherwise normal platelets Tx: Prednisone

What is the most common cause of thrombocytopenia in children and adults and it usually occurs after what? Involves antibody (Ig__) against what?

Immune Thrombocytopenic Purpura (ITP) Happens after viral infections (e.g., HIV, HCV) Autoimmune - hence, "IMMUNE" in the name - IgG antibodies against GpIIb-IIIa receptors - splenic macrophages phagocytose platelets. May be idiopathic or 2° to autoimmune disorders (eg, SLE), viral illness (eg, HIV, HCV), malignancy (eg, CLL), or drug reactions. ITP is also the most common cause in adults... same Ab's invovled

++++++++++++++++++++++++++ Type 3 Hypersensitivity Reaction: AKA Immune Complex Disease: - Common underlying mechanism - 3 main examples, each with slightly different mechanisms

Important feature is that the individual is continuously exposed to an antigen, allowing immune complexes develop, which causes the Type 3 Hypersensitivity reaction. It's very gradual. This can be due to: 1. Administration of a large amount of antigen - Snake Venom can cause a T3H reaction. Gradually forms globs of immune cells. 2. Persistent infection - Antigen stays in the system for a long time and forms immune complexes. - Ex: Bacterial Endocarditis 3. A chronic Autoimmune Disease - Ex: SLE - IgG antibodies form immune complexes. Here, the antigen is host tissues -- IgG against a variety of tissues, and IgG immune complexes form. These cause the symptoms of the disease, such as glomerulonephritis, arthritis, and the characteristic butterfly rash on face.

Clinical presentation of Polymyositis and Dermatomyositis What is the only way they differ in presentation? Which type of T-cell is involved in each type> (MNEM) Treatment

In a Nutshell: > Polymyositis and Dermatomyositis present the same except Dermatomyositis includes a rash. > Distal and Facial muscles and sensation are spared in both. Proximal muscles are weak, but do NOT hurt. - Polymyositis an autoimmune myopathy characterized by symmetric proximal muscle weakness and no rash. - Dermatomyositis is an autoimmune myopathy characterized by symmetric proximal muscle weakness and a rash Muscle Biopsy: - PM: CD8+ T cells endomysial infiltrate - DM: CD4+ T cells perifascicular infiltrate with vasculitis > MNEM: alphabetical order: 4 before 8, D before P. Both: - Manifest as symmetric proximal skeletal muscle weakness (e.g., shoulders, thighs, and neck) - Insidious onset (3-6 months) - SPARES facial muscles and distal muscles and sensation. - Minimal muscle pain -- just weakness. - Dysphagia due to esophageal dysmotility due to involvement of the upper third of esophageal (skeletal muscle). - Raynaud's phenomenon: What Tess has. - Pulmonary: diaphragmatic weakness, interstitial pneumonitis which can cause fibrosis - Cardiac: arrhythmias, cardiomyopathy, CHF Dermatomyositis is the above plus various rashes. Treatment: Steroids E.g., MedBullets for Dermato: A 53-year-old woman with a past medical history of thyroid disease presents to her physician's office for weakness. She reports that she has been feeling weak, has difficulty climbing the stairs, and combing her hair every morning. Today, she also has difficulty arising out of her chair. Physical exam reveals decreased strength, especially in the shoulders. She also has an impressive lilac periorbital rash and a sunburn on her cheeks. She is sent for further laboratory workup and counseled on the likely initiation of high-dose steroids.

Chronic Renal Failure: 1. What are the 2 most common causes? 2. What type of anemia do they have (___cytic) and why? 3. Which vitamin will become deficient? 4. What cardiovascular findings are usually present? 5. What type of renal casts will they have? 6. MAD HUNGER mnemonic for manifestations / consequences of CRF.

In the Name: Kidneys have literally failed, chronically. They don't work anymore, which results in the characteristic group of sxs that are all related to lacking renal function. 1. DM and HTN (both of which are chronic processes) - Whereas NSAIDs and ischemia are common cause of acute renal failure since they are more acute processes. 2. Normocytic, from decreased EPO production. 3. Vitamin D deficiency (leads to renal osteodystrophy, osteomalacia, osteoporosis, easy bone fractures) 4. Hypertension (Na+ and H2O retention - kidneys can't filter it out), CHF, atherosclerosis 5. WAXY casts 6. Consequences (MAD HUNGER): -Metabolic Acidosis - Dyslipidemia (especially triglycerides) - High potassium - Uremia - Na+/H2O retention (HF, pulmonary edema, hypertension) - Growth retardation and developmental delay - EPO deficiency (anemia) (EPO made by kidneys) - Renalosteodystrophy (from low Vit D)

++++++++++++++++++++++++++++++ What is the pathogenesis of Scleroderma? (What happens?) What are the most common symptomatic manifestations of scleroderma?

Inappropriate, extensive fibrosis of tissues, organs, and blood vessels. NOT an inflammatory disorder. Immune system dysfunction may either injure and/or stimulate the fibroblasts and vascular endothelium. Inappropriate fibroblast activation causes collagen production and deposition (scar tissue) which disrupts normal organ architecture and function. Additionally, vascular endothelial cells proliferate within the intima of small vessels (small arteries, arterioles and capillaries) causing luminal narrowing and tissue ischemia. Raynaud's phenomenon (fingers or toes turn blue then red with reperfusion) occurs in >90% of patients with scleroderma. Sclerodactly (crooked fingers from scar tissue on joints) is common too. Ulcers on the fingertips are very common. Dysphagia is very common as the esophagus can become dysmobile. Skin is the most common organ system involved and defines the disease. There are several stages of skin involvement: Earliest- edematous skin Fibrotic stage- thick hard skin Atrophic- skin is tight Limited PSS- affects hands, distal forearm, face Diffuse PSS- entire extremity, trunk, face

Leukocyte Adhesion Deficiency (LAD) is caused by a loss of CD__ on cell surfaces.

Loss of CD18 Failure of leukocytes to adhere to vascular endothelium and migrate to site of infection. Leads to: • Recurrent bacterial infection • Impaired wound healing

Diagnose: A 46-year old woman presents to her primary care physician for a dark spot on the back of her right hand. She states that the spot first appeared about 2 years ago and has slowly been growing. It does not burn, itch, or sting. She has a history of regular suntanning and minimal sunscreen use. On exam, there is a 2-cm, asymmetric, thin, brownish-blue plaque with somewhat ill-defined borders and an irregular pattern of coloration. Dermatopathology

MELANOMA Highly malignant, high metastatic potential, deep invasion.

+++++++++++++++++++++++++ Name the 3 cancers caused by each type of MEN (Multiple Endocrine Neoplasia) mutation: - MEN-1 - MEN-2a - MEN-2b MNEMONIC

MEN: Multiple Endocrine Neoplasia These are the genetic mutations that cause Familial Primary Hyperparathyroidism - So they cause elevated calcium MNEM: MEN-1 to MEN-2a to MEN-2b goes PPP, PPM, PMM (first letters of cancers they cause) ▶ 3 P's of MEN-1: PPP: Tumors in: ٭Pituitary: ↑ GH or Prolactin ("Goliath") ٭Parathyroid: HyperPTism ٭Pancreas: ↑ Insulin, Glucagon, or VIP ▶ MEN-2: PPM ٭Parathyroid: HyperPTism ٭Pheochromocytoma ٭MTC: Medullary Thyroid Cancer ▶ MEN-2b: "Abe Lincoln" - PMM ٭Pheochromocytoma ٭MTC: Medullary Thyroid Cancer ٭Marfanoid habitus The MEN-2's: MEN-2a and MEN-2b: ✔ Mutation on Chromosome 10q ✔ MEN-2 mutations are activated by the RET receptor. - Pheochromocytoma and Marfanoid habitus also involve RET mutations.

+++++++++ HIGH YIELD, EASY POINTS +++++++++ T cells recognize short linear peptides from an antigen, which are presented on the surface by ___? What are superantigens?

MHC - MHC I or MHC II MHC displays a piece of antigen DNA on the surface of infected cells so that T cells will see it and attack that cell. MHC class I molecules are expressed by most cell types. MHC class II molecules are expressed by professional antigen-presenting cells (macrophages, dendritic cells, B cells). Epitopes recognized by T cells are linear, and are processed and presented by MHC class I or class II molecules. - Except for superantigens, which are bacterial antigens that can bind directly to MHC class II and TCR, triggering strong inflammatory responses. Epitope = the part of an antigen molecule to which an immune cell attaches itself.

+++++++++ HIGH YIELD, EASY POINTS +++++++++ MHC class I presents antigens to CD__ T-cells. MHC class II presents antigens to CD__ T-cells. ^ MNEMONIC - Which MHC class presents endogenous antigens? - Which MHC class binds and presents exogenous antigens?

MHC class I presents endogenous antigens to CD8+ T-cells (Cytotoxic T-cells). MHC class II presents exogenous antigens to CD4+ T-cells (Helper T-cells). MNEM: 4 x 2 = 8 x 1 - Or just remember that it's reverse numerical order. MHC I goes with CD8, MHC II goes with CD4. MHC I presents endogenous antigens -- antigens that were made inside of the cell. MHC II binds and presents exogenous antigens -- antigens floating around. Note: MHC I and II are in all cells, but especially important in Antigen-Presenting Cells (APCs) -- makes sense -- cells that present antigens.

Urticaria Pigmentosa: - Appearance of skin - Due to __ cells

Mast Cells

What are the clinical manifestations of a stroke involving each of these major circle of willis vessels? - MCA - ACA - PCA - AICA - PICA - AICA and PICA

Memorize Image Most often, questions will describe sxs and ask where the stroke was. MCA strokes (most common) typically cause motor and sensory deficits in the face/upper extremities, speech deficits (dysarthria and aphasia in dominant, usually left hemisphere strokes, and solely dysarthria in nondominant, usually right hemisphere strokes), or inability to attend to one side of space (nondominant, usually right hemisphere strokes). They affect the basal ganglia, resulting in symptoms of contralateral hemiparesis of both the upper and lower extremity, usually in the absence of sensory symptoms. - MCA supplies motor and sensory cortexes and basal ganglia. Anterior cerebral artery occlusions would show signs of motor and sensory impairment of the lower extremities w/wo urinary incontinence. A posterior cerebral artery stroke would result in contralateral hemianopia with macular sparing. An individual with an anterior inferior cerebellar artery (AICA) occlusion would have facial paralysis, Horner syndrome, vertigo, ataxia, and more. Patients with AICA would likely have the classic "crossed symptoms" associated with brainstem stroke, including ipsilateral loss of pain/temperature sensation on the face (spinal trigeminal nucleus) with contralateral loss of pain/temperature sensation of the rest of the body. An individual with a posterior inferior cerebral artery (PICA) occlusion would also show signs of dysphagia, hoarseness, impaired gag reflex, and possibly hiccups—the so-called "lateral medullary syndrome." Ipsilateral facial pain and temperature sensation is typically lost as well as contralateral body pain and temperature sensation.

A 55 year old man falls off of his bike on the Big Dam Bridge. He has a tingling sensation in his left and right arms and goes to the doctor the next day. A X-ray reveals compression fractures throughout his cervical vertebrae with numerous "punched-out" lytic lesions in his vertebral bodies and skull. Electrophoresis reveals an M-spike. What is the most likely diagnosis?

Multiple myeloma Punched-out lesions Compression fractures M-spike on electrophoresis

In general, which two ions are high outside of a cell and which ion is high inside of a cell to create a concentration gradient?

Na+ and Ca2+: high outside the cell. K+: high inside the cell. This is the normal concentration gradient. Inside is relatively negative! This is established by the Na+/K+ Pump -- In 2K+ and Out 3 Na+. Also a Na+/Ca2+ exchanger that moves Na+ and Ca2+ in the same direction, out of the cell. Creates more positive charges outside of the cell.

What medication is used for an opiate overdose? What medication is used for Alcohol and Opiate Use Disorders?

Naloxone for opiate overdose; fast-acting Naltrexone is long acting and delayed onset and used for etoh and opiate use disorder tx but not for OD's.

+++++++++++++++++++++++++++++++++ Nephritic vs. Nephrotic Syndromes: - UA will show Blood or Protein in urine? - What kind of casts are found in the urine? - Which one involves hypertension, oliguria (reduced urine output), signiificant drop in GFR (inc. Cr), and edema due to sodium retention? Why? - Which one involves generalized edema, hypoalbuminemia, and hyperlipidemia. Why?

Nephritic syndrome. Patients with nephritic syndrome have red blood cells (hematuria) and red blood cell casts in the urine. There is usually hypertension and edema due to renal sodium retention. Nephritic syndrome is also associated with proteinuria, though it is less than the 3.5 g/d seen in nephrotic syndrome. There is often a drop in the glomerular filtration rate (GFR) and urinary output. This leads to an increase in the serum urea and creatinine concentrations. - Think of this as nephritis - inflammation - that's why there's blood. - Examples: ✔ Acute poststreptococcal glomerulonephritis ✔ Rapidly progressive glomerulonephritis ✔ IgA nephropathy (Berger disease) ✔ Alport syndrome ✔ Membranoproliferative glomerulonephritis Nephrotic syndrome. Patients with nephrotic syndrome have heavy proteinuria (greater than 3.5 g/d). Normally, we should have less than 30 mg/d of protein in the urine, so this is a big change! Patients develop edema due to renal sodium retention from impaired glomerular function. Loss of important regulatory proteins in the urine give rise to hyperlipidemia and hypercoagulability. The GFR is generally preserved early in disease. However, with time, sclerosis (scarring) of the glomerulus may result in decreased GFR and urinary output. - Examples: ✔ Focal segmental glomerulosclerosis ✔ Minimal change disease ✔ Membranous nephropathy ✔ Amyloidosis ✔ Diabetic glomerulonephropathy

Folic acid / Folate is given to pregnant or trying mothers to prevent what?

Neural Tube Defects Neural tube defects are birth defects of the brain, spine, or spinal cord. They happen in the first month of pregnancy, often before a woman even knows that she is pregnant. The two most common neural tube defects are spina bifida and anencephaly. Sexually active women in childbearing age should take folate because NTDs occur early in pregnancy.

PT vs. PTT: - PT tests what 5 clotting factors? - PTT tests all clotting factors except for what two? - Warfarin is a common cause of isolated increase in which one since it blocks Vitamin __. - Hemophilia A and B are the most common causes of isolated increase in which one?

PT (Extrinsic) "The Rat WEPT" Tests factors 1, 2, 5, 7, 10 Common cause of isolated increase in PT is warfarin Warfarin (Rat poison) blocks Vit K derived Ca2+ dependent clotting factors PTT (Intrinsic) Tests all factors all except 7 & 13 Most well known cause of isolated increased PTT is hemophilia A and B Vitamin K deficiency (decreased 2,7,9,10,C and S) and liver disease causes increased PT and PTT (effects 7...the link, really it effects more than this, but this is the most important one) PT: extrinsic part of clotting pathway; how long it takes for clot to form after being exposed to TF. aPPT (PTT): intrinsic part of clotting (i.e., the zymogen factors) -- you measure how long it takes pts blood to clot after adding silica and phospholipid.

A 65-year-old man presents with worsening left-sided hearing loss and mild headache. He reported that his symptoms began a few months prior to presentation. He denies starting any new medications but has noticed he can no longer fit the hat he has owned for many years. Physical examination is unremarkable. Laboratory testing is notable for an elevated serum alkaline phosphatase. Calcium, Phosphate, and PTH are normal. 1. Diagnose 2. Why is there hearing loss? 3. What is the status of osteoblasts and osteoclasts in this condition? (Hallmark etiology) 4. These patients are at high risk for what type of cancer? 5. Describe the roles of RANK and RANKL in bone remodeling, including which one is on each type of bone remodeling cell and what the default is. MSK Pathology

Paget Disease of the Bone - Inc. osteoblast and clast activity = CONSTANT BONE REMODELING - Long bone "chalkstick" fractures - High Osteosarcoma risk. - Head grows - may have hearing loss Elevated Alkaline Phosphatase is the ONLY abnormal lab in Paget's. - Normal Calcium, Phosphate, and PTH levels. Adult bone constantly undergoes bone remodeling, which is accomplished by: 1. Osteoblasts: - contain RANKL (which binds RANK on osteoclasts) and Osteoprotegerin (OPG) - OPG inhibits RANKL from binding to RANK on osteoclasts, thus inhibiting bone remodeling and promoting bone formation (the default). 2. Osteoclasts: - contain RANK (receptor activator for NF-κB) transmembrane receptors, which respond to RANK ligand (RANKL) on osteoblasts and osteocytes to differentiate and activate in order to resorb bone.

A 58-year-old man with a 74-pack-year smoking history comes to the physician because of a 4-month history of anorexia, weight loss, generalized weakness, and painless jaundice. One week ago, he had pain in his left calf associated with swelling and warmth. Today, the patient reports his left calf has healed, but his right forearm is now swollen and painful. Physical examination reveals a painless enlargement in the right upper quadrant of his abdomen. This is a classic presentation for what cancer?

Pancreatic Cancer Painless enlargement of the gallbladder, known as Courvoisier sign, is also an indication of a malignant process in the pancreas. Pain in the calf associated with swelling and warmth, as well as current swelling and pain in the forearm, or things like that, suggest pancreatic adenocarcinoma with migratory thrombophlebitis, associated with Trousseau syndrome. Trousseau syndrome is classically associated with pancreatic cancer, although it can be present in other malignancies. It is classically defined as superficial thrombophlebitis, which implies superficial venous thrombosis and a state of hypercoagulability as the root of the problem. Even without SVT, however, Trousseau syndrome secondary to malignancy increases the risk of clotting events, including deep vein thrombosis (DVT) and subsequent pulmonary embolism. DVTs can cause pain, swelling, and warmth of the extremities, as seen in this patient.Generally, risk factors for DVT include Virchow triad, which consists of stasis (eg, obesity, surgery, or congestive heart failure), endothelial injury (eg, trauma or recent fracture), and hypercoagulable states (eg, pregnancy, oral contraceptive use, coagulation disorders, or malignancy).

++++++++++++++++++++++++++++ Know the neurotransmitter, nucleus, area of the brain for various vignettes: Parkinson's disease: 1. Identify the NT abnormality. 2. Damage to what 3 parts of the brain? One area involved in #1, other two involved in smoothed movements?

Parkinson's Disease: 1. Dopamine: Low => Jerky, uncoordinated movements. - Also, Ach is high (MNEM: Ach helps things move, Parkinson's pts move around a lot) and Serotonin is low. 2. Brain Damage: ✔ Substantia nigra (midbrain) - makes dopamine ✔ Caudate and Basal ganglia - loss of smooth movements With Parkinson's, be sure to treat EPS (later)

Gluconeogenesis Overview: - 3 bypass steps - Rate-limiting enzyme - Anabolic or Catabolic? - Occurs in what organs? - Basic overview of the 9 steps

Pathway responsible for converting non-carbohydrates to glucose 3 bypass steps: 1) Pyruvate to Phosphoenolpyruvate (PEP) 2) Fructose-1,6-bisphosphate to Fructose-6-phosphate (F-1,6-BPase) 3) Glucose-6-phosphate to Glucose (G6Pase) Rate-limiting enzyme: Fructose-1,6-Bisphosphatase Gluconeogenesis is an anabolic pathway and requires energy The major substrates are glucogenic amino acids (all but lysine and leucine), lactate, glycerol, and propionate Gluconeogenesis occurs predominately in the liver and kidneys. Gluconeogenic mechanisms remove lactate from muscle and erythrocytes (Cori cycle) Gluconeogenesis preserves glucose homeostasis. Produces glucose when there isn't enough. 9 Steps: 1) Pyruvate ➔ Phosphoenolpyruvate (PEP). 2) PEP ➔ 2-Phosphoglycerate (2-PG). 3) 2-PG ➔ 3-PG 4) 3-PG gets another phosphate added ➔ 1,3-BPG. 5) 1,3-BPG ➔ Glyceraldehyde. 6) Glyceraldehyde combines with another 1,3-bisphosphoglycerate ➔ Fructose 1,6-BP. 7) Fructose 1,6-BP loses a phosphate ➔ Fructose 6-Phosphate (F6P). 8) F6P ➔ Glucose 6-Phosphate (G6P). 9) G6P loses the phosphate to form the final product, glucose.

For simplicity, think of LVEDV as the same thing as ...

Preload

What is the classic defense mechanism of Paranoid Personality Disorder?

Projection E.g., "You're angry with me, I can tell" (when really they're the angry one)

How do prostaglandings affect the renal arterioles?

Prostaglandins Dilate Afferent arterioles (NSAIDs block it)

Protein C and Protein S: - Both are ___-__ dependent! - Protein C inactivates what two factors? - Role of Protein S (simple) - Deficiency of either would therefore lead to ... - Warfarin decreases these, so to prevent warfarin skin necrosis, you must bridge with ___.

Protein C and S are a vitamin K dependent protease made in the liver Prot C inactivates factor Va and factor VIIIa Protein S acts as a cofactor for protein C to inactivate FVa and FVIIIa Deficiency of either can cause increased risk of thrombosis -- clotting cascade isn't deactivated. Low Prot C or S can cause warfarin skin necrosis when starting warfarin, so must bridge with heparin

Scleroderma Treatments: Symptomatic Relief - Main drug that treats Pulmonary HTN - Immunosuppression - Hypertension - If there's renal involvement - If there's lung involvement

Pulmonary HTN: Use Epoprostenol, a prostaglandin that works by dilating / relaxing blood vessels and increasing the supply of blood to the lungs, reducing the workload of the heart. - e.g., from MedBullets stem for Pulm HTN: "....The diagnosis is confirmed with a right heart catheterization and she is started on epoprostenol." 1. Immunosuppressive therapies: - purpose: to prevent progression of sclerosis, especially if pulmonary system is involved. i.e., prevents it from getting worse. - Use: Methotrexate, Mofetil, or Cyclophosphamide if the first two didn't work. 2. Hypertension: Use Vasodilators - Epoprostenol (prostaglandin; vasodilator; for Pulm HTN) - Calcium Channel Blockers - ARBs - Etc. 3. Renal involvement: - Use ACE-Inhibitors - Protects from death due to sclerodermal renal crisis or failure. 4. Lung Involvement / Interstitial Pulmonary Fibrosis: - Use Mycophenolate Mofetil or Cyclophosphamide - If there's pulmonary hypertension, see Image

What type of arthritis is described? › Morning pain, gets better with use ("Morning Stiffness") › DIPs are spared, but can have Bouchard's nodes (PIPs) › Swan Neck Deformity (Crooked fingers) › Ulnar deviation › Baker's cyst (behind knee) › Associated with HLA-DR4 › Subluxation › Involves formation of Pannus (proliferative granulation or scar tissue in the joints) E.g., A 36-year-old woman presents to her physician's clinic with pain and swelling in the hands and wrists. She reports morning stiffness that lasts for approximately 1 hour and improves throughout the day with use. She also reports symptoms of general fatigue and depression. On physican exam, she has decreased grip strength and 4/5 strength on wrist extension. There is tenderness to palpation and swelling of the proximal interphalangeal joints and metacarpophalangeal joints, though the distal interphalangeal joints appear normal. MSK Pathology

Rheumatoid Arthritis Rheumatoid spares the DIPs (whereas Osteoarthritis includes the DIPs) > MNEM (alphabetical): The "D" in DIPs is closer to "Osteo" than "Rheumatoid," so if it says it affects the DIPs, think of which one is alphabetically first: Osteoarthritis. Swan Neck Deformity and Ulnar Deviation are classic Rheumatoid. (MNEM: Swan and Ulnar are alphabetically closer to Rheumatoid) Rheumatoid pain is bad when they wake up, but better with use (i.e., "Morning Stiffness") - Bad in the morning, Better throughout the day and with use. - Makes sense -- with rest, immune complexes deposit in joints and makes them stiff, but they loosen up as they moved around throughout the day. (whereas osteoarthritis pain gets worse with use but feels better in the morning or after rest) Pathogenesis of Rheumatoid Arthritis: Autoimmune; Type III HS (Immune Complex Deposition on joints) - inflammation causes growth of a pannus (proliferative granulation or scar tissue in the joints), which erodes articular cartilage and bone.

Diagnose: A 45-year-old patient dresses oddly and sleeps outside to absorb the full gravitational pull of the moon.

Schizotypal Personality Disorder I just made this card to memorize the classic STEP description of Schizotypal Personality Disorder. NBME stems are "cat lady"-like with odd dress and beliefs, relatively close to psychotic but not overtly delusional. - Dr. Spollen Display idiosyncratic speech patterns, eccentric beliefs, paranoid tendencies, perceptual illusion, unusual appearance, inappropriate affect, stilted language, and social anxiety. Genetic and phenotypic overlap with schizophrenia (so a "type of schizophrenia" -- on the schezophrenia spectrum).

++++++++++++++++++++++++++++ A 50-year-old woman presents to her physician's office for a routine checkup. Once at the office, she reports that she has been generally doing well but recently noticed that her fingers tend to turn blue in the cold. She reports feeling a general skin tightening in her face and hands, which makes forming a fist difficult. She also notes that she has had increased acid reflux lately and requests a medication for that. Her past medical history includes autoimmune thyroid disease and alopecia areata. Physical exam reveals sclerodactyly, and tight, hardened skin with limited mobility in her fingers. Her physician sends her for additional autoimmune workup. 1. Diagnose 2. Famous MNEMONIC for symptoms 3. Caused by what antibody? 4. This disease is caused by widespread ___ deposition on skin, organs, etc. MSK Pathology

Scleroderma &/or CREST Syndrome - CREST is basically a form of scleroderma In CREST and Scleroderma there is progressive collagen deposition through out the body affecting the skin, lungs, kidneys, vessels, and GI tract. Antibodies and MNEMONICS: CREST is c/b anti-Centromere antibodies (centromeres are needed in cell division) Scleroderma is c/b Anti-Scl-70 1. Calcinosis of skin 2. Raynaud phenomenon: ↓ blood flow to skin from either cold temperatures or stress, which causes blue coloration and turns red when reperfused. 3. Esophageal dysmotility 4. Sclerodactyly 5. Telangiectasia If there are skin changes in the fingers and face, it's CREST > scleroderma.

The effects of Scleroderma on the Kidneys: - List the characteristics of a Sclerodermal Renal Crisis - Patients who experience a Sclerodermal Renal Crisis usually have what antibody? - Treatment for renal involvement in scleroderma

Scleroderma Renal Crisis Only occurs in diffuse disease and may be related to steroid use. 80% cases within first 5 years have a renal crisis Associated with anti-RNA polymerase III antibody Sclerodermal Renal Crisis is characterized by: - Accelerated hypertension - Nephritic syndrome (proteinuria and RBCs) - Microangiopathic hemolysis - Rapidly progressive renal failure Treatment: ACE-Inhibitors - Protect from death due to sclerodermal renal crisis or failure.

A 40-year-old woman with no significant past medical history is bothered by several brown "moles." She started noticing them in the past year and notes that they have not changed in size. There is one mole in particular that is bothersome to her, cosmetically, because it is large. She denies any family history of skin cancer. Physical examination shows several dark brown, flat, plaque like lesions that appear to be "pasted on." 1. Diagnose this very common lesion. 2. What is the "sign of Leser-Trelat?" Dermatopathology

Seborrheic Keratosis Common benign neoplasm of semi-older folks. Sign of Leser-Trelat = Sudden appearance of a bunch of SKs; indicates a possible underlying malignancy (GI, lymphoid, etc) FIRST AID: Seborrheic Keratosis • Flat, pigmented, sometimes greasy squamous epithelial proliferation of immature keratinocytes with keratin-filled cysts (horn cysts). • Black, brown, tan • Appear "stuck on" or "pasted on" • Lesions occur on head, trunk, and extremities. • Benign • Leser-Trélat sign = rapid onset of multiple seborrheic keratoses, indicates possible malignancy (eg, GI adenocarcinoma).

++++++++++++++++++++++++++++++++++++ How do each of the following change acutely vs. long-term in High Altitude? - PAO2 and PaO2 - PACO2 and PaCO2 - Systemic Arterial pH - Hb concentration - Hb % Sat - Systemic Arterial O2 Cont

See Image

How does each of the following lead to malabsorption: - Tropical Sprue - Whipple Disease - Celiac Disease - Disaccharidase Deficiency - A-Beta-lipoproteinemia - Pancreatic insufficiency

See Image

Know the general histological appearance of each type of glomerular changes: - Increased mesangial matrix - Crescent formation - Mesangial hypercellularity - Glomerular sclerosis

See Image

MNEM for vascular damage to the brainstem and what cranial nerves will be affected

See Image

Obstructive vs. Restrictive Lung Diseases - RV FRC TLC FEV1 FVC FEV1/FVC Flow-volume loop changes for each.

See Image

Davenport Diagram: Plasma HCO3- vs. Plasma pH How will each axis / variable be affected by: 1. metabolic alkalosis with partial respiratory compensation. 2. respiratory acidosis (hypoventilation) with partial renal compensation 3. Hyperventilation 4. metabolic acidosis with partial respiratory compensation. 5. acute metabolic acidosis or alkalosis with no change in CO2.

See Image A is metabolic alkalosis with partial respiratory compensation. - High HCO3- because it's metabolic alkalosis, slightly higher pH because it's alkalosis, but respirations have partially corrected that. B is respiratory acidosis (hypoventilation) with partial renal compensation. C: This line represents effects of acute metabolic acidosis (C) or alkalosis with no change in CO2. D is metabolic acidosis with partial respiratory compensation. - Metabolic acidosis, so bicarb is low. pH is low since it's acidosis with only partial compensation. E: This line represents effects of acute changes in ventilation. Hyperventilation is E - Blowing off a lot of CO2, so pH is higher (alkalosis). - Bicarb is low to try to fix this alkalosis and b/c it's being converted to CO2, which is needed.

Emphysema vs. Chronic Bronchitis in terms of: PaO2 PaCO2 & pH Cyanosis? Cor Pulmonale? Histology

See Image Emphysema: Skinny. Need O2. Alveoli lose elasticity, so they can't push air all the way out. Chronic Bronchitis: Heavy. Cyanotic/Blue. Also need O2. Caused by chronically inflamed airways / mucosa proliferation. Cor pulmonale is defined as an alteration in the structure and function of the right ventricle (RV) of the heart caused by a primary disorder of the respiratory system (Chronic Bronchitis). Pulmonary hypertension is often the common link between lung dysfunction and the heart in cor pulmonale

High vs. Low Grade Lymphoma - Small or large cells? - Histiocytes present? - Mitoses present? - Is the staining (color) homogenous or heterogenous? - Size of nucleoli? - Which one responds to chemo?

See image

Name the causes (MNEM: GET SMASHED) and complications of pancreatitis.

Sharp, sudden epigastric pain that radiates to the back. Caused by pancreas inflammation that over-releases pancreatic digestive enzymes. In turn, those enzymes autodigest the pancreas. Causes (GET SMASHED) G - gallstones (common) E - Ethanol (common) T - Trauma (e.g., car wreck) S - Steroids M - Mumps A - Autoimmune S - Scorpion sting, snake bite H - Hypercalcemia/Hyperlipidemia E - ERCP D - Drugs (sulfa) Outcomes - With chronic pancreatitis, there's deficiency in the fat-soluble vitamins (ADEK), due to malabsorption. - Steatorrhea and Flatulence. - DIC - ARDS - Fat necrosis (saponification) - Hypocalcemia (used in soap deposits) - Pseudocyst - Chronic pancreatitis - Hemorrhage - Infection - MSOF, death

Diagnose: A 36 year old patient has a traumatic birth of twins. She has a prolonged hospital course (as do the twins) and during the process required multiple transfusions of packed RBC's, pressors and fluid support. The patient was discharged home with her children after 3 weeks in the hospital and is coming to you for her 1 month checkup. Her only complaint is that she is having problems breast feeding her children due to insufficient milk.

Sheehan's Syndrome (post partum necrosis of the pituitary gland) Caused by hemorrhage during labor, which results in permanent ischemia and death of the pituitary. Prolactin is not released.

What is the difference between Somatic Symptom Disorder and Conversion Disorder?

Somatic Symptom Disorder is when psychological distress is expressed in the form of physical symptoms. The psychological distress in somatization is most commonly caused by a mood disorder that threatens mental stability. Involves excessive preoccupation with the symptom(s), unlike Conversion DO, which often involves la belle indifférence. - Ex: A 27-year-old woman presents to her primary care physician due to headache, chest pain, and food intolerance. These complaints have been very distressing for her and have been present for approximately 8 months. She previously had seen a headache specialist, gastroenterologist, and obtained a number of electrocardiograms in the emergency department. Their respective thorough work-up was negative. On physical exam, the patient appears healthy and is otherwise unremarkable. Conversion disorder occurs when the somatic presentation involves any aspect of the central nervous system over which voluntary control is exercised. La belle indifférence is often seen in conversion disorder, where they aren't very distressed about the problem; unlike Somatic Symptom DO, which involves excessive preoccupation with the symptom(s). Also, stems for this will often mention a major stressor around the time of symptom onset. - Ex: A 25-year-old woman presents to the emergency room reporting inability to move her right leg since this morning. When the patient is asked to raise the leg, there is no downward pressure felt under the left heel (Hoover's sign). Neurologic exam is otherwise unremarkable. She reports that she is supposed to visit her stepfather, who sexually abused her as a child, later that day.

+++++++++++++++++++++++++++ Classic associations (colors) and what's High Risk after a Heart Attack: - Days 2-4 - Days 5-10 - Sudden death after 1 week would be due to... - Weeks later

Study IMAGE Day 1 - Can't really notice changes Days 2-4 - Arrhythmia, re-infarction Days 5-10 - Yellow and soft heart, rupture Rupture presents as sudden death at 1 week Patients will have sudden, usually catastrophic drop in cardiac output Weeks (7) - Grey\white heart, aneurysm

Diagnose: A 50-year-old male presents with an unprovoked convulsive episode that lead to right-sided hemiparesis. On physical exam, a port wine stain is appreciated, affecting the ophthalmic and maxillary division of the trigeminal nerve on the left side of the face. Head CT shows brain calcifications.

Sturge-Weber Disease ✔ AVM's of small vessels (capillaries) ✔ Port wine stain (trigeminal nerve distribution) ✔ Leptomeningeal angiomatosis (meningeal AVM)

A 47 year old woman presents to her family practice doctor with a painful lump in the center of her neck. She relates having had a cold 2 weeks prior. Lab tests show that she has an elevated T4 and decreased TSH. What is the most likely diagnosis?

Subacute Granulomatous Thyroiditis / de Quervain Thyroiditis (most common cause of a painful thyroid) Very tender thyroid gland following viral illness is key. She recently had a cold. This usually follows a URI or some other infection. This is a temporary condition. Should go from hyperthyroid to hypothyroid to normal. Early on may be hyperthyroid (similar to Hashimoto's) Later, symptomatology is hypothyroid (though this is not permanent)

Diagnose: • Slowly progressing encephalitis • Intranuclear inclusions in the neurons and oligodendrocytes • Assoicated with measles virus • Presents with seizures

Subacute Sclerosing Panencephalitis (SSPE) Subacute sclerosing panencephalitis (SSPE) is a rare complication of measles that occurs within a range of 7-10 years after the initial infection and is initially characterized by personality changes, lethargy, difficulty in school, and odd behavior. It can progress to dementia, severe myoclonic jerking, and then to eventual flaccidity and decorticate rigidity. SSPE is always fatal, with most patients living 1-3 years after diagnosis. No cure for SSPE exists. Patients with SSPE often have high titers of measles antibodies in their serum and CSF, which can be detectable as oligoclonal bands e.g., A Salvadoran woman brings her 9-year-old son to the pediatrician. She states her son had been very energetic and well liked by all of his teachers and had received excellent grades in school. Over the past 6 months, however, he has become withdrawn and lethargic, and his performance in school has deteriorated. Physical examination reveals occasional myoclonic jerking motions and some sensory deficits. The mother has the child's medical chart with her, and a nurse's note from a clinic in El Salvador describes that the patient had a peculiar rash when he was 11 months old. The pediatrician orders a brain biopsy that is positive for a hemagglutinin-containing RNA virus. ASSOCIATED WITH MEASLES!

Define each of these glomerular deposit locations: 1. Subepithelial deposits 2. Subendothelial deposits 3. Intramembranous deposits 4. Mesangial deposits Define each pattern of glomerular deposits: 1. Diffuse deposits 2. Focal deposits 3. Segmental deposits 4. Global deposits

Subepithelial immune deposits localize between the epithelium and GBM, Subendothelial deposits localize between the endothelium and GBM Intramembranous deposits localize within the GBM Mesangial deposits localize to the mesangial matrix ------------------------- Diffuse - affecting the majority of glomeruli, >50%. Focal - only some of the glomeruli, <50%. Segmental - only part of a glomerulus involved. - The best example of a segmental disease would be FSGS Global - affecting all regions of any given glomerulus.

++++++++++++++++++++++++++++++++++++++++ What spinal tracts are affected by each of these conditions? 1. Syringomyelia 2. Multiple Sclerosis 3. Tabes dorsalis (Tertiary syphilis) 4. B12 deficiency

Syringomyelia: Central canal cyst compresses mainly the Spinothalamic tract - Result is loss of pinprick and temperature, but dorsal column is still intact, so vibration and proprioception are maintained. Movement (corticospinal tracts) can also be affected Multiple Sclerosis: Random and asymmetric lesions of central white matter. Tends to present randomly. Tabes dorsalis: • Tertiary syphilis • Affects dorsal columns (MNEM: dorsalis) • Thus, loss of proprioception and vibration, but spinothalamic tract is still intact, so temperature and pinprick are intact. • Movement (corticospinal tracts) is intact • i.e., loss of proprioception and vibration as the only problem B12 deficiency: • Affects dorsal columns and corticospinal tracts • Therefore, it affects proprioception and vibration, as well as motor functions. But pinprick and temperature will be intact.

+++++++++++++++++++++++ Name each heart murmur Systolic Murmurs: 1. Crescendo-Decrescendo systolic ejection murmur and a soft S2. Might have accompanying SAD: Syncope, Angina, Dyspnea. 2. Holosystolic, high-pitched "blowing murmur" heard loudest at the apex. 3. Late systolic crescendo murmur with midsystolic click 4. Holosystolic, harsh-sounding murmur. Loudest at tricuspid area (T). Diastolic Murmurs: 5. High-pitched "blowing" early diastolic decrescendo murmur. Best heard at base (aortic root dilation) or left sternal border (valvular disease). Long diastolic murmur, hyperdynamic pulse, and head bobbing when severe and chronic. Wide pulse pressure. - MNEM for 4 most common causes: BEAR 6. Mid-to-late diastolic rumble heard after an opening snap in diastole. Snap occurs after S2, followed by rumble. Often presents with hoarse voice and/or dysphagia. - Often a late complication of what condition? Continuous Murmurs: 7. Continuous, machine-like murmur that's loudest at S2. Often due to congenital rubella or prematurity.

Systolic Murmurs: MR. S AS + MVP and VSD 1. Aortic Stenosis - HOCM also has a crescendo-decrescendo sound, but should include other clues (e.g., sudden death in a young athlete). 2. Mitral Regurgitation - Holosystolic because blood is also shooting up into the loose mitral valve during systole. - Often an early sign of Rheumatic Fever. 3. Mitral Valve Prolapse - Due to sudden tensing of chordae tendineae (like a parachute, chordae being the strings) as mitral leaflets prolapse into the Left Atrium. Chordae cause Crescendo with Click. 4. Ventricular Septal Defect - Holosystolic because blood moves across the ventricular septum to the right ventricle during systole / contraction. Diastolic Murmurs: MS. D AR 5. Aortic Regurgitation - It's the sound of blood regurgitating back down from the aorta into the LV during diastolic filling. - Regurgitation of blood from aorta + Normal blood filling => LV is overwhelmed and can lead to systolic HF. - Hee Hoo Hee Hoo Hee Hoo - Causes include Bicuspid aortic valve, Endocarditis, Aortic root dilation, and Rheumatic fever (BEAR). 6. Mitral Stenosis - Opening snap is the mitral valve being forced open during diastole to fill the left ventricle. Then the low rumble is the blood being pushed through the narrow valve. - Pressure in LA has to be much larger than Pressure in LV during diastole to push blood through. - Often a late (and highly specific) sign of Rheumatic Fever. - Often presents with hoarseness and dysphagia -- Left Atrium chronically dilated and can balloon out, compressing the left recurrent laryngeal nerve (hoarseness). Continuous Murmur: 7. Patent Ductus Arteriosus - MNEM: "PDAs (Public Displays of Affection) are continuously annoying."

What is malingering? How is it different from Factitious / Munchausen disorder?

The DSM-5 describes malingering as the intentional production of false or grossly exaggerated physical or psychological problems. Motivation for malingering is usually external (e.g., avoiding military duty or work, obtaining financial compensation, evading criminal prosecution, or obtaining drugs). Malingerers engage in many of the same activities as people with factitious disorder. They exaggerate or make up symptoms of an illness, either physical or psychiatric. Factitious disorder is an actual mental health condition with no clear cause. While both intentionally lie (unlike Conversion DO which is unintentional) and make up symptoms, malingerers do it for personal gain. Also, Factitious DO is sometimes done to another person, such as their child, while malingering is only done to yourself. To know which one it is, look for any sign that the person did it for personal gain.

A 17-year-old boy is drinking large amounts of water and urinating frequently. BMI 20, BP 115/85, blood glucose 85 mg/dl, Ca 9.0 mg/dl. Urine volume remains high after a four hour water deprivation test, but decreases following ADH / vasopressin / desmopressin administration.

The boy is suffering from neurogenic (central) DI If a patient is suffering from nephrogenic DI, salt restriction, thiazides, and NSAIDS are used for treatment

+++++++++++++++++++++++ Distinguishing adrenal enzymes involved in conversion of Cholesterol to each -- which enzymes are needed to make each? 1. Aldosterone (mineralocorticoids) 2. Cortisol (glucocorticoids) 3. Testosterone (androgens)

The most common adrenal enzyme deficiencies are 21-hydroxylase 11-hydroxylase 17-hydroxylase Cholesterol ⇨ Progesterone, then progesterone can go to two paths: • Progesterone -21-> -(11) -> Aldosterone • Progesterone -17-> -21-> -11-> Cortisol Cholesterol ⇨ Pregnenolone ⇨⇨ -17-> Androgens • Aldosterone needs 21, but doesn't have to use 11. • Cortisol needs all three: 17, 21, and 11. • Androgens only need 17. DOUBLE CHECK THIS WITH IMAGE TO MAKE SURE I INTERPRETED IT CORRECTLY

What is retic (reticulocyte count) a measure of? Normal is __ to __ % Greatly helps classify what?

The retic count is a direct measure of the rate of production of red cells in the bone marrow The normal retic count is 0.5% - 2% Greatly helps classify anemias -- anemia of DESTRUCTION vs. anemia of UNDERPRODUCTION. - Destruction -- retic will be high to try to increase - Underproduction -- retic will be low as few RBCs are being produced

+++++++++++++++++++++++++++++++ A 7 year old boy presents for his annual checkup. His mother's only complaint is that he has multiple, raised, cauliflower like lesions on his feet and hands. She wonders if it arose because he doesn't like bathing. - What is the diagnosis? - What pathogen causes this? - Is there risk that they could progress to cancer?

They are Verruca (= Cutaneous Warts) and likely due to HPV infection. - HPV causes raised warts on the skin. - The technical name of these cutaneous warts is verruca. This is what I had on my finger in college. They do not progress to cancer / SCC. They are harmless and stable. They can be frozen off. - Verruca are very common and are usually seen on the hands and feet. - They start as small white plaques (left) and progress to raised lesions with a rough surface and occasional scale. - They are caused by HPV (viral lesions are hot test topics) and are treated by topical therapy. - There is no risk of progression to SCC. - If a pt has numerous warts/verruca, they may have an underlying immunodeficiency.

+++++++++++++++++++++++++++++++++++ Key events after an MI, in terms of wound healing. At each of these points s/p MI, name the: (a) gross appearance of the cardiac tissue (b) histological findings (c) deadly complications 1. First 24 hours -- really, just know the complication here. 2. 1-3 days 3. 3-14 days 4. Weeks later

This card describes that process and the evolution of the affected / ischemic heart tissue after an MI. First 24 hours (a) and (b) not much change, but the area of the heart supplied by the occluded coronary artery will slowly die, leading to complications. (c) ARRHYTHMIA is the leading cause of sudden death immediately after a heart attack. Give the pt anti-arrhythmics. 1-3 days later: (a) red border, but center is turning yellow/tan from ischemia (b) Invasion of neutrophils due to necrosis. Necrosis always leads to an immune response. Neutrophils go to abnormal tissue and start eating the necrosed tissue. (c) Still ARRHYTHMIA 3-14 days later: (a) Tissue is Yellow (b) Granulation tissue is present, trying to restore tissue as a part of the wound-healing process. (c) RUPTURE of the weakened area. Sudden death, usually around a week s/p MI. Weeks later: (a) Firm, grey-white scar tissue. (b) Fibrosis (scar tissue) (c) ANEURYSM ----- from old card ----- Eventually, NECROSIS forms at the heart. Necrosis always causes an immune response. So the NEUTROPHILS are like "Hey, that's weird, we need to go there." That happens at about 12 hours. Then over days, neutrophils keep coming and GRANULATION TISSUE forms, eventually causing scarring (FIBROSIS). The heart's gross appearance changes some and you can actually see color changes. In that chart, know the general time frames, like "It's been 4 days, so this patient is at increased risk for rupture because of the pale-yellow granulation tissue."

What general type of condition does this patient have? A 50 year old male with a history of intravenous drug use (but none for last 5 years), hepatitis B, & hepatitis C presents with two months of fever, two days of left upper quadrant abdominal pain, several weeks of numbness of his right thigh, dragging of his left foot, and a 10 pound weight loss. Physical Exam: - BP= 190/100, HR= 100, T=101, RR= 18 - Livedo reticularis (lace-like purplish discoloration of the skin) of hands and thighs - RUQ slightly tender to palpation without rebound - Decreased sensation of his right thigh - Mild weakness on dorsiflexion of his left foot

This is going to be some kind of vasculitis. Hepatitis and other infections can sometimes (rarely) lead to immune complex formation and deposition around small cutaneous vessels. Large Vessel Vasculitis: - Takayasu's arteritis - Temporal arteritis Medium Vessel Vasculitis: - Polyarteritis Nodosa - Kawasaki's disease Small Vessel Vasculitis: ANCA + - Eosinophilic Granulomatosis with polyangiitis (Churg-Strauss Syndrome) (p-ANCA) (MNEM: P-osinophilic) - Granulomatosis with polyangiitis (Wegener Granulomatosis) (c-ANCA) - Microscopic polyangiitis ANCA - - Henoch Schonlein Purpura - Cryoglobulinemic vasculitis - Leukocytoclastic vasculitis

A 10-year-old boy is brought to the pediatrician because he has been vomiting every few hours for the past 2 days. He denies any recent travel or contact with sick persons. He has a history of asthma, which is well controlled with 1-2 puffs of salbutamol per day. His temperature is 98°F (36.7°C), blood pressure is 99/65 mm Hg, pulse is 88/min, respiratory rate is 10/min, and oxygen saturation is 92% on room air. On examination, he appears lethargic and uncomfortable; however, no wheezing is appreciated on lung examination, and his abdomen is soft and nontender. What will be the status of: - blood pH - PCO2 - HCO3-

This patient presents with episodic vomiting and lethargy. A history of several episodes of vomiting, especially if profuse, should suggest a primary metabolic alkalosis caused by the loss of hydrochloric acid from the stomach. Metabolic alkalosis is characterized by a pH >7.45 and high HCO3-. Alkalosis is from vomiting stomach acid. HCO3- is high mainly because this is the hallmark of metabolic alkalosis, but also because it can be converted to CO2 to lower the blood pH. PCO2 will be high because of the low RR, likely as a compensation for the metabolic alkalosis.

A 21-year-old woman with past medical history of SLE presents with fever, fatigue, and altered mental status. On physical exam, a petechial rash is seen on her upper extremities. Her temperature is 103°F (39.4°C). Her labs are notable for Hgb 7.3 g/dL, platelets 17,000/mm3, and creatinine of 3.4 mg/dL. Peripheral blood smear shows schistocytes. 1. Diagnose 2. Know the Pentad of symptoms 3. This disease is caused by a deficiency of ___. 4. Peripheral Blood Smear -- 3 findings

Thrombotic Thrombocytopenia Purpura (TTP) 1. Microangiopathic hemolytic anemia 2. Acute renal failure 3. Thrombocytopenia 4. Fever 5. Neurologic abnormalities Similar to HUS but with fever and neurologic symptoms. Deficiency of ADAMTS13 - ADAMTS13 cleaves very high molecular weight multimers of vWF. - If the larger, more active, multimers of vWF cannot be broken down by ADAMTS13, they remain attached to endothelium and this promotes platelet aggregate formation. Peripheral Blood Smear: Schistocytes Polychromasia Thrombocytopenia

A 38 year old woman is flown into the emergency department on Friday afternoon with symptoms of mental status changes, oligouria, fever, decreased platelets, and schistocytes on her peripheral smear. 1. What is the most likely diagnosis? 2. She is deficient in ___, which leads to?

Thrombotic Thrombocytopenic Purpura (TTP) Deficient in ADAMTS 13, which cleaves vWF. This leads to huge aggregates of vWF, which leads to platelet aggregation and diffuse thrombosis. Triad of thrombocytopenia (platelets), microangiopathic hemolytic anemia (Hb, schistocytes, LDH), and acute kidney injury (Cr). Plus unique sxs of fever and neurologic symptoms (mental status change).

Deficiency or Antibodies to ADAMTS13 = What diagnosis?

Thrombotic thrombocytopenic purpura (TTP) This is a Thrombotic Microangiopathy (along with HUS - Hemolytic Uremic Syndrome), which are all characterized by thrombocytopenia, microangiopathic hemolytic anemia, and renal insufficiency Congenital

A 36 year old woman notices a "full feeling" in her neck. She also complains of increasing weakness during the day that is better in the morning. She is diagnosed with myasthenia gravis. What else should this patient be evaluated for?

Thymoma Thymomas are highly associated with myesthenia gravis

DiGeorge Syndrome and SCID are both associated with hypoplasia of what organ? Myasthenia gravis involves a ___ on this organ.

Thymus Location of T-cell maturation (positive and negative selection) Myasthenia Gravis involves thymomas - tumors on thymus.

Result of damage to Pons cranial nerves: -5 -6 -7 -8

Trigeminal (CN5): ipsilateral loss of pain, temperature, and touch on the face as far back as the anterior two-thirds of the scalp, but not the angle of the jaw. Abducens (CN6): ipsilateral weakness of abduction (lateral movement) of the eye (lateral rectus muscle). Facial (CN7): ipsilateral facial weakness. Auditory (CN8): ipsilateral deafness, possibly with balance problems.

A 40 year old man appears in your emergency having been bitten on the lower leg 1 hour previously by a cottonmouth during a fishing trip. He is in considerable pain and the wounded limb is edematous. Based on the severity of the bite and symptoms, you conclude that treatment with anti-venom is warranted. You first perform a skin-test with a small amount of the anti-venom. The skin test is negative, so you proceed with the treatment and he recovers. Two weeks later he calls you and says that he has developed lower back and joint pains, a generalized rash that itches, and a fever of 102. What type of Hypersensitivity reaction is this? i.e., this reaction was mediated by what?

Type 3 Hypersensitivity Reaction: AKA Immune Complex Disease As a consequence of antivenom administration, the patient's immune system reacts by producing antibodies that attach to foreign antibodies (antivenom), resulting in the formation of an immune complex. • Serum sickness is a common consequence of anti-venom treatment • Onset is 2-4 weeks • Deposition of immune complexes and inflammation of glomeruli and joints • Deposition of complexes along vessels - vasculitis • Release of inflammatory cytokines - fever

A 17-year-old female presents to the emergency department with complaints of subjective fever and crampy abdominal pain for the past week. Today, she had an episode of bloody bowel movement, which has never happened before. A physical examination demonstrates mild conjunctival pallor but is otherwise unremarkable. ESR and CRP are elevated.

Ulcerative colitis Key Features: IMAGE - Colon only - Starts at rectum and moves proximally - NO granulomas - Cancer risk - Primary sclerosing cholangitis

++++++++++++++++++++++++++++++++++++++++++ High Yield Spinal Cord Levels and Dermatomes - Name the spinal cord level of each dermatome: Upper extremity: • Thumb (radial) • Middle finger • Pinky (ulnar) Body: • Nipple Line • Umbilicus • Inguinal Ligament

Upper extremity: C6/C7/C8 • Thumb (radial) - C6 • Middle finger - C7 • Pinky (ulnar) - C8 Body: • T4: Nipple Line • T10: Umbilicus • L1: Inguinal Ligament

A 15 yr old girl is brought to clinic due to sudden onset of generalized swelling. On exam she has anasarca. Has been using a medication for the last 1 month for severe case pimples. Labs CBC normal, Renal panel with slightly increased creatinine from her previous visits (1.3 from 1) , urine shows 4+ proteinuria (UPC 6 gms/day) and no blood. Albumin 1.5 gm 1. Diagnosis: a. IgA nephropathy b. Infection related GN c. Minimal change disease d. Diabetic nephro-sclerosis. e. Membranous 2. What would you see on Light Microscopy? What about EM?

c. Minimal change disease Most common cause of nephrotic syndrome in children. Normal glomerulus on PAS and JMS - no change here. The ONLY change is complete foot process effacement seen on electron microscopy- that's the minimal change - This allows protein, but not blood, to leak through into urine. Treatment: Excellent response to corticosteroids (Prednisone).

Need-to-know important side effects of antihypertensives - Name the antihypertensive drug (class): - angioedema, cough; CONTRAINDICATED IN PREGNANCY - bronchoconstriction - orthostatic hypotension - Butterfly rash - specific drug - Hyperglycemia - specific drug - Increased hair growth - specific drug

• ACEI: angioedema, cough, C/I in pregnancy! Fetus needs AngII. • BB: bc, mask db sx, unopposed α-constriction • αB: first dose, ortho hypo • Vasodilator: reflex tachy -Hydralazine: butterfly rash -Diazoxide: hyperglycemia -Minoxidil: hair growth

Galactosemia: - Caused by deficiency in what enzyme? - What accumulates? - Sxs that result from this accumulation

• Genetic deficiency of galactose 1-P uridyltransferase that exchanges galactose 1-P for glucose on UDP-glucose. Most Q's seem to focus on this. • Accumulation of galactose 1-P induces tissue damage in kidney, liver and brain • This causes cataract formation, growth failure, and mental retardation. Untreated, this is a severe disease that can lead to death from liver damage • Galactokinase deficiency causes a milder form of galactosemia USMLE-Rx Example: A 6-day-old female neonate who has been vomiting for the past 2 days is brought to the emergency department by her mother. During this time, the patient has also experienced diarrhea and difficulty breastfeeding. On physical examination, the physician notes a lethargic newborn with yellow sclerae and hepatomegaly. The anterior fontanelle appears to be sunken, and capillary refill is delayed. Blood work reveals an alanine aminotransferase (ALT) of 56 U/L and aspartate aminotransferase (AST) of 64 U/L. The patient is switched from breast milk to a special formula, and her symptoms fully resolve in 3 days.

Result of damage to Medulla cranial nerves: -9 -10 -11 -12 Define Lateral Medullary Syndrome (HIGH YEILD) What is the Nucleus Ambiguus?

• Glossopharyngeal (CN9): ipsilateral loss of pharyngeal sensation (dysphagia) • Vagus (CN10): ipsilateral palatal weakness • Spinal accessory (CN11): ipsilateral weakness of the trapezius and stemocleidomastoid muscles • Hypoglossal (CN12): ipsilateral weakness of the tongue (toungue deviation). Lateral Medullary Syndrome is when the Nucleus Ambiguus (lateral side of the medulla, CNs 9, 10, 11) is damaged. Hoarseness and dysphagia are fairly specific for lateral medullary syndrome (Wallenberg syndrome) because it points to a lesion of the nucleus ambiguus (CNs 9, 10, 11). The PICA supplies blood to structures of the lateral medulla (vestibular nuclei, spinal cord tracts) and the inferior cerebellar peduncle.

Medical manifestations of Anorexia and Bulimia (mostly in terms of hormone and electrolyte disturbances)

• Low Estrogen/LH/FSH (aka, hypogonadism) in females (leads to amenorrhea) • Low Testosterone in males • Low Noradrenaline leads to Hypotension, Hypothermia, Bradycardia • Hypokalemia may lead to Cardiac Arrhythmias (cause of death) • Hypophosphatemia, Hypomagnesemia (refeeding syndrome concerns) • Hyponatremia • Increased Cortisol and Growth Hormone • Leukopenia, mild anemia • Lanugo (fine downy body hair); Hypercarotenemia • Osteoporosis • Increased ventricular-brain ratio (from starvation) • If induced vomiting (mostly seen in Bulimia on NBME): - Metabolic alkalosis with elevated serum bicarb - Hypokalemia, Hypochloremia

Patient interviewing, consultation, and interactions with the family: • What will always be a good answer to what to do if a patient seems stressed, depressed, trying to cope, etc. • What do you do if a parent has a child with behavioral problems? What is this particular therapy term called? • Confidentiality concerns with teenagers involving sexual activity, birth control, and pregnancy, etc (HIGH YIELD question topic); lean towards what solution? • Suicide risk assessment: The test writers are always looking for what?

• Pt is stressed, depressed, trying to cope, etc: - Lean towards "ask the patient about..." answers - Ask them about fears, understanding, frustration, death/dying issue - Ask them about the source of their feelings and how they are trying to cope. - i.e., don't ignore it or blow it off or say "it'll be fine." • Child is having behavioral problems: - "Parental Training" => Help parents set limits • Confidentiality concerns w/ teens concerning sexual stuff: - Frequent question and clinical scenario - Lean towards confidentiality - this is a private matter and the doctor shouldn't disclose this to the parents if the teen does not want them to know. Even with birth control -- you don't need your parents' permission. • Suicide Risk Assessment: - Usually they want "ask them directly" if any doubt -- asking them up front usually works. - "hospitalize", or even "commit" if high risk" - E.g., hospitalize &/or commit patients with suicidal plans, or command hallucinations, or anorexia with complications, such as with electrolyte disturbance / bradycardia. - Trick: If you think you could be liable for a patient dying, hospitalize them.

Chediak-Higashi Syndrome: - Etiology: Defect in what process and what type of immune cell? - Common symptoms

• Rare autosomal recessive disorder (LYST gene) • Oculocutaneous albinism • Defects in vesicle fusion • Impaired phagocytosis due to inability of endosomes to fuse with lysosomes • Impaired NK cell cytotoxic function • 85% develop "accelerated" lymphoproliferative phase Presentation: - Partial Albinism: Silvery hair, bright eyes - Recurrent pyogenic infections, especially of the skin and soft tissue - Staph and Strep - Pancytopenia: easy bruising, recurrent infections, bleeding

List the: - SNS agonists - SNS blockers - PNS agonists

• SNS agonists: Epi(a,b1,b2), NE(a,b1), PE(a1), Iso(b1,b2), Dobutamine(b1), Albuterol(b2) • SNS blockers: PT,PB, Prazosin(a1), Atenolol(b1), Met(b1), Propranolol(b), Labetalol(a1,b) • PS agonists: ACh(M), Bethanechol(M), Neostigmine (Indirect), Atropine (MB), Mecamylamine (GB) • Others: vasoconstrictors, vasodilators, RAAS, NO, ANP

How are ACE-Is and ARBs different? - Serum AngII level - Serum bradykinin level - Exogenous administration of Ang II could only be blocked by which?

• Serum Ang II level is low with ACEI and high in ARB • Serum bradykinin level is high with ACEI → dry cough • Exogenous Ang II effect will only be blocked by ARB

A 60-year-old man is brought to ED after losing sensation in his fingers and toes. He jumped into a lake 1 hour ago as part of a polar plunge competition. The lake temperature was 4.4°C (40°F). His core body temperature is 35°C (95°F). Physical examination shows pale extremities that are non-receptive to sharp touch, dull touch, and two-point discrimination. The compensation will be ↑ or ↓: - Central blood volume - Baroreceptor firing - ANP

↑ Central blood volume ↑ Baroreceptor firing ↑ ANP When the body is cold, it vasocontricts peripheral vessels to try to keep blood towards the center / core. - Central blood volume will therefore increase. - Baroreceptors fire when there's high BP in the carotid. More blood will be in the carotid since it's at the core. - ANP is released when the atrium is stretched. More blood will be in the heart (core) so ANP will increase. • ANS responds to baroreceptor stretch (usually @ carotid) • RAAS responds to renal perfusion • ANP responds to atrial stretch (central blood volume) • ADH: 1) osmolarity, 2) very low volume (may override the osmolarity response and mess up plasma sodium levels!)

+++++++++++++++++++++++++++++++ What will be the effect of 11ß-Hydroxylase Deficiency? - Cortisol, and how this presents - Aldosterone, and how this presents - ACTH, and how this presents - Androgens, and how this presents - Blood Pressure - Plasma Na+ - Plasma K+ (The Tricky One)

↓ Cortisol ↓ Aldosterone (but high BP b/c high 11-deoxycorticosterone raises BP) ↑ Androgen / Testosterone ↑ ACTH production from ↓Cortisol -- pigmentation The key to understanding this is to know that 11-deoxycorticosterone is high, resulting in aldosterone effects (HTN, high Na+, low K+), but aldosterone is still low. Presentation: - ambiguous female genitalia at birth e.g., clitoral enlargement and labial fusion - Hypokalemia and Hypernatremia, despite low aldo, due to 11-deoxycortisone haing mineralocorticoid effects. - hypertension, secondary to increased 11-deoxycorticosterone. Patients have ↑ 11-deoxycorticosterone, which has mineralocorticoid properties and results in high blood pressure, but aldosterone is still low. So the only difference between 21 and 11 is that 11 has HIGH BP, HYPERnatremia, and HYPOkalemia. 21 is the opposite. Blood Pressure: High in 17 and 11 deficiency Low in 21 deficiency MedBullets Example: A 1-year-old male is found to have high blood pressure on multiple visits to your office. On examination, the patient has normal genitalia. Further laboratory workup reveals low serum aldosterone and high serum testosterone. Which of the following is most likely to be elevated in this patient? A. 17-hydroxylase B. 21-hydroxylase C. 11-hydroxylase D. 5'-deiodinase E. 11-deoxycorticosterone

+++++++++++++++++++++++++++++++ What will be the effect of 17α-Hydroxylase Deficiency? - Cortisol, and how this presents - Aldosterone, and how this presents - ACTH, and how this presents - Androgens, and how this presents

↓ Cortisol ↑ Aldosterone ↓ Androgen production ↑ ACTH production; pigmentation Presentation: - Ambiguity of male genitalia at birth; puberty delay in both sexes - Hypertension with hypernatremia and hypokalemia due to excess aldosterone. - Diffuse skin pigmentation due to ↑ACTH. Blood Pressure: High in 17 and 11 deficiency Low in 21 deficiency MedBullets Example: A 17-year-old girl is brought to the pediatrician by her father for evaluation. He is concerned that she has not undergone puberty yet, while all of her classmates at school have. The patient herself feels well overall, with no specific complaints. Examination shows vital signs of T 98.9, HR 71, and BP 145/92. The physician notes undeveloped breasts and normal external and internal female genitalia in Tanner I stage of development. Her body mass index is within normal limits, she is in the 40th percentile for height, and she is agreeable and pleasant during the interview. Which of the following additional findings is likely present in this patient? A. Aromatase enzyme deficiency B. Hypokalemia C. Increased levels of sex hormones D. XY karyotype E. Hypercortisolism

1. A healthy, young, regular runner is sitting quietly. Just thinking about running increases the blood flow to leg muscles. What is responsible for this vasodilation? A. Adenosine B. Endothelin C. Epinephrine D. Nitric oxide E. Norepinephrine 2. Use MNEMONICS to name the biggest vasodilators and the biggest vasoconstrictors (body molecules, not medications) Cardio Review

✔ C. Epinephrine Remember that Epi is a Vasodilator in some places (via ß2) and a Vasoconstrictor in other places. MAJOR VASODILATORS (α,ß2) "Evan Has Pretty Big Arteries" - EPINEPHRINE - via ß2 receptor at rest - Histamine - Prostaglandin E (PGE) - Bradykinin - Adenosine MAJOR VASOCONSTRICTORS "Narrow Every Artery Everywhere" - Norepinephrine - NE does not like ß2, but Epi does. - Endothelin - Angiotensin II - EPINEPHRINE - via alpha 1 vasoconstriction > Epinephrine acts on alpha receptors causing vasoconstriction and on beta receptors causing vasodilation. NOT Adenosine. - Adenosine in CV: > A2 receptor in coronary arteries: ↑cAMP → vasodilation > A1 receptor in AV node: ↓cAMP → ↓ AV conduction

+++++++++ HIGH YIELD, EASY POINTS +++++++++ Psychotic Disorders: Diagnose each: 1. Psychotic symptoms, such as delusions, hallucinations, and disorganized speech / behavior, lasting less than 1 month (i.e., X number of days). Stem often mentions it following a major stressor. 2. Psychotic symptoms, such as delusions, hallucinations, and disorganized speech / behavior, lasting 1-6 months (e.g., "for the last 5 months). 3. Psychotic symptoms, such as delusions, hallucinations, and disorganized speech / behavior, lasting 6 months or longer. 4. Can present as either schizophrenia + depression symptoms or schizophrenia + Manic symptoms. Must have at least 2 weeks of schizophrenic symptoms without mood disorder. 5. One or more fixed, persistent, unswerving belief(s) that may or may not affect some aspects of daily functioning. Overall, these patients will have relatively intact functioning (e.g., working) and they speak and look mostly normal. No psychotic symptoms, no negative symptoms, no mood symptoms like depression or mania. They just have that fixed, illogical, and pathological (bad for their mental health) belief. 6. Psychotic symptoms that are only present during episodes of depression. 7. Psychotic symptoms that are only present during episodes of mania or a patient who is constantly manic and psychotic.

"KNOW THESE TIME COURSES," especially the differences between 1-3 below. - Dr. Spollen 1. Brief psychotic disorder (more than 1 day, but less than 1 month -- i.e., ~ 2-30 days) - Usually after extreme stressor 2. Schizophreniform disorder (1-6 months) - Just like schizophrenia, but less than 6 months - "If it's 5 months and 29 days, it's still schizophreniform." - Almost like pre-schizophrenia, but schizophrenia is when you hit the 6 month mark. So schizophreniform may or may not become schizophrenia. 3. Schizophrenia (> 6months) Diagnosis requires ≥ 2 of the following active symptoms, including ≥ 1 from symptoms #1-3: 1. Delusions 2. Hallucinations, often auditory 3. Disorganized speech 4. Disorganized or catatonic behavior 5. Negative symptoms 4. Schizoaffective Disorder - Schizophrenia (hallucinations, delusions) plus a mood disorder (depression or manic symptoms) - Must have at least 2 weeks of schizophrenic symptoms without mood disorder. If mood disorder is invariably present, diagnosis is bipolar disorder with psychotic features. - Before diagnosing schizophrenia, check for depression or bipolar / mania. 5. Delusional disorder: - Relatively intact functioning (eg, working), sound and look mostly normal (no disorganized speech or behavior, no negative symptoms), except for delusions. - Hallucinations are absent or not prominent and, if there are hallucinations, they're always related to delusions. - First Aid: ≥ 1 delusion(s) lasting > 1 month, but without a mood disorder or other psychotic symptoms. Daily functioning, including socialization, may be impacted by the pathological, fixed belief but is otherwise unaffected. Can be shared by individuals in close relationships (folie à deux). 6. Major Depressive Disorder with Psychotic Symptoms - Psychotic symptoms occur exclusively during periods of depressive mood disturbance - There is no 2+ week period of no mood DO symptoms (which is the difference from schizoaffective DO) 7. Bipolar Disorder with Psychotic Symptoms - Psychotic symptoms occur exclusively during periods of mania, or a schizophrenic person without the 2+ week period of no mood symptoms required for a schizoaffective diagnosis.

USMLE 2015 Sample Question A 22-year-old man comes to the emergency department because of the recent onset of torticollis and uncontrollable facial grimacing. He began therapy with a new drug 24 hours ago. Which of the following drugs is the most likely cause? (A) Amitriptyline (B) Diazepam (C) Fluoxetine (D) Haloperidol (E) Levodopa

(D) Haloperidol Famously causes extrapyramidal symptoms (Parkinson-like)

One day after a 10-km race, a previously healthy 42-year-old man has dark urine. Urinalysis: Specific gravity 1.010 Glucose negative Blood positive Nitrites negative Microscopic examination WBC negative RBC negative Which of the following is the most likely cause of these findings? (A) Acute glomerulonephritis (B) Hypovolemia (C) Renal infarct (D) Renal vein thrombosis (E) Rhabdomyolysis

(E) Rhabdomyolysis Strenuous physical exercise plus dark urine = rhabdomyolysis.

MoA of Adenosine Drug of Choice for ... Usefulness during an MI?

- Activates inhibitory alpha 1 receptors on the AV node to slow AV node conduction - Drug of choice in terminating reentrant SVTs. - Vasodilator effects: dilates coronary arteries to reduce resistance in the healthy coronary arteries during an MI.

A 26-year-old man is brought to the emergency department by ambulance 30 minutes after being shot in the leg. He is unconscious and appears markedly pale. His pulse is 120/min, respirations are 16/min, and blood pressure is 80/60 mmHg. Compared with a healthy adult, will he have increased or decreased: - Arterial receptor firing rate - Systemic vascular resistance (SVR) - Pulmonary vascular resistance (PVR)

- Decreased arterial receptor firing rate -- fire when there's high BP (DC) - Increased SVR - Increased PVR

+++++++++++++++++++ KNOW +++++++++++++++++++ Coagulation Cascade - Factors - PT - PTT

.

Histology of Chronic Pancreatitis

.

Name the cancer associated with each tumor marker: - AFP - Beta-HCG - CA-125 - Alkaline phosphatase - TRAP - CA 19-9 - Calcitonin

.

Which glomerular disease involves deposition of these immune complexes (Type 3 hypersensitivity): 1. Type IV Collagen 2. Phospholipase A2 receptor (PLA2R)

1. Goodpasture's Syndrome: Type IV Collagen 2. Membranous Nephropathy: PLA2R

What 3 skin malignancies are associated with UV exposure? Dermatopathology

1. Melanoma (intense / excessive as child) 2. SCC (chronic) 3. Basal Cell Carcinoma (chronic)

Bone Tumor Quiz! Diagnose each: 1. Tumor associated with FAP / Gardner's syndrome 2. Most common benign bone tumor 3. Benign Cartilagenous tumor found IN the bone 4. Benign bone tumor that causes intense pain that is eliminated by NSAIDS 5. Benign tumor that often arises in knee and mimics malignancy. It is locally aggressive and has a "soap bubble" on X-ray MSK Pathology

1. Osteoma (new bone only, skull) 2. Osteochondroma (bone with spiculated (on X-ray) cartilage cap) 3. Enchondroma (chondrosarcoma is the malignant version) 4. Osteoid osteoma 5. Giant cell tumor

Vulvar Cancers: Diagnose from stain results: 1. S100 +, Mucin - 2. S100 -, Mucin +

1. S100 +, Mucin - = Melanoma of vulva ◦ Highly lethal ◦ May metastasize ◦ S100 +, Mucin - ◦ May not be pigmented 2. S100 -, Mucin + = Paget Disease of vulva ◦Locally destructive ◦Low metastatic rate ◦S100 -, Mucin + ◦Usually not associated with other carcinoma (unlike Paget's in breast)

A 27-year-old male who recently returned from combat duty in Iraq is having trouble readjusting to civilian life. He has been home for a few months but remains extremely irritable and jumpy with recurring nightmares. He has noticed that whenever a car backfires or a plane passes, his mind flashes back to a time in Iraq when his unit was attacked and two of his fellow soldiers were seriously injured. Since being home, he has been sleeping poorly and spends much of his time in the basement, which is soundproof. He is very concerned because his symptoms have not improved with time. 1. What medication class do you give him? 2. What medication can help with his nightmares?

1. SSRI 2. Prazosin for nightmares

Pneumonia Pathogen Flash Quiz: 1. Most common community acquired? 2. Associated with alcoholics & elderly nusing home patients 3. Most common atypical pneumonia? 4. Most common in setting of cystic fibrosis

1. Strep pneumoniae 2. Klebsiella pneumoniae (strep pneumo is still most common in this group though) 3. Mycoplasma pneumoniae 4. Pseudomonas aeruginosa

What is generated from each acetyl-CoA that goes into the Citric Acid Cycle? - How many NADH molecules? - How many FADH2 molecules? - How many GTP molecules? What process are those molecules used for?

3 NADH 1 FADH2 1 GTP per each acetyl-CoA The abstracted electron pairs from the NADH and FADH2 are used to make ATP by oxidative phosphorylation (electron transport chain)

The Nucleus Solitarius controls what special sense? This comes from CNs __, __, and __. It is located at what level of the brainstem?

7, 9, and 10 are for taste (ant. 2/3, post. 1/3, & epiglottis) Always think of the Solitary nucleus / Nucleus Solitarius with taste and 7/9/10. Located in the medulla. So it can be found only in medulla cross sections. It makes an L shape (IMAGE). The bottom hook (9 and 10) also do sensation for the GUT. 7 does not participate in that. But taste is the main thing.

An 8-year-old boy is brought to the ophthalmologist by his parents after his teacher recommends that the child's vision be evaluated. The child has had difficulty seeing in class and is unable to do his school work. His parents note that the boy is much taller than many of his classmates. On examination the ophthalmologist notices a lens that is not located centrally and refers the patient to his pediatrician for further work up. During this visit he is observed to have sideways curvature of the spine. Genetic testing reveals a mutation on chromosome 15. Which of the following is the most likely cardiovascular complication to be seen in this patient? check A. A decrescendo diastolic murmur B. Aortic stenosis C. Hypertension D. Ischemic stroke E. Venous thrombosis

A. A decrescendo diastolic murmur This patient with a dislocated lens associated with vision difficulties, taller than average stature, and scoliosis, shows signs and symptoms of Marfan syndrome, a connective tissue disorder. Together with ectopia lentis (a displaced lens), aortic involvement is a primary hallmark of Marfan syndrome and may be manifest with dilation of the aortic root (aortic aneurysm) leading to aortic regurgitation that is detectable on auscultation as a decrescendo diastolic murmur. Aortic dissection and mitral valve prolapse may also be seen. In addition, Marfan syndrome can be characterized by scoliosis greater than 20 degrees (as seen in this patient), an arm span exceeding height (ratio >1.05), reduced upper-to-lower body segment ratio, arachnodactyly, and dural ectasia. Marfan syndrome is caused by a mutation of the fibrillin gene, located on chromosome 15. Fibrillin is one of the major components of microfibrils, which are essential to the formation of elastin in the aorta, suspensory ligaments of the lens of the eye, and other connective tissues. Decreased microfibril formation results in the release of excessive growth factor and decreased elasticity in many tissues.

A person develops endocarditis one month after a cystoscopy. It is catalase-negative, non-hemolytic, gram-positive cocci. What is the pathogen? A. E faecalis B. S aureus C. S bovis D. S epidermidis E. S pneumoniae

A. E faecalis Recent GI/GU infx or procedure: Enterococcus (E. faecalis) Enterococcus can get into bloodstream and is common in GI/GU areas. KNOW THAT CHART IN THE IMAGE!!

A 15-year-old girl presents for a CBC. She is of North African descent and is a lifelong ovo-lacto vegetarian, who recently became vegan. She has no prior medical history and there is no family history of anemia. RBC = 3.5 (3.8-5.2) Hemoglobin = 10 g/dL (11.5-16.0 g/dL) Hematocrit = 30% (34-47%) MCV = 70 fL (80-100 fL) MCHC = 29 g/dL (30-36 g/dL) Reticulocyte count = 0.5% (0.5-1.5%) WBC = 6.0 (4-10) What is the most appropriate next step? A. Trial of iron therapy for probable iron deficiency anemia B. Hemoglobin electrophoresis for probable thalassemia C. Evaluation of serum B12 and folate for probable megaloblastic anemia D. Systemic workup for infection and autoimmune diseases to explain her anemia of chronic disease E. Bone marrow biopsy to evaluate probable aplastic anemia

A. Trial of iron therapy for probable iron deficiency anemia This is classic Iron Deficiency Anemia - Microcytic, Vegan / Vegetarian Diet, If there's microcytic anemia, especially in a female, you should think IDA unless you can prove otherwise. Image was given too -- hyperchromatic

What is meant by Incomplete Penetrance?

An individual who has the genotype for a disease may not exhibit the disease phenotype at all, even though he or she can transmit the disease gene to the next generation. Ex: In image, you can see skipped generations. Autosomal dominant, but may not penetrate enough to show up in the phenotype. But can still transmit the defect to the next generation. Shows up sporadically. Penetrance rates are estimated by examining a large number of families to determine what proportion of obligate carriers (AD) or homozygotes (AR) develop the disease phenotype.

A 40-year-old woman with rheumatoid arthritis has felt tired and weak for several months. She attributed this to her autoimmune disease. However, in the past week, she has begun feeling very short of breath even walking to the grocery store; something she was able to do easily before. Her husband also noticed that she has been very pale. On physical exam, she has conjunctival pallor but an otherwise baseline physical exam. LABS: Low RBCs Low MCV Decreased serum iron Increased ferritin Decreased TIBC 1. Diagnose this Anemia. 2. Explain these diagnostic lab findings. 3. Explain the pathogenesis of this type of anemia.

Anemia of Chronic Inflammation/Disease Normocytic or microcytic anemia (microcytic in longstanding disease) Pathophysiology: Chronic inflammation (such as from CKD or an autoimmune disease like SLE) increases release of hepcidin from the liver via inflammatory cytokines. > This inhibits ferroportin, which prevents iron transfer from macrophages to erythroid precursors. Thus serum iron is low (doesn't make it into RBCs), but Ferritin (intracellular storage form of Fe) is high, which means they have enough iron (ruling out IDA). Low TIBC may mean: Anemia due to red blood cells being destroyed too quickly (hemolytic anemia), decrease in red blood cells from the intestines not properly absorbing vitamin B12 (pernicious anemia), etc. Your first step is to determine what the abnormal lab values mean. The low RBCs and low MCV means it's a microcytic anemia. The low serum iron level, low total iron-binding capacity (TIBC), and elevated ferritin level suggest anemia of chronic disease (ACD). ACD is further supported by the patient's fatigue. In chronic disease, inflammation leads to the release of hepcidin, an acute-phase reactant that impairs the transfer of iron from macrophages to erythroid precursors. Both SLE and CKD cause decreased serum erythropoietin levels. EPO administration is the proper treatment for this patient as it will stimulate RBC proliferation in bone marrow and exogenous administration can help treat CKD-induced ACD.

O2 saturation curve of Hemoglobin

As deoxygenated hemoglobin binds O2, significant structural changes take place throughout the molecule, and the affinity for O2 increases The O2 saturation curve is sigmoidal, denoting cooperative binding kinetics - after one O2 binds, subsequent O2 molecules bind more readily These allosteric properties result from specific interactions between the subunits

A 62-year-old man comes to the physician because of dizziness, especially after getting up from bed in the morning. He is a non-smoker with a 5-year history of mild hypertension. His treatment regimen is hydrochlorothiazide and a salt-restricted diet. On physical examination, his mucous membranes are dry and his skin shows slightly decreased turgor. When he stands, his heart rate increases. Why does HR increase when standing up? A. Brain needs more blood B. Heart has less blood to pump C. Legs are further away from the heart D. More leg muscles are in use

B. Heart has less blood to pump - blood is in legs now - heart compensates for CO by increasing HR. When you stand up, blood flow increases to the legs (due to gravity), meaning less blood to the heart (decreased preload). When blood to the heart is reduced, blood coming out of the heart (stroke volume) is also reduced (= Frank-Starling Mechanism). The heart must maintain CO since the stroke volume will be lower, so it increases heart rate.

+++++++++++++++++++++++++++++++++++++ What mutation is common in CML? What genetic test can quickly detect this?

BCR-ABL fusion FISH - targets and highlights a particular gene

A 58-year-old woman who has not had healthcare for past 15 years comes to a free clinic and is diagnosed with hypertension and mild congestive heart failure. Appropriate pharmacological treatments are started. During a follow-up after 3 months, elevated levels of blood glucose and uric acid are noted. What is the most likely agent that caused these adverse effects? A. Digoxin B. Furosemide C. Hydrochlorothiazide D. Lisinopril E. Metoprolol F. Spironolactone

C. Hydrochlorothiazide Thiazides are the first line antihypertensives. Can cause Hyperglycemia. Antihypertensive treatment choice may in part be based on co-morbidities. Besides being first-line drug, diuretics may help with the congestive heart failure as well.

Diagnose: A 67 year old male presents to the emergency department complaining of chest pain. On physical exam you note hypotension, JVD, tachycardia and a decrease in pulse wave during inspiration. His EKG shows Electrical alterans (beat to beat variation in peak height).

Cardiac Tamponade

What is the most common cancer that spreads to the liver?

Colon cancer GI blood drains to liver

Repro Question 20 1. Maternal antibodies are able to cross the placenta and circulate within the fetal blood and protect the child after its born from pathogens while the child's immune system is still developing. For how long after the child is born will these protective maternal-derived antibodies last in the child's serum? A. One month B. 1-3 months C. 6 months D. 6-12 months E. 24 months 2. Only maternal Ig__ is able to cross the placenta.

D. 6-12 months Maternal antibodies are transferred from mother to child and protect neonates and infants during the time of maturation of their immune system (6-12 months). Maternal IgG crosses the placenta, but not IgM and IgA.

A 36-year-old woman presents after sudden onset-right neck pain that occurred about three weeks ago after a traumatic fall. Since then, she has noted continued pain from her neck that radiates down her right arm into her hand. Examination of the right arm shows weakness of elbow extension, wrist extension, and forearm pronation. Reflexes are +2 in the bilateral biceps, bilateral brachioradialis, and left triceps. The right triceps reflex is absent. Examination of sensation reveals reduced sensation of light touch and pinprick along the right middle finger. What is the most likely diagnosis? A. Median Neuropathy B. Ulnar Neuropathy C. Radial Neuropathy D. C7 radiculopathy E. C8 radiculopathy

D. C7 radiculopathy C7 is responsible for the triceps reflex. C7 dermatome is middle finger.

Which has the fastest conduction? A. Atrial myocyte B. AV node C. SA node D. Purkinje fiber E. Ventricular myocyte

D. Purkinje fiber Conduction = movement of electrical activity from one cell to the next. SA node fires the fastest though. But that doesn't matter if the later parts aren't ready.

An old man has poorly-controlled hypertension but no symptoms. Most likely auscultatory finding? A. Crescendo-decrescendo ejection murmur B. Fixed S2 split C. Holosystolic murmur D. S4

D. S4 Hypertension often slowly leads to LVH. Mnemonic is 4H - S4 for Hypertrophy. S4 indicates right / diastolic HF. buh-DUH..buh.....buh-DUH..buh

Which of the following plays a role in affinity maturation of antibody binding after antigenic stimulation of B cells? A. Generation of junctional diversity B. VDJ recombination C. Isotype switching D. Somatic hypermutation E. Alternative RNA splicing

D. Somatic hypermutation • Somatic hypermutation occurs during B cell proliferation post-stimulation, introducing point mutations in the CDRs of antibodies Somatic hypermutation (or SHM) is a cellular mechanism by which the immune system adapts to the new foreign elements NOT: • B cells expressing higher affinity antibodies are selected for expansion - isotype switching does not influence antibody affinity • VDJ recombination and generation of junctional diversity take place before antigen stimulation • RNA splicing isn't involved in somatic hypermutation or affinity maturation

A 2-year-old girl inhales a peanut that lodges in a peripheral airway. What physiological change would you expect to see in the peanut-occluded pulmonary unit? A. Decreased PCO2 B. Increased PO2 C. Increased blood pH D. Vasoconstriction E. Increased hemoglobin concentration

D. Vasoconstriction In the occluded lung units, there is no or very little ventilation, so the alveolar air is hypoxic, leading to hypoxic vasoconstriction.

Which of the following anti-hypertensives could cause severe bradycardia if taken in excess? A. Captopril B. Hydrochlorothiazide C. Minoxidil D. Verapamil

D. Verapamil Verapamil is a CCB works on the heart!

1. Name the most common germ cell tumor of both the testis and ovaries. (different name in M vs. F) (most common testicular tumor overall in males) 2. What is a big risk factor for this testicular tumor in males?

Dysgerminoma (Females) Seminoma (Males) In males, presents as painless enlargement of the testes. Fried-Egg appearance on histology. Cryptorchidism (undescended testicle/s) is a big risk factor for Seminomas.

A 5-year-old boy was admitted to the hospital with pneumonia. His mother reports that since approximately 1 year of age he has suffered from recurrent bouts of otitis media, sinusitis, and pneumonia requiring treatment with antibiotics. Because of your concern of a possible primary immunodeficiency, you obtain a complete blood count which shows a normal absolute lymphocyte count, but low absolute neutrophil count (neutropenia). In addition you obtain quantitative immunoglobulins which show an IgG level of 25 (normal range 600-1500 mg/dl), absent IgA, and an increased IgM level of 300 mg/dl (normal range 75-150 mg/dl). A lymph node biopsy shows absence of secondary follicles and germinal centers. What is the most likely diagnosis? A. X-linked severe combined immunodeficiency disease (SCID) B. X-linked agammaglobulinemia C. Common variable immune deficiency D. X-linked chronic granulomatous disease E. X-linked hyper IgM syndrome

E. X-linked hyper IgM syndrome • This is a classic presentation of Hyper-IgM syndrome • Loss of isotype-switched immunoglobulins and increased IgM because the B cells can't make anything else. • The lack of germinal centers is also characteristic, but not definitive.

Classic x-ray appearance of: - Emphysema - Chronic Bronchitis Include appearance/orientation of heart in each.

Emphysema has a really low diaphragm, more space, and is more clear. Heart appears more vertical than horizontal. Chronic Bronchitis has infiltrates throughout the lung, the diaphragm is higher and rounder, and the heart appears more horizontal than vertical.

+++++++++++++++++++++++++ A 27 year old woman presents to her doctor complaining of painful intercourse and painful defecation. Additionally, she states that her and her husband have been trying to get pregnant for well over a year. She wonders if the symptoms are related to her perceived sterility. She has chocolate cysts on her uterus. What is the most likely diagnosis?

Endometriosis Endometrial glands and stroma occurring outside of the uterine cavity = diagnostic of endometriosis Chocolate cysts = Endometriosis Classic cause of infertility.

++++++++++++++++++++++++++++++ Flow vs. Volume curve changes for: - Obstructive lung diseases - Restrictive lung diseases

FIRST, note that volume increases as you move to the left. TLC is on the Y-axis. Also, the zero point for Flow (Y-axis) is in the middle, between inspiration and expiration. Obstructive: COPD, Asthma - Shorter and to the left - Higher Residual Volume (can't exhale all air) - Higher TLC - Decreased flow (shorter range on Y-axis) - Can't exhale as much, but volume is never down to zero. Restrictive: Fibrosis, etc. - Slightly shorter but still tall; to the right - Lower Residual Volume - Lower TLC - Can't inhale as much, but can exhale plenty.

Diagnose this vitamin deficiency: HIGH Homocysteine Methylmalonic acid levels are normal Bloodwork reveals megaloblastic anemia. Risk Factors: Unpasteurized goat milk in infants Alcoholism Overcooking of foods Chronic diarrhea and tropical sprue Celiac disease Methotrexate Pregnancy Chronic hemolytic anemias (SS, SC, HS)

FOLATE DEFICIENCY (Folic acid / Vit B9)

What are the main differences between Minimal Change Disease and Focal Segmental Glomerulosclerosis?

MCD • Mostly seen in children • Usually corticosteroid-responsive • Normal light microscopy FSGS • Mostly adults • Often Steroid Refractory • Focal sclerosis on LM

Anorexia: Treatment options

Medical treatment/admission: for dehydration, starvation and electrolyte imbalances - Inpatient treatment, sometimes for months, for those >20% under expected body weight - Psychiatric commitment may be required if they refuse Cognitive-behavioral therapy Family therapy - This and schizophrenia only disorders with family therapy indication on NBME exams

Which glomerular disease shows spikes in the GBM and IgG deposits on immunofluoresence?

Membranous Glomerulopathy IgG deposits on the GBM, causing spikes. Caused by antibodies (IgG) to the PLA2 Receptor LM—diffuse capillary and GBM thickening IF—granular due to immune complex (IC) deposition EM—"Spike and dome" appearance of subepithelial deposits

+++++++++++++++++++++++++++++++++++++++++++++ Brainstem: What cranial nerves and arteries are at each level? (Worm Diagram) - Midbrain - Pons - Medulla

Midbrain: 3, 4 - PCA Pons: 5, 6, 7, 8 - Basilar a. - AICA Medulla: 9, 10, 11, 12 - PICA - ASA Medial CN nuclei: Motor Lateral nuclei are Sensory

A 58 year old, floridly diabetic man, who has had many episodes of DKA, and an A1c of 13, presents to your emergency department with a black discoloration of his face, severe sinus pain, and a bloody discharge from his left eye. What is the most likely infectious organism?

Mucormycosis Think, Mucormycosis attacks Mucous membranes around face. Common in diabetics.

Best treatments for gout

NSAIDs are first line -- any NSAID should work. This will slow the inflammatory process occuring in the toe / joint. Allopurinol is also effective, and definitely should be started if the pt has tophi (nodules).

What organ produces and secretes HCO3- rich fluid into the duodenum?

Pancreas

Effect of Scleroderma on the Lungs: - Alveoli - Expected DLCO, lung volumes

Pulmonary disease occurs in 70% of Scleroderma patients. Serositis Interstitial lung disease: - Alveolitis- inflammation in alveoli - Interstitial fibrosis- presumably due to alveolitis - Pulmonary Hypertension- 30 - 35% in diffuse PSS up to 50% in limited PSS Low DLCO Low lung volumes on PFTs Respiratory failure Most pulmonary hypertension patients will develop dyspnea.

++++++++++++++++++++++++++ For each, is it a sign of Pericarditis or Cardiac Tamponade? - Pulsus Paradoxus - Kussmaul Sign - JVD present - ECG shows electrical alterans - ECG shows ST elevations - Muffled/Distant Heart sounds on auscultation - Friction rub on cardiac auscultation

Pulsus Paradoxus: Cardiac Tamponade - BP decreases more during inspiration (due to exaggerated L=>R blood shunting during inspiration) Kussmaul Sign: Pericarditis - JVD gets worse during inspiration Jugular Venous Distention (JVD): BOTH - Both cause back flow of blood from heart into the jugular because there's less room in the heart. ECG: - Electrical alterans: Cardiac Tamponade - ST elevations: Pericarditis Heart Sounds: - Muffled: Cardiac tamponade - Friction rub: Pericarditis

Angina: - Stable - Prinzmetal's - Unstable

Stable (atherosclerosis, pain with exertion) Prinzmetal's (vasospasm, pain at rest) Unstable (thrombosis, pain increasing)

Structure of a collagen molecule Common diseases caused by collagen deficiency

Three separate a chains super-twisted about each other, in a right-handed sense, with the glycines in the shared interior. - Osteogenesis imperfecta - Ehlers-Danlos syndrome - Scurvy (Vitamin C deficiency) - Menkes disease

Autoimmune Hemolytic Anemia (AIHA) - Warm vs. Cold - which Ig is elevated in each? - what type of test detects Ig's

Warm type: IgG Cold type: IgM - Caused by mycoplasma and EBV Detect Ig with Coomb's

A 35 year old man presents to his doctor with complaints of urinary urgency and frequency. He thinks he has a low grade fever and he states that it does hurt when he pees and that he can't sit in hard chairs because it is tender "in that area." He also reports lower back pain. DRE reveals an enlarged, tender prostate. What bacteria most commonly cause Acute Prostatitis in: - Young men - Older men Dr. Quick GU Review

Young - Chlamydia and Neisseria - STDs -- more likely in young people. Old - E. Coli (or Pseudomonas, Klebsiella) Of note: Chronic prostatitis is more common than acute. It is usually abacterial, although some cases are caused by recurrent infection. It can occur in bicycle riders.

Classic translocation of Mantle Cell Lymphoma Classic buzzword for MCL: ___ lymphadenopathy

t(11;14) - Activates Cyclin-D1 BUZZword: Painless lymphadenopathy

All about Multiple Myeloma - Most common translocation. - MNEMONIC for features and symptoms: CRAB - Should see what on electrophoresis? - Antinody results

t(11;14) - same one that causes mantle cell lymphoma involves production of monoclonal antibodies: IgG C - hyperCalcemia - corrected Ca > 11.5 - caused by the breakdown of bone R - Renal insufficiency: Cr > 2 - Light chain cast nephropathy - Pyelonephritis - urinary urgency; dysuria; nocturia A - Anemia: Hb < 10 - normocytic and normochromic - Fatigue, pallor - BM infiltration - Decreased EPO from renal failure - Rouleaux B - Bone lesions (pain) and Bone marrow infiltration: - Commonly back pain (progressively worsening back pain) - Osteopenia, lytic lesions, pathologic fractures, or severe pain Should see an M-spike on serum electrophoresis - M-Spike for Multiple Myeloma The increased total protein and the increased calcium levels seen in this patient's lab values are also signs of multiple myeloma. The increased breakdown of bone experienced by patients with multiple myeloma results in increased serum levels of calcium. Multiple myeloma is a neoplastic proliferation of plasma cells, which produce immunoglobulins. The molecule most commonly produced by the plasma cells is immunoglobulin G or IgG. In multiple myeloma, the proliferation of plasma cells results in production of large amounts of IgG that causes increased total protein. E.g., You see a 67-year-old man for a routine yearly physical exam and he complains that he feels tired all the time and he has a tingling sensation in his feet at times. He is generally healthy, and the only medication that he takes on a daily basis is a long-acting NSAID for increasingly bothersome back pain. Physical exam is remarkable only for pallor and some tenderness to percussion over the lumbar spine. Screening labs reveal a normal WBC and platelet count, a mild normocytic normochromic anemia with a Hb of 9.1 g/dL, an elevated serum calcium of 12.1 mg/dL, and an elevated serum creatinine of 2.3 mg/dL.

21-Hydroxylase vs. 11-Hydroxylase vs. 17-Hydroxylase Deficiencies: - 17-ketosteroids - 17-Hydroxyprogesterone - 17-Hydroxycorticoid - Mineralocorticoids - Presentation

.

CO vs. EDV (or RAP) plot: - Increased Contractility or Decreased Afterload - Increased Afterload or Decreased Contractility

.

Cardiac Function Curve -- What does increased Preload look like on a SV (or CO) vs. EDV (or RAP) plot?

.

High Yield CV Equations to know for STEP

.

>95% of scleroderma patients have an antinuclear antibody. What are the 3 antibodies that can be positive in Scleroderma?

>95% of patients with PSS have an ANA 1. anti-Centromere antibody indicates patients with limited PSS. 2. anti-Scl-70 (anti-topoisomerase 1) antibody indicates patients with diffuse PSS and is associate with a worse prognosis. This antibody is directed against topoisomerase I, an enzyme that participates in the initial uncoiling of DNA prior to transcription. 3. anti-RNA polymerase III antibody- associated with renal crisis

A 75-year-old man presents to his primary care physician with worsening breathlessness after walking up the stairs and the need to sleep propped up by an increasing number of pillows. On physical exam, the physician finds an early diastolic decrescendo murmur heard best at the left sternal border and crackles over both lower lung fields. The patient is sent for an echocardiogram, which shows an enlarged left ventricle. Which is the most likely etiology for this patient's symptoms? A. Acute myocardial Infarction B. Aortic valve regurgitation C. Aortic valve stenosis D. Chronic hypertension E. Mitral valve regurgitation

B. Aortic valve regurgitation Early diastolic decrescendo murmur = AR It's the sound of blood regurgitating back into the LV after systole, thus in early diastole. Also c/w enlarged left ventricle -- blood is flowing back into the LV from the aorta.

What is consanguinity? It increases the risk for getting genetic diseases with what inheritance pattern?

Consanguinity = Inbreeding Increases the risk for Autosomal Recessive disease

+++++++++++++++++++++++++++++++++++++ Schistoma (blood fluke) infections are a major risk factor for ___ cancer.

Bladder cancer!!!

A 5 year old boy presents to the ED with fever, vomiting, and cough. Physical Exam shows: - abscess on his right neck that is red and painful -gingivitis Chest X-ray: large "cotton ball" appearing lesions. Fine needle aspiration showed the numerous aspergillus fungal hyphae when aspirate stained. Culture confirmed the presence of Aspergillus fumigatus. He was admitted and placed on antifungal therapy, however, his vomiting continued. An upper GI series was performed and demonstrated narrowing at the distal portion of the esophagus. 1. Diagnose 2. This patient has a genetic defect of what oxidizing agent, resulting in failure to oxidize and kill pathogens? 3. Classic signs of this disease

Chronic Granulomatous Disease This is the prototypical phagocytic disorder (along with leukocyte adhesion deficiency and chediak higashi) This immunodeficiency state represents a defect in phagocytic cells. These cells can ingest but cannot kill bacteria due to a failure to form bacteriocidal oxygen radicals by the cell due to lack of NADPH. • Gene defect in NADPH system (2/3 XL, 1/3 AR) • Phagocytic cells ingest but do not kill bacteria due to failure to generate oxidative burst Classic Symptoms: • Recurrent fungal and bacterial infection with catalase positive bacteria (Staph aureus, gram negatives, fungi) • Granulomas of skin, liver, lungs, lymph nodes, GI, GU - Lymphadenopathy, HSM, gum disease

Repro Question 23 A 30-year-old female patient arrives at your office because of missed menstrual periods for 2 months. Her history indicates regular menstrual periods in the past. During physical examination, you suspect that she may be pregnant. Which laboratory values would be compatible with your diagnosis? A) Low plasma progesterone and high LH B) High prolactin, low LH and low progesterone C) High urinary estradiol and low progesterone D) High urinary hCG and high plasma progesterone

D) High urinary hCG and high plasma progesterone Early pregnancy is associated with high levels of hCG (basis of pregnancy test) and high levels of progesterone due to "hCG rescue" and maintenance of the corpus luteum, which continues secreting progesterone to maintain the uterus / prevent menstruation.

A patient who recently received a bone marrow transplant presents to your clinic with mental status changes. What is the most likely infectious cause? A) Paracoccidiomycosis B) Coccidiodomycosis C) Histoplasmosis D) Cryptococcus

D) Cryptococcus Key will be an immunocompromised patient with mental status changes. Pathology findings: They have a thick capsule that will not stain with india ink (see picture)

What Complex (I-IV) of the Electron Transport Chain is inhibited by each of these poisons? - Arsenic (Both arsenate and arsenite) - 2,4-Dinitrophenol - Rotenone - Antimycin A - Sodium Azide - Carbon monoxide - Cyanide - Oligomycin

DRAAACO DR A ACO

Which glomerular disease is described? - Leading cause of end-stage renal disease (ESRD) in the United States. - Micro albuminuria - Proteinuria- usually nephrotic range - Nephrotic syndrome - 10% can have hematuria - Acute renal failure uncommon but CKD is very common - Mesangial Expansion seen on LM - Nodular glomerulosclerosis -- large K-W nodules seen on histology

Diabetic nephropathy Hyperglycemia => glycation of tissue => mesangial expansion, GBM thickening, and increased permeability. Hyperfiltration (glomerular HTN and decreased GFR) => glomerular hypertrophy and glomerular scarring (glomerulosclerosis) => further progression of nephropathy. LM—Mesangial expansion, GBM thickening, eosinophilic nodular glomerulosclerosis (Kimmelstiel-Wilson nodules)

Laryngeal Papillomatosis: - Presentation - Caused by what pathogen? - Histology? - Appearance on scope?

Difficulty breathing Ex: A 15 year old presents to the emergency department with difficulty breathing Human Papilloma Virus - HPV 6 or 11 Papillomas in the larynx can be seen with a scope (Image).

A 56-year-old woman comes to the physician because of shortness of breath. She has smoked 2 packs of cigarettes daily for 37 years. Pulmonary function tests show the following pulmonary volumes and capacities: • Functional residual capacity 4.5 L • Inspiratory reserve volume 1.5 L • Inspiratory capacity 2.0 L • Vital capacity 3.0 L Which of the following is the residual volume of this patient? A. 1.5 L B. 2.0 L C. 2.5 L D. 3.0 L E. 3.5 L F. 4.0 L

E. 3.5 L TLC = FRC + IC = 4.5 + 2 = 6.5 TLC = VC + RV = 6.5 RV = 6.5 - VC = 6.5 - 3 = 3.5

++++++++++++++++++++++++++++++++ HPV Viral Proteins: - E6 blocks - E7 blocks

E6 blocks p53 E7 blocks RB-E2F 6 before 7, P before R It is important to recognize that p53 exerts a positive effect on p21, which is a cell cycle inhibitor! If you lose p53 then you lose p21 which leads to cell cycle progression Also note that E7 directly inhibits p21 as well, making it an important target!

How to recognize an Autosomal Dominant pedigree

Each child of a carrier of the abnormal allele stands a 50 percent chance of inheriting the allele and the associated disease. Males and Females are equally affected. Fathers can pass it to sons and daughters. Remember, these are autosomal chromosomes, not sex chromosomes. Has nothing to do with how many X chromosomes you have. Frequently passed on, unlike recessive.

What causes Hemolytic Disease of the Newborn?

Either Rh reaction or an ABO reaction (O type mother with AB type fetus) Rh neg. mothers should be given RhoGAM: Anti-D IgG Pathogenesis/Mechanism: • Rh factor is a red blood cell antigen • When an Rh-negative mother is pregnant with an Rh-positive fetus, the mother is exposed to Rh-positive red blood cells and leads to maternal antibody production (IgG) against the foreign Rh antigen • As IgG can cross the placenta, subsequent pregnancy with Rh-positive fetus will result in fetal alloimmune-induced hemolytic anemia • Breakdown of red blood cells causes elevation of bilirubin => Jaundice.

What is meant by Variable Expression?

Even when penetrance of a condition is complete, the severity of the disease may vary greatly. Possible causes: environmental factors, modifier genes.

A 54-year-old man has chronic hepatitis C infection with an elevated serum alanine aminotransferase. His liver architecture now includes bridging fibrosis with hepatocyte regeneration. A clone of hepatocytes has an acquired mutation leading to loss of growth control. These cells also have poor MHC class I expression. This abnormal clone of hepatocytes is most likely to be destroyed via which of the following cells? A. B lymphocyte B. CD4 T lymphocyte C. CD8 T lymphocyte D. Macrophage E. Neutrophil F. NK cell

F. NK cell NK cells are programmed to respond and destroy cells that fail to properly display MHC class I antigen. If a virus downregulates MHC expression, or a neoplastic cell functions abnormally in MHC expression, then such abnormal cells are attacked by the NK cells.

++++++++++++++++++++++++++ If you ever get a question about a young person who receives a drug then becomes anemic the answer is what enzyme deficiency?!?

G6PD deficiency Results in lack of glutathione, which is needed to remove ROS's. Thus, ROS's proliferate and destroy RBCs. Causes jaundice too since RBCs are being broken apart.

The effects of Scleroderma on the GI tract: - Mouth - Esophagus - Intestines

GI is the 2nd most common organ system involvement, after skin. Face Decreased oral aperture (can't open mouth as wide) Esophagus - Esophageal dysmotility [decreased peristalsis of the lower 1/3 (smooth muscle portion)] causing dysphagia. Small Intestine Chronic obstruction due to decreased peristalsis Diarrhea due to bacterial overgrowth GI bleeding from telangiectasias Malabsorption Large Intestine Diverticula GI bleeding from telangiectasias Liver Primary biliary cirrhosis

What biological compound causes the classic signs of rubor, calor, and tumor as it causes vasodilation (rubor and calor) and then vascular permeability (tumor) by contracting vascular endothelial cells? What type of cell releases this compound?

Histamine - Mast cells release histamine (in response to insult: infection immune, burn, trauma, necrosis, etc.)

A 36-year-old female has been visiting her primary care physician very frequently over the past seven months because she is afraid she has an underlying illness. She has been experiencing mild, nonspecific abdominal pain off and on throughout this time. Despite reasssurances by her doctor, as well as a normal abdominal ultrasound and abdominal CT scan, she is still convinced something is being missed. Because of her fear, she has become socially withdrawn and spends much of her time at home researching diseases on the internet. What is the most likely diagnosis?

Illness Anxiety Disorder (Hypochondriac) ✔ Somatic symptoms are absent or mild ✔ Preoccupation with having or acquiring a serious illness (will say diagnosis: "I have cancer/heart disease/etc.") ✔ High anxiety and easily becomes alarmed about health ✔ Excessive or maladaptive health behavior, such as hospital visits.

How to tell if it's Left or Right Heart Failure

In heart failure, blood is not pumped forward so it backs up. Congesting the lungs (backing up from the left heart) or the systemic circulation (backing up from the right heart). Left heart failure = pulmonary congestion: orthopnea, paroxysmal nocturnal dyspnea, pulmonary edema, pleural effusion, heart failure cells. General dyspnea can be from many things but orthopnea points to HF. Cough and hemoptysis were also mentioned by Dr. Smith. Right heart failure = systemic congestion: jugular venous distension, hepatomegaly (nutmeg liver), peripheral edema Right HF is usually from Left HF. Right HF resulting from a primary disorder of the respiratory system is called "cor pulmonale".

Oxygen Dissociation Curve: - MNEM for what causes a right shift: CADET, face RIGHT - Anemia causes a shift to what direction - Which direction of shift means a higher O2 unload? Lower?

Increase in pretty much anything -- 2,3-DPG, H+, temp, etc. -- causes a right shift. Easy to remember that. "CADET, face Right!" for CO2, Acid, 2,3-DPG, Exercise, Temperature Severe anemia = reduced O2 affinity (right shift) enables a higher O2 unload, a critically important compensation.

+++++++++++++++++++++++++++++ A patient presents with a fever, diffuse myalgias, abdominal pain, a peripheral neuropathy, as well as hypertension and a recent weight loss of 10 pounds. The patient has a past medical history of hepatitis B and C. Angiography demonstrates a "string of pearls appearance" in the renal artery, as well as in several other organ systems with the exception of the pulmonary arteries.

Polyarteritis Nodosa ✔ Medium sized muscular arteries ✔ Necrotizing immune complex mediated ✔ Affects renal and visceral vessels ✔ Aneurysm and constrictions on arteriogram ✔ Very rare p-ANCA association Most common association? Hep B.

Takayasu Arteritis (Pulseless Disease): - Presentation - Pathogenesis

Rare vasculitis of aorta and large branches that affects adolescent girls and young (< 40 year old) women (especially japanese). Transmural thickening of arteries Affects aorta and large branches: Aortic arch causes aortic insufficiency or affects branches Abdominal aorta causes claudication, HTN Common carotid artery causes strokes Renal artery causes HTN Subclavian artery causes arm claudication, pulseless arm Vertebral artery causes dizziness

A 20-year-old woman presents to her primary care physician's office for discolored fingers when in the cold. She reports that this is not painful, but the sight of her blue fingers bothers her. She reports that it becomes white first, blue, and finally red when she rewarms her hands. Although she has tried wearing gloves, she still reports having this discoloration. She has no significant past medical history. Her physician prescribes her nifedipine.

Raynaud's Disease Discoloration in extremities due to an exaggerated physiologic response to cold temperatures or emotional stress. Caused by reduced blood flow to extremities. Called Raynaud's phenomenon when due to SLE / CREST

Diagnose this vaginal cancer: - Infants and children under 5 years of age. - Rapidly-growing polypoid grape-like mass on anterior wall of vagina. - Histology shows Strap cells (rhabdomyoblasts) trying to recapitulate skeletal muscle

Sarcoma Botryoides

Mnemonics for effects and features of a Dilated Heart vs. a Hypertrophic Heart.

See IMAGE

10 key points: Name the main, distinguishing features of Types 1-4 Hypersensitivities.

See Image

Middle cerebral artery (MCA) stroke - Clinical Picture

See Image Most common stroke MCA supplies A LOT of the brain, including the motor and sensory cortices and the language centers.

You have been referred a 25-year-old woman that has suffered a seizure. Temp 36.9 C HR 67 BP 127/87 Na 129 mEq/L (LOW) K 3.5 mEq/L Ca 9.7 mg/dl glucose 90 mg/dl Plasma osmolality 262 mOsmol/kg (LOW) Urine osmolality 277 mOsmol/kg (HIGH) 1. Diagnose 2. Treatment

The woman is suffering from SIADH. Na+ is lost in the urine and water is excessively reabsorbed: - High urine osmolality (due to more Na+ and less water in urine) - Low serum Na+ - Low plasma osmolality (due to low Na+ and more water) So concentrated and salty urine, and dilute blood. Treatment is fluid restriction and salt administration + ADH antagonists: conivapton, tolvaptan

Leukocyte extravasation (getting out of vessels) process - MNEM - Lick the Rat DM

• Rolling - E & P selectin (vessel) \ Sialyl Lewis (WBC) • Adhesion • Tight binding - ICAM-1 \ LFA-1 (WBC integrin) • Diapedesis - PECAM-1 (vessel and WBC) • Migration - WBC drawn to microbes by LICK (LTB4, IL-8, C5a, Kallikrein)

DSM-5 criteria for Paranoid Personality Disorder (So you can be able to rule it out or know for sure on STEP)

• Tendency to be suspicious, mistrustful, hypervigilant, and preoccupied with being exploited or betrayed • Hostile, irritabile, avoidant (b/c don't trust others) • Can have anxiety, often secondary to the paranoid beliefs. • Self-fulfilling prophecies: mistrustful behavior causes others to act in an overly cautious or deceptive way. • Ex: "You doctors and the government are always trying to screw over people like me."

1. A 27-year-old woman presents to her primary care physician due to headache, chest pain, and food intolerance. These complaints have been very distressing for her and have been present for approximately 8 months. She previously had seen a headache specialist, gastroenterologist, and obtained a number of electrocardiograms in the emergency department. Their respective thorough work-up was negative. On physical exam, the patient appears healthy and is otherwise unremarkable. Diagnosis? (A) Illness Anxiety Disorder (B) Somatic Symptom Disorder (C) Conversion Disorder (D) Factitious Disorder (E) Malingering 2. A 25-year-old woman presents to the emergency room reporting inability to move her right leg since this morning. When the patient is asked to raise the leg, there is no downward pressure felt under the left heel (Hoover's sign). Neurologic exam is otherwise unremarkable. She reports that she is supposed to visit her stepfather, who sexually abused her as a child, later that day. Diagnosis? (A) Illness Anxiety Disorder (B) Somatic Symptom Disorder (C) Conversion Disorder (D) Factitious Disorder (E) Malingering

1. (B) Somatic Symptom Disorder - Somatization is the psychological mechanism whereby psychological distress is expressed in the form of physical symptoms. The psychological distress in somatization is most commonly caused by a mood disorder that threatens mental stability. 2. (C) Conversion Disorder - Conversion disorder occurs when the somatic presentation involves any aspect of the central nervous system over which voluntary control is exercised.

What cortisol synthesis pathway enzyme is likely deficient for this unique set of symptoms: High blood pressure (hypertension) Low levels of cortisol and androgens in the blood Delayed sexual development / lack of androgens (amenorrhea in a young teen female is a CLASSIC presentation)

17α-Hydroxylase Cortisol and Androgens are low. Aldosterone is not low here - see Image -- cholesterol can take that downwards pathway to get to aldosterone without using 17α-Hydroxylase.

21-hydroxylase deficiency results in deficiency of what 2 hormones? In "adrenogenital hyperplasia" due to 21-hydroxylase deficiency: a. Adrenocorticotropic hormone (ACTH) secretion is decreased b. Cortisol production is decreased c. Androgen production is decreased d. The plasma concentration of dehydroepiandrosterone-sulfate (DHEA-S) is decreased e. The urinary 17-ketosteroid concentration is decreased

21-Hydroxylase Deficiency effects: ↓ Cortisol ↓ Aldosterone ↑ ACTH release (attempt to increase cortisol and aldo) b. Cortisol production is decreased STEP1 always asks about enzyme deficiencies in the cortisol synthesis pathway. The main enzyme deficiency is 21-hydroxylase deficiency. These pts tend to have hypotension and hyperkalemia due to low aldosterone.

+++++++++++++++++++++++++++++++ What will be the effect of 21-Hydroxylase Deficiency? - Cortisol, and how this presents - Aldosterone, and how this presents - Androgens, and how this presents - ACTH, and how this presents

21-Hydroxylase Deficiency is the most common adrenal enzyme deficiency. • ↓ Cortisol (leading to ↑ ACTH release) • ↓ Aldosterone => hyponatremic hypotension, hyperkalemia • ↑ Androgens / testosterone - results in ambiguity of female genitalia at birth precocious puberty in both sexes. - this occurs because more precursors are shunted into the testosterone synthesis pathway. • ↑ ACTH => diffuse skin pigmentation Blood Pressure: High in 17 and 11 deficiency Low in 21 deficiency MedBullets Example: A 5-year-old male visits his pediatrician for a check-up. His height corresponds to the 99th percentile for his age, and pubic hair is present upon physical examination. Serum renin and potassium levels are high, as is 17-hydroxyprogesterone. Which of the following is likely deficient in this patient? A. 17a-hydroxylase B. 11ß-hydroxylase C. 21-hydroxylase D. Aromatase E. 5a-reductase

What hormones will be deficient, and will there be hypertension or hypotension for: - 21-hydroxylase deficiency - 11ß-Hydroxylase deficiency

21-Hydroxylase deficiency: ↓ Cortisol ↓ Aldosterone ↓ BP (Hypotension) 11-ß-Hydroxylase deficiency: ↓ Cortisol ↓ Aldosterone ... but ..... ↑ BP (Hypertension), ↑Na+, and ↓K+ -- because 11-deoxycorticosterone is high and has mineralocorticoid effects -- acts just like aldosterone. Be able to determine the likely enzyme deficiency. Method: - Check BP. >> If hypotensive: 21-hydroxylase. >> If hypertensive: 11ß-Hydroxylase or 17. ACTH synthesis will be high for both of these since cortisol is deficient. Causes hyperpigmentation.

ELANE mutations are associated with developing what blood problem?

A couple types of Neutropenia (lack of neutrophils) So they'll get sick a lot. - Severe Congenital Neutropenia - Cyclic Neutropenia

+++++++++++++++++++++++++++++++ Repro Question 24 Which of the following is a unique property of SERMs? A) Act as agonists in some tissues and antagonists in other tissues B) Activate a unique plasma membrane-bound receptor C) Have both estrogenic and progestational agonist activity D) Inhibit the aromatase enzyme required for estrogen synthesis E) Produce estrogenic effects without binding to estrogen receptors

A) Act as agonists in some tissues and antagonists in other tissues SERMs = Selective Estrogen Receptor Modulators e.g., Tamoxifen, which is an estrogen antagonist in the breasts, but agonist in the uterus. - B is incorrect because SERMs bind to estrogen receptors as natural estrogen ligand - C is incorrect because they do not bind to progesterone receptors and have no progesterone activity. - D is incorrect because SERMS have no androgenic activity - E is incorrect because SERMS bind to estrogen receptors to elicit their agonist or antagonist activity

A 65-year-old woman who has smoked 2 packs per day for forty years comes to your practice complaining of a chronic cough, dyspnea, hemoptysis, and difficulty rising from a chair. A CXR and biopsy reveals small cell lung cancer. All of the following would be consistent with her diagnosis EXCEPT? A. Decreasing muscle strength with repetitive stimulation B. Blurry vision C. Antibodies to presynaptic calcium channels D. Dry mouth E. Orthostatic hypotension

A. Decreasing muscle strength with repetitive stimulation - This would describe myasthenia gravis Small cell lung cancer is associated with Lambert Eaton Syndrome, which is caused by antibodies to presynaptic calcium channels at the NMJ. Symptoms improve with use of the muscles throughout the day. Also involves autonomic symptoms (dry mouth), reduced reflexes Type II hypersensitivity reaction

A 38 year old man has been taking antibiotics for 2 weeks for a severe case of pneumonia. He calls his doctors office complaining to the nurse of worsening fever and a rash. He has also noticed that he has only urinated 2x in the last two days and the urine is dark yellow/brown. What is the most likely diagnosis? MNEM for most common causes - 5 P'S

Acute Interstitial Nephritis / Tubulo-interstitial nephritis (TIN) - Drug-induced hypersensitivity reaction involving the interstitium and tubules of the kidneys. - i.e., Basically allergic reaction to a medication that occurs in the kidneys. Presents as Oliguria, Fever, and a Rash days to weeks after starting a drug, usually NSAIDs, penicillin, PPIs, diuretics, etc. May see eosinophils in the urine (makes sense -- allergic reaction and eosinophil proliferation). MNEM: 5 P'S that cause AIN. Pee (diuretics) Pain-free (NSAIDs) Penicillins (and cephalosporins) Proton pump inhibitors RifamPin Sulfa drugs

Name the one start codon and the 3 stop codons

AUG - mnem - I started in AUGust UAA - U Are Annoying UGA - U Go Away UAG - U Are Gone

Which glomerular disease is described? - Nephritic - Usually kids - C3 deposits - Subepithelial deposits

Acute Post-Streptococcal Glomerulonephritis C3 is from the infection; forms subendothelial deposits (Image)

A 60-year-old woman tells you her hands are getting larger and she can no longer wear her wedding ring. She also indicates she is having some problems with her peripheral vision and is worried about driving. You believe there may be some prognathism and ask her to bring you a picture of when she was younger. Routine screening reveals hyperglycemia.

Acromegaly - ↑ in GH after closure of the epiphyseal plates Gigantism - ↑ GH before closure of the epiphyseal plates ↑ IGF-1, no ↓ of GH following glucose challenge, MRI of pituitary GH is used to treat GH deficiency in children, to ↑linear growth in Prader-Willi Syndrome and Turner Syndrome, and to ↓ wasting in HIV patients IGF-1 (mecasermin) is used to treat Laron Dwarfism In addition to being used for acromegaly, octreotide can also be used to treat carcinoid syndrome, grastinoma, and glucagonoma. Insulinomas are normally removed surgically.

+++++++++ HIGH YIELD, EASY POINTS +++++++++ +++++++++ YOU NEED TO MEMORIZE THESE +++++++++ IMPORTANT CYTOKINES TO KNOW: MNEMONIC: "Hot T-bone stEAK": What illness symptoms or what products are produced by each of these cytokines in an immune response? - IL-1 - IL-2 - IL-3 - IL-4 - IL-5 - IL-6 Neutrophils are recuited by IL-_ to clear infections (MNEM) Interferon-γ (IFN-γ): - Inhibits what type of cell? - Activates what cell to kill pathogens? - Activates __ cells in the case of a virus. - Promotes IL-4 Functions: MNEM: 2 BEG 4 Help - Induces differentiation of T-cells into what type of helper T-cell -- Th1 or Th2? - Promotes production of __ cells. - Enhances Ig class switching from IgM to Ig __ and Ig __.

Acute (IL-1, IL-6, TNF-α), then Recruit (IL-8, IL-12). FYI: They're called interleukins because they go from one leukocyte to another. Acute symptoms: "Hot T-bone stEAK": IL-1: fever (hot). IL-2: stimulates T-cells. IL-3: stimulates Bone marrow. IL-4: stimulates IgE production. IL-5: stimulates IgA production. IL-6: stimulates aKute-phase protein production Recruiters: IL-8 recruits neutrophils to clear infections. ("Clean up on aisle 8") IL-12 recuits natural killer cells and Th1 CD4 cells. Interferon-γ (IFN-γ): ✔ Secreted by NK cells and Th1 CD4+ Helper-T cells ✔ Decreases Th2 cell production - Inhibits differentiation of Helper T-cells into the Th2 type. ✔ Stimulates macrophages - kill phagocytosed pathogens and induce granuloma formation. ✔ Activates Natural Killer cells - kill virus-infected cells. ✔ Promotes Ig class switching to IgG production. ✔ Increases MHC expression and antigen presentation by all cells. IL-4: 2 BEG 4 Help Induces differentiation of T-cells into Th2 Helper T-cells (i.e., Th2 CD4+ T-cells). Promotes growth of B-cells. Enhances class switching to produce IgE and IgG from IgM in B-cells. IL-5: Contributes to proliferation of activated B cells; promotes class switching to IgA (the A Hot T-Bone stEAK)

A 70 year old myeloma patient is receiving a transfusion when he starts to complain of back pain and a feeling that he is going to die (impending doom). The nurse immediately stops the transfusion and initiates a transfusion reaction workup. 1. What type of transfusion reaction is this man experiencing? 2. What type of test would confirm? 3. What is the mechanism of this acute reaction?

Acute hemolytic transfusion reaction What tests should you order? - Coombs' test \ Indirect Coombs' test (AKA DAT) Urinalysis What will the urine look like and why? - It will be red to brown in color because of hemoglobinuria Why do AHTR's happen? - Blood group AG incompatibility or atypical RBC antibodies - Those would cause a pretty immediate reaction.

+++++++++++++++++++++++++++ MI Complications: - Most common cause of death; can occur 1-3 days post-MI - Can cause sudden death 7-10 days later - Friction rub 3-5 days post-MI - Autoimmune fibrinous Pericarditis weeks after an MI is called...

Arrhythmia - 1-3 days, most common cause of death Cardiogenic shock - big MI's Rupture - death at 7-10 days Fibrinous pericarditis - friction rub 3-5 days Dressler's - several weeks, autoimmune fibrinous pericarditis

What is a thyroid storm? Treat with what medications?

Acute, life-threatening form of hyperthyroidism/thyrotoxicosis that may present w/ Afib, fever, and delirium. Treat with: PPPP 1. Propythiouracil (to reduce thyroid hormones) 2. Propanolol (for HR) 3. Prednisone (glucocorticoid) 4. Potassium iodide - utilizes the Wolff-Chaikoff effect: increase free iodide to reduce iodide uptake by the thyroid and thus reduce T3/4 production; tricks the feedback loop. MedBullets Vignette: A 34-year-old woman with a history of hypothyroidism presents to the emergency department with altered mental status. According to the husband, she woke up this morning generally confused and was unable to answer questions. She had 3 episodes of nonbloody, nonbilious emesis shortly after and has been agitated since. She was recently discharged from the hospital status post-elective cesarean section. Her temperature is 99.5°F (37.5°C), blood pressure is 160/100 mmHg, pulse is 142/min, and respirations are 22/min. Laboratory studies show low TSH.

Name this condition: Distress and some mild level of dysfunction (etoh use, work) following a significant stressor (break-up, job loss, test failure).

Adjustment Disorder Having a variety of problems due to a specific stressor. NOTE: Different from Bereavment, which is a normal reaction to the death of a loved one and involves thoughts about death and dying, like possibly about "joining" the deceased; significant suicidal thoughts are not normal.

Advantages and Disadvantages of Cytogenomic Arrays

Advantages: - Far superior for genomic copy number changes (gains or losses) that are below resolution of G-banding. - Remember resolution dependent on: > Size of DNA probes on the arrays > Distance between probes - Equivalent to thousands of FISH at once - Detection of tandem duplications as sensitive as deletions - Does not require dividing cells (no culture) - SNPs can detect loss of heterozygosity Disadvantages: - Inability to detect BALANCED rearrangements - Inversions, balanced translocations - Cannot detect low level mosaicism - Cannot detect single point mutations - Not all copy number changes are clinically significant (benign, pathogenic, or of unknown significance). - Relatively expensive

How to recognize an X-linked Dominant pedigree

Affected males will pass it to ALL daughter and no sons. Affected females can pass it to sons or daughters. Difference from X-Linked Recessive: Sons and daughters can be affected (in XLR, only sons are affected --since they only have one X chromosome -- while females can be carriers) Males and females may be affected. The expression in heterozygous females may be variable. Often the clinical expression is more consistent and severe in hemizygous males than in heterozygous females, with some conditions causing lethality in males. Like X-linked recessive, males cannot pass it to sons because they give Y-chromosome to sons. No male carriers -- males only have one X-chromosome, so if they have it, it will be dominant. Affected males will give it to ALL DAUGHTERS and NO SONS.

What does withdrawal from alcohol, benzos, and barbituates look like, clinically? Treatment?

Alcohol/Benzos/Barbiturates: - Intoxication: slurred speech, behavioral disinhibition, memory impairment; coma and even death with very high etoh levels - Withdrawal: anxiety, increased HR & BP, tremors (DTs), nausea; if severe, seizures, hallucinations, delirium, rarely death; - Treatment: Benzos, classically Lorazepam. E.g., MedBullets: A 38-year-old man is brought to the emergency department after being in a motor vehicle accident. The patient fractured his femur and required surgery. On postoperative day 2, the patient reports trouble falling asleep, palpitations, and feeling anxious. Medical history is significant for hypertension and alcohol use disorder. His blood pressure is 165/98 mmHg, pulse is 105/min, and respirations are 20/min. Physical exam is significant for tremulousness and diaphoresis (sweating). He is started on lorazepam.

++++++++++++++++++++++++++++++++++++++ Diagnose: A 14-year-old boy presents with hematuria, problems with vision, and deafness. Approximately 2 weeks prior to symptom development, he recovered from a viral upper respiratory tract infection. Family history is significant for deafness and early initiation of lisinopril in his father. His temperature is 99°F (37.2°C), blood pressure is 140/90 mmHg, pulse is 75/min, and respirations are 18/min. Physical examination is signifcant for anterior lenticonus and sensorineural hearing loss. Urinalysis demonstrates dysmorphic red blood cells and red blood cell casts. Electron microscopy shows alternating thick and thin GBM with 'basketweave' appearence.

Alport syndrome (Hereditary nephritis) Nephritic, so there's blood in the urine. - genetic disorder (X- linked) involving mutations in genes that encode for type IV collagen in GBM. - typically presents with microscopic hematuria, mild proteinuria, and hearing loss - Several family members may be involved. - patients may also present with eye abnormalities (retinopathy and anterior lenticonus), sensorineural hearing loss, and rarely, leiomyomatosis. Alternating thick and thin GBM with 'basketweave' appearence because it's caused by autoimmune antibodies against Type IV Collagen.

Which glomerular disease is described? Clinical presentation - Weakness and weight loss, bone pain (systemic involvement) - Nephrotic syndrome, orthostatic hypotension, and peripheral neuropathy Diagnosis: - Congo red positivity on kidney biopsy - Apple green birefringence under polarized light - Mesangial expansion - Immunofixation of urine and serum - Bone marrow biopsy- congo red stain - Biopsy of affected organ- rectal biopsy diagnostic in > 80%

Amyloidosis - Systemic disease characterized by organized deposits of specific proteins in the glomeruli - Can affect virtually any organ system including the liver, heart, skin, and nervous system. Hence secondary glomerular disease AL amyloidosis is due to the accumulation of circulating monoclonal light chains. 90% associated with muLtiple myeLoma. AA amyloidosis is due to the accumulation of serum amyloid A (SAA), which is produced in patients with chronic inflammatory conditions (e.g.rheumatoid Arthritis).

++++++++++++++++++ KNOW ++++++++++++++++++ Valvular Disease Summary: Preload, Afterload, and Stroke Volume changes, AND PV loops for: - AS - MR - AR - MS Draw the table for P, A, & SV and fill in each column Then draw the PV loops for each.

Aortic Stenosis: Difficult to push through aorta - High Preload - High Afterload - Low Stroke Volume Mitral Regurgitation: Backflow into left atrium during systole, flows back into LV during diastole. - High Preload - Low Afterload - High Stroke Volume Aortic Regurgitation: Backflow from aorta to LV - High Preload (LV has even more blood) - High Afterload - High Stroke Volume Mitral Stenosis: Difficult to fill LV - Low Preload - Low Afterload - Low Stroke Volume PV loop changes are consistent with all of this. E.g., high afterload and low SV in AS -- PV loop is tall and thin.

A 15-year-old girl presents to the emergency room with a nosebleed that has not stopped for hours. She also has bleeding in her gums. She was recently started on carbamazepine for a newly diagnosed epileptic condition. Lab results reveal decreased counts in all leukocyte counts. Reticulocyte count is decreased as well. Carbamazepine is discontinued and a bone marrow biopsy is obtained.

Aplastic Anemia Usually not just anemia, but pancytopenia - APLASTIC = Bone Marrow problem -- NO TYPES OF BLOOD CELLS are made sufficiently. Reticulocyte: Immature / New RBC Histology: Bone marrow is just empty.

++++++++++++++++++++++++++++++++++++++++++ High Yield Spinal Cord Levels - What spinal cord level is responsible for each reflex: Arms: • Biceps • Brachioradialis • Triceps Legs: • Patellar • Ankle Jerk

Arms: • Biceps (C5/6) • Brachioradialis (C6) • Triceps (C7) Legs: • Patellar (L3/L4) • Ankle Jerk (L5/S1)

A 3-year-old boy is brought to the clinic by his mother due to a severe ear infection. She is upset because he just recovered from one about two months ago. On exam, he has multiple telangiectasias on his face and walks with difficulty. He is clumsy when walking and his trunk seems unstable. 1. Diagnose 2. Caused by a mutation in what gene? 3. Classic symptoms 4. Labs - What protein is often elevated?

Ataxia Telangectasia Immune deficiency disorder Mutation: ATM Sxs: infections, progressive ataxia, skin signs - telangiectasias Labs: Increased alpha-fetoprotein (AFP) levels

+++++++++++++++++++++++++++ A distressed young woman visits the emergency department because of acute-onset, severe abdominal pain. She consumed 10 alcoholic drinks yesterday and, on awakening today, had two bouts of diarrhea, gradually worsening epigastric pain, and two episodes of vomiting. She denies any medical problems but admits to taking furosemide for 4 days as a means of losing weight for an upcoming family reunion that is causing her a lot of anxiety. Her temperature is 37.5°C (99.5°F), respiratory rate is 25/min, blood pressure is 145/82 mm Hg, and oxygen saturation is 100% on room air. Physical examination reveals periumbilical tenderness to palpation. The result of a stool guaiac test is negative. Laboratory studies show: Na+: 138 mEq/L K+: 3.6 mEq/L Cl-: 105 mEq/L Glucose: 88 mg/dL HCO-3: 22 mEq/L Arterial blood gas analysis reveals: pH: 7.55 HCO3-: 20 mEq/L (LOW, nl = 22-28) PCO2: 25 mm Hg (LOW, nl = 35-45) Which of the following is the most likely cause of her primary acid-base disturbance? A. Electrolyte imbalance due to diuretic use B. Hyperventilation secondary to pain and anxiety C. Hypoventilation due to alcohol-induced respiratory depression. D. Prolonged diarrhea due to severe abdominal pain E. Vomiting due to alcohol toxicity

B. Hyperventilation secondary to pain and anxiety To analyze acid-base disturbances, first look at the pH; is it acidotic or alkalotic? Second, look at the PCO2. If pH and PCO2 change in the same direction, it's metabolic. If pH and PCO2 change in opposite directions, it's respiratory, like this patient. First, figure out the primary acid/base disturbance. The pH is 7.55, so it's alkalosis. Then, figure out which of the options would cause alkalosis. It's not metabolic because HCO3- is low. HCO3- must be high to call it metabolic alkalosis. However, the RR is high and the PCO2 is low, so that's probably causing the alkalosis, making it respiratory alkalosis. This patient has an increased pH with decreased [PaCO2], indicating that she is experiencing a primary respiratory alkalosis. This is supported by her tachypnea on exam. In both respiratory acidosis and alkalosis, there are two phases of HCO3 compensation: 1. Right away, there is a prompt, small change in serum HCO3- that occurs in the same direction as the PCO2 change as a result of carbonic anhydrase activity. 2. After a longer period (hours to days), the kidneys respond by increasing or decreasing serum HCO3- by increasing or decreasing HCO3- reabsorption / secretion). This compensation in serum HCO3- is also in the same direction as the PCO2. In contrast, in a metabolic acidosis or alkalosis, respiratory compensation is immediate rather than delayed. This results in a change in PCO2 in the same direction as HCO3-. NOT: Prolonged diarrhea produces a metabolic acidosis due to loss of bicarbonate from the gut. Even though HCO3- would decrease and carbon dioxide (CO2) would be low due to respiratory compensation, the patient would have a low pH, not a high pH. Furosemide (Loop Diuretic) causes alkalosis (one of the classic OHH DANG sxs of loop diuretics) as they can cause hypovolemia, which causes activation of aldosterone. This would manifest as a high pH (alkalosis), yet both high HCO3- and high CO2 (due to compensation), rather than the low HCO3- and CO2 seen in this patient. Hypoventilation causes CO2 retention, leading to a respiratory acidosis. The pH would be low and CO2 would be high. Vomiting causes a metabolic alkalosis due to loss of HCl from the stomach. Although pH and CO2 may increase (due to compensation), HCO3- would also be high.

A 54-year-old woman has noted that during the past month her fingers become pale and painful upon exposure to cold. On exam she has mild dyspnea, but no wheezing. Her blood pressure is 170/110 mmHg. Her antinuclear antibody test is positive with a titer of 1:256 with a nucleolar pattern. Her serum urea nitrogen is 15 mg/dL with creatinine of 1.1 mg/dL. Which of the following autoimmune diseases is she most likely to have? A. Discoid lupus erythematosus (DLE) B. Systemic sclerosis C. Polymyositis-dermatomyositis D. Sjogren's syndrome E. Rheumatoid arthritis

B. Systemic sclerosis A nucleolar pattern of antinuclear antibody deposition is specific for systemic sclerosis. Scleroderma, an autoimmune disease, can involve 3 antibodies: • anti-Scl-70: a/w systemic scleroderma; in 30% of pts • anti-centromere autoantibodies: a/w limited scleroderma (CREST syndrome); in 50% of pts. • antinuclear antibodies: in ~ 90-95% of affected patients; nucleolar pattern is specific for systemic sclerosis Complications of scleroderma include renal hypertension, though signs of renal failure may not occur unless the pt progresses to a hypertensive emergency. She also has features of Raynaud phenomenon and Pulmonary fibrosis (dyspnea). NOT: a) DLE is characterized mainly by skin pathology c) Renal disease is not a feature of polymyositis-dermatomyositis d) Sjogren's mostly affects salivary and lacrimal glands e) Renal disease is not a feature of rheumatoid arthritis, which mostly affects small joints

+++++++++++++++++++++++++++++++ Anti-Arrhythmia Drug Classes: 1. Na+ Channel Blockers - 1A, 1B, 1C 2. Beta blockers 3. K+ Channel Blockers 4. Calcium Channel Blockers Drugs in each class and MoA (heart or nodes, and what they prolong)

Beta Blockers and CCBs affect the nodes to change HR. Na+ Channel Blockers and K+ Channel Blockers work on the heart muscle to change HR. 1. Na+ Channel Blockers (Class IA, IB, & IC): Slow depolarization of cardiac myocytes. 2. Beta Blockers (Class II): Slow Depolarization at Nodes 3. K+ Channel Blockers (Class III): Prolongs Action Potentials in cardiac myocytes. 4. CCBs (Class IV): Slows depolarization at Nodes

Diagnose: A 22-year-old Vietnamese woman presents for a routine GYN exam. Her menstrual cycle is normal, and there is no evidence of other bleeding. Guaiac is negative (not losing blood in stool). LABS: Hgb is 11 (12-16) RBC is 5.8 (3.5-5.5) MCV of 70 (80-100) Normal RDW of 10. Normal WBC Normal Platelets Hemoglobin electrophoresis: Increase in Hgb A2 and HgbF

Beta Thalassemia Pathology: Beta subunit of Hgb isn't made. HgbA2 makes up for it. - Therefore, it makes sense that RBCs are high. Hgb is defective, EPO is high to try to increase O2 delivery to tissues. Hgb A2 and Hgb F increase. THAT's THE BUZZWORD! Often have hepatosplenomegaly There's a NORMAL RDW in thalassemia Low RDW = IDA - Red blood cell distribution width (RDW) is a measure of the range of variation in RBC volume / size.

- Classic presentation of Hemophilia - Hemophilia A involves a decrease in coag. factor ___. - Hemophilia B involves a decrease in coag. factor ___. - Both present with an increased PT or PTT?

Both present with increased PTT (isolated) Easy to remember: - Hemophilia A - decreased factor 8 - Hemophilia B - decreased factor 9 Clinical presentation: - Massive hemorrhage out of proportion to insult - Hemarthroses - Easy brusing - Bleeding from around gums - Death E.g., MedBullets A 4-year-old boy is brought into the emergency room after he is pushed on the playground. After falling to his knees, he develops grossly swollen and painful joints bilaterally, where the trauma occurred. Aspiration of the knee reveals frank blood. Platelet count and PT are normal. However, PTT is increased.

Repro Question 15 Your primigravid patient at 41 weeks' gestation presents for her weekly prenatal care visit with complaints of decreased fetal movement. Amnionic fluid index reflects oligohydramnios. On examination, her cervix is not dilated. Which class of agents is used clinically to open the cervix? A. Oxytocin B. Beta mimetics C. Prostaglandins D. Non-steroidal antiinflammatory drugs

C. Prostaglandins Prostaglandins are used to enhance cervical effacement (thinning) and dilation, reducing initial induction failures, shortening the induction-delivery interval, reducing oxytocin use, and lowering the rate of cesarean section because of failure to progress. Oxytocins are for increasing myometrial contractions and hence, labor induction.

Repro Question 21 Which of the following statements regarding the menstrual cycle is FALSE? A. The follicular phase is characterized by an increase in FSH and estradiol. B. The follicular phase tends to vary among women, between 10-20 days. C. The luteal phase tends to vary among women, between 10-20 days. D. Progesterone levels are higher in the luteal phase than in the follicular phase.

C. The luteal phase tends to vary among women, between 10-20 days. The luteal phase generally lasts for 14 days and is pretty consistent amongst women. Once ovulation occurs (LH surge), the CL exists for a finite amount of time and if there's no conception, the CL deteriorates and regress. That's why it doesn't vary much -- limited by the corpus luteum. The follicular phase is the phase that can vary.

Repro Question 6 Which of the following pathways in the hypothalamic-pituitary-ovarian axis is described incorrectly? A. The hypothalamus releases GnRH, which stimulates the pituitary to release FSH and LH. B. The pituitary releases FSH and LH to stimulate ovarian cells. C. The ovary releases progesterone during the luteal phase which exhibits positive feedback on the hypothalamus and pituitary. D. Estradiol, released by the ovary, exhibits mostly negative feedback on the hypothalamus and pituitary except mid-cycle when its positive feedback leads to the LH surge. E. Inhibin A is released by the ovary as a negative feedback to the pituitary.

C. The ovary releases progesterone during the luteal phase which exhibits positive feedback on the hypothalamus and pituitary. Progesterone, released by the corpus luteum, exhibits negative feedback rather than positive feedback on the HP axis during the luteal phase. This keeps FSH and LH low during the second half of the cycle.

A 18-year-old boy presents to his primary care physician for abdominal pain, diarrhea, and chronic fatigue. He reports that he has nonbloody diarrhea for the past few weeks. He could not identify any "bad" foods, and nobody else around him has the same symptoms. His aunt has a history of inflammatory bowel disease, but he is unsure which specific disease. He is noted to be underweight on physical exam. Laboratory results show elevated erythrocyte sedimentation rate (ESR) and C-reactive protein. He is scheduled for a colonoscopy with biopsy to confirm the diagnosis.

Crohn's Disease Key features: IMAGE - Entire GI tract, but there are Skip lesions and it spares the rectum. - Cobblestone mucosa - Fistulas

+++++++++ EASY POINTS +++++++++ What is the role of IL-2 in the production of CD8+ Killer T-cells? IL-2 is made by what cell? They activate ___ and ___ to fight infections. IL-4, IL-5, IL-6, IL-10, IL-13 are made by what cell? They are released so that they can recruit ___ to fight parasitic infections and to promote Ig class switching from IgM. Th1 cells are inhibited by IL-__ and IL-__. Th2 cells are inhibited by ___.

CD8+ T cells love IL-2, so they need help from Th1 CD4+ Helper T cells, which make IL-2 in vast amounts. Th1 CD4+ T-cells make IL-2, which activates macrophages and CD8+ Killer T-cells. Th2 CD4+ Helper T-cells produce IL-4, IL-5, IL-6, IL-10, IL-13, which are released to: - recruit eosinophils for parasitic infections - promote Ig Class switching in B cells from IgM. Th1 cells are inhibited by IL-4 and IL-10. Th2 cells are inhibited by IFN-γ (interferon gamma). FYI: They're called interleukins because they go from one leukocyte to another.

+++++++++++++++++++++ CO = __ x __ Blood Flow (Q) = __ /__ MAP = __ x __ = __ x __ x __ If given a blood pressure, MAP = __ systolic + __ diastolic

CO = HR x SV Q = ΔP/R - The magnitude of blood flow is directly proportional to the pressure difference. - More resistance means less blood flow. MAP = CO x TPR or MAP = (HR x SV) x TPR MAP = ⅓ systolic P + ⅔ diastolic P > Easier: DDS/3 - (Dia + Dia + Sys) / 3

Paget's Disease and Postmenopausal Osteoporosis can both be treated with what hormone analog?

Calcitonin: Decreases bone resorption (<= mainly remember that!) when serum calcium gets too high. - "Tones Down" calcium Since Calcitonin (CT) inhibits osteoclastic bone resorption, it has been widely used to treat metabolic bone disorders, such as Paget's disease of bone, malignancy-associated hypercalcemia, and osteoporosis. Paget's Disease: - characterized by increased bone turnover. - Calcitonin is used clinically to decrease this turnover and decrease bone pain. Postmenopausal Osteoporosis: - characterized by increased bone turnover and Calcitonin is one of the anti-resorptive agents used for treatment. Tones down calcium, so it will tone down calcium resorption from bone. - Bisphosphonates (-dronate's) are first line though

A 34 year-old woman presents to the ED with intractable vomiting. She states that she is pregnant, around 4 months, but that she had not had morning sickness and the vomiting had just started and has not stopped for 4 days. She has noted spotting and that prompted her to come in. Her serum hCG was 500,000. An ultrasound showed a snowstorm like appearance and no fetus. What is your diagnosis?

Complete molar pregnancy Too much male DNA; egg implants correctly but it is not viable.

Glycolysis Overview - What is the rate-limiting enzyme? - What is the final product? - Purpose is to produce a net total of how many ATP, as well as 2 molecules of what reducing agent? - Where does it occur? USMLE-Rx Question: Which of the following has a role in the regulation of the rate-limiting step of carbohydrate metabolism? A. Acetyl-coenzyme A B. Alanine C. Citrate D. Glucose-6-phosphate E. Reduced nicotinamide adenine dinucleotide

Central pathway of glucose catabolism in almost all organisms. Rate Limiting Enzyme: PFK-1 One 6-carbon glucose degraded in a series of enzyme-catalyzed steps to two 3-carbon pyruvate molecules Free energy released in the form of ATP and NADH (reducing agent) under aerobic conditions Occurs in the cytoplasm. Yields: 2 ATP 2 NADH 2 H+ (produces acid) Steps 1 and 3 = - 2ATP Steps 7 and 10 = + 4 ATP Net "visible" ATP produced = 2 USMLE-Rx Question: C. Citrate Glycolysis involves the oxidation of glucose to pyruvate, which then goes on to enter the Krebs cycle. The rate-limiting step of glycolysis is the conversion of fructose-6-phosphate to fructose-1,6-bisphosphate by the enzyme phosphofructokinase. Citrate is an intermediate produced in the Krebs cycle that has a role in the regulation of glycolysis by inhibiting the action of phosphofructokinase.

+++++++++++++ HIGH YIELD +++++++++++++++++ A 62-year-old man presents to his primary care provider for an urgent care visit. He reports feeling very fatigued recently. He denies any change in bowel movements or any blood in his stool. On physical exam, he has mild splenomegaly. Blood tests come back with increased leukocytosis with neutrophilic dominance, increased platelets, and increased basophils. Concerned, his primary care provider refers him to hematology-oncology. His peripheral smear shows myeloid cells. 1. Diagnose 2. Likely Mutation 3. Treatment and what pathway it inhibits

Chronic Myeloid Leukemia (CML) Always associated with the t(9;22) BCR-ABL1 fusion gene -- "Philadelphia chromosome" is the fusion of BCR on chromosome 22 with the ABL1 from chromosome 9. Imatinib mesylate is a tyrosine kinase inhibitor that acts on the Philadelphia chromosome fusion protein product BCR-ABL, which is a constitutive tyrosine kinase that enables the unchecked proliferation of myeloid cells in CML. - Median age is 67 years but can occur at any age - Hypermetabolic symptoms - Splenomegaly, often massive - CBC shows an elevated WBC with a left shift >> Basophilia and eosinophilia are common >> Typically have thrombocytosis >> Anemia is usually mild (fatigue) - Bone marrow is packed with myeloid elements

+++++++++++++++++++++++++++++++++ A 50-year-old man presents to his primary care physician with yellowing skin and increased abdominal girth. He believes that he has gained weight and is worried about obesity. He also reports being concerned about increased breast size. He denies having a history of alcohol abuse, but when he is questioned further, he admits to drinking a bottle of vodka daily to cope with stressors in his life. On exam, he has spider angiomas on his abdomen, jaundice, and gynecomastia.

Cirrhosis (Liver scarring from repeat injuries; Liver no longer functioning properly) IMAGE: KNOW THESE SYMPTOMS!!! FOUND IN MANY QUESTION STEMS!

12 month old male infant seen for routine evaluation. Has been well recently. Has mild pallor on exam, but otherwise unremarkable. • Screening CBC: Hgb 6.4 g/dl (low), Hct 19% (low), RBC 3.1 million/m3 (low), and reticulocytes 0.2% (low). • MCV 59 fL (low), RDW 16.8 (high) So this is an underproduction anemia. But what type? Breast fed until age 9 months, then switched to cow's milk. Currently consumes ~32 oz cows milk per day (high)

Classic case of Iron Deficiency Anemia

Granulomatosis with Polyangiitis (Wegener's Granulomatosis): - Classic triad of areas involved - Clinical presentation - Labs show increase in which ANCA?

Classic triad: 1. Upper airway involvement 2. Lower airway involvement 3. Renal involvement Clinical Presentation: Image Labs: c-ANCA, which targets PR3.

Which personality disorders are in each cluster? 1. Cluster A 2. Cluster B 3. Cluster C

Cluster A: "Odd, Eccentric" • Paranoid • Schizoid • Schizotypal > MNEM: Cluster A - "What A weird dude..." - These are the ones with weird beliefs or behaviors - All of them are living in their own worlds. Cluster B: "Dramatic, Eratic, Sporatic" • Antisocial • Narcissistic • Borderline • Histrionic > MNEM: Beware of Cluster B - These are ones you don't want to be around. Cluster C: "Anxious or Fearful" • Obsessive-Compulsive • Avoidant • Dependent > MNEM: Cluster C = Compulsive, Careful, or Clingy - These all have an element of anxiety to them -- anxiety about order, social anxiety, and anxiety about being alone, respectively.

A 26-year-old man with no significant past medical history presents with a prolonged period of cough and repeated sinus infections. He also reports persistent watery diarrhea. His physician orders immunoglobulin levels, which come back with decreased IgA and IgG, but normal IgM. Plasma cells are also decreased - Low RBCs, WBCs, Neutrophils, and Platelets. 1. Which immunoglobulin deficiency does this patient have? 2. Pathogenesis: The problem is with what process? 3. What are common comorbidities? 4. What is the definitive treatment?

Common Variable Immunodeficiency Disorder (CVID) This is one of the Immunoglobulin Deficiencies (Immunodeficiency). In addition to increased infections, may also see autoimmunity, granulomatous disease, malignancy Pathogenesis: Decreased isotype-switching from IgM to other Ig's in B-cells. Expected Labs: • Patients may have normal to low B-cell numbers • Low IgG and IgA • Normal or Low IgM (as opposed to hyper-IgM syndrome) • Possibly low RBCs, WBCs, Platelets, etc. (anemia, neutropenia, thrombocytopenia are often comorbid) • Most cases are diagnosed at in 20's or 30's. • Clinically, recurrent sinopulmonary bacterial infections • Comorbid autoimmune disease: anemia, thrombocytopenia, neutropenia, rheumatoid arthritis • Malignant disease: gastric carcinoma, EBV-associated lymphoma • Granulomatous disease ("sarcoid-like") Treatment: IVIG - immunoglobulin replacement

Briefly describe each of these genetic tests: - Comparative genomic hybridization (CGH) - Array Comparative genomic hybridization - Targeted vs. Whole Genome arrays

Comparative genomic hybridization A technique that is used to detect chromosome gain or loss by hybridizing DNA from a patient and from a control cell that are differentially labeled with unique fluorescence Array CGH - High-resolution CGH technique. - DNA from pt one color and control is diff color—mix two samples and hybridize to microarray chip which has short single strands of DNA in the wells - These unique sequences (in the wells) are chosen to be uniformly distributed throughout the genome - Look at ratio of light emitted - NEED PATIENT AND REFERENCE / CONTROL DNA Targeted Array - Focus on Specific Areas: - Subtelomeres - Pericentromeres - Microdeletion/Microduplication regions Whole Genome Array - Analyze entire Genome: - e.g., BAC arrays (32,000 BACs) - Oligo arrays (44,000 to 2M probes)

A 27-year-old woman presents to your office complaining of right arm numbness and weakness. Her past medical history is unremarkable. Her family history, however, is significant for the unexpected death of both her parents at age 59 in a motor vehicle accident last week. On physical exam, her bicep, brachioradialis, and patellar reflexes are 2+ bilaterally. CNS imaging shows no abnormalities. What is the most likely diagnosis?

Conversion Disorder Unintentionally producing unreal symptoms or deficits affecting voluntary motor or sensory function (sxs must be "neurologic" - like blind, deaf, even pseudoseizures) Commons presentation include pseudoseizures, blindness, deafness, sensory loss, paralysis or gait issues On test, may show "la belle indifference" (pt not too concerned about the problem), but that isn't really useful.

What is Cor Pulmonale? How does it progress?

Cor Pulmonale = isolated right heart failure Due to pulmonary hypertension (hence, "pulmonale") RV having to push harder due to pulmonary HTN Leads to right ventricular hypertrophy Eventual (isolated) right sided heart failure

Diagnose and Name the Enzyme Deficiency: An infant is brought to the pediatrician by his parents because they are concerned about the yellow color of his skin and general behavior changes. They report that he has seemed more tired and weak with his arms just flopping down by his side instead of him reaching for his toys. The parents are known to be first cousins.

Crigler-Najjar Syndrome Hereditary accumulation of unconjugated bilirubin. Defect in UDP-glucuronosyltransferase (UGT), which normally catalyzes the conjugation of bilirubin and glucuronic acid within hepatocytes This is why it presents with neonatal jaundice

++++++++++++++++++++++++++++++++ A patient presents complaining weight gain, especially in their face and neck. Upon examination you note that she has increased facial hair and on her abdomen faint purple lines can be noted running up and down her flanks. What is the most likely cause if ACTH is high vs. low?

Cushing Syndrome/Disease = High Cortisol 2 Causes of High Cortisol Levels: - Most common cause is steroid therapy, or ectopic release by an adrenal or lung tumor (will have low ACTH due to feedback inhibition) - If ACTH is high, the etiology is pituitary-based (excess ACTH release) and it's called Cushing's disease. - Cushing Syndrome: Too much cortisol, and resulting low ACTH. From exogenous steroid use or an adrenal/lung tumor. Will have high Aldo too. - Cushing Disease: Too much cortisol b/c of high ACTH. Will have normal aldo (released from adrenal, but controlled by RAAS, not ACTH). Patients with Cushing's experience HTN, hyperglycemia, hypokalemia, metabolic alkalosis, muscle weakness, striae, etc.

DiGeorge Syndrome: - Defects in what 3 organs? - Deletion in chromosome ___, resulting in what classic set of symptoms - Malformation of which 2 pharyngeal pouch numbers? - What type of T-cell will be markedly reduced and why? - B-cells?

Defects in: - thymus - parathyroid - heart One of the CATCH-22 diseases: A/w chrom. 22 - Cleft palate - Abnormal facies - T-cell deficiencydue to thymic aplasia - Cardiac abnormalities - Hypocalcemia, due to parathyroid aplasia • Malformation of the 3rd and 4th pharyngeal pouches during embryogenesis • 80% have thymic hypoplasia - CD4+ T cells markedly reduced • B cell numbers normal to high • T cell-dependent antibody responses impaired

Diagnose: A soft-spoken woman comes to the clinic. When asked what she thought about the treatment plan, she replied "whatever you say." She reveals being in an abusive relationship, but says that he's "all she has."

Dependent Personality Disorder I just made this card to memorize the classic STEP description. STEP vignettes often mention a patient saying to MDs "whatever you say" or staying with someone abusive ("he's all I have"). - Dr. Spollen Constantly seek reassurance and care from others. Fear being assertive. Pushovers. Excessively rely on others. Extremely uncomfortable being alone.

A 53-year-old woman with a past medical history of thyroid disease presents to her physician's office for weakness. She reports that she has been feeling weak, has difficulty climbing the stairs, and combing her hair every morning. Today, she also has difficulty arising out of her chair. Physical exam reveals decreased strength, especially in the shoulders. She also has an impressive lilac periorbital rash and a sunburn on her cheeks. She is sent for further laboratory workup and counseled on the likely initiation of high-dose steroids.

Dermatomyositis Same thing as polymyositis but involves rashes as well. Both Dermatomyositis and Polymyositis manifest as symmetrical proximal skeletal muscle weakness (e.g., shoulders, thighs, and neck).

Diagnosis of Cushing Syndrome: Cushing's vs. Ectopic ACTH secretion (tumor) - Low dose dexmethasone test, and interpretation of results. - ACTH measurement, and interpretation of results. - High dose dexmethasone test, and interpretation of results. A 50-year-old female is evaluated by her physician for recent weight gain. Physical examination is notable for truncal obesity, wasting of her distal musculature and moon facies. In addition she complains of abnormal stretch marks that surround her abdomen. The physician suspects pituitary adenoma. Which of the following high-dose dexamethasone suppression test findings and baseline ACTH findings would support his view? A. Cortisol suppression, normal baseline ACTH B. Cortisol suppression, high baseline ACTH C. No cortisol suppression, high baseline ACTH D. No cortisol suppression, low baseline ACTH E. Elevation of cortisol above pre-test levels, high baseline ACTH

Diagnosis of Cushing Syndrome - Determining why cortisol is elevated. Dexmethasone is a corticosteroid. Pituitary Adenoma releasing ACTH => high cortisol = Cushing's. Ectopic secretion of ACTH is usually from small cell carcinoma of the lung Measure 24 hr urinary cortisol or give low dose (1 mg) dexamethasone. If there is Increased cortisol in urine and no suppression of plasma cortisol by 1 mg dexamethasone, this is evidence of Cushing syndrome. Measure ACTH. - If ACTH ↓, ACTH-independent (e.g. adrenal tumor). - If ACTH↑, ACTH-dependent (i.e., either Cushing disease or ectopic ACTH). To differentiate, give high dose (8 mg) dexamethasone or CRH. - For Dexamethasone, if cortisol ↓, Cushing disease (i.e. production is suppressible), if not, ectopic ACTH (i.e. production is NOT suppressible). - For CRH, if both ACTH and cortisol ↑, Cushing Disease, if not, ectopic ACTH. Answer: B. Cortisol suppression, high baseline ACTH - ACTH will be high regardless in Cushing's (pituitary adenoma) - Cortisol should be suppressed if it's Cushing's.

A patient undergoing pulmonary function testing is asked to inspire maximally and then exhale as hard as possible while a spirometer measures forced expiratory volume in 1 second (FEV1), total volume, and forced vital capacity (FVC), yielding the curve labeled "Control" above. Normally, FEV is about 80% of FVC. After the patient inhales an unknown drug, a second recording is taken (the curve labeled "+ Drug" above). The drug decreased blood pressure but increased HR. The graph shows lung volume over time. The control is low, but with the added drug, the curve gets higher (volume decreases more slowly over time). i.e., FEV1 and FVC were lower after the drug. BP dropped and HR increased after the drug. So what kind of agonist would do this? Stimulation of which of the following receptor types is responsible for the change in FVC? A. Alpha-1 adrenergic B. Beta-1 adrenergic C. Beta-2 adrenergic D. Muscarinic-2 cholinergic E. Muscarinic-3 cholinergic

E. Muscarinic-3 cholinergic - Bronchoconstrictor - Drop in BP from non-innervated M3 receptors; reflex tachycardia.

A 2-week-old boy born at 28 week gestation. Vital signs are normal. A continuous murmur is heard at the left heart border. The problem is a congenital abnormality involving a derivative of which embryologic structure? A. First aortic arch B. Second aortic arch C. Third aortic arch D. Fourth aortic arch E. Sixth aortic arch

E. Sixth aortic arch Sixth arch gives rise to the ductus arteriousus and pulmonary artery. Continuous murmur = Patent Ductus Arteriosus

A 50-year old man presents with subacute weakness. On examination, he has symmetric proximal muscle weakness as well as a decreased grip strength. His muscles are painless to palpation. No rashes or skin changes are seen on examination. Prior empiric with steroids did not produce any improvement. A serum creatine kinase is over 10,000 U/L. What findings on muscle biopsy would be most consistent with these clinical findings? A. Ragged red fibers on a trichrome stain B. Significantly reduced dystrophin staining C. Diffuse lymphocytic infiltrate with perifascicular atrophy D. Fiber-type grouping E. Vacuoles in muscle fibers with basophilic rims

E. Vacuoles in muscle fibers with basophilic rims - i.e., Rimmed Vacuoles This is the classic description of Becker Muscular Dystrophy. Dystrophin staining will only be slightly reduced (Duschenne is more severe). Becker Muscular Dystrophy presents after teenage years while Duschenne presents in childhood.

A 4-year-old boy is brought to the physician because of purplish patches on his arms and thorax. The mother says that the boy also has a history of recurrent pneumococcal pneumonia and red, itchy rashes on the inside of his elbows and knees that leak fluid and crust over when scratched. Laboratory studies show thrombocytopenia and a selective decrease in IgM. Which of the following is the most likely underlying condition? A. Ataxia telangiectasia B. Common variable immunodeficiency C. Hyper-IgM syndrome D. Severe combined immunodeficiency E. Wiskott-Aldrich syndrome

E. Wiskott-Aldrich syndrome This child has Wiskott-Aldrich syndrome, which is an X-linked, partial combined immunodeficiency disorder (WAS gene mutation). It presents clinically as the triad of thrombocytopenic purpura, eczema, and recurrent opportunistic infections with organisms having polysaccharide capsules, such as Streptococcus pneumoniae. MNEM: WATER -- Wiskott-Aldrich Syndrome causes: - Thrombocytopenic purpura (low plts, easy bruising, purpura and petechiae) - Eczema - Recurrent infections Patients characteristically have decreased serum IgM, normal IgG, elevated IgA/IgE, and T-cell deficits (which appear later in life). NOT: • Ataxia telangiectasia (choice A) is a combined partial T- and B-cell deficiency that presents with ataxia, telangiectasia, and IgA and IgE deficiencies. • Common variable immunodeficiency (choice B) is a humoral deficiency that typically presents in the late teens and early twenties as a decrease in immunoglobulin levels. • Hyper-IgM syndrome (choice C) is an X-linked immunodeficiency that is a mutation in CD40L on activated CD4 Th cells that results in a deficiency in class-switching. • Severe combined immunodeficiency disease (choice D) presents early in life with severe and recurrent infections with all pathogens.

++++++++++++++++++++++++ Important uterine cancer info: 1. Endometrioid adenocarcinoma: - Results from ___ - Associated with increase in what hormone? - Associated with what mutation? 2. Serous "Papillary" Carcinoma: - Spreads through what mechanism, so it might present how? - Associated with what mutation? - Histology will show ___ bodies. 3. Clear cell adenocarcinoma - Associated with what uterine condition?

Endometrioid adenocarcinoma: - Comes from hyperplasia (↑ estrogen) - Recapitulates endometrial glands - PTEN mutations Serous "papillary" carcinoma: - Spreads through exfoliation to the peritoneal cavity (presents with ascites), thus worse prognosis (uterine cases can exit via the tube) - high grade - p53 mutations - Psammoma bodies on histology (Image) Clear cell adenocarcinoma: - high grade - Associated with endometriosis (uterine tissue outside of the uterus)

+++++++++++++++++++++++ LV Pressure-Volume Loop -- Which parts are: Contraction (IsoV) Ejection Relaxation (IsoV) Filling Know where on the loop the Mitral and Aortic Valves open and close. Know where end systole and end diastole are!

End Systolic is at the top left corner. End Diastolic is at the bottom right corner.

++++++++++ HIGH YIELD ++++++++++ ENDOCARDITIS 1. Most common bacteria that causes: - Acute Endocarditis (develops rapidly) - Subacute Endocarditis (develops slowly, like over months) 2. A 45-year old man had fever and symptoms for 3 months. A systolic murmur at LSB. Echo shows a stenotic bicuspid aortic valve and bacterial vegetation. What type of bacteria is most likely? A. Gram-negative B. Coagulase+, catalase+ C. Coagulase-, catalase+ D. Alpha hemolytic strep E. Beta hemolytic strep 3. What pathogen causes Rheumatic Fever? 4. What pathogen is it if they recently had any kind of GI/GU infection or procedure (e.g., cystoscopy)?

Endocarditis is inflammation of the heart valve, secondary to infection. Acute: S. aureus Subacute (btw acute and chronic): Strep viridans (Alpha hemolytic strep) or Staph epidermis. Acute is much worse than subacute. Requires surgery, empiric therapy, etc. D. Alpha hemolytic strep - Because 3 months is subacute. - Strep viridans = Alpha hemolytic strep Rheumatic Fever: Group A Strep - Strep Pyogenes (Beta hemolytic Strep) Recent GI/GU infx or procedure: Enterococcus (E. faecalis) E.g., A 50-year-old man presents to the emergency room for a fever that has persisted for several days. He denies any history of intravenous drug use or any congenital heart disease. Physical exam reveals nailbed splinter hemorrhages, Osler nodes on his fingers, and Janeway lesions on his palms and soles (these are from emboli). Heart auscultation reveals a new murmur. An echocardiogram shows vegetations on the mitral valve.

Name the main steps in the progression of an Atheroma (3 main steps that are tested on)

Endothelial damage ➜ Macrophage / LDL accumulation ➜ Fatty streak ➜ Smooth muscle migration ➜ Fibrous plaque (scar tissue) ➜ Complex atheroma Which 3 could be test questions? 1. Endothelial damage 2. Smooth muscle migration 3. Complex atheroma Atheroma- Degeneration of the walls of the arteries caused by accumulated fatty deposits and scar tissue, and leading to restriction of the circulation and a risk of thrombosis.

What is the definitive treatment for anaphylaxis?

Epinephrine also antihistamines The α-adrenergic vasoconstrictive effects of epinephrine reverse peripheral vasodilation, which alleviates hypotension and also reduces erythema, urticaria, and angioedema The β-adrenergic properties of epinephrine cause bronchodilation, increase myocardial output and contractility, and suppress further mediator release from mast cells and basophils.

A 35 year old woman presents to the emergency department with flu like symptoms and a vesicular rash that has a targetoid appearance. Older lesions had sloughed and left a raw, red base. She came in because the lesions were spreading to her mouth and nose. -- OR -- A 12-year-old girl was recently treated for walking pneumonia and was found to be positive for mycoplasma. A few days into her illness, she developed multiple raised lesions over her palms, back of hands, and extensor forearms. These lesions were targetoid and mildly itchy. - What is the diagnosis? - What preceding exposure most often causes this? - What other symptoms would you expect with this? Dermatopathology

Erythema Multiforme - possibly as a part of Steven-Johnson Syndrome (SJS) or Toxic Epidermal Necrolysis (TEN), a more extreme version of SJS as it covers >30% of the body ("2 disorders on the same scale"). Classic: Targetoid Lesions Usually drug-induced: - e.g., Sulfas, Beta lactams, Phenytoin SJS includes erythema multiforme, fever, bulla formation (fluid-filled blisters), skin necrosis, mucous membrane involvement, and sloughing. TEN is more severe than SJS (TEN if it covers >30% of body), but both can be fatal.

Difference between Primary and Secondary Erythrocytosis Primary: - C/b constitutive activity of what signaling pathway? - EPO levels - Most likely diagnosis? Secondary: 2 types: Hypoxia-dependent vs. Hypoxia-independent - EPO levels in each - Most likely diagnosis if pt is not hypoxemic? (Mutation in what protein, leading to...)

Erythrocytosis = Too many RBCs. Primary erythrocytosis: Primary is a problem with the stuff that actually makes RBCs: EPO. - Constitutive JAK2 activity despite low plasma EPO levels. EPO uses JAK/STAT to make RBCs, but here, JAK2 is stimulating this pathway on its own and EPO is low because you don't need more RBCs. - Most common cause is Polycythemia Vera >>> an activating JAK2 point mutation. - Genetic mutations in the Epo signaling pathway is another cause Secondary erythrocytosis: 2 Types; both depend on external factors. 1. Hypoxia dependent = appropriately high EPO levels to increase RBCs. - this is normal - e.g., Altitude -- low O2, need more RBCs so EPO is "appropriately high" 2. Hypoxia independent = inappropriately high EPO levels - this is abnormal - a mutant / non-functioning Von Hippel Lindau protein (VHL), which is a tumor suppressor protein, is a likely cause. ^ prevents ubiquitin-mediated proteolysis of HIFa (hypoxia-inducible factor), causing persistently elevated EPO levels. ^ i.e., Decreased levels of von Hippel-Lindau (VHL) tumour suppressor protein are associated with up-regulation of hypoxia-inducible factor (HIF), leading to increased EPO levels, tumour proliferation, and angiogenesis.

Membranoproliferative Glomerulonephritis and Dense Deposit Disease: - Nephritic or Nephrotic? - Most common cause of Type I MPG - Type II MPG (Dense Deposit Disease) is associated with ___ Nephritic factor. - Histology of Type I vs. Dense Deposit Disease - GBM splitting leads to what appearance on histology for both types?

Etiologies: • Type I: Hepatitis B or C. • Dense deposit disease (MPG Type II): Associated with C3 Nephritic factor Histology: - Membrane proliferation - Widening of the capillary loops, often with a double-contoured / tram track appearance due to GBM splitting Specifically... •Type 1 MPGN -- subendothelial and mesangial immune deposits. • Dense deposit disease -- irregular ribbon-like intramembranous deposits FIRST AID: MPGN is a nephritic syndrome that often co-presents with nephrotic syndrome. Type I may be 2° to hepatitis B or C infection. May also be idiopathic. Subendothelial IC deposits with granular IF. Type II is associated with C3 nephritic factor (which is an IgG antibody that stabilizes C3 convertase => persistent complement activation => decreased C3 levels). Intramembranous deposits, also called dense deposit disease Both types: mesangial ingrowth GBM splitting => "tram-track" on H&E and PAS stains

Diagnose the disease based on the newborn screening of Hgb types: 1. FS 2. FSC 3. FAS (more A than S) 4. FSA (more S than A) 5. FA + Barts Remember -- these are listed in order of amount -- FAS means Hgb F > A > S

FS SS disease or S-Beta thalassemia FSC SC disease FAS Sickle Trait (more A than S) FSA S beta+ thalassemia (more S than A) FA + Barts (l4) Alpha thal trait Barts is not present later in life but Hb A2 levels will be low

Acromegaly is diagnosed by what test?

Failure to supress GH after oral glucose tolerance test. GH has gluconeogenic effects. Adding glucose should decrease GH through feedback inhibition. If it doesn't, GH is being released on its own (pituitary tumor).

Pathophysiology of Familial Hypocalciuric Hypercalcemia: - Cause: Mutation in ... - Effect: accidental over-release of ___. - Calcium levels in these patients are ___. - PTH levels are ... - Urinary excretion of Calcium is high or low? ^ Key feature of FHH that distinguishes it from Primary Hyperparathyroidism.

Familial Hypocalciuric Hypercalcemia • Problem: Mutation of the Ca2+ sensing receptor [CaSR] on the parathyroids. i.e., the parathyroid glands cannot sense Ca2+ levels. • Result: PTH is over-released despite high Ca2+ (thinks there's low serum Ca2+ due to defective Ca2+ receptor on parathyroid glands) and Ca2+ is reabsorbed in the kidneys. • This leads to hypocalciuria and hypercalcemia, even though the original calcium level was actually fine. • PTH levels are normal to high regardless of serum calcium levels. Parathyroids are flying solo, unregulated. • Urine excretion of calcium is low -- key feature that distinguishes primary hyperparathyroidism from FHH. - Hence, Hypocalciuric Hypercalcemia

Fetal erythropoiesis occurs in what 4 locations? MNEMONIC Erythropoiesis uses what signaling pathway?

Fetal Erythropoiesis occurs in: • Yolk sac • Liver • Spleen • Bone marrow Young Livers Synthesize Blood Erythropoiesis uses the JAK/STAT pathway (IMAGE)

Antidiuretic Hormone (ADH): - Function - Famous ADH agonist used to treat Central DI and Von Willebrand disease.

Function: ↑ permeability of kidney collecting duct cells to water by increasing the number of AQP channels. Also stimulates vascular smooth muscle contraction to further increase blood pressure. ADH agonist - desmopressin, used for central DI and von Willebrand disease Diseases involving ADH: - Diabetes insipidus - Central (no production) and Nephrogenic (no response) - SIADH

What histological changes are expected with MIs? Don't necessarily need to recognize the histology, but know the buzz words: - First few hours - 12-24 hrs - 7-10 days - 8 weeks

First few hours: WAVY FIBERS (big BUZZword) Then neutrophils => phagocytosis (clean up dead tissue) => granulation tissue (form new tissue) => scar tissue formation.

Which glomerular disease is described: - 30%-35% of cases of adult nephrotic syndrome - More prevalent among African Americans and Hispanics. - >50% of patients progress to end-stage renal disease (ESRD) within 5 years of the diagnosis - Associated with HIV, heroin abuse, Interferon treatment, loss of renal parenchyma, tissue/scarring. - Does NOT resolve with steroids

Focal Segmental Glomerulosclerosis (FSGS)

Acute Tubular Necrosis (ATN): - Definition - How are kidneys affected? - ATN is the most common cause of ___. - Histology will show presence of what? - What type of medication is the most common cause of Nephrotoxic ATN?

Follows prerenal azotemia, which is decreased blood flow to the kidneys (e.g., heart failure). It's characterized by damage to the epithelium of the kidney's collecting tubules. Most often caused by ischemia or presence of toxins in the urine. Results in oliguria. Epithelial cells slough off and clog the tubules, drastically reducing GFR... - BUN:Cr ratio <15 (very high creatinine) - Decreased Na+ reabsorption (FENa >2%) - Inability to concentrate urine (urine osm < 500) ATN is the most common cause of Acute Renal Failure Histology: Damaged tubal epithelial cells slough and make casts (granular / epithelial). - Brown, granular casts are seen in urine Aminoglycosides are the most common cause of Nephrotoxic ATN. - MNEM: Aminoglycosides - kidneys are on the sides.

+++++++++++++++++++++++++++++++++++++ X-Linked Agammaglobulinemia (XLA): - The problem is with __- cells and thus is characterized by a complete lack of ___. (Hint: In the name)

Genetic defect: mutated B-cell tyrosine kinase (btk) - Hence the "X-Linked" part. B-cells fail to mature beyond the pre-B stage, and are absent from blood => No serum immunoglobulin. B-cells make antibodies (IgA, E, M, etc.). In XLA, B-cells don't develop all the way so there are no immunoglobins at all. Hence, Agammaglobulinemia - no immunoglobulins in the blood. This is one of the Immunoglobulin Deficiencies (Immunodeficiency). Should have: ✔ Low IgA, IgE, IgG, and IgM (i.e., all are low). ✔ Low B-cell count. Ex: A 6-year-old boy presents to the hospital with a severe upper respiratory infection requiring hospitalization. Chart review reveals that he has presented multiple times to the emergency room and primary care physician's office for a variety of infections, including otitis media, upper respiratory infections, pneumonia, and sinusitis. When detailing family history, it is found that his maternal uncle died of an infection as a child. Lab findings include decreased levels of IgG, IgM, and IgA.

Diagnose: A 38 year old woman has a 1 week history of diarrhea. 10 days later she begins to complain of weakness and tingling in her feet and ankles. This is shortly followed by weakness in her hands.

Guillain Barre Syndrome - autoimmune attack of peripheral Schwann cells due to molecular mimicry - symmetric weakness usually begins in distal limbs and moves proximally - stem often includes diarrhea What other service might you have to consult if her condition deteriorates? Consult pathology for a plasmapheresis. Pulmonary medicine - Progression leads to pulmonary compromise, which might need ventilator support. Will also accept cardiology, as some patients have severe autonomic dysfunction as well as PMR for rehab

A 68-year-old male presents with sudden onset seizures. The patient is noted to have an ill-defined mass, arising in the cortical hemispheres and crossing the corpus callosum. There are areas of necrosis present on the MRI. What is the diagnosis and how long will he live?

Glioblastoma (GBM) (grade IV astrocytoma), probably less than 1 year These can cross the corpus callosum; Butterfly lesion

Lung problems / pulmonary symptoms + Kidney problems / Glomerulonephritis (Hematuria) = What diagnosis? E.g., A 62-year-old man presents to his primary care doctor complaining of recent-onset hemoptysis. He has not had any fevers, night sweats, or weight loss. He recently traveled to Italy with his wife. He has a 5 pack-year smoking history. On review of systems, he reports that his urine has been "red-tinged" for several months. Urinalysis reveals RBC casts. He is referred to a nephrologist and undergoes a renal biopsy. Immunofluorescence staining of a glomerulus shows tagged fluorescent antibodies distributed in a linear pattern. Which of the following is the most likely underlying pathogenesis of his disease? A. Type III hypersensitivity reaction B. Antibodies to collagen type IV C. Presence of C-ANCA D. Mycobacterial infection E. Metastatic malignancy

Goodpasture's Syndrome - AKA: Anti-glomerular basement membrane antibody disease -Antibodies to GBM and alveolar basement membrane - Characterized by linear deposits of (lgG) along the GBM - It's Goodpasture syndrome when alveolar hemorrhage is present. This is one of the 2 types of Rapidly Progressing Glomerulonephritis (RPGN) B. Antibodies to collagen type IV - Goodpasture's is caused by an antibody to Type IV Collagen

What is the only difinitive way to diagnose Gout?

Gout is diagnosed ONLY by aspirating synovial fluid from the toe or tophus (nodule) and finding Monosodium Urate crystals. Negative birefringence

Diagnose: A 52 y/o previously healthy male presents with a 2 week history of fever and rash. No recent travel or medication use. -Labs: Hb: 6.8, Plt: 213,000, BUN: 54, Cr: 4.3 -CXR: Pneumonia -UA: 30 protein, Large blood, 25-50 RBCs -Urine sediment shows dysmorphic RBCs, single RBC cast - Serology: c-ANCA: 5120 Anti GBM : Neg

Granulomatosis with Polyangiitis (Wegener's granulomatosis) c-ANCA This involves Rapidly Progressing Glomerulonephritis (RPGN), just like Goodpasture's does. Look for crescent shapes -Characterized by the absence of immune deposits within the kidney . - Most patients exhibit an underlying small vessel vasculitis.

HFrEF vs. HFpEF - Changes to Left Ventricular P/V Loop

HFrEF = Systolic HF - There is decreased contractility, by definition (CAN'T PUMP) - Thus, PV Loop will be shifted to the right, top of the curve is shifted down. - FILLING MORE BUT PUSHING OUT THE SAME AMOUNT (inc. EDV, same SV) - So DOWN RIGHT for reduced EF HFpEF = Diastolic HF - There is decreased compliance, by definition (CAN'T FILL) - So it fills less and pushes out less, making the pressure always higher and the filling volume lower. - The bottom of the PV loop is shifted up and the curve is shifted left. - FILLS LESS AND PUMPS LESS (dec. EDV and SV) - So LEFT UP

HLA Class I Deficiency vs. HLA Class II Deficiency: - What type of T-cell is lacking in each? - Which one presents with chronic recurrent bacterial infections, skin ulcerations, and vasculitis? - Which one has normal B-cells, but still very low serum immunoglobulins (e.g., IgE, IgM, IgA, etc.)?

HLA encodes for MHC, so class I vs. II is basically MHC I vs MHC II deficiency. Remember, MHC I goes with CD4+ T-cells and MHC II goes with CD8+ T-cells. - MNEM: 4 x 2 = 8 x 1 HLA class I deficiency: - Mutations in TAP, failure to express cell surface class I molecules - Lack of CD8+ T cells, due to lack of positive selection in thymus - Chronic respiratory bacterial infections, Skin ulceration with vasculitis HLA class II deficiency: - Bare lymphocyte syndrome - Mutations in class II-specific transcription factors CIITA or RFX - Lack of CD4+ helper T cells, CD8+ T cells normal - B-cells normal, but very low serum Ig's Treatment for both is Bone Marrow Transplant or stem cells.

Diagnose and Treat: A 50-year-old man presents to his primary care physician for an annual visit. He has no complaints. His past medical history includes hypertension, hypercholesterolemia, and rheumatoid arthritis. He goes for routine blood tests today, which reveals marked pancytopenia. Physical exam reveals splenomegaly. A peripheral blood smear is done, showing cells with hair-like projections. He has elevated levels of Tartrate-Resistant Acid Phosphatase (TRAP). Flow cytometry is positive for CD25 and CD11c.

Hairy Cell Leukemia Treatment: Cladribine Elevation in Tartrate-Resistant Acid Phosphatase (TRAP) is characteristic of Hairy Cell Leukemia. Hairy cell leukemia is a low-grade B-cell neoplasm that typically manifests with bone marrow involvement, leading to anemia, thrombocytopenia, and leukopenia. As a result, patients can show symptoms of fatigue, easy bruising, and recurrent infection. Hairy cell leukemia can also cause massive splenomegaly which may present as abdominal fullness or discomfort. TRAP is not positive in other B-cell neoplasms, so it can be useful in the diagnosis of HCL. However, flow cytometry is also a widely used diagnostic test because HCL cells are positive for CD25 and CD11c.

Which antipsychotic famously causes Extrapyramidal Symptoms (EPS), resembling parkinsonism? What are the main extrapyramidal symptoms?

Haloperidol MNEM: Haloperidol causes extrapyramidal symptoms EPS = Parkinson's-like symptoms, which include: - Dyskinesia/Akinesia - impaired initiation of voluntary movement - Dystonia - twisting and repetitive movements or abnormal fixed postures - Resting Tremor - often repetitive "pill rolling" movement - Tardive Dyskinesia - involuntary, repetitive movements of facial, tongue, and neck muscles. - Akathisia - restlessness; inability to stay still

A 47 year old female presents to her doctor complaining of weakness, weight gain, fatigue, and sensitivity to cold. The doctor notes that she has dry, brittle hair and sounds hoarse. There is some swelling around the patients eyes. Her heart rate is 55 BPM. She has a family history of autoimmune disease. Labs reveal hypothyroidism. What is the most common form/cause of hypothyroidism? How would you diagnose this? -- What immunoassays? What malignancy is associated with this?

Hashimoto Thyroiditis A condition characterized by the autoimmune destruction of the thyroid. Antibody assay to look for antithyroglobulin, antiperoxidase, and antimicrosomal antibodies. What malignancy is associated with Hashimoto's thyroiditis? - B-cell lymphoma

Name the two components of each type of Hgb: HbA HbF HbS HbA2 HbA1c

HbA - normal adult - α2 β2 HbF - fetal hemoglobin - α2 γ2 HbS - sickle cell hemoglobin - α2 S2 HbA2 - minor adult form - α2 ẟ2 HbA1c - glycated hemoglobin in diabetes

A 42-year-old woman with a history of polycystic ovarian syndrome and diabetes presents to her primary care physician's office for abdominal pain. She reports that she has experienced this for the past few months. On physical exam, her skin is noted to be hyperpigmented. She also has hepatomegaly. Given these findings, her physician sends her for laboratory testing, which shows elevated transaminases, iron, and ferritin. She is scheduled for regular phlebotomy.

Hemochromatosis Iron deposits! Deposits in (triad) 1. Liver (cirrhosis \ carcinoma) 2. Pancreas (bronze diabetes) 3. Heart (CHF)

A 5-year-old boy presents to the emergency room for abdominal pain and rash on his legs. He also reports having intermittent joint pains. A few weeks ago, he had an upper respiratory infection. On physical exam, he has nontender palpable purpura on his buttocks and lower extremities. His abdomen is mildly tender. Urine studies show proteinuria and hematuria. 1. Diagnose 2. This is caused by deposition of which Ig?

Henoch-Schonlein Purpura AKA IgA vasculitis Most common systemic childhood vasculitis Affects small vessels Follows Upper Respiratory Infection Triad of systems (3): ✔ Skin (palpable purpura) ✔ Joints (arthralgia) ✔ GI (pain, blood in stool) ✔ GU (hematuria, proteinuria from IgA deposition)

After attending a few parties, a 19 year old college student decides to end the evening by heading to Jack in the Box for a double cheeseburger. 2 weeks later he presents to the student health clinic with abdominal pain and a fever. The doctor notes that he is slightly yellow and his AST and ALT are elevated. What is your diagnosis?

Hepatitis A Fecal to oral route. 2 weeks is very acute. 1/3 of Acute Hepatitis is caused by Hepatitis A

A 23-year-old woman with past medical history of multiple allergies leading to facial swelling presents with marked difficulty breathing, abdominal pain, and swelling of the face and neck. On physical exam, her extremities are also noted to be markedly swollen. She is immediately sedated for intubation to maintain patency of airway. Chart review reveals newly-started ACE-inhibitor. She is given epinephrine, fluids, and fresh frozen plasma. 1. Diagnosis 2. This patient likely has deficiency of C__ esterase inhibitor 3. What are the classic symptoms at presentation? 4. What treatment options are available?

Hereditary Angioedema C1 esterase inhibitor deficiency. - One of the complement deficiency disorders • Inherited disorder that presents with abdominal pain, facial swelling, airway edema • Treatment options include - Androgens - Epsilon amino-caproic acid - Recombinant C1 esterase inhibitor now available - Acute management includes airway support, IV fluid resuscitation, and Fresh Frozen Plasma (FFP).

+++++++++++++++++++++++++++++ Brain Anatomy of Language - Function and Location of: - Heschl's gyrus - Wernicke's Area - Angular gyrus - Broca's area And result of damage/lesion in each area

Heschl's gyrus: - Primary auditory cortex - tonotopic organization - If lesioned, would not have any language abilities -- comes before broca's and wernicke's areas. Wernicke's Area: - Superior temporal gyrus - Comprehension, extracting meaning from language Angular gyrus: - links written words with language (reading) Broca's area: - left inferior frontal lobe - Speech production

++++++++++++++++ HIGH YIELD, EASY POINTS ++++++++++++++++ Lysosomal Storage Diseases Know the symptoms, accumulated product, and enzymatic deficiency responsible for: • Tay-Sachs Disease • Niemann-Pick Disease • Gaucher Disease • Fabry Disease (X-linked recessive) • Krabbe • Metachromatic leukodystrophy MNEMONICS - Letters in name of DZ and what enzyme is deficient.

Hexosaminidase A is deficient in Tay-Sachs disease (MNEM: tAy-saX - heXosaminidase A) - GM2 ganglioside accumulates. - Lysosomes have onion skin appearance. - Presents with CNS degeneration, blindness, and cherry red spot on macula. No hepatosplenomegaly (unlike Niemann-Pick). Sphingomyelinase is deficient in Niemann-Pick disease. - Sphingomyelin accumulates. - Histiocytes look "foamy." - Presents with hepatosplenomegaly (unlike Tay-Sachs), anemia, cherry red spots on macula. β-Glucocerebrosidase is deficient in Gaucher disease. - Glucocerebroside accumulates in phagocytes. - Presents with hepatosplenomegaly, aseptic necrosis of heads of long bones, mild anemia. α-galactosidase A is deficient in FAbry disease. - Ceramide trihexose accumulates - X-Linked Recessive - Presents with peripheral neuropathy (especially in hands and feet) and angiokeratomas (small purple blemishes on skin) β-galactocerebrosidase is deficient in Krabbe disease. - Galactocerebroside accumulates - Presents with hyperactive reflexes, optic atrophy, and developmental delay. Arylsulfatase A is deficient in MetAchromAtic Leukodystrophy. - Cerebroside sulfate accumulates - Presents with demyelination in CNS and PNS, resulting in ataxia and dementia.

+++++++++++++++ UNDERSTAND THIS!!! +++++++++++++++ The Neuromuscular Junction (NMJ): - What occurs before the synapse? What disease result from antibodies to the steps here? - What happens after the synapse? What disease results from antibodies here? (MNEM: Alphabetical order for names of diseases)

How the NMJ works: (1) Action potential causes calcium influx into the pre-synaptic neuron. (2) Pre-synaptic motor neurons containing Ach-filled vessicles release Ach into the NMJ in response. (3) Ach binds to nicotinic Ach receptors on the target muscle, causing depolarization. (4) Muscle contracts MNEM: Alphabetical Order for the big 2: - Antibodies against Pre-synaptic Ca2+ channels causes Lambert-Eaton. - Antibodies against Post-synaptic Ach receptors causes Myasthenia gravis. Antibodies to Pre-Synaptic Motor Neuron Voltage-Gated Ca2+ Channels: = Lambert Eaton - For the pre-synaptic motor neuron to release Ach, there must be a Ca2+ influx via voltage-gated Ca2+ channels. - Antibodies binding these channels will prevent release of Ach and thus will prevent muscles from contracting. - Symptoms improve with use of the muscles / throughout the day. Antibodies to Post-Synaptic Muscle Ach Receptors: = Myasthenia gravis - for the muscle to contract, Ach must bind to post-synaptic nicotinic Ach receptors. - antibodies binding these Ach receptors will make the muscle less responsive to Ach. - Fluctuating symptoms, worsen with fatigue, improve with rest. Also... Antibodies to Pre-synaptic Ach-Releasing Vessicles: = Botulinism - Attacks the SNARE complex, which is required for Ach release. - Causes flaccid paralysis because Ach is not released -- muscles never get the green light to contract.

What defense mechanism is the hallmark of Obsessive Compulsive Personality Disorder? What are these patients like? How is their behavior described?

Intellectualization Obsessive-compulsive personality disorder is characterized by an excessive preoccupation with control, order, and perfectionism. - USMLE-Rx • Perfectionistic, constricted, and excessively disciplined • Behavior is rigid, formal, emotionally cool, distant • Overly conscientious, scrupulous, and inflexible about matters of morality, ethics, or values. • Use intellectualization as a defense mechanism • Often in med students/doctors • Ex: Person with attention to detail so excessive they cannot complete tasks

Adaptive Immunity: 2 Types 1. Humoral Immunity: - Involves production of __ cells, which produce antibodies that bind antigens resulting in their destruction. 2. Cell-Mediated Immunity: - Involves __ cells, which directly recognize and kill host cells that present antigens on their surfaces.

Humoral Immunity: B-cells; produce antibodies, which can fight the antigen. Cell-Mediated Immunity: T-cells; directly recognize and attack cells that present antigens to their surface. Remember: BOTH are ADAPTIVE IMMUNITY - Remember, adaptive immunity involves B and T cells. It responds 1-2 weeks after an infection and serves to fight infections and make antibodies that can fight the infection or protect you from it later.

++++++++++++ "HIGH YIELD" EASY POINTS +++++++++++++ Mucopolysaccharidoses: - Hunter syndrome - Hurler syndrome 1. Which enzyme is deficient in each? 2. What 2 same products accumulate in both of those? 3. Which one is X-linked Recessive? (MNEM) 4. Presentations - How do they differ? (MNEM)

Hunter Syndrome: XLRecessive - Deficiency: Iduronate sulfatase (X-linked recessive) - Accumulations: Dermatan sulfate and Heparan sulfate - MNEM: Hunters see clearly and aggressively aim for the X Hurler Syndrome: - Deficiency: Iduronidase - Accumulations: Dermatan sulfate and Heparan sulfate Presentations: - Hurler Syndrome: Developmental delay, gargoylism, airway obstruction, corneal clouding, hepatosplenomegaly. - Hunter Syndrome: Same as Hurler, but add aggressive behavior and there's no corneal clouding. Hunters see clearly and aggressively aim for the X. Hunter and Hurler syndromes can cause skeletal abnormalities and mental retardation Note that Hunter syndrome is X-linked recessive - Hunters see clearly and aggressively aim for the X Hurler and Hunter syndromes are the only mucopolysaccharidoses currently listed in First Aid USMLE-Rx Example: 1. A 2-year-old boy is brought to the physician by his parents due to concerns about progressive hearing loss. Developmental progress has plateaued since his last visit approximately 9 months ago. The child exhibits a prominent forehead and enlarged tongue. Hepatosplenomegaly and an abdominal hernia are also noted on physical examination. Laboratory analysis shows a deficiency in iduronate sulfatase. - Hunter Syndrome

A 6-month-old boy presents with a past medical history of skin infections presents with difficulty breathing, high fever, and nonproductive cough. He is immediately started on broad-spectrum antibiotics. On further questioning, his family history includes a maternal uncle dying at age 20 from overwhelming pneumonia. Sputum is induced with hypertonic saline inhalation, and eventual staining shows Pneumocystis. Serum studies show a normal B-cell count, elevated IgM, and low IgA, IgE, and IgG. He is immediately started on IVIG. 1. Diagnose 2. Pathogenesis: What is defective and what does this lead to? (Easy)

Hyper-IgM Syndrome Pathogenesis Summary: B-cell levels are normal and making IgM normally. However, the CD40 ligand required for Ig class switching from IgM to the other antibodies is defective. Thus, IgM doesn't switch to IgG, IgA, or IgE, so those 3 are deficient. IgM builds up so it is elevated. This is one of the Immunoglobulin Deficiencies (Immunodeficiency). Should have: ✔ Low IgE, IgA, IgG ✔ Normal IgM ✔ Normal B-cell count. Details: • Usually (70%) X-linked. • Deficiency in serum IgG, IgA, and IgE, but elevated IgM. • Normal B cell numbers. • Caused by mutations in CD40 ligand. • B cell CD40 engagement by T cell CD40-L is required for Ig class switching and germinal center formation. • Autosomal disease may be due to mutations in CD40, or in B cell activation-induced cytidine deaminase.

QRS axis -- Directions of Lead I, Lead II, and Lead avF What is QRS axis if Lead 1 is positive and avF doesn't have much activity?

I: Right II: Down right avF: Down Normal axis: Down right. So if lead I has upward peaks, the axis is to the right. If it has downward peaks, it's to the left. Lead avF upward peaks means it goes down. If they point downwards, it goes up. If Lead I is positive and Lead avF doesn't have much activity, that would be a normal axis.

Key Lab / CBC Findings to look out for in: 1. Iron Deficiency Anemia 2. Anemia of Chronic Disease (autoimmune) 3. α or β-Thalassemia 4. Lead poisoning anemia MCV Serum Fe TIBC Serum ferritin RDW RBC count Sideroblasts present?

IDA: INCREASED TIBC - The iron binding capacity is high because there's not much iron in the body. ACD: Decreased TIBC with decreased serum Fe Thalassemias: NORMAL Serum Fe NORMAL TIBC NORMAL Serum Ferritin NORMAL RDW INCREASED RBC count Lead poisoning: - Increased Serum Fe - SIDEROBLASTS are present

Diagnose this disease: - Arises from a defect in targeting of enzymes to the lysosome due to a deficiency of N-acetyl glucosamine phosphotransferase in the Golgi apparatus (frame shift or chain termination mutation) - Fibroblasts from affected individuals show dense cytoplasmic inclusion bodies due to impaired lysosome function. - Autosomal Recessive - Clinical: coarse facial features, death by age 8, severe psychomotor retardation, skeletal abnormalities. USMLE-Rx Example: An 18-month-old African-American boy is brought to the pediatrician because of failure to thrive and developmental delay. The infant is small with coarse facial features, corneal clouding, and an enlarged liver. He also has limited mobility in both his elbows and knees. Clinical molecular biologists are consulted to determine the patient's suspected biochemical disease. Fibroblasts cultured from the patient are found to have deficiencies in specific lysosomal enzymes with corresponding elevations in the patient's bloodstream. Blood and urine tests are negative for mucopolysaccharides. On electron microscopy, the fibroblasts show prominent vacuolization and inclusions in the cytoplasm. - Which defect is associated with this patient's most likely condition? A. Defect in protein tagging at the Golgi complex B. Defect in the breakdown of glycogen C. Defect in the breakdown of glycosaminoglycans D. Defect in the breakdown of sphingomyelin E. Defect in the metabolism of an essential amino acid

I-cell disease (Mucolipidosis type II) "I" stands for Inclusion Lack of N-acetylglucosaminyl-1-phosphotransferase causes a defect in protein tagging in the Golgi complex. Without this recognition tag, lysosomal enzymes are unable to be directed to primary lysosomes and instead are transported to the extracellular space, causing excess amounts of these enzymes to be detected in the blood. USMLE-Rx Example: A. Defect in protein tagging at the Golgi complex

+++++++++++++ KNOW THIS - HIGH YIELD +++++++++++++++ A 32-year-old man presents to the emergency department with worsening lower extremity weakness. The patient was in his usual state of health until approximately one week prior to presentation, where he developed increased difficulty climbing the stairs. Now he is unable to stand. A few weeks ago he developed bloody diarrhea, which he attributes to drinking unpasteurized milk. On physcal examination, he has bilateral lower extremity 1/5 strength with absent patellar reflexes. 1. What is the diagnosis? 2. What causes this disease? 3. How does it typically present? 4. Physical exam: - symmetric/bilateral or unilateral problems? - sensory, motor, or both? 5. What is the timing/onset of this disease? 6. Typically preceded by an infection, especially by what pathogen? 7. Is it congenital or acquired? 8. What is his prognosis?

IMAGE: Explains the etiology. I personally made that slide. - Impulse travels down peripheral nerve, but reaches the demyelinated region and slows down or stops. Unable to move muscles or feel sensation if it has to cross a damaged nerve. - It's a peripheral nerve problem, not a central nervous problem. The brain is still sending the motor and sensory impulses, but due to demyelination, the don't make it to the muscles. And ascending sensory impulses don't make it back up to the brain. 1. Guillain-Barré Syndrome - GBS is a rapid-onset muscle weakness caused by the immune system (autoantibodies) attacking the myelin of the peripheral nervous system, making it hard for action potentials to get all the way down to the distal limbs. - It's the primary cause of ACUTE polyneuropathy / peripheral neuropathy. Peripheral neuropathies are characterized by symmetric / bilateral weakness and loss sensation. 2. Autoimmune - antibodies attack myelin on peripheral nerves (see Image), making it hard for action potentials to get all the way down to the distal limbs. 3. Typically presents as symmetric, ascending weakness and sensory loss. Usually begins in distal limbs and moves proximally. Often affects Cranial Nerves as well -- facial paralysis. 4. Exam: symmetric weakness, areflexia, & sensory loss, typically in distal limbs and moves proximally. - both motor and sensory loss, starting in distal limbs and moves proximally. - e.g., something like "Achilles and patellar reflexes are 1+ (bad), 2+ reflexes in the brachioradialis and biceps, 3/5 strength on foot dorsiflexion and plantarflexion, and 4/5 strength on knee extension. > Look for weakness, diminished reflexes, and/or loss of sensation in distal limbs, like achilles, patellar, feet, etc. > But don't rule it out if it's in the proximal limbs too because eventually it moves proximally. 5. Acute Onset: The symptoms may develop over hours to a few weeks. Antibodies attack nerves near distal limbs first and then quickly moves proximally to other nerves. It can eventually affect the lungs, causing difficulty breathing. Look for something like "he was fine until a week ago," or something similar. The antibodies attack quickly. 6. Typically occurs after a Campylobacter infection. - MAKES SENSE: an illness could form harmful antibodies. - Curved, oxidase-positive, gram-negative bacteria that can be grown at 42°C. - This explains his "bloody diarrhea" 7. Acquired demyelinating polyneuropathy -- not congenital. Everything was going great until the day the peripheral nerves were attacked by the immune system. 8. Prognosis: Good! Almost all patients survive and achieve complete recovery in several weeks. USMLE-Rx -- Another Example Stem: A 28-year-old man presents to the emergency department complaining of increasing muscle weakness. He states he first noticed that he had some mild weakness in his feet and legs 4 days ago. He initially attributed the weakness to "being tired," but became concerned when he began to have difficulty walking. He states that his symptoms have progressed over the past few days, and the weakness has now spread to his arms and hands. Other than a recent episode of diarrhea and vomiting, the patient has been in good health. His neurologic exam is notable for 1+ Achilles and patellar reflexes, 2+ reflexes in the brachioradialis and biceps, 3/5 strength on foot dorsiflexion and plantarflexion, and 4/5 strength on knee extension.

How to recognize an Autosomal Recessive pedigree

If any affected child has 2 unaffected parents, the disease must be autosomal recessive. An affected individual must inherit a recessive allele from both parents, so both parents must have an allele. Must be homozygous for it to show up. If heterozygous, can still be a carrier. Dominant means that a single copy of the disease-associated mutation is enough to cause the disease. This is in contrast to a Recessive disorder, where two copies of the mutation are needed to cause the disease. Remember, males can be carriers here because these are autosomal chromosomes, not sex chromosomes. Has nothing to do with how many X chromosomes you have. So males and females are equally affected.

ITP vs. TTP vs. DIC in levels of each: - Platelets - Hemoglobin - MAHA (Microangiopathic Hemolytic Anemia) present? - Hemolysis labs (LDH, haptoglobin, Tbili) - Peripheral smear - Creatinine - PT/PTT - Fibrinogen - D-Dimer

ITP: acquired immune-mediated destruction of otherwise normal platelets by IgG antibodies directed against platelet surface glycoproteins such as GPIIb/IIIa. - isolated thrombocytopenia (other labs should be normal) Thrombotic Thrombocytopenic Purpura (TTP): deficiency of ADAMTS13, which breaks apart large vWF proteins, resulting in large vWF multimers → increased platelet adhesion → platelet thrombosis - Labs: Anemia (low Hgb), thrombocytopenia (low Plts), ↑ LDH, ↑ Cr, negative Coomb's, ↑ bleeding time, normal PT/PTT DIC: over-activation of coagulation cascade, which leads to thrombi production, which leads to coagulopathy. Fibrinolysis excessively occurs at sites of thrombi, and fibrin degeneration products (e.g., D-dimers) are elevated and can interfere with coagulation. - i.e., widespread clotting and degradation of those clots. - Labs: thrombocytopenia (platelets are low), elevated PT and PTT, low fibrinogen, elevated D-dimers (from clot degradation). MAHA = Microangiopathic Hemolytic Anemia -- anemia caused by destruction of erythrocytes from physical shearing as a result of passage through small vessels occluded by systemic microthrombi. MAHAs are characteristically accompanied by thrombocytopenia in the absence of defects in coagulation. Hemolytic Uremic Syndrome (HUS), DIC, and Thrombotic Thrombocytopenic Purpura (TTP) are the prototype MAHAs Fibrinogen is a soluble protein present in blood plasma, from which fibrin is produced by the action of the enzyme thrombin. Low in DIC. - Fibrin forms a fibrous mesh that impedes the flow of blood. D-dimer is a fibrin degradation product (or FDP), a small protein fragment present in the blood after a blood clot is degraded by fibrinolysis. It is so named because it contains two D fragments of the fibrin protein joined by a cross-link. Increased in DIC and can be increased in TTP.

How does 2,3-BPG affect the O2 saturation curve?

Increases oxygen unloading in tissues so it causes a right shift High altitudes. Conditions that cause hypoxia (deficiency of oxygen) such as anemia, smoking and high-altitude, increase BPG levels in the red blood cells

+++++++++ HIGH YIELD, EASY POINTS +++++++++ What cells are involved in the Innate Immune System vs. the Adaptive Immune System? Which type is always present and active, very fast and ready to go (activated immediately to try to kill pathogen), but is not specific and has no memory? Which type must be induced to turn on, is very slow (1-2 weeks after an infection), but much more potent, highly specific, and has memory?

Innate: • Neutrophils, Macrophages, NK cells • Always present / on • Fast -- immediate response; tries to kill the pathogen. • Not very specific • No memory • Activates inflammation • Uses receptors that recognize patterns associated with pathogens to discriminate self vs. non-self Adaptive: • Lymphocytes: B cells, T cells • Must be induced • Slow -- 1-2 weeks after infection, but much more potent response. • Initiated in secondary lymphoid tissues • Highly specific • Has Memory

Your mother calls you in the morning. She has recently noticed a small amount of bloody fluid leaking from her left nipple. She is 42 years old. She ask you what it could be and states that she is afraid to go to the doctor. What is it most likely?

It is most likely a benign condition, specifically an intraductal papilloma (and go see a doctor) First Aid: small fibroepithelial tumor within lactiferous ducts, typically beneath areola. Most common cause of nipple discharge (serous or bloody discharge). Slight risk for cancer.

Types of Vasculitis: - Large Vessel Vasculitis (2) - Medium Vessel Vasculitis (2) - Small Vessel Vasculitis -- 3 that are ANCA+ and 3 that are ANCA- High yield: Proper workup is to give them ___ BEFORE doing a biopsy.

Large Vessel Vasculitis: - Takayasu's arteritis - Temporal arteritis Medium Vessel Vasculitis: - Polyarteritis Nodosa - Kawasaki's disease Small Vessel Vasculitis: ANCA + - Eosinophilic Granulomatosis with polyangiitis (Churg-Strauss Syndrome) (p-ANCA) - Granulomatosis with polyangiitis (Wegener Granulomatosis) (c-ANCA) - Microscopic polyangiitis ANCA - - Henoch Schonlein Purpura - Cryoglobulinemic vasculitis - Leukocytoclastic vasculitis High Yield: Particularly for Temporal / Giant Cell arteritis First give steroids (IMMEDIATELY) Then do a biopsy The reason for giving steroids right away is to prevent blindness in Temporal Arteritis. MNEM: Giant Cell Arteritis -- Give Corticosteroids Ammediately - and Gaw Claudication Always

++++++++++++++++++++++++++++++++++++++++++ RHEUMATIC FEVER: - J♡NES Criteria - Often follows what? - Caused by what pathogen? - It's a Type __ Hypersensitivity reaction caused by antibodies to ___ protein. - Usually causes what type of murmur? - Histology of cardiac tissue will show ___ bodies. Which valvular abnormalities is NOT commonly associated with rheumatic heart disease? A. Fusion of commissures B. Perforation of valve leaflets C. Thickening of chordae

J - Joints (migratory polyarthritis) ❤ - Endocarditis, fusion of commissures, thickening of chordae N - Nodules - Subcutaneous E - Erythema marginatum (ring-like rash) S- Sydenham chorea (jerky, dance-like movements in extremities) Often follows strep throat / pharyngitis Type II hypersensitivity (immune mediated); not directly caused by bacteria; antibodies to M protein cross-react with self antigens (Molecular Mimicry). Causes Mitral Regurgitation. Early, there's Mitral Regurg (holosystolic). Late, there's Mitral Stenosis (opening snap) Caused by Group A Strep: Strep Pyogenes Histology: Aschoff Bodies -- Look for cardiac tissue with an area containing those pink circles. B. Perforation of valve leaflets - This would be characteristic of acute endocarditis (S. aureus) - The other two are characteristic of Rheumatic Fever. - Fusion of commissures - individual parts of valve fuse. E.g., A 25-year-old woman presents to her primary care physician's office for her annual physical. She recently immigrated from a developing country and reports having multiple episodes of pharyngitis in the past. On physical exam, there is a holosystolic murmur at the apex. A follow-up echocardiogram shows mitral regurgitation, concerning for rheumatic heart disease.

++++++++++++++++++++++++++++ What disease has the classic symptoms of CRASH and Burn: Conjunctivitis Rash Adenopathy (= lymphadenopathy) Strawberry tongue Hands (peeling skin) Fever ("Burn")

KAWASAKI DISEASE Necrotizing vasculitis of small & medium vessels (coronary arteries - aneurysm)

Kwashiorkor vs. Marasmus - appearance of child - what is deficient - which one involves low albumin?

Kwashiorkor - Belly protrudes - Protein malnourishment - Decrease in albumin (due to low protein) leads to loss of oncotic pressure and edema (hence the squashy) - Due to edema, body weight may be normal(ish) - Skin and hair changes, along with anemia and immunodificiency Marasmus - Skinny - Calorie malnourishment - Emaciated extremities - Big head (compared to body) - Anemia (Fe) - Immunodeficiency

If a biopsy of a glomerulus shows wire-looping, what is the diagnosis? It will also show Full House Staining on IF -- IgA, IgG, IgM , C3, C1q, kappa, lambda This involves what type of nephritic syndrome? MNEMONIC

LUPUS / SLE MNEM: Wire-Lupus Involves Diffuse proliferative glomerulonephritis Renal complications are the #1 cause of death in SLE patients. Often due to SLE (think"wire lupus"). DPGN and MPGN often present as nephrotic syndrome and nephritic syndrome concurrently. LM—"wire looping" of capillaries IF—granular; EM—subendothelial, sometimes subepithelial or intramembranous IgG-based ICs often with C3 deposition Lupus Symptoms: RASH OR PAIN: Rash (malar or discoid) Arthritis (nonerosive) Serositis (eg, pleuritis, pericarditis) Hematologic disorders (eg, cytopenias) Oral / nasopharyngeal ulcers (usually painless) Renal disease Photosensitivity Antinuclear antibodies Immunologic disorder (anti-dsDNA, anti-Sm, antiphospholipid) Neurologic disorders (eg, seizures, psychosis)

Gross appearance of heart tissue after an MI (to solidify what a heart attack does to the heart). - What will the heart tissue look like hours after an MI? - What will heart tissue look like after 2-3 days (subacute MI)? - What will heart tissue look like weeks to years later (healed)? - Appearance of a rupture.

Left Picture: - Red is normal, White is dead tissue. - The tiny white asterisk over the dark spots shows where a rupture occured, which is what killed this patient. Dead heart tissue can easily rupture, especially if reperfused. Right Picture: - Purple box: Acute MI. Hours s/p MI. Only partially ischemic so far. - Green box: Subacute MI. Days s/p MI. Tissue has now turned yellow. - Yellow box: Healed MI. Weeks to years s/p MI. The tissue is dead but risk for rupture is low. Tissue has turned white because it isn't getting any oxygen or blood.

A 3-year-old female presents with an abnormal "dark spot" in certain parts of the body and large nodules all over her back. The mother reports that these spots began to develop sometime during the last year, but have progressively worsened. The mother also states that her husband has similar skin findings and a history of seizures. On exam, 7-8 uniformly hyperpigmented macules are noted on the chest, back, and buttock. There is freckling in the axilla and inguinal region. 1. What is the diagnosis? 2. These are derived from what type of nervous system cell?

Neurofibromatosis Schwann cell (peripheral nerve support cell, not the neuron itself) These patients present with: Neurofibromas Optic gliomas Café au lait spots Lisch nodules Pheochromocytomas Image: This is an extreme example of neurofibromatosis.

Pancoast Tumor: - Where does this tumor occur? - Famously causes ___ syndrome - How does it affect the brachial plexus? What symptoms do you see?

Lung Cancer -- Carcinoma that occurs in apex of lung It Causes: Horner syndrome - due to proximity of the tumor to the superior cervical sympathetic ganglion chain can result in destruction/compression of the chain yielding characteristic: > ptosis > miosis > anhidrosis ^ (Eye is droopy, pinned, and dry) Thoracic outlet syndrome - due to proximity of the tumor to the thoracic outlet of the brachial plexus can result in compression of the C8, T1, and T2 nerve roots yielding characteristic: - axillary pain - shoulder pain especially along the cervical border of the scapula - pain along the ulnar nerve distribution - fourth and fifth digit pain, numbness, and muscle atrophy

MARFAN SYNDROME Now the USMLE places emphasis on cadiac and eye manifestations 1. Main cardiac manifestation 2. Main eye manifestation 3. What is the classic genetic mutation that causes it?

MARFAN SYNDROME Autosomal dominant connective tissue disorder Main Things to Remember: 1. Aortic Root Dilation / Aneurysm 2. Ectopia Lentis: Lens in wrong location 3. Mutation in the Fibrillin-1 gene (FBN1) - Fibrillin is an extracellular matrix protein found all over Other signs/sxs: -Wrist Sign -Thumb Sign -Pectus (Carinatum > Excavatum) -Flat Feet -Pneumothorax -Reduced upper segment:lower segment -Increased arm span:height -Skin Striae -Scoliosis -MV prolapse MedBullets Example: A 30-year-old man with a marfanoid habitus presents for genetic counseling. His father, paternal uncle, and paternal great-grandfather died of sudden cardiac deaths. His father, specifically, suffered from an aortic dissection. As part of this patient's work-up, he recently had cardiac imaging, which reveals a 5 cm aortic aneurysm.

++++++++++++++++++++++++++++++++++ That amazing ANEMIA chart -- Memorize and you can get all anemia questions right! Draw it out a few times. Microcytic Anemia (MCV <80) -- 5 causes Normocytic Anemia (MCV 80-100): - Low reticulocyte count -- 3 causes - Normal retic -- 1 cause - High retic -- many causes Macrocytic Anemia (MCV >100): - Megaloblastic Anemia -- Low Retic -- 4 causes

MEMORIZE IMAGE

Congenital birth defects: Disruptions Major causes of disruptions - to be able to recognize them in stems

Major factors responsible for disruptions: - vascular (occlusion, hemorrhage) - ischemia - ionizing radiation - infection - early amnion rupture Examples include some facial clefts and missing digits or limbs. A disruption is due to a destructive force acting upon an otherwise normal developing structure. Anomalies caused by disruptive forces can present a particularly distinctive appearance because of the loss of tissue and aberrant differentiation of adjacent tissues with which adhesions may have developed. Everything was growing normally until some external event happened and interfered with normal growth or some other process. See Image.

++++++++++++++++++++++++++ Brachial Plexus - Simplified Musculocutaneous, Axillary, Median, Radial, & Ulnar nerves -- what are their spinal roots. What are the symptoms of damage to each of those nerves? Lateral, Posterior, and Medial branches

Memorize Read it Right to Left C5/C6 => Lateral Cord => Musculocutaneous and Median nerves (LMM) C7 => Posterior Cord => Axillary and Radial nerves (PAR) C8/T1 => Medial Cord => Ulnar and Median nerves (MUM) MNEMONICS: Alcohol-Related - Reach To Drink Cold Beer: Roots, Trunks, Divisions, Cords, Branches. - Most Alcoholics Must Really Urinate: Branches/Nerves are Musculocutaneous, Axillary, Median, Radial, Ulnar, in order. Injuries: - Musculocutaneous: cannot flex elbow, cannot supinate arm. So elbow will be extended and arm will be pronated (like in Erb's Palsy). Can't flex your muscles. - Axillary: usually from injury to the surgical neck of the humerus. Can't abduct between 15-90 degrees. So they can lift their arms only slightly, but not all the way. - Median: Carpal Tunnel Syndrome. Loss of sensation to lateral 3.5 fingers, so pinky, ring and lateral half of middle finger. Distal lesion causes Median Claw hand: When asked to open hand, index and middle fingers stay flexed. Proximal lesion causes blessed hand: When asked to make a fist, thumb, index and middle fingers stay extended, like you're blessing someone. Different from Ulnar Claw Hand because thumb is also affected. - Radial: Supplies Triceps, Brachioradialis, and Wrist Extensors. Injury results in Wrist Drop. Sensory loss over posterior arm and forearm, as well as below the fingertips of the medial 3 1/2 fingers. - Ulnar: Ulnar Claw Hand: Can't extend 4th and 5th digits, but you can still flex the thumb (unlike median nerve damage).

Renal Blood Flow maintenance: - Define the Myogenic Response - Define the Tubuloglomerular Feedback Mechanism - What happens to RBF and GFR during sympathetic activation (e.g., exercise)? - During SNS activation (exercise), what happens to blood flow to: - Brain - Heart - Kidneys - Skeletal Muscles - Mesen. arteries

Myogenic response: detects pressure not RBF per se. More relevant in protecting kidneys from high pressure Tubuloglomerular feedback: ensures a constant delivery of NaCl to the distal tubules not RBF or GFR per se Strong sympathetic activation decreases RBF and GFR: running away from the present danger or maintaining BP in hemorrhage is more important in the short term When ↓RBF, RAAS works hard to keep GFR afloat

Highest Yield Cardiac Tumors: Myxoma vs. Rhabdomyoma - Which is the most common ♡ tumor in kids? Benign or malignant? - Which is the most common ♡ tumor in adults? Benign or malignant? - Which one is usually described as a "ball valve" obstruction in the Left Atrium and often involves multiple syncopal episodes due to LA obstruction? - Which one is associated with Tuberous Sclerosis? - How is the histology of a Myxoma usually described? (MNEM)

Myxoma - Most common ADULT heart tumor - MALIGNANT - First Aid: Myxomas are usually described as a "ball valve" obstruction in the left atrium and are associated with multiple syncopal episodes. - "Ball valve" obstruction of Left Atrium causes syncope (less blood to brain) - Called ball valve because it's a ball shape sitting around the mitral valve. - MNEM: Called a "myxoma" as it's composed of mixed tissues and myxoma cells. Composed of mesenchymal cells and associated endothelial, smooth muscle, and fibroblastic cells set in a matrix. - Firt Aid: Histology shows a gelatinous material with myxoma cells immersed in glycosaminoglycans. Rhabdomyoma - Most common heart tumor in CHILDREN - BENIGN - Most frequent 1° cardiac tumor in children (associated with tuberous sclerosis). - Histology: hamartomatous growths - Tuberous sclerosis: rare disease that causes benign tumors to grow all over the place: in the brain, skin, kidneys, eyes, heart, or lungs. BUZZword for tuberous sclerosis is a hypopigmented macule / spot (ash leaf spot).

A lawyer loudly demands the best hotel room money can buy because he is their best customer. Diagnosis?

Narcissistic Personality Disorder I just made this card to memorize the classic STEP description. STEP vignettes are often lawyers - Dr. Spollen ✔ Egocentric, grandiose ✔ Crave admiring attention and praise and place excessive emphasis on beauty, power, fame, and/or wealth. They just want to be better than others. ✔ Feel "entitled" to special rights, attention, privileges, and consideration ✔ Quick to mention their degrees or power, put others down (to make themselves seem relatively better)

Acute Post-streptococcal Glomerulonephritis - Clinical presentation - Preceded by what? - Histology - Nephritic or Nephrotic?

Nephritic - so blood, HTN Most frequently seen in children. 2-4 weeks after group A streptococcal infection of pharynx or skin. Resolves spontaneously in most children; may progress to renal insufficiency in adults. Type III hypersensitivity reaction. Presents with peripheral and periorbitaledema, tea or cola-colored urine, HTN. ⊕strep titers/serologies, complement levels (C3) due to consumption. LM—glomeruli enlarged and hypercellular IF—("starry sky") granular appearance ("lumpy-bumpy") due to IgG, IgM, and C3 deposition along GBM and mesangium EM—subepithelial IC humps

Electrical activity of the nodes: - Contraction (firing) occurs when what ion flows in? - Relaxation occurs when what ion flows out?

Nodes fire when Ca2+ flows in - Depolarization, because inside becomes less negative (not polarized) Relaxes when K+ flows out - Repolarization, because inside becomes more negative (polarized) In heart muscle, the action potential is more jerky -- contraction occurs when Na+ moves in and relaxation occurs when Ca2+ and K+ move out.. MNEMs: - Nodes -- Milk (Ca2+) going into NOSE - Muscle -- Jerky -- Beef JERKY is salty -- Na+ influx.

Bacterial vs. Viral Meningitis CSF findings: - Cell count - Differential: What type of immune cell will be present? - CSF glucose - CSF protein - Gram stain + or -

Normal CSF should essentially be acellular and have no inflammation Cell count is high in bacterial because bacterial cells are counted. Low in viral because viruses kill cells. CSF glucose is low in bacterial meningitis because bacteria use up glucose and produce lactose. Gram stain will only be positive if it's bacterial.

Result of damage to midbrain cranial nerves: -3 -4

Oculomotor (CN3): impaired adduction, supradduction and infradduction of the ipsilateral eye with or without a dilated pupil. The eye is turned DOWN AND OUT. Trochlear (CN4): eye unable to look down when the eye is looking in towards the nose (superior oblique).

What type of arthritis is described? › Pain isn't so bad in the morning, but gets worse with use throughout the day. › Heberden's nodes (DIPs included - near fingertips) › Bouchard's nodes (PIPs) › Osteophytes (small growths on various bones, can be painful) › Eburnation (polished, shiny bone) › Subchondral cysts › NO swan neck deformity or ulnar deviation. Those would 100% rule this out. e.g., A 56-year-old woman presents to her primary care physician with pain in her hands. The pain began approximately 1 year prior to presentation and has progressively worsened. She describes the pain being worse in the evening and improves in the morning. She has also noticed swelling in her knuckles. On physical exam, there is bone deformity noted on the distal and proximal interphalangeal joints, as well as tenderness upon palpation of the affected joints. MSK Pathology

Osteoarthritis - includes the DIPs - Heberden's nodes near fingertips. (whereas Rheumatoid spares the DIPs) > MNEM (alphabetical): The "D" in DIPs is closer to "Osteo" than "Rheumatoid," so if it says it affects the DIPs, think of which one is alphabetically first: Osteoarthritis. Eburnation is polished, shiny bone that results from friction. Makes sense that it would go with Osteoarthritis, which results from friction. Caused by loss of cartilage. Osteophytes are associated with Osteoarthritis. These are small little nodules that grow on bone. They can cause pain (if they pinch or compress a nerve). Osteoarthritis pain gets worse with use and throughout the day. - Worse at night, Better in the morning. - Makes sense -- bones rubbing together would cause increasing pain throughout the day. (whereas rheumatoid is bad when they wake up, but better with use) Pathogenesis of Osteoarthritis: (First Aid) Mechanical—wear and tear destroys articular cartilage, which degenerates the joints, and causes inflammation without adequate repair. Chondrocytes (which make cartilage) mediate degradation and inadequate repair. (whereas Rheumatoid is autoimmune, Type III HS)

A 9-month-old boy is seen by his pediatrician for failure to thrive and for growth retardation. He has experienced multiple long bone fractures since birth. His mother has recently noticed that he has been bruising and bleeding more easily than usual. He has also developed chronic nasal stuffiness. An x-ray image shows diffuse symmetric sclerosis of bones. 1. Diagnose 2. The hallmark of this disease is abnormal function of which bone cell type? 3. ^This leads to failure of bone ___. 4. Why do these patients have bleeding problems, namely pancytopenia (anemia, thrombocytopenia, etc.)? MSK Pathology

Osteopetrosis Failure of bone absorption = thick, but brittle bones. - Bones are thick, but very likely to fracture because the normal process of remodeling and resorption is disrupted. Caused by failure of osteoclasts - that's why bones get so big; failure of bone resorption by osteoclasts. Can have Anemia or even Pancytopenia because overgrowth of bone fills up the bone marrow. Can also have nerve problems / pain if large bones compress a nerve. STEMS CAN FOCUS JUST ON PANCYTOPENIA! The only osteopetrosis USMLE-Rx Q was this one: A mother brings her 2-month-old daughter to the emergency department because of lethargy and a fever of 39.2°C (102.6°F). The mother is also concerned because the infant is not gaining weight. Physical examination reveals that the infant has an increased head circumference and prominent hepatosplenomegaly. Mastoid and paranasal sinus malformations are also noted during the course of the workup. Results of laboratory tests are as follows: HGB: 7.9 g/dL, Hct: 28%, WBC: 373/μL, MCV: 78.2 Reticulocyte count: 6.5% Despite fluid resuscitation and initiation of antibiotic therapy, the infant dies. On autopsy, histologic analysis of the bone marrow reveals a lack of a medullary canal, with persistence of the primary spongiosa, and deposition of dense, sclerotic bone. Which malfunctioning cells are the cause of this patient's disease process? - Osteoclasts

Diagnose: A 7-year-old girl is brought to the emergency department after experiencing a wrist fracture. The patient fell down to the ground and has not hit her head. This has never happened before. The patient's parents reports their child states she has pain in bones and at times feels weak. On physical exam, there is tenderness to palpation of the wrist with a waddling gait. Laboratory testing is notable for a decreased serum calcium and phosphate level, and increased alkaline phosphatase and parathyroid hormone. ↓ Calcium ↓ Phosphate ↑ PTH ↑ Alkaline Phosphatase MSK Pathology

Osteomalacia (adults) Rickets (kids) VITAMIN D Deficiency Failure of bone mineralization = soft bones. Low VitD ➔ Low Ca2+ ➔ Increased PTH ➔ Bone reabsorption

A 68-year-old woman presents to her primary care physician with lower back pain of acute onset. She denies any trauma to the spine or any radiation of pain. Her last menstrual period was when she was 51-years-old. On physical exam, she has tenderness to palpation at the level of L4-L5, as well as a loss of lumbar lordosis. A dual-energy x-ray absorptiometry (DEXA) scan reveals a T-score of -2.7. 1. Diagnose 2. Due to reduction of ___ bone resulting from decreased levels of what hormone? 3. Often presents with compression fractures where and Colles fractures where? 4. Prophylaxis for preventing this disease 5. Treatment (medications) MSK Pathology

Osteoporosis Reduction of trabecular bone due to decreased estrogen. ↓ Estrogen => ↑ bone reabsorption. Can present with compression fractures of the vertebral column, causing acute back pain, loss of height, and kyphosis. Can also can present with fractures of femur and radius (Colles fractures). Prophylaxis: regular weight-bearing exercise and adequate Ca2+ and vitamin D intake throughout adulthood. Treatment: bisphosphonates, teriparatide, SERMs

++++++++++++++++++++++++++++ Know the neurotransmitter, nucleus, area of the brain for various vignettes: Panic Disorder & PTSD (overactive "fight or flight"): 1. What NT is too high? 2. What part of the brain do you suspect is overactive? 3. Which part of the brain is overactive? 4. What 2 classes of medications can you give them for treatment: Short-term (acute attack) and Long-term (prevent attacks)? 5. CO2 / False Suffocation Theory of Panic DO -- Effect of sodium Lactate or bicarbonate infusion (which raises the CO2 in the blood) on panic DO pts.

Overactive SNS: Panic DO and PTSD: 1. Norepineprine is too high; etiology of panic in both conditions. - also GABA and serotonin are deficient in anxiety overall, but NE causes the panic feeling - overactive SNS - makes sense. 2. Brain Damage: Locus Coeruleus - located in pons (brainstem) - makes NE -- makes sense: NE and SNS activity. 3. The Amygdala is overactive - key player in panic attacks - "fight or flight" response. 4. SSRIs 1st line for long term tx and prevention. Benzos for acute attack / ER use to end the attack quickly. 5. Lactate/CO2 sensitivity -- Lactate and Bicarconate increase blood [CO2] and can produce panic sxs in patients with Panic DO and PTSD. • Thus, CO2 induces panic attacks only in people with Panic DO. • Might be why you breath heavily during panic attacks -- blow off CO2. • Excess CO2 tells the brain that you need more oxygen. So even though they are getting plenty of oxygen, they feel like they are suffocating. • More evidence: People with panic DO often have chronic hyperventilation (need more O2 in general). • So even though they are getting enough O2, something in their brain is saying BREATHE FASTER, YOU'RE RUNNING OUT OF BREATH!! GET MORE OXYGEN!!

A 63-year-old male is brought to the physician by his wife for the evaluation of a tremor. The tremor is worse at rest, and decreases in severity with purposeful movement. The patient reports to having difficulty with initiating voluntary movement, and his wife states that the patient's movements have been slow. On exam, there is seborrheic dermatosis on the nasolabial folds. There is a "pill-rolling" resting tremor accentuated when the patient is asked to perform mental calculations. Resistance to passive movement at the elbow joint is noted. On gait testing, there was difficulty with initiating gait, as well as the patient taking short steps when walking forward. 1. Diagnose 2. Key symptoms of this disease - MNEM 3. This patient's disease involves accumulation of ___ Bodies. 4. This patient has damage to what two areas, and low amounts of what NT, causing his jerky movements? 5. What is the best, #1, first line medication to treat Parkinson's?

Parkinson's Disease Absolute Diagnostic Criteria: ✔ Bradykinesia - REQUIRED - Bradykinesia = slowed movements, difficulty initiating movements. ✔ At least one of the following: 1. Rigidity in movement 2. Resting tremor: Not required. 1/3 pts don't have a tremor, which is called "Atypical Parkinson's" 3. Postural instability: e.g., push them and they can't keep themselves balanced. Parkinson's Key Symptoms: "Man, Parkinson's Really Blows" - Muscle rigidity - Postural instability - posture is off - Resting tremor: "pill-rolling" - Bradykinesia (slow movement; Required Symptom) Non-Motor Symptoms: - Autonomic symptoms - Orthostatic hypotension - Overactive Bladder - Constipation (common) - Anosmia (loss of smell) (common) - REM Behavioral Disorder (Dec. REM sleep with resulting behavioral changes) - Cognitive impairment (confusion, dementia in late stage, hallucinations) - Hallucinations, mostly visual; <10% of pts; Tx is reassurance or clozapine / quetiapine if really bothersome. 3. Associated with Lewy Bodies - But different from "Lewy Body Dementia" 4. Brain Damage: ✔ Substantia nigra (midbrain) - makes dopamine ✔ Caudate and Basal ganglia - loss of smooth movements ✔ Low Dopamine - caudate and basal ganglia need dopamine to smoothen movements. Treatment: Dopamine agonist - Monitor for schiophrenia-like sxs. - L-Dopa has been the best treatment for >150 years. It's the only medication available that replaces the lost dopamine. - Less than 30% of pts are unresponsive to even the highest doses of L-Dopa, but there are other options, like deep brain stimulation. - Of note: As Parkinson's progresses, more neurons die and an increased dosage of L-Dopa is usually needed when this happens, since there would be even less natural dopamine available. - L-Dopa Side Effects: Sleep problems, Manic sxs (like spending all money, impulsivity, etc.), positive Schizo sxs like hallucinations, and Dyskinesias. NEVER GIVE THEM AN ANTIPSYCHOTIC, which would just reverse the dopaminergic benefits of L-Dopa.

Diagnose: A patient presents with a fever, diffuse myalgias, abdominal pain, a peripheral neuropathy, hypertension, and a recent weight loss of 10 pounds. The patient has a past medical history of hepatitis B and C. Angiography demonstrates a "string of pearls appearance" in the renal artery, as well as in several other organ systems with the exception of the pulmonary arteries.

Polyarteritis Nodosa (PAN) - A medium-sized vessel Vasculitis 25% patients are hepatitis B sAg positive - Age: 40-60 years. - Males and females equally involved. - 25% of patients with PAN are (Hep BsAg+). - Pathology: Fibrinoid necrosis of small to medium sized artery walls. Involves all organs and tissues (except the lungs). > Skin- nodules, palpable purpura, ulcerations, etc. > Heart- coronary arteritis can cause manifestations including tachycardia, MI, and CHF (with SOB). > GI tract- (up to 50% pts)- pain, bleeding, perforation. > Renal- vasculitis may cause renal dysfunction and/or HTN. > Nervous System- CNS and PNS (mononeuritis multiplex). > Testicles, liver, breast, eye, ovary, gallbladder can also be involved.

A 40-year-old gentleman presents with episodic headaches and palpitations. Attributing it to his tendency to worry excessively, he put it off for several months until he began to sweat episodically too. In the clinic, his blood pressure is found to be elevated at 160/120 mmHg. Plasma metanephrines are elevated and an abdominal CT scan reveals an adrenal mass. 1. Diagnose 2. Proper treatment regimen

Pheochromocytoma - from chrommaffin cells, most common adrenal medullary tumor in adults - NE, Epi, or both, sustained hypertension, but sometimes episodic - HTN, heart palpitations, headache (main symptom), anxiety, sweating, pallor and glycosuria - measure i.e. metanephrine in plasma and VMA in urine - Tx: α-blocker, then β-blocker, then surgery ^ i.e., fix HTN, then Tachycardia, then remove the tumor. ^ Phenoxybenzamine is the most common α-blocker used. Similar to Neuroblastomas, which derive from neural crest cells, but these are the most common adrenal medullary tumors in children, whereas pheochromocytomas occur in adults.

What medication can treat a patient with anticholinergic syndrome (i.e., with severe muscarinic effects), such as overdosing on atropine, an Alzheimer's patient (low Ach), muscarinic mushroom poisoning, organophosphorus poisoning, etc.? (sxs of atropine OD = what muscarinics do = palpitations, dilated pupils, difficulty swallowing, hot dry skin, thirst, dizziness, restlessness, tremor, fatigue, and problems with coordination).

Physostigmine (Ach-Esterase Inhibitor) - so it increases Ach by inhibiting the Ach-degrading enzyme. It is used clinically to reverse the effects of anticholinergic symptoms Atropine is an anticholinergic that helps reduce saliva, mucus, and other secretions in your airway during a surgery. Atropine is also used to treat spasms in the stomach, intestines, bladder, or other organs. Atropine is sometimes used as an antidote to treat certain types of poisoning. - Other source: Atropine is indicated for blocking severe or life threatening muscarinic effects (e.g., as an antidote for organophosphorus or muscarinic mushroom poisoning, and to treat bradyasystolic cardiac arrest). Physostigmine is an Acetylcholine-Esterase Inhibitor used for reversing anticholinergic syndrome induced by anticholinergics, such as Atropine. Thus, it increases / restores Ach by blocking the Ach-degrading enzyme. Patients with Alzheimer's are at increased risk for being sensitive to anticholinergic medicines due to their low Ach.

How would an Aspirin overdose affect respiratory rate, blood pH, and blood bicarbonate?

Point E in IMAGE Salicylates crosses blood brain barrier as salicylic acid and lower the pH of central chemoreceptors that control ventilation. So the central chemoreceptors are fooled into thinking that there is too much CO2, and ventilation is increased. Acute hyperventilation raises pH and lowers HCO3 slightly (point E) because as CO2 is blown off, HCO3 is converted into CO2.

Polyneuropathy / Peripheral Neuropathy: - What is it? - Affects proximal or distal limbs? - Symmetric or Asymmetric? - How does it affect sensation? - How does it affect motor function / muscle movement? - Specific examples / causes. - An acute polyneuropathy is most likely what dx?

Polyneuropathy is any damage or disease that affects the peripheral nerves (peripheral neuropathy) in roughly the same areas on both sides of the body, featuring both weakness and numbness. Look for symmetrical weakness and numbness in distal limbs. - Affects distal limbs -- hands, feet (Image) - Symmetrical / Bilateral symptoms (Image) - Distal limbs lose sensation - vibration, proprioception, pinprick, and temperature. - Distal limbs are weak / flaccid and there are reduced reflexes. Due to loss of lower motor neurons. - Examples: Vitamin B12 deficiency, Guillain-Barré syndrome - Acute polyneuropathy: Usually Guillain-Barré syndrome (rapid-onset muscle weakness caused by the immune system damaging / demyelinating the peripheral nervous system)

Polycythemia Vera: - What is the best lab test for help with a PCV diagnosis - What mutation has been associated with the vast majority of PCV cases? - What causes the flushing and itchiness after hot showers (classically tested symptom)? - They will have an enlarged ___.

Polycythemia Vera = general increase in red blood cells - EPO level -- will be decreased -- don't need to make more RBCs. - JAK2 mutation - Increased mast cells (all granulocytes are increased) - Enlarged Spleen (Splenomegaly) FIRT AID: Polycythemia Vera - Disorder of RBCs, usually due to acquired JAK2 mutation. - May present as intense itching after shower. Rare but classic symptom is erythromelalgia (severe, burning pain and red-blue coloration) due to episodic blood clots in vessels of the extremities. - DECREASED EPO (vs 2° polycythemia, which presents with endogenous or artificially increased EPO). - Treatment: phlebotomy, hydroxyurea, ruxolitinib (JAK1/2 inhibitor).

A 50-year-old man complains of open, non-healing blisters on the dorsal surfaces of his hands. These blisters form when he is in the sun. He has a history of untreated chronic hepatitis C infection. While he tries not to drink, he admits to having one glass of wine over the holidays. He denies any abdominal pain. 1. Diagnose 2. What enzyme is deficient? In what pathway? 3. What accumulates? 4. Primary presentation 5. Common triggers 6. Stem might include history of Hepatitis __.

Porphyria Cutanea Tarda - most common porphyria. Blistering response to sunlight - Eschars, hyperpigmentation, mutilation Defective Enzyme: Uroporphyrinogen decarboxylase Accumulated Product: Uroporphyrin (tea-colored urine) Presenting Symptoms: Blistering cutaneous photosensitivity and hyperpigmentation. Exacerbated with alcohol consumption. Associated with hepatitis C (C for Cutanea) Triggers: 1. Iron overload (HFE mutations) 2. Hepatitis C infection 3. Alcohol Blistering cutaneous photosensitivity caused by hepatotoxic triggers (e.g., Hep C, alcohol) Autosomal dominant or sporadic defect in heme synthesis

Patient presents with poor balance, constricted pupil, alternating hemianalgesia, and reduced sensation of the face. What vessel is potentially responsible for these symptoms?

Posterior Inferior Cerebellar a. This is Lateral Medullary Syndrome (Wallenburg Syndrome) dysphagia (specific) hoarsness (specific) ↓ gag reflex vertigo ↓ pain and temperature sensation of the: -- ipsilateral face -- contralateral body Horner's syndrome

What is the most likely cause of these symptoms in a young patient: > Men - impotence, loss of libido > Women - amenorrhea, galactorrhea

Prolactinoma (pituitary tumor) Prolactin is a GnRH antagonist

Diffuse Large B-cell Lymphoma (DLBCL) - Flow cytometry is positive for CD__ - Involves activation of BCL-__, preventing apoptosis and cell differentiation.

Proliferation of very large B cells Most common form of Non-Hodgkin's Lymphoma. CD20 + BCL-6 is activated, preventing apoptosis and cell differentiation => cancer. - no specific translocation is associated with it.

What are the causes and effects on liver of: - Prehepatic blood flow obstruction (and what are the 2 vessels that supply the liver?) - Intrahepatic/Posthepatic blood flow obstruction Which type does not involve hepatomegaly? Which type causes a nutmeg liver?

Pre-Hepatic: Two blood supplies that could be occluded: 1. Hepatic artery - from PAN, transplant rejection 2. Portal Vein - from inflammation, PCV, tumor infiltration • Portal HTN, air in vein, splenomegaly, ascites • NO HEPATOMEGALY - no blood congestion; ischemia is the problem here. Intrahepatic/Posthepatic: • Centrilobular necrosis, Cirrhosis, Sickle cell, Budd-Chiari Syndrome • Centrilobular necrosis is c/b heart failure/ischemia • Nutmeg liver - from congestion • Painful hepatomegaly due to blood congestion in liver, inc. transaminases, may progress to cirrhosis

A 36-year-old woman at 34 weeks gestation presents to the emergency room for abdominal pain and headaches. She reports noticing these symptoms last night but attributed it to eating some take-out last night. On physical exam, she has tenderness to palpation in the epigastrium. Her blood pressure is 166/115 mmHg. She begins having tonic-clonic seizures. She is immediately prepped for delivery via cesarean section and started on anti-seizure medications.

Preeclampsia / Eclampsia Hypertension + Proteinuria + Seizures

Eosinophilic Granulomatosis with Polyangiitis (Churg Strauss syndrome) (Allergic Granulomatosis/Angiitis) - Presentation - Labs - Treatment

Presentation: 1. Increased allergy symptoms (esp asthma) 2. Eosinophilia (>10% WBC) - required - hence asthma and Eosinophilic G w/ P. 3. Eventually: Vasculitis typically develops as asthma / allergies improve. Labs: - c-ANCA, PR3 - Eosinophilia (>10% WBC) - Sinus opacifications - Chest x-ray: patchy or nodular lung infiltrates Treatment: high dose steroids - since this is inflammatory / autoimmune

What is the most common cause of and what will you expect of calcium levels in: 1. Primary Hyperparathyroidism 2. Secondary Hyperparathyroidism 3. Hypoparathyroidism (several)

Primary Hyperparathyroidism > High PTH > Primary means it's a problem with the thyroid itself. > Most common cause is a parathyroid adenoma. > Also, MEN-1 & 2 mutations cause Familial Primary Hyperparathyroidism. > Expect high calcium level, low phosphate > Groans, Stones, Overtones Secondary Hyperparathyroidism > High PTH > Secondary means something else is reducing effects of PTH. > Most common cause is chronic renal failure. Vitamin D is not activated and Ca2+ isn't reabsorbed in intestines or kidneys. As a result, excess PTH release. > Usually results in hyperplasia of the parathyroids (enlarged) > Expect low calcium level, high phosphate Hypoparathyroidism > Low PTH > Causes: DiGeorge, autoimmune parathyroid damage, and surgical excision of parathyroids. > Low PTH, Low Ca2+ > Hypocalcemia: Chvostek and Trousseau signs

A 25-year-old man presents to the clinic for an annual check-up. He is otherwise healthy and has no significant concerns. Basic laboratory studies demonstrate a sodium level of 129 mg/dL (LOW) and potassium level of 5.9 mg/dL (HIGH). A physical examination demonstrates hyperpigmentation of the oral mucosa. Review of systems were positive for generalized fatigue, 5-lbs unintentional weight loss, and some headaches.

Primary adrenal insufficiency (Addison's disease) - Often develops slowly. Both cortisol and aldosterone are decreased. - There is hyperpigmentation because of increased ACTH. - Autoimmune destruction of adrenal glands and TB are primary causes.

++++++++++++++++++++ Types of Adrenal Insufficiency: - Primary Adrenal Insufficiency (Addison's Disease) - Secondary Adrenal Insufficiency - Tertiary Adrenal Insufficiency - Waterhouse-Friderichsen Syndrome 1. Cortisol Levels 2. Aldosterone Levels 3. Hyperpigmentation? 4. Etiology / Cause

Primary adrenal insufficiency (Addison's disease) - Often develops slowly. Both cortisol and aldosterone are decreased. Due to adrenals not releasing them. - There is hyperpigmentation because of increased ACTH trying to increase cortisol. - Autoimmune destruction of adrenal glands and TB are primary causes. Secondary adrenal insufficiency: Occurs when ACTH secretion is decreased, so adrenals don't get the signal to release cortisol. No hyperpigmentation and aldosterone secretion is not decreased (ACTH controls cortisol release only, but both are released by adrenals -- RAAS controls Aldo release). Tertiary adrenal insufficiency: Occurs when there is an abrupt withdrawal of chronic exogenous glucocorticoids. ACTH and cortisol are depressed. There is no hyperpigmentation and aldosterone is not affected. Waterhouse-Friderichsen syndrome: Acute adrenal insufficiency following hemorrhage associated with septicemia (often Neisseria meningitidis). Dr. Quick: Syptoms of Adrenal Insufficiency: Hypotension (mineral and glucocorticoid loss = sodium loss = hypotension) Hyperpigmentation (ACTH stimulates melanocytes) Hyponatremia & Hyperkalemia due to loss of aldosterone (Na lost and K retained) Hypoglycemia - decreased cortisol (gluconeogenic) Metabolic acidosis (loss of proton pump) (c/w high K+) Eosinophilia from loss of cortisol

Primary hyperparathyroidism: - Usually caused by what? - What happens to plasma calcium? - What happens to plasma PO43-?

Primary hyperparathyroidism is usually due to a single adenoma, which causes increased secretion of PTH, resulting in high serum Ca2+. Normally, the high serum Ca2+ would feedback to decrease PTH, but the tumor cells still produce PTH. -- 20% of hypercalcemic patients have bone metastases that produce hypercalcemia by eroding bone. Results in: - Increased total plasma calcium and increased ionized Ca2+ in the plasma. - Decreased plasma PO43- and increased urinary PO43- - Urinary Ca2+ can vary >> Decreased urinary Ca++ due to reabsorption of Ca++ at the level of the kidney tubule >> Increased urinary Ca++ when the plasma Ca++ increases in concentration to the point that the filtered load is greater than the kidney can reabsorb

Levels of protein structure: - Primary structure - Secondary structure - Teriary structure - Quaternary structure

Primary: Sequence of amino acids Secondary: Presence of alpha-helices or beta-sheets Tertiary: Unique three dimensional folding of the molecule Quaternary: Interactions of a protein with other subunits of enzymes

A 70 year old man presents to his family practice doctor complaining of right-sided flank pain that extends to his back. The doctor is able to palpate the patients right kidney, but not his left. The patient's urine dipstick reveals microscopic hematuria. He is diagnosed with Renal Cell Carcinoma. - What is the most common type of RCC? - What are the 4 common PEARaneoplastic syndromes? - What will the kidney look like grossly and what will be present on histology? - 3 big risk factors for RCC - RCC originates in what part of the kidneys? - RCC often involves the ___ vein and can extend to the ___. - RCC most often metastasizes where?

Renal Cell Carcinoma: -Clear cell RCC is the most common type (80%) -MNEM: PEARaneoplastic: Excess secretion of PTH, EPO, ACTH, & Renin -Grossly, bright yellow (due to lipids). -Microscopically, you see clear cells that are clear because they contain lipid and glycogen. -Associated with: > Smoking > Von Hippel Lindau (chromosome 3 -- RCC is 3 letters) > PCKD (adult type) -Originates in the proximal tubules. -PATTERN: Invades renal vein ➙ IVC ➙ heart ➙ circulation ➙ hematogenous spread ➙ bones, lungs, heart. > Thus, it can spread to the heart!!! - Hematogenous metastasis to the: > Lung (50-60%), multiple, cannonballs > Bone > Lymph Nodes > Skin

++++++++++++++++++++++ A 2-month-old baby boy is brought in for an urgent visit to the pediatrician. He has had several ear infections in his short lifetime and now seems to be struggling with a cold. On physical exam, his tongue is noted to be coated with white film. His scalp and face are covered with a flaky dandruff-like substance. Immediately concerned, his pediatrician orders a chest radiograph, which shows absence of thymic shadow. Flow cytometry is ordered. 1. Diagnose 2. Caused by an absence of what type of immune cell? 3. Most common genetic mutation?

Severe Combined Immunodeficiency (SCID) ↓ Lymphocyte count (< 3000/μL) Chest radiography with no thymic shadow Flow cytometry: absent T-cells abnormal function of B-cells Most common genetic mutation is in the gamma chain of IL-2, 4, 7, 9, 11, 15 receptors. Thus, T-cells will be absent. Other forms have mutations in Adenosine Deaminase (ADA) gene or JAK3 gene.

+++++++++++++++++++++++++++++++++++ Diagnose: A 50-year-old woman presents to her physician's office for a routine checkup. Once at the office, she reports that she has been generally doing well but recently noticed that her fingers tend to turn blue in the cold. She reports feeling a general skin tightening in her face and hands, which makes forming a fist difficult. She also notes that she has had increased acid reflux lately and requests a medication for that. Her past medical history includes autoimmune thyroid disease and alopecia areata. Physical exam reveals sclerodactyly, and tight, hardened skin with limited mobility in her fingers. Her physician sends her for additional autoimmune workup.

SCLERODERMA YOU NEED TO KNOW THIS Scleroderma is the S and Raynaud's is the R in CREST Syndrome CREST Syndrome: Calcinosis, Raynaud's phenomenon, Esophageal dysmotility, Sclerodactyly, and Telangiectasia. Pathogenesis: - deposition of collagen on skin and internal organs - proliferation of scar tissue in microvasculature, leading to a vasculopathy - chronic inflammation with activation of humoral and cellular immunity -- increased release of inflammatory cells initiates and propagates the fibrotic process Esophageal dysmotility atrophy of smooth muscles in esophagus can cause ↓ lower esophageal sphincter pressure and dysmotility, leading to increased dysphagia and acid reflux

++++++++++++++++++++++++++ A 40 year-old woman with no previous medical illnesses is referred with progressive pain and stiffness of her hands and forearms for approximately 2 years. Originally her hands were slightly puffy, but now they feel tight and achy. She also complains of fatigue, slight dyspnea with exertion, difficulty swallowing and a painful sore on her fingertips. She denies joint swelling, chest pain, hair loss, skin rash, abdominal pain, proximal weakness or numbness. On skin examination: the patient has a slightly decreased oral oriface with a purse string mouth, tight skin of the fingers, hands and distal forearm and a few partially healed fingertip ulcers with loss of skin folds over the PIP joints and slightly decreased finger ROM.

Scleroderma KNOW THIS!!! Sclero- hard Derma- skin Inappropriate fibrosis of tissues and blood vessels. Systemic Sclerosis (SSC)=Primary Systemic Sclerosis (PSS)=Scleroderma Important and Distinguishing Features: Presence of: - Tight skin - Dyspnea on exertion - Dysphagia - Sore on her fingertip And the absence of: - Joint swelling - Hair loss - Skin rash Raynaud's phenomenon (fingers or toes turn blue then red with reperfusion) occurs in >90% of patients with scleroderma. Antibodies: - >95% of patients with PSS have an ANA - anti-Centromere antibody indicates patients with limited PSS. - anti-Scl-70 (anti-topoisomerase 1) antibody indicates patients with diffuse PSS and is associate with a worse prognosis. This antibody is directed against topoisomerase I, an enzyme that participates in the initial uncoiling of DNA prior to transcription. - anti-RNA polymerase III antibody- associated with renal crisis

++++++++++++++++++++++++++++++++ Visual Field Defects: What will the vision impairment be if each is damaged: 1. Optic nerve 2. Optic chiasm 3. Optic tract 4. Meyer's loop (optic radiations) 5. Superior occipital lobe 6. Inferior occipital lobe

See Image

Effect of nitroprusside or any other vaso/venodilator on the LV Pressure Volume Curve/Loop and on end systolic V and P.

Shorter and Wider Nitroprusside is a vasodilator AND venodilator. Vasodilators will decrease the afterload, so the loop will be shorter and extend to the left more. Venodilators will decrease the preload (more room in the veins), decreasing the LV volume so the curve won't go as far to the right. End systolic volume and pressure in the left ventricle will be lower

Compare the appearance of Sickle Cells to Schistocytes "Don't Confuse These"

Sickle Cells can cause crises because they group together and occlude. Schistocytes (MNEM: shaped like an eye - Schistoceyets") - These are fragmented red blood cells that can take on different shapes. They can be found as triangular, helmet shaped, or comma shaped with pointed edges. They are most often found to be microcytic with no area of central pallor.

Classic histological appearance of a Meningioma (picture it)

Spindle cells concentrically arrange in a whorled pattern. Psammoma bodies (laminated calcifications) Most common in females (has estrogen receptor positivity). They typically do not invade the brain parenchyma, but instead can be found pushing into the brain.

A 35-year-old male presents for follow-up. He started taking isoniazid 2 months ago for latent tuberculosis. Physical examination is notable for decreased sensation to pinprick in his lower extremities. A bone marrow aspirate is obtained and after staining with Prussian blue, RBCs with dark rings around the outer border are noted. LABS: - Decreased MCV (microcytic) - Inc. serum iron - Inc. ferritin (primary form of iron stored inside of cells) - Decreased TIBC (with Inc. transferrin saturation) - Dimorphic RBC population 1. Diagnose this Anemia 2. Explain these diagnostic lab results, like the high Fe, microcytic, etc. 3. Name and appearance of the classic bone marrow aspirate findings. And... 4. Why do these abnormal RBCs form? 5. What would be the diagnosis if the bone marrow aspirate showed basophilic stippling?

Sideroblastic Anemia Bone marrow aspirate will show Ring Sideroblasts. May also show Pappenheimer bodies (iron accumulates) and Dimorphic RBC population in the bone marrow aspirate with sideroblastic anemia. Ring Sideroblasts (IMAGE) are insoluble iron complexes in mitochondria of erythroid precursors. Definitely know the histological appearance of ring sideroblasts. Here, iron levels are high or normal, but it accumulates in the mitochondria of immature RBCs in the bone marrow. Sidero- is from the Greek sideros, meaning iron - So you might imagine that sideroblastic anemia is another anemia caused by an iron problem. But it isn't, exactly. Sideroblastic anemia is actually caused by an inability to form the structure that holds iron (the protoporphyrin ring). There's enough iron around, and it is available to developing red cells, but there's nowhere to put it! So the iron accumulates in cytoplasmic granules, eventually forming a ring of granules around the nucleus of the red cell precursor. Such cells are called ringed sideroblasts—and they are the basis for the name sideroblastic anemia. LABS: - Inc. serum iron - Inc. ferritin (primary form of iron stored inside of cells) - Dec. TIBC (with Inc. transferrin saturation) - Dec. MCV (microcytic) or Dimorphic RBC population Basophilic stippling is present in lead poisoning (precipitates of ribosomes and mitochondria).

++++++++++ HIGH YIELD ++++++++++ Sendond Messengers for Smooth MM. SMOOTH MUSCLE VASODILATORS: 1. The ß2 receptor causes both vasodilation and bronchodilation using what second messenger? 2. The α1 receptor causes vasoconstriction using what 2 second messengers to increase __ levels, which causes vasoconstriction. 3. Nitrous Oxide (NO) causes vasodilation through what second messengers.

Skeletal Muscles: • ß2 vasodilation: ↑cAMP Smooth Muscles: • α1 vasoconstriction: IP3→↑Ca++ • NO → ↑cGMP in SM → vasodilation -----Another Slide----- • α1 vasoconstriction / venoconstriction • ß2 vasodilation (skeletal muscle arteries) • α2 presynaptic block of NE release • ß1 heart stimulation (↑ CO via HR and contractility) • ß2 bronchodilation

What type of esophageal cancer is associated with: - Smoking and drinking - GERD and Barrett's esophagus

Squamous cell carcinoma of the esophagus is associated with smoking and drinking whereas adenocarcinoma is associated with reflux and Barretts

A 36 year old female presents to her doctor for a check up. She is noted to be obese with a BMI of 38. Blood work performed at the time shows her AST and ALT to be elevated (2x the normal level). Her alkaline phosphatase is normal. She stated during her history that she only drinks socially and does not do any drugs. A liver biopsy is performed to determine the cause of her elevated enzymes. What is the diagnosis and what will the liver look like?

Steatohepatitis, the liver will have numerous fat droplets present in the hepatocytes (fatty liver) "Steato" means fat-related.

Stranger anxiety: What age range can it be diagnosed? (thus, pt in stem will be a baby in this age range) School phobia in a kid (terrified of going to school): Treatment - E.g., a child is very nervous and refuses to go to school

Stranger anxiety: 6-12 months (6 months to 1 year old) - Dr. Spollen said they want you to know that this is "not PTSD" -- so remember to eliminate PTSD if it's an option and the pt is 6-12 months old / exhibits stranger anxiety. School "Phobia" in a kid: E.g., a child is very nervous and refuses to go to school - Send them to school! - Essentially, this is exposure therapy - A phobia is a strong fear of something for no obvious reason (so not from bullying, etc.; just irrational fear and avoidance, just like a spider phobia, etc.)

T-Cell Maturation: Explain what stays in positive selection and what is deleted in negative selection of T-cells

Summary: - Positive Selection: Thymus Cortex; T cells w/ TCR that bind self MCH survive. - Negative Selection: Thymus Medulla; apoptosis of T cells that react to self antigen. Prevents autoimmune reactions. Positive selection in the thymus keeps T-cells that expresses MHC class I and class II molecules. T-cells that interact with class I or class II molecules are signaled to proliferate and mature, resulting in single-positive CD4 or CD8 thymocytes. The choice of CD4 or CD8 is determined according to whether the TCR interacts with class II or class I, respectively. Thymocytes that survive positive selection undergo negative selection by thymic macrophages and dendritic cells, which deletes T cells with high affinity for self-MHC. Naive T cells that survive both positive and negative selection leave the thymus and enter the blood.

+++++++++ HIGH YIELD, EASY POINT +++++++++ Superantigens are antigens that produce intense, massive immune responses by binding directly to ___ and ___, which over-stimulates the release of ___.

Superantigens bypass the normal T-cell activation through MHC presentation of an antigen. They do this by directly binding both MHC and T-cell Receptors, over-activating both. This causes a dramatic release of inflammatory cytokines by inappropriately activated T-cells, resulting in a massive immune / inflamatory response.

Horner's syndrome: -clinical signs -difference from occulomotor palsy

Sympathetic trunk / cervical ganglion damage -- so you get parasympathetic sxs: PARTIAL ptosis, pupil constriction, vasodilation, and absence of sweating. - i.e., parasympathetic sxs MNEM for ptsosis: here, makes sense that it's parasympathetic. You open your eyes wide when scared; they droop when calm. This is damage at the level of the medulla. Difference from occulomotor (CN3) palsy: -There should be NO eye mm./movement deficits. CN3 damage = down and out. But not Horner's.

Birth Defects - Name each category of multiple concurrent defects: - multiple anomalies of 2 or more organ systems with a common cause - patterns of birth defects that occur together with a high frequency but no specific common cause - a series of anomalous findings attributable to an early abnormality of embryogenesis with a cascading effect

Syndromes - multiple anomalies of 2 or more organ systems with a common cause - E.g., Turner Syndrome -- all the sxs are due to the chromosomal abnormality. - E.g., Goodpasture Syndrome -- antibodies against basement membrane, which affects kidneys and lungs, leading to hemoptysis and kidney failure. Associations - patterns of birth defects that occur together with a high frequency but no specific common cause - E.g., MURCS association: Müllerian duct anomalies Renal anomalies Cervico-thoracic Somite dysplasia - Common link = altered blastema (pluripotent stem cells) of lower mesoderm with altered spatial relationship. Sequences - series of anomalous findings attributable to an early abnormality of embryogenesis with a cascading effect - E.g., cleft palate can result from early hypoplasia of the mandible.

Features of Systolic Heart Failure vs. Diastolic Heart Failure 3 Main Causes for each

Systolic HF: 1. Heart disease, such as MI, stroke - Most important - Compensation for MI is to increase volume to maintain CO => cannot be sustained and heart is stretched out. 2. Chronic volume overload, such as valve regurge 3. Dilated Cardiomyopathy 2 and 3 stretch out the heart, making its walls weaker => reduced contractility. Dialated heart. Heart can't pump as well. Reduced contractility, by definition. S3 sound - diastolic gallop Diastolic HF: 1. Left Ventricular Hypertrophy 2. Restrictive Cardiomyopathy 3. Pericardial contriction or tamponade Hypertrophic heart Can't fill as well because less space for blood. S4 sound

Murmur Basics: 1. What are the 2 systolic murmurs? 2. What are the 2 diastolic murmurs? (MNEMONIC)

Systolic murmur -Normal systole is A opening and M closing -Abnormal: A stenosis, M regurgitation - ASD, MVP, HOCM, VSD - MR. S AS Diastolic murmur -Normal diastole is A closing and M opening -Abnormal: A regurgitation, M stenosis - MS. D AR

Diagnose: Classic in Japanese women who are < 40 years of age Weak pulse and decreased blood pressure in the upper extremities Fatigue, malaise, weight loss, and arthralgias MNEM: FAN MY SKIN On Wednesday Fever Arthritis Night sweats MYalgia SKIN nodules Ocular disturbances Weak pulses

Takayasu's Arteritis (Pulseless Disease) - Granulomatous thickening of aortic arch and proximal great vessels - Affects medium and large vessels - Increased ESR - Asian females less than 40 years old - It's the thickening that makes it pulseless MNEM: FAN MY SKIN On Wednesday Fever Arthritis Night sweats MYalgia SKIN nodules Ocular disturbances Weak pulses

Diagnose: - Elderly (> 50 years of age) patient with a headache in the temporal region, often worse with chewing - Pain upon palpation of the temporal artery - 50% will present with jaw claudication - Abnormalities in vision may occur (e.g., diplopia or blindness) when the ophthalmic artery is involved - May be associated with polymyalgia rheumatica SXS: unilateral headache, jaw claudication, vision problems (opthalmic artery)

Temporal (Giant Cell) Arteritis SXS: unilateral headache, jaw claudication, vision problems (opthalmic artery). Pain upon palpation of the temporal artery. - Medium and large arteries - Most common vasculitis - Focal, skip lesions, granulomatous - Elderly females, carotid artery branches (i.e. temporal artery)

Name the cancer based on these buzz words: - Eye lid droop - Episodic hypertension - Baby with a sacral tumor - New onset seziure or vomiting in AM (B9 and Malignant) - Easy bleeding

Teratoma: tumor made up of several different types of tissue, such as hair, muscle, teeth, or bone. GMB = Glioblastoma Multiforme

Relapsing and Remitting: A 29 year old woman presents to her family practice doctor with a sudden onset of speech problems, vision problems and transient incontinence. She is diagnosed with multiple sclerosis. She is treated with immunosuppressant therapy and a LP is performed. What CSF electrophoresis finding is diagnostic for Multiple Sclerosis?

The one on the RIGHT. The electropherogram on the right is abnormal as it has increased number of bands in the upper half of the gel. This is called "oligoclonal banding" that can be seen in the CSF. This is diagnostic of MS. MS is an autoimmune disease leading to inflammation and demyelination of the CNS. Clinical tests include: ▫ Spinal tap with CSF electrophoresis ▫ Increased Protein & IgG in CSF ▫ MRI (periventricular plaques)

Lichen Sclerosis: - Presentation - Increases risk for what vulvar cancer

White plaque on vagina in a post-menopausal woman. Epidermal thinning and dermal "hyalinization" too. Slight risk for Vulvar SCC - HPV is the #1 risk for vulvar cancer Lichen Simplex Chronicus - Any age - White, scaly, thickened vulva (due to hyperkeratosis) - Hyperkeratosis, hypergranulosis, acanthosis - Due to "itch scratch cycle"

A 33-year-old female is referred to a neurologist by her primary care physician for unsteady gate, forgetfullness, and recent episodes of Tourette-like spells where she will fling one arm out and above her head seemingly unprovoked. One physical exam, the neurologist notices that her irises appear multicolored with concentric rings around the perpiphery. He is concerned for a metabolic disease and orders several laboratory and radiologic studies. In the interim he has advised her to avoid eating chocolate and shellfish.

Wilson's Disease Copper deposits! Liver and Brain are affected.

Fragile X Syndrome: - Caused by what type of mutation on what gene - Main symptoms - What test can you use to detect it?

Trinucleotide repeat (CGG) on FMR1 gene of the X-chromosome. It's X-linked. - FMR1 protein is underexpressed as a result - MNEM: Fragile X causes retardation so these kids need an fMRI (FMR1 gene) and it's X-linked. Sxs: - Mental Retardation: most common cause of retardation in boys, and 2nd most common cause overall. Tends to resemble autism (no eye contact, flapping hands, etc.) - Long slender facies, large protuberant ears, lax joints, low tone, macro-orchidism • Detected with Southern blot or PCR MedBullets Example: A 13-year-old boy is brought to his pediatrician. He has cognitive delays, a prominant jaw, large soft ears, and large testicles.

What type of hypersensitivity reaction can cause bleeding from the lungs and the kidneys, usually at the same time? What is this syndrome called? Describe the pathogenesis of this syndrome.

Type II - Antibody dependant Goodpasture syndrome Anti-basement membrane IgG damages the basement membranes in the glomerular basement membrane and the basement membranes in the lungs leading to hemorrhage in both locations (which is the clinical scenerio you should be looking for). Hemoptysis + Hematuria

A 4 year old girl has severe pruritic dermatitis on her legs and hands. The parents indicate that two days ago, they did some hiking and their daughter wandered off the trail into some low brush a couple of times chasing after their dog. The family often takes this path but this is the first time that their daughter has developed the dermatitis. What type of Hypersensitivity is this?

Type IV Hypersensitivity (Delayed-type Hypersensitivity) - Mediated by CD4 Helper T-cells (memory cells, c/w delayed response) She likely has poison ivy. • Delayed response (24-48 hours) • Requires previous exposure to allergen • Low molecular weight molecules (haptens) • Attach to skin proteins and present as new epitopes

Refresher on Blood Typing and who can receive from who - Universal RBC donors and recipients (based on RBC antigens) - Universal Plasma donors and recipients (based on plasma antibodies)

Universal red cell donor: O negative - No A, B, or Rh antigens on their red cells Universal red cell recipient: AB positive - No A or B antibodies in their serum ___________________________ Universal plasma donor: AB - No Anti-A or Anti-B antibodies in their serum - Rh is less important since there are few red cells in the plasma Universal plasma recipient: O - No Anti-A or Anti-B antigens on their red cells

All of these are clues to which vitamin deficiency? HIGH Homocysteine HIGH Methylmalonic acid Dietary Strict vegan (but most take supplements) Elderly with "tea and toast" diets GI - Chronic diarrhea - Gastric/duodenal resection - Terminal ileum resection - Chronic proton pump inhibitor therapy (PPI - Prilosec, Nexium, etc.)

VITAMIN B12 DEFICIENCY (Cobalamin) Aborption occurs in the GI tract and requires Intrinsic Factor (IF) which requires gastric acid

Intrauterine exposure to diethylstilbestrol (DES) causes what cancer?

Vaginal Clear Cell Adenocarcinoma

ANTICOAGULANTS Warfarin inhibits what? Tissue Plasminogen Activator (t-PA) - MoA Unfractionated Heparin and Low Molecular Weight Heparin (which one is more reversible?) Direct Factor Xa Inhibitors (-aban) Clopidogrel (Plavix) inhibits ___

Warfarin Vitamin K And therefore, Factors 2, 7, 9, & 10 and Proteins C and S TPA Mechanism of Action: increase conversion of plasminogen to plasmin which degrades fibrin Indications: limit clot formation and remove existing clots. - Used for: > myocardial infarction > ischemic stroke > pulmonary embolism Unfractionated Heparin (UFH) - cofactor for the activation of Antithrombin - immediate effect for PE, acute coronary syndrome, dialysis - short half-life - UFH requires monitoring with aPTT or heparin anti-Xa - reversible with protamine Low-molecular weight heparins (LMWH) - cofactor for the activation of Antithrombin - Longer half life (4 hours) - Not easily reversible - LMWH doesn't require routine monitoring Direct Factor Xa Inhibitors - Directly block Factor Xa to block clot formation. - Do NOT require monitoring - But NOT easily reversible Clopidogrel (Plavix) Irreversibly inhibits P2Y12

Transcortical aphasias are caused by ___ strokes

Watershed Strokes An ischemia, or blood flow blockage, that is localized to the border zones between the territories of two major arteries in the brain. Watershed locations are those border-zone regions in the brain supplied by the major cerebral arteries where blood supply is decreased. Mesial Frontal and Temporal-parietal junction are the two transcortical areas.

A 34-year-old man presents to his doctor's office for a rash of several weeks. He reports also having allergic rhinitis in the past few weeks, which was strange given that he normally has allergic rhinitis in the spring. Additionally, he reports having had childhood asthma, but in the past few months, he has had to use his inhaler for episodes of asthma. On physical exam, there is palpable purpura and urticarial plaques on the lower extremities and several skin nodules. Wheezes are heard on lung exam. Additional laboratory tests reveal peripheral eosinophilia with elevated IgE levels and c-ANCA. A chest X-ray shows nodular densities in the lungs. A UA reveals RBCs (hematuria).

Wegener Granulomatosis (Granulomatosis with Polyangiitis) Triad of findings: ✔ Necrotizing vasculitis (small vessels) ✔ Necrotizing granulomas of lung and upper airway ✔ Necrotizing glomerulonephritis Symptoms (lots of blood): ✔ Perforated nasal septum ✔ Hemoptysis ✔ Hematuria ✔ Cough, SOB, ear and sinus infections Clinical Diagnosis: 1. Urine - RBC casts, blood 2. Serum - c-ANCA 3. Chest X-ray - lung nodular densities Treatment - steroid & immunosuppressant

Posterior Cerebral Artery stroke: -clinical signs -what structures are at risk? (one CN and one long tract) -define mydriasis

Well it's a cerebral artery, so it'll be up high. It can affect: - CN III -- mydriasis, FULL ptosis, failure of all but LR & SO. So eye is DOWN & OUT & CLOSED and pupil is BLOWN. - Lateral corticospinal tract (contralateral weakness) Mydriasis = blown/dialated pupil

INFLAMMATORY CELLS: Histology: - Histiocyte - Neutrophil - Eosinophil - Lymphocyte - Basophil \ Mast cell - Plasma cell

What is each inflammatory cell associated with? Neutrophils - Bacteria, necrosis (MI) Eosinophils - Parasites, fungus Histiocytes (granuloma) - TB Mast cell - Histamine release (allergies) Lymphocyte - Viral Plasma cell - Viral and bacterial

++++++++++++++++++++++++++++++++++++++ An 8-year-old boy is brought to his pediatrician for easy bruising. On physical exam, he is found with petechiae and purpura in multiple areas over his body, as well as bruises over his arms. Eczematous patches are also found on his flexural surfaces. Laboratory results reveal thrombocytopenia to 30,000/mm3. Further questioning reveals a past medical history of multiple hospital stays due to pneumonia and otitis media infections as well as recurrent epistaxis. 1. Diagnose 2. What gene is mutated? (literally an acronym of the DZ name) 3. What is the classic triad of sxs seen in this disease? (MNEM)

Wiskott-Aldrich Syndrome Mutation: WAS gene - literally, an acronym for the name of the DZ - results in defective T cell signalling and interactions with APCs - also impairs phagocytosis and chemotaxis Immune deficiency disorder - Clinical Triad: WATER - Wiskott-Aldrich causes: ✔ Thrombocytopenic Purpura (low plts, easy bruising, purpura, petechiae) ✔ Eczema ✔ Recurrent infections

How to recognize an X-linked Recessive pedigree

X-linked mutant genes are fully expressed in males, who have only a single X chromosome, i.e., are hemizygous for X-linked genes. Fathers must transmit their Y chromosome to their sons, thus there is no male-to-male transmission of X-linked genes. Females can be carriers but won't be affected since it's recessive. Not all sons are affected since the mother can pass the mutated or a normal X chromosome.

Heart failure patient. The compensation will be ↑ or ↓: - RAAS (Renin, Angiotensin, Aldosterone) - ADH - ANP

↑ RAAS ↑ ADH ↑ ANP All 3 are going to increase. HF means reduced CO. Not meeting demand. Similar to shock, RAAS and SNS are going to kick in to increase CO. RAAS kicks in because kidney perfusion decreases. ADH will kick in to try to increase volume to improve CO. ANP detects blood volume in the heart, and since there's a lot of blood inside the heart in HF, it will think there's high blood volume and will increase to try to fix that. • ANS responds to baroreceptor stretch (usually @ carotid) • RAAS responds to renal perfusion • ANP responds to atrial stretch (central blood volume) • ADH: 1) osmolarity, 2) very low volume (may override the osmolarity response and mess up plasma sodium levels!)


Conjuntos de estudio relacionados

Module 10 Review - Volume and Surface Area (copy)

View Set

Chapter 7 Biology: Cellular Respiration

View Set

Prep U chapter 57 management of patients with burn injury

View Set